Download as pdf or txt
Download as pdf or txt
You are on page 1of 410

Solutions to end-of-chapter problems

Basics of Engineering Economy, 3rd edition


Leland Blank and Anthony Tarquin

Chapter 1
Foundations of Engineering Economy

1.1 In an economic analysis of alternatives, list three items that are typically included as part of
the evaluation.

Purchase cost, useful life, maintenance & operating costs, salvage value, inflation rate, and
interest rate.

1.2 What is the name of the alternative that represents the status quo?

The alternative that represents the status quo is called the do-nothing alternative.

1.3 What is the primary evaluation criterion used in engineering economic analysis?

The primary evaluation criterion is cost.

1.4 List three evaluation criteria besides initial cost that might be used for selecting the best
automobile to purchase.

Fuel rating, crash protection, acceleration, body styling, color, environmental impacts, and
resale value.

1.5 Identify the following factors as either economic (tangible) or noneconomic (intangible);
first cost, leadership, taxes, salvage value, morale, inflation, profit, acceptance by others,
interest rate.

First cost: economic; leadership: non-economic; taxes: economic; salvage value: economic;
morale: non-economic; inflation: economic; profit: economic; acceptance by others: non-
economic; interest rate: economic.

1.6 Identify the following factors as either tangible or intangible: sustainability, installation cost,
transportation cost, simplicity, resale value, rate of return, dependability, deflation rate, and
ethics.

Sustainability: intangible; installation cost: tangible; transportation cost: tangible;


simplicity: intangible; resale value: tangible; rate of return: tangible; dependability:
intangible; deflation rate: tangible; ethics: intangible.

Copyright 2021 © McGraw-Hill Education. All rights reserved. No reproduction or distribution without the prior
written consent of McGraw-Hill Education.
1
1.7 What is meant by the term time value of money?

Time value of money means that there is a certain worth in having money, and that worth
changes as a function of time.

1.8 Interest is a manifestation of what general concept in engineering economy?

Interest is a manifestation of the time value of money.

1.9 Of the fundamental dimensions length, mass, time, and electric charge, which one is the
most important in economic analysis? Why?

The most important fundamental dimension in economic analysis is time because of time
value of money.

1.10 Identify at least five areas of personal finances wherein you might use economic analysis in
the future.

Examples are: house purchase; car purchase, credit card (which ones to use); personal loans
(and their rate of interest and repayment schedule); investment decisions of all types; when
to sell a house or car.

1.11 In economic calculations, how is the time value of money recognized?

The time value of money recognized through payment of interest

1.12 A small Voice over Internet Protocol (VoIP) provider borrowed $2 million for new
equipment and repaid the principal of the loan plus $275,000 interest after 1 year. What
was the interest rate on the loan?

Interest rate = (275,000/2,000,000)*100


= 13.75%

1.13 In order to reduce so-called back-office costs, such as payroll and insurance costs, by
$45 million per year, trucking giant Yellow Corp. agreed to purchase rival Roadway LLC
for $966 million in cash. If the savings are realized as planned, what is the rate of return on
the investment?

Rate of return = (45/966)*100


= 4.7%

1.14 A small analytical laboratory borrowed $40,000 (the original amount or the principal) at an
interest rate of 8% per year to purchase a used Agilent gas chromatograph. If the lab repays
the loan plus interest in 1 year, how much must it pay at the end of the year?

Copyright 2021 © McGraw-Hill Education. All rights reserved. No reproduction or distribution without the prior
written consent of McGraw-Hill Education.
2
Amount due in one year = 40,000 + 40,000(0.08)
= 40,000 + 3,200
= $43,200

1.15 If the amount due now on a loan taken 1 year ago at an interest rate of 10% per year is
$18,000, what was the principal of the loan?

Amount due = principal + (principal)(interest rate)


18,000 = P + (P)(0.10)
1.1P = 18,000
P = $16,363.64

1.16 If you expect to receive an inheritance of $100,000 one year from now, how much money
would you be able to borrow now at an interest rate of 8% per year if you pledge the total
amount of the inheritance against the loan?

Future amount = principal + (principal)(interest rate)


100,000 = P + (P)(0.08)
1.08P = 100,000
P = $92,592.59

1.17 Assume you just invested $130,000 in a high-rise condo venture in downtown Austin, one
of the top 10 hottest real estate markets in the country. The developers promise a rate of
return of 21% per year for as long as you have your money invested. (a) How much interest
will you receive at the end of 1 year, and (b) what is the total amount you will have after
1 year?

(a) Interest = (principal)(interest rate)


= 130,000*0.21
= $27,300

(b) Total amount = Principal + Interest


= 130,000 + 27,300
= $157,300

1.18 Emerson Processing borrowed $900,000 for installing energy-efficient lighting and safety
equipment in its manufacturing facilities. The terms of the loan were such that the company
would pay interest only at the end of each year for 4 years, after which the company would
have to pay the annual interest and entire loan principal after 5 years. If the interest rate on
the loan was 12% per year and the company paid only the interest for 4 years, (a) what was
the amount of each of the four interest payments, and (b) what was the amount of the final
payment at the end of year 5?

(a) Amt paid first four years = 900,000(0.12)


= $108,000

Copyright 2021 © McGraw-Hill Education. All rights reserved. No reproduction or distribution without the prior
written consent of McGraw-Hill Education.
3
(b) Final payment = 900,000 + 900,000(0.12)
= $1,008,000

1.19 A start-up chemical company has an average cost of capital of 15% per year. Additionally,
it has a long-term goal of making at least a 20% per year rate of return on all investments;
however, because of market opportunity the ROR can be reduced for the current project by
3%. If the company acquired $50 million in venture capital, how much did it have to earn
in the first year?

Expected MARR = 20% + 15% – 3% = 32%


Earnings = 50,000,000(0.32)
= $16,000,000

1.20 Bradley Instruments borrowed $3,500,000 from a private equity firm for expansion of its
manufacturing facility for making carbon monoxide monitors/controllers. The company
repaid the loan after 1 year with a single payment of $3,885,000. What was the interest rate
on the loan?

interest rate = [(3,885,000 – 3,500,000)/3,500,000](100)


= 11% per year

1.21 A friend of yours borrowed $550 for video game purchases and repaid the principal of the
loan plus $75 interest after 1 year. (a) What was the interest rate on the loan? (b) During
the same year, you borrowed $8000 for lawn mowing equipment to make money on the
side and paid it off in 1 year for a total of $8600. How does your interest rate compare to
that of your friend?

(a) Interest rate = (75/550)(100)


= 13.64%

(b) Interest rate = (600/8000)(100)


= 7.5%

You paid slightly more than ½ as much as your friend.

1.22 A design-build engineering firm completed a pipeline project wherein the company
realized a profit of $2.3 million in year 1 and $750,000 in year 2. If the amount of money
the company had invested was $6 million both years, what was the rate of return on the
investment for each year?

Year 1, ROR = (2.3/6)(100) = 38.3%

Year 2, ROR = (0.75/6)(100) = 12.5%

Copyright 2021 © McGraw-Hill Education. All rights reserved. No reproduction or distribution without the prior
written consent of McGraw-Hill Education.
4
1.23 Which of the following 1-year investments has the highest rate of return: $12,500 that
yields $1125 in interest, $56,000 that yields $6160 in interest, or $95,000 that yields $7600
in interest?

ROR = (1125/12,500)(100) = 9%
ROR = (6160/56,000)(100) = 11%
ROR = (7600/95,000)(100) = 8%

Therefore, the $56,000 investment has the highest rate of return

1.24 A new engineering graduate who started a consulting business borrowed money for 1 year
to furnish the office. The amount of the loan was $23,800, and it had an interest rate of
10% per year. However, because the new graduate hadn’t built up a credit history, the bank
made him buy loan default insurance for 5% of the loan amount. In addition, the bank
charged a loan setup fee of $300. What was the effective interest rate the engineer paid for
the loan?

Interest on loan = 23,800(0.10) = $2,380


Default insurance = 23,800(0.05) = $1190
Set-up fee = $300
Total amount paid = 2380 + 1190 + 300
= $3870
Effective interest rate = (3870/23,800)(100)
= 16.3%

1.25 How many years does it take for an investment of $280,000 to total at least $425,000 if the
return is 15% per year (a) only on the original amount each year, and (b) on the total
accumulated amount each year?

(a) Interest paid each year = 280,000(0.15) = $42,000


Amt. at end of year 1 = 280,000 + 42,000 = $322,000 < $425,000
Amt. at end of year 2 = 280,000 + 84,000 = $364,000 < $425,000
Amt. at end of year 3 = 280,000 + 126,000 = $406,000 < $425,000
Amt. at end of year 4 = 280,000 + 168,000 = $448,000 > $425,000

Time = 4 years

(b) Amt. at end of year 1 = 280,000(1.15) = $322,000 < $425,000


Amt. at end of year 2 = 322,000(1.15) = $370,300 < $425,000
Amt. at end of year 3 = 370,300(1.15) = $425,845 > $425,000

Time = 3 years

Copyright 2021 © McGraw-Hill Education. All rights reserved. No reproduction or distribution without the prior
written consent of McGraw-Hill Education.
5
1.26 At an interest rate of 15% per year, an investment of $100,000 one year ago is equivalent to
how much now?

Amt. now is F = 100,000 + 100,000(0.15)


= $115,000

1.27 As a principal in the consulting firm where you have worked for 20 years, you have
accumulated 5000 shares of company stock. One year ago, each share of stock was worth
$40. The company has offered to buy back your shares for $225,000. At what rate of return
would the firm’s offer be equivalent to the worth of the stock last year?

Let i = ROR

5000(40)(1 + i) = 225,000
1 + i = 225,000/200,000
1 + i = 1.125
i = 0.125 (12.5% per year)

1.28 At an interest rate of 8% per year, $10,000 today is equivalent to how much (a) 1 year from
now, and (b) 1 year ago?

(a) Equivalent future amount = 10,000 + 10,000(0.08)


= 10,000(1 + 0.08)
= $10,800

(b) Equivalent past amount = 10,000/1.08


= $9259.26

1.29 A medium-size consulting engineering firm is trying to decide whether it should replace its
office furniture now or wait and do it 1 year from now. If it waits 1 year, the cost is
expected to be $16,000. At an interest rate of 10% per year, what is the equivalent cost
now?

Equivalent cost now = 16,000/1.10


= $14,545.45

1.30 A design-build engineering company that usually gives year-end bonuses in the amount of
$8000 to each employee is having cash flow problems. The company said that although it
could not give bonuses this year, it would give each individual two bonuses next year; the
regular one of $8000 plus an amount equivalent to the $8000 they would have gotten this
year. If the interest rate is 6% per year, what will be the total amount of bonus money an
employee should receive next year?

Total bonus next year = 8,000 + 8,000(0.06)


= $16,480

Copyright 2021 © McGraw-Hill Education. All rights reserved. No reproduction or distribution without the prior
written consent of McGraw-Hill Education.
6
1.31 University tuition and fees can be paid using one of two plans:
Early-bird: Pay total amount due one year in advance and get a 10% discount
On-time: Pay total amount due when classes start
If the cost of tuition is $18,000 per year, determine (a) the tuition if paid in advance, and
(b) the equivalent amount of the early-bird savings compared to on-time payment.

(a) Early bird: payment = 18,000 – 18,000(0.10)


= $16,200

(b) Equivalent future is F = 16,200(1 + 0.10)


= $17,820
Savings = 18,000 – 17,820
= $180

1.32 An engineer told you that a project is economically acceptable when its rate of return
equals or exceeds the corporation’s cost of capital. Is this correct? Explain your answer.

The engineer is correct, if the MARR is exactly equal to the cost of capital. The engineer is
wrong, if a return greater than zero is required. Usually, the inequality ROR ≥ MARR >
cost of capital is used, and the MARR is established higher than the cost of capital so that
profit, risk and other factors are considered.

1.33 The MARR is not calculated like an ROR. What is the MARR used for in economic
analysis?

The MARR is used to make accept/reject decisions on projects and alternative selections.

1.34 Dawn is preparing a home office to perform subcontract projects for midsized architect
firms. She plans to use $15,000 of her own funds which currently generate a return of
4% per year. The remainder of financing will be provided by a $10,000 bank loan carrying
a 9% per year interest rate. She hopes to realize a return of 3% above the average cost of
capital to establish her office, and she realizes that the factors of inflation and risk should
also be considered. Her decision is to add another 2% per year to compensate for these
elements. What is the MARR she should use when evaluating projects?

Fraction of equity capital = 15,000/25,000 = 0.60


Fraction of debt capital = 10,000/25,000 = 0.40

WACC = 0.60(4%) + 0.40(9%)


= 6%

MARR = WACC + return above WACC + risk and inflation add-on


= 6% + 3% + 2%
= 11% per year

Copyright 2021 © McGraw-Hill Education. All rights reserved. No reproduction or distribution without the prior
written consent of McGraw-Hill Education.
7
1.35 Determine the WACC, MARR, and estimated ROR for the following situation at
Himalayan Air, Inc., which owns and operates oxygen bars across the United States and
Europe. The management plans a second major expansion of outlets with a hoped-for
return of 20% above cost of capital the first year of full operation. The first was very
successful with an excess revenue of $7 million on a $20 million investment during the first
year, leading to an ROR of 35%. Being more conservative due to market competition, the
return on the new expansion is estimated at 80% of the previously observed ROR. This
expansion will be funded using a 50-50 debt-equity mix. The equity funds currently return
6% per year and the debt financing will cost 9.7% per year. Is the expansion ROR expected
to exceed the MARR?

WACC = 0.50(6%) + 0.50(9.7%)


= 7.85%

MARR = 7.85% + 20% = 27.85%

Estimated expansion ROR = 0.8(35%) = 28%

Conclusion: Since 28% > 27.85%, the expansion is barely expected to meet the MARR
criterion. Any over-estimate in the ROR will make the expansion financial questionable.

1.36 In an effort to increase its customer base, a company set the project MARR at exactly the
WACC. If equity capital costs 9% per year and debt capital costs 11.75% for the project,
what is the equitydebt percentage mix of capital required to make the WACC = 10%?

Let X = percent equity capital

WACC = 10% = X(9%) + (1 – X)(11.75%)


(9 – 11.75)X = 10 – 11.75
X = 1.75/2.75
= 0.636 (63.6%)

Mix is 63.6% equity and 36.4% debt capital

1.37 Will and Ben Ice Cream plan to build a new mixing plant to serve customers in Mexico.
Because the company is in good financial shape with equity funds returning 11% per year,
the bank will charge an interest rate of 8% per year for the loan. An MARR that is 5% over
the WACC is required to proceed with the project, which sets the MARR at 14%. What
percentage of debt financing can the company assume to meet its MARR requirement?

Let X = percent debt capital

WACC = 14 – 5 = 9%

Copyright 2021 © McGraw-Hill Education. All rights reserved. No reproduction or distribution without the prior
written consent of McGraw-Hill Education.
8
WACC = 9% = X(8%) + (1 – X)(11%)
(8 – 11)X = 9 – 11
X = 0.667 (66.7%)

Mix is 66.7% debt and 33.3% equity capital

1.38 Raydeck International set aside a lump sum investment 4 years ago in order to finance a
plant expansion now. The money returned 10% per year simple interest. How much did the
company set aside if the investment is now worth $850,000?

F = principal + interest
= P + Pni

850,000 = P + P(4)(0.10)
1.4P = 850,000
P = $607,143

1.39 Dynamic Data LLC, a company that customizes software for construction cost estimates,
repaid a loan obtained 3 years ago at 7% per year simple interest. If the amount that
Dynamic Data repaid was $120,000, calculate the principal of the loan.

F = P + Pni
120,000 = P + P(3)(0.07)
1.21P = 120,000
P = $99,173.55

1.40 The Aberdeen Fixed Rate Fund pays a dividend of 10% per year simple interest. If you
invest $240,000 today, what total amount will you have accumulated at the end of 3 years?

F = 240,000 + 240,000(0.10)(3)
= $312,000

1.41 If EP Electronics sets aside $1,000,000 now into a contingency fund, how much will the
company have in 2 years if it doesn’t use any of the money and the account grows at a rate
of 10% per year?

F1 = 1,000,000 + 1,000,000(0.10)
= 1,100,000

F2 = 1,100,000 + 1,100,000(0.10)
= $1,210,000
or
F2 = 1,000,000(1.10)2
= $1,210,000

Copyright 2021 © McGraw-Hill Education. All rights reserved. No reproduction or distribution without the prior
written consent of McGraw-Hill Education.
9
1.42 Rambus Diagnostics has extra funds to invest for future capital expansion. If the selected
investment pays simple interest, what interest rate would be required for the amount to
grow from $60,000 to $90,000 in 5 years?

90,000 = 60,000 + 60,000(5)(i)


300,000 i = 30,000
i = 0.10 (10% per year)

1.43 You plan to place your savings into a high-yield account at your company’s Employee
Credit Union. How long will it take to double your money at 5% per year simple interest?

2P = P + P(0.05)(n)
P = P(0.05)(n)
n = 20 years

1.44 If your money earns compound interest at a generous 20% per year, how long will it take
$50,000 to accumulate to at least $85,000? Solve using two methods: (a) year-by-year
accumulated total, and (b) Equation [1.6].

(a) Amount Year 1 = 50,000 + 50,000(0.20) = $60,000< $85,000


Amount Year 2 = 60,000 + 60,000(0.20) = $72,000 < $85,000
Amount Year 3 = 72,000 + 72,000(0.20) = $86,400 > $85,000

Time = 3 years

85,000 = 50,000(1.20)n
1.70 = 1.20n
log 1.70 = n log 1.20
0.23045 = n(0.07918)
n = 2.9 years

1.45 In order to finance a new product line, a company that makes high-temperature ball
bearings borrowed $1.8 million at 10% per year compound interest. If the company repaid
the loan in a lump sum amount after 2 years, what was (a) the amount of the payment, and
(b) the amount and percentage of interest based on the original principal?

(a) Payment = 1,800,000(1 + 0.10)2


= $2,178,000

(b) Interest = 2,178,000 – 1,800,000


= $378,000

Percent of principal = (378,000/1,800,000)(100)


= 21%

Copyright 2021 © McGraw-Hill Education. All rights reserved. No reproduction or distribution without the prior
written consent of McGraw-Hill Education.
10
1.46 Fill in the missing values (A through D) for a loan of $10,000 if the interest rate is
compounded at 10% per year.
End of Interest Amount Owed End-of-Year Amount Owed
Year for Year After Interest Payment After Payment
0 — — — 10,000
1 1000 11,000 2000 9000
2 900 9900 2000 A
3 B C 2000 D

Follow calculations in Example 1.8

A is 9900 – 2000 = $7900


B is 7900(0.10) = $790
C is 7900 + 790 = $8690
D is 8690 – 2000 = $6690

1.47 Companies frequently borrow money under an arrangement that requires them to make
periodic payments of “interest only” and then pay the principal all at once at the end of the
loan period. If Cisco International borrowed $500,000 (identified as loan A) at 10% per
year simple interest and another $500,000 (identified as loan B) at 10% per year compound
interest and paid only the interest at the end of each year for 3 years on both loans, (a) on
which loan did the company pay more interest, and (b) what was the difference in interest
paid between the two loans?

(a) Loan A: interest/year = 500,000(0.10) = $50,000


Loan B: interest/year = 500,000(0.10) = $50,000

The same amount of interest was be paid on each loan

(b) There was no difference paid between the two loans

1.48 The NFL should update its online security systems now at a cost of $5 million to avert
cyberattacks by hackers. However, since there has been no breach thus far, the tendency is
to put off the upgrade and take the risk. If these costs are being inflated at 12% per year,
estimate the cost (a) next year, and (b) 2 years from now.

(a) Cost next year = 5,000,000(1.12) = $5,600,000

(b) Cost two years from now can be found using either of two relations.

Method 1: Cost in 2 years = 5,600,000(1.12)


= $6,272,000
or

Method 2: Cost in 2 years = 5,000,000(1.12)2

Copyright 2021 © McGraw-Hill Education. All rights reserved. No reproduction or distribution without the prior
written consent of McGraw-Hill Education.
11
1.49 Suppose you want to purchase a Dodge Ram 1500 pickup that costs $32,000 now, but you
don’t have the money. If the cost increases by 5% per year, and you plan to buy it for cash
in 5 years, estimate the future cost using two methods: (a) based on Equation [1.7], and
(b) the increased percentage shown in Figure 1.3.

(a) Cost in 5 years = 32,000(1 + 0.05)5


= 32,000(1.276)
= $40,832

(b) Cost in 5 years = 32,000 + 32,000(0.28)


= $40,960

1.50 You and your sister went shopping today for jeans. The pair she bought costs $49.95 and is
identical to a pair you purchased 2 years ago for $37.87. (a) Determine the numerical and
percentage increase in cost. (b) Determine the annual inflation rate of the cost increase and
compare your result to the increase indicated in Figure 1.3 (upper).

(a) Numerical increase = 49.95 – 37.87 = $12.08


Percentage increase = (12.08/37.87)(100) = 32%

(b) Use Equation [1.7]

Increase in cost: (1 + f)2 – 1 = 0.32


1 + f = 1.320.5
f = 0.1489 (14.89% per year)

Figure 1.3 indicates a cost increase of 32% when f = 15% per year, which is the same as
14.89%, rounded up.

1.51 An arc welder with AI software onboard costs $4500 today, $3750 last year, and $3400 one
year prior to that. Determine the 1-year rate of inflation for each year.

Last year: 4500 = 3750(1 + f)


1 + f = 1.20
f = 0.20 (20%)

Year before: 3750 = 3400(1 + f)


1 + f = 1.103
f = 0.103 (10.3%)

1.52 When the yield on a U.S. Treasury Bill, considered a “safe” investment, is 3% per year,
investors expect the inflation rate to be approximately what value?

When the yield is 3% per year on U.S. government bonds, investors expect the inflation rate
to be near zero.

Copyright 2021 © McGraw-Hill Education. All rights reserved. No reproduction or distribution without the prior
written consent of McGraw-Hill Education.
12
1.53 You have the $1200 to cover the discounted sales price of your favorite Shimano mountain
bike, but no safe storage area if you owned it today. The projected cost increase each year
is 5%. Determine the purchasing power of your $1200 if you wait (a) 2 years, and
(b) 5 years to spend it.

Based on Figure 1.3 Based on Equation [1.8]


(a) 2-year reduction in PP is 9% (a) PP = 1200/(1.05)2
PP = 1200(1 – 0.09) = $1092 = $1088

(b) 5-year reduction in PP is 22% (b) PP = 1200/(1.05)5


PP = 1200(1 – 0.22) = $936 = $940

1.54 Assume you received $500 for your birthday today and placed it in your no-interest
checking account for safe keeping. Determine the purchasing power 3 years in the future if
(a) f = 3.5% per year, and (b) f = 10% per year. (Round answers to the nearest dollar.)
(c) How much more dollar purchasing power was lost with the higher inflation rate?

(a) PP3 = 500/(1.035)3


= 500/1.1087
= $451

(b) PP3 = 500/(1.10)3


= 500/1.3310
= $376

(c) Additional lost PP = 451 – 376 = $75

1.55 McDougal’s LTD is a food products wholesaler operating in Costa Rica and Mexico where
the inflation rate has historically averaged 3% and 5% per year, respectively. The company
maintains a cash amount of $500,000 in each country’s banks in case of emergency needs.
(a) Use Figure 1.3 to estimate the purchasing power after 1 year and 2 years of the
$500,000 in each country. (b) (Spreadsheet exercise) If assigned by the instructor, use a
spreadsheet and the approach of Example 1.9 to plot the purchasing power over a 10-year
period for both inflation rates.

(a) Costa Rica: PP after 1 and 2 years at f = 3% per year

PP1 = 500,000(1 – 0.03) = $485,000

PP2 = 500,000(1 – 0.06) = $470,000

Mexico: PP after 1 and 2 years at f = 5% per year

PP1 = 500,000(1 – 0.05) = $475,000

PP2 = 500,000(1 – 0.09) = $455,000

Copyright 2021 © McGraw-Hill Education. All rights reserved. No reproduction or distribution without the prior
written consent of McGraw-Hill Education.
13
(b) (Spreadsheet exercise)
Develop the relations = 500000/(1 + f)^n where f = 0.03 and 0.05, respectively and
n = 0, 1,…, 10; then insert a line chart of the PP results. The chart should look something
like the one below.

1.56 Continental Airlines operates in and out of many countries. Country A has a low inflation
rate of 3% per year while country B has a high rate of 30% per year. A $1 million fund is
maintained in each country for emergency purchases to repair disabled aircraft. (a) Use a
formula to determine the purchasing power after 2, 4, and 5 years if the funds are not
utilized. (b) (Spreadsheet exercise) Use a spreadsheet to plot the diminishing purchasing
power curves, if assigned by your instructor. (c) With the dramatic effect of 30% inflation,
if you were president of Continental Airlines, how would you manage this situation?

(a) Country A with f = 3%

PP2 = 1,000,000/(1.03)2 = $942,596


PP4 = 1,000,000/(1.03)4 = $888,487
PP5 = 1,000,000/(1.03)5 = $862,609

Country B with f = 30%

PP2 = 1,000,000/(1.30)2 = $591,716


PP4 = 1,000,000/(1.30)4 = $350,128
PP5 = 1,000,000/(1.30)5 = $269,329

(b) (Spreadsheet exercise)


Develop the relations = 1000000/(1 + f)^n where f = 0.03 and 0.30, respectively and
n = 0, 1, …, 5; then insert a line chart of the PP results. The chart should look something
like the one below.

Copyright 2021 © McGraw-Hill Education. All rights reserved. No reproduction or distribution without the prior
written consent of McGraw-Hill Education.
14
(c) No answer is provided since it is meant to be a discussion question.

1.57 Every engineering economy problem will involve at least how many symbols?

It will involve at least four symbols.

1.58 Thompson Mechanical Products is planning to set aside $150,000 now for possibly
replacing its large synchronous refiner motors whenever it becomes necessary. If the
replacement isn’t needed for 7 years, how much will the company have in its investment
set-aside account if it achieves a rate of return of 11% per year?

P = $150,000; F = ?; i = 11%; n = 7

1.59 Atlas Long-Haul Transportation is considering the installation of Valutemp temperature


loggers in all of its refrigerated trucks for monitoring temperatures during transit. If the
systems will reduce insurance claims by $100,000 two years from now, how much should
the company be willing to spend now, if it uses an MARR of 12% per year?

P = ?; F = $100,000; i = 12%; n = 2

1.60 Determine the amount of money FrostBank might loan a housing developer who will repay
the loan 2 years from now by selling eight lots at $240,000 each. Assume the bank’s
interest rate is 10% per year.

P = ?; F = 8(240,000) = $1,920,000; n = 2; i = 0.10

1.61 Bodine Electric, based in Des Moines, Iowa, USA, makes gear motors with a three-stage,
selectively hardened gearing cluster that is permanently lubricated. If the company borrows
$20 million for a new distribution facility in Europe, how much must it pay back each year
to repay the loan in six equal annual payments at an interest rate of 10% per year?

P = $20,000,000; A = ?; n = 6; i = 0.10

Copyright 2021 © McGraw-Hill Education. All rights reserved. No reproduction or distribution without the prior
written consent of McGraw-Hill Education.
15
1.62 DubaiWorks manufactures angular contact ball bearings for pumps that operate in harsh
environments. The company invested $2.4 million in a process that resulted in net profits of
$760,000 per year for 5 consecutive years. What rate of return did the company make?

P = $2,400,000; A = $760,000: n = 5; i = ?

1.63 How many years will it take for your investment to be 1.5 times as much as what you
deposit now, if the return is 5% per year?

P = P; F = 1.5P; i = 5%; n = ?

1.64 Vision Technologies, Inc. is a small company that uses ultra-wideband technology to
develop devices that can detect objects (including people) inside of buildings, behind walls,
or below ground. The company expects to spend $100,000 per year for labor and $125,000
per year for supplies before a product can be marketed. What is the total equivalent future
amount of the company’s expenses at the end of 3 years at 15% per year interest?

A = $225,000; n = 3; i = 15% per year; F = ?

1.65 Fifteen years ago your grandfather invested $10,000 in a stock fund to pay for your college
education. Each year thereafter, he deposited $3000 into the fund. If the investments grew
at a rate of 8% per year over the 15 years, how much is in the fund now?

P = $10,000; A = $3,000; i = 0.08; n = 15; F = ?

1.66 What does the term end-of-period convention mean? What does it not mean?

End-of-period-convention means end-of-interest period. It does not mean end of


the calendar year.

1.67 Identify the following as cash inflows or outflows to a commercial air carrier: fuel cost,
pension plan contributions, fares, maintenance, freight revenue, cargo revenue, extra-bag
charges, cost of snacks, web-based advertising, landing fees, and seat-preference fees.

Fuel cost - outflow, pension plan contributions - outflow, passenger fares - inflow,
maintenance - outflow, freight revenue - inflow, cargo revenue - inflow, extra-bag charges –
inflow, snacks - outflow, web-based advertising - outflow, landing fees - outflow, seat
preference fees – inflow

1.68 Identify the following as cash inflows or outflows to a privately owned water company:
well drilling, maintenance, water sales, accounting, government grants, issuance of bonds,
energy cost, pension plan contributions, heavy equipment purchases, used-equipment sales,
stormwater fees, and discharge permit revenues.

Copyright 2021 © McGraw-Hill Education. All rights reserved. No reproduction or distribution without the prior
written consent of McGraw-Hill Education.
16
Well drilling: outflow; maintenance: outflow; water sales: inflow; accounting: outflow;
government grants: inflow; issuance of bonds: inflow; energy cost: outflow; pension plan
contributions: outflow; heavy equipment purchases: outflow; used-equipment sales:
inflow; stormwater fees: inflow; discharge permit revenues: inflow.

1.69 Many credit unions use semiannual interest periods to pay interest on customer savings
accounts. For a credit union that uses June 30 and December 31 as its semiannual interest
periods, determine the end-of-interest period total deposit amounts that will be recorded for
the cash flows shown below.
Month Deposit, $
Jan 50
Feb 70
Mar —
Apr 120
May 20
June —
July 150
Aug 90
Sept —
Oct —
Nov 40
Dec 110

End-of-period amount for June = 50 + 70 + 120 + 20


= $260
End-of-period amount for Dec = 150 + 90 + 40 + 110
= $390

1.70 List three different types of cost estimation and give an example of each type different than
those mentioned in this chapter.

Product – motorcycle; Project – bridge upgrade; operation – delivery truck, driver and fork
lift; system – develop and implement a pension program for all employees.

1.71 Identify the type of cost estimation that is associated with each of the following: new
suspension bridge, bicycle manufacturing, identity theft deterrent by a nationwide bank
corporation, monthly lubrication of heat-sensitive bearings, writing a textbook on
engineering economy, 3-D printer.

New suspension bridge - project, bicycle manufacturing - product, identity theft deterrent
by a nationwide bank corporation - system, monthly lubrication of heat-sensitive bearings -
operation, writing a textbook on engineering economy - project, 3d printer - product.

Copyright 2021 © McGraw-Hill Education. All rights reserved. No reproduction or distribution without the prior
written consent of McGraw-Hill Education.
17
1.72 Construct a cash flow diagram for the following cash flows: $25,000 outflow at time zero,
$9000 per year inflow in years 1 through 5 at an interest rate of 10% per year, and an
unknown future amount in year 5.

Assuming down is negative: down arrow of $25,000 in year 0; up arrows in the amount of
$9000 in years 1 through 5; i = 10% per year; arrow in year 5 identified as F=? (either
direction).

1.73 Construct a cash flow diagram to find the present worth in year 0 for the following series at
i = 15% per year.
Year Cash Flow, $
0 −19,000
1–4 +8,100

Assuming down is negative: up arrow in year 0 identified as P = ?; down arrow of $19,000


in year 0; up arrows in the amount of $8100 in years 1 through 4; i = 15% per year.

1.74 Construct a cash flow diagram to find the rate of return on an investment of $60,000 made
today and accumulated to an amount of $95,000 in 5 years.

Assuming down is negative: down arrow of $60,000 in year 0; up arrow of $95,000 in year
5; i = ?

1.75 Write the engineering economy symbol that is displayed when each of the following
spreadsheet functions are entered.
a. PV
b. PMT
c. NPER
d. IRR
e. FV
f. RATE

(a) PV is P; (b) PMT is A; (c) NPER is n; (d) IRR is i; (e) FV is F; (f) RATE is i

1.76 State the purpose for each of the following built-in spreadsheet functions.
a. PV(i%,n,A,F)
b. FV(i%,n,A,P)
c. RATE(n,A,P,F)
d. IRR(first_cell:last_cell)
e. PMT(i%,n,P,F)
f. NPER(i%,A,P,F)

(a) PV(i%,n,A,F) finds the present value P


(b) FV(i%,n,A,P) finds the future value F
(c) RATE(n,A,P,F) finds the compound interest rate i for equal periodic payments A

Copyright 2021 © McGraw-Hill Education. All rights reserved. No reproduction or distribution without the prior
written consent of McGraw-Hill Education.
18
(d) IRR(first_cell:last_cell) finds the compound interest rate i
(e) PMT(i%,n,P,F) finds the equal periodic payment A
(f) NPER(i%,A,P,F) finds the number of periods n

1.77 For the following spreadsheet functions, (a) write the values of the engineering economy
symbols P, F, A, i, and n, using a ? for the symbol that is to be determined, and (b) state
whether the displayed answer will have a positive sign, a negative sign, or it can’t be
determined from the entries.
1. = FV(8%,10,3000,8000)
2. = PMT(12%,20,−16000)
3. = PV(9%,15,−1000,600)
4. = NPER(10%,−290,12000)
5. = FV(5%,5,500,−2000)

(a) (1) F = ?; i = 8%; n = 10; A = $3000; P = $8000


(2) A = ?; i = 12%; n = 20; P = $–16,000; F = 0
(3) P = ?; i = 9%; n = 15; A = $–1000; F = $600
(4) n = ?; i = 10%; A = $–290; P = 0; F = $12,000
(5) F = ?; i = 5%; n = 5; A = $500; P = $–2000

(b) (1) negative


(2) positive
(3) positive
(4) positive (years)
(5) can’t determine if 5% per year will cover the 5 withdrawals of $500

1.78 In a built-in spreadsheet function, if a certain parameter does not apply, under what
circumstances can it be left blank, and when must a comma be entered in its place?

For built-in spreadsheet functions, a parameter that does not apply can be left blank when it
is not an interior one. For example, if no F is involved when using the PMT function, it can
be left blank because it is an end parameter. When the parameter involved is an interior one
(like P in the PMT function), a comma must be entered in its position.

1.79 Emily and Madison both invest $1000 at 10% per year for 4 years. Emily receives simple
interest and Madison gets compound interest. Use a spreadsheet and cell reference formats
to develop relations that show a total of $64 more interest for Madison at the end of the
4 years. Assume no withdrawals or further deposits are made during the 4 years.

Spreadsheet shows relations only in cell reference format. Cell E10 will indicate $64 more
than cell C10.

Copyright 2021 © McGraw-Hill Education. All rights reserved. No reproduction or distribution without the prior
written consent of McGraw-Hill Education.
19
1.80 Assume you pay the reduced amount of $4750 for a corporate stock that has a market value
of $5000. The stock pays an annual dividend of 4% of its market value. Since this is
primarily a dividendpaying stock, you estimate that you will sell the stock 10 years from
now at the current $5000 market value. Develop spreadsheet functions that display the
following results:
a. Total amount of the dividends paid to you.
b. Equivalent future worth after 10 years of the dividends at i = 6% per year.
c. Present worth now (year 0) of the original purchase price, dividends and proceeds when
the stock is sold after 10 years, if i = 6% per year. Observe the sign on the result and
explain what it means to you financially.
d. Rate of return on this entire investment over a 10-year period. (Note: The answer
displayed here should confirm your response to the question in part c.)

(a) The function = 10*0.04*5000 displays the total dividends of $2000

(b) The function = – FV(6%,10,200) displays an F of $2636.16

(c) The function = – PV(6%,10,200,5000) – 4750 displays a P of $–486.01. This means you
did not make 6% on the investment.

(d) The function = RATE(10,200,-4750,5000) displays a i value of 4.64%, which verifies


that the return is less than 6% per year.

1.81 A Harley-Davidson bike you hope to own in the future costs $12,000 today. If the inflation
rate is expected to be 4.5% per year, determine the estimated cost using a spreadsheet
function for each of the next 5 years.

Use the FV function with i = f. Enter the year numbers 1-5 in cells A2 through A6 and
develop the functions = – FV(4.5%,year_cell,12000) where ‘year_cell’ is a reference to the
cell containing the year numbers 1-5. To the nearest dollar, the displays should be:
$12,540; $13,104; $13,694; $14,310; and $14,954.

Copyright 2021 © McGraw-Hill Education. All rights reserved. No reproduction or distribution without the prior
written consent of McGraw-Hill Education.
20
1.82 Explain the relation between a common moral and a personal moral.

A common moral is a fundamental belief held by virtually all people. A personal moral is
the translation of a common moral into that which an individual believes and uses as
guidance for their own decisions and actions.

1.83 What is one primary use of a code of ethics for a specific discipline of professional
practice?

A code of ethics can be used as an evaluation measure for the decision and actions of an
individual who works in the discipline that honors the code.

1.84 Yesterday, Carol, an engineer with Hancock Enterprises, was at lunch with several work
friends. Joe, a person Carol has known for a year or so from similar lunches, proudly
mentioned that he got a free flight and tickets to a major league playoff game two weeks
from now in a distant city. Joe happened to also mention the company: it is Dryer. Carol is
aware that Dryer is one of the prime bidders on a major contract to be evaluated by
Hancock next month. Upon inquiry, Carol learned that both she and Joe are on the bid
evaluation committee. Carol suspects that someone in Dryer has offered Joe the tickets as a
consideration for Joe’s favorable evaluation of their bid.
a. Carol has determined that she could do one of several things about the situation:
recommend to Joe directly that he refuse the tickets; show Joe the NSPE Code of Ethics
for Engineers and let him make his own decision; go to Joe’s supervisor and tell her of
the situation; go to her supervisor and inform him of her suspicion; write an e-mail to
Joe with a copy to Carol’s supervisor recommending that Joe consider the ethical
dilemma involved for him; do nothing. Considering only these actions, select one you
think is the best and explain why you chose it.
b. Identify other options for Carol’s response at this time and determine if one of them is
better than her options outlined above.

(a) Assuming that Carol’s supervisor is a trustworthy and ethical person himself, going to
her supervisor and informing him of her suspicion is probably the best of these options.
This puts Carol on record (verbally) as questioning something she heard at an informal
gathering.
(b) Another good option is to go to Joe one-on-one and inform him of her concern about
what she heard him say at lunch. Joe may not be aware he is on the bid evaluation team
and the potential ethical consequences if he accepts the free tickets from Dryer.

1.85 Stefanie is a design engineer with an international railroad locomotive manufacturing


company in the state of Illinois. Management wants to return some of the engineering
design work to the United States rather than export all of it to India, where their primary
design work has been completed for the last decade. This transfer will employ more people
locally and could improve the economic conditions for families in and around Illinois.
Stefanie and her design team were selected as a test case to determine the quality and
speed of the design work they could demonstrate on a more fuel-efficient diesel

Copyright 2021 © McGraw-Hill Education. All rights reserved. No reproduction or distribution without the prior
written consent of McGraw-Hill Education.
21
locomotive. None of her team members or she has done such a significant design job
themselves, because their jobs had previously entailed only the interface with the
subcontracted engineers in India. One of her team members had a great design idea on a
key element that will improve fuel efficiency by approximately 15%. She told Stefanie it
came from one of the Indiangenerated documents, but that it would probably be okay for
the team to use it and remain silent as to its origin, since it was quite clear the U.S.
management was about to cancel the foreign contract. Though reluctant at first, Stefanie
did go forward with a design that included the efficiency improvement and made no
mention of the origin of the idea during the oral presentation or documentation delivery. As
a result, the Indian contract was canceled and full design responsibility was transferred to
Stefanie’s group.
Consult the NSPE Code of Ethics for Engineers and identify sections that are points
of concern about Stefanie’s decisions and actions.

Many sections could be identified; however, III.9.a and b are likely the most applicable.
Some others are: I.2 and 5, and II.2.a and b.

1.86 All of the following are examples of cash outflows except:


a. Asset salvage value
b. Income taxes
c. Operating cost of asset
d. First cost of asset

Answer is (a)

1.87 An example of an intangible factor is:


a. Taxes
b. Cost of materials
c. Morale
d. Rent

Answer is (c)

1.88 At an interest rate of 10% per year, the equivalent amount of $10,000 one year ago is
closest to:
a. $8264
b. $9095
c. $11,000
d. $12,000

Amount one year ago = 10,000/(1+ 0.10) = $9090.90


Answer is (b)

Copyright 2021 © McGraw-Hill Education. All rights reserved. No reproduction or distribution without the prior
written consent of McGraw-Hill Education.
22
1.89 In most engineering economy studies, the best alternative is the one that:
a. Will last the longest time
b. Is most politically correct
c. Is easiest to implement
d. Has the lowest cost

Answer is (d)

1.90 The time it would take for a given sum of money to double at 4% per year simple interest is
closest to:
a. 30 years
b. 25 years
c. 20 years
d. 10 years

2P = P + P(n)(0.04)
1 = 0.04n
n = 25
Answer is (b)

1.91 The compound interest rate per year at which amounts of $1000 one year ago and $1345.60
one year hence are equivalent is closest to:
a. 8.5% per year
b. 10.8% per year
c. 20.2% per year
d. None of the above

Move both cash flows to year 0 and solve for i

1000(1 + i) = 1345.60/(1 + i)
(1 + i)2 = 1345.60/1000
(1 + i) = 1.16
i = 0.16 (16%)
Answer is (d)

1.92 The simple interest rate per year that will accumulate the same amount of money in 2 years
as a compound interest rate of 20% per year is closest to:
a. 20.5%
b. 21%
c. 22%
d. 25%

For compound interest at 20%, in 2 years F = P(1.20)(1.20) = 1.44P

For simple interest, F = P + Pni = P(1 + ni) = P(1 + 2i)

Copyright 2021 © McGraw-Hill Education. All rights reserved. No reproduction or distribution without the prior
written consent of McGraw-Hill Education.
23
Equate the two relations.
P(1 + 2i) = 1.44P
(1 + 2i) = 1.44
i = 0.22 (22%)
Answer is (c)

1.93 A company that utilizes carbon fiber 3-D printing wants to have money available two years
from now to add new equipment. The company currently has $650,000 in a capital account
and it plans to deposit $200,000 now and another $200,000 one year from now. The total
amount available in two years, provided it returns a compounded rate of 15% per year, is
closest to:
a. $1,354,125
b. $1,324,125
c. $1,325,500
d. $1,050,000

Amount available = total principal in year 0 + interest for 2 years + principal added year 1
+ interest for 1 year
= 850,000(1+0.15)2 + 200,000 (1+0.15)
= 1,124,125 + 230,000
= $1,354,125
Answer is (a)

1.94 If inflation has been steady at 3.2% per year for 4 years, the reduction in purchasing power
over the 4 years is closest to:
a. 6%
b. 12%
c. 14%
d. 22%

Purchasing power reduction = [1 – 1/(1.032)4](100)


= [1 – 0.8816](100)
= 0.1184 (11.84%)
Answer is (b)

1.95 For the spreadsheet built-in function PV(i%,n,A,F), the only parameter that can be
completely omitted is:
a. i%
b. n
c. A
d. F

Only a parameter at the end of the string can be omitted without an entry or comma; it is F
in the PV function.
Answer is (d).

Copyright 2021 © McGraw-Hill Education. All rights reserved. No reproduction or distribution without the prior
written consent of McGraw-Hill Education.
24
Solutions to end-of-chapter problems
Basics of Engineering Economy, 3rd edition
Leland Blank and Anthony Tarquin

Chapter 2
Factors: How Time and Interest Affect Money

2.1 (a) Look up the numerical value for the following factors from the compound interest factor
tables.
(b) Use spreadsheet functions to display the numerical value for the first three factors.
1. (F∕P,10%,7)
2. (A∕P,12%,10)
3. (F∕A,2%,50)
4. (P∕G,15%,20)
5. (A∕G,30%,15)

(a) Tabulated factors


1. (F/P, 10%, 7) = 1.9487
2. (A/P, 12%,10) = 0.17698
3. (F/A,2%,50) = 84.5794
4. (P/G,15%,20) = 33.5822
5. (A/G,30%,15) = 3.0344

(b) Spreadsheet functions from Table 2.5


1. = – FV(10%,7,1) displays 1.9487
2. = – PMT(12%,10,1) displays 0.17698
3. = – FV(2%,50,1) displays 84.5794

2.2 How much can Azco Autosystems, Inc. afford to spend now on an energy management
system, if the software will save the company $21,300 per year for the next 5 years? Use an
interest rate of 10% per year.

P = 21,300(P/A,10%,5)
= 21,300(3.7908)
= $80,744

2.3 What is the future worth in year 8 of a present sum in year 0 of $23,000 at an interest rate of
10% per year? Solve using (a) tabulated factor values, and (b) a built-in, single-cell
spreadsheet function.

(a) Hand solution


F = 23,000(F/P,10%,8)
= 23,000(2.1436)

Copyright 2021 © McGraw-Hill Education. All rights reserved. No reproduction or distribution without the prior
written consent of McGraw-Hill Education.
1
= $49,302.80

Copyright 2021 © McGraw-Hill Education. All rights reserved. No reproduction or distribution without the prior
written consent of McGraw-Hill Education.
2
(b) Spreadsheet solution
The function = – FV(10%,5,23000) displays $49,302.54

2.4 Determine the amount of money Western Bank should be willing to loan a developer who
will repay the loan in a lump sum amount of $840,000 two years from now at the bank’s
interest rate of 10% per year.

P = 840,000(P/F,10%,2)
= 840,000(0.8264)
= $694,176

2.5 Xavier is an engineer who believes in “save now and play later.” He wants to retire from full
employment in 20 years with $1.5 million. At 10% per year interest, how much does he
have to invest each year (starting in year 1) to reach the $1.5 million goal? Solve by
tabulated factors and spreadsheet.

Hand solution
A = 1,500,000(A/F,10%,20)
= 1,500,000(0.01746)
= $26,190 per year

Spreadsheet solution
Function = –PMT(10%,20,1500000) displays an A value of $26,189.44 per year

2.6 During a recession, the price of goods and services goes down because of low demand. A
company that makes Ethernet adapters is planning to expand its production facility at a cost
of $1,000,000 one year from now. However, a contractor who needs work has offered to do
the job for $790,000 if the company will do the expansion now instead of one year from
now. If the interest rate is 15% per year, what is the difference in the present worth of the
two amounts?

Equivalent present amount = 1,000,000/(1 + 0.15)


= $869,562

Difference = 869,562 – 790,000


= $79,562

2.7 The Moller Skycar M400 is a flying car known as a personal air vehicle (PAV). The cost is
$985,000, and a $100,000 deposit holds one of the first 100 vehicles. Assume a buyer pays
the $885,000 balance 3 years after making the $100,000 deposit. At an interest rate of 10%
per year, determine the effective total cost of the PAV in year 3 using (a) tabulated factors,
and (b) a single-cell spreadsheet function.

(a) F = 885,000 + 100,000(F/P,10%,3)


= 885,000 + 100,000(1.3310)
= $1,018,000

Copyright 2021 © McGraw-Hill Education. All rights reserved. No reproduction or distribution without the prior
written consent of McGraw-Hill Education.
3
(b) Spreadsheet function is = –FV(10%,3,100000) + 885,000.
Display is $1,018,000

2.8 What is the present worth of a future payment of $19,000 in year 7 if the interest rate is 10%
per year using (a) the tabulated factor values in your book, (b) TVM functions on a financial
calculator, and (c) built-in functions on a spreadsheet?

(a) P = 19,000(P/F,10%,7)
= 19,000(0.5132)
= $9750.80

(b) If the calculator function is PV(10,7,0,19000), display is P = $–9750.00

(c) The spreadsheet function = – PV(10%,7,19000) displays $9750.00

2.9 Determine the amount of money a bank will loan a developer now who repays the loan by
selling seven view lots at $120,000 each 2 years from now. Assume the bank’s interest rate
is 10% per year. Use (a) the tabulated factor values in your book, (b) TVM functions on a
financial calculator, and (c) built-in functions on a spreadsheet.

(a) Total for 7 lots is 7(120,000) = $840,000


P = 840,000(P/F,10%,2)
= 840,000(0.8264)
= $694,176

(b) If the calculator function is PV(10,2,0,840000), display is P = $–694,214.88

(c) The spreadsheet function = – PV(10%,2,840000) display $694,214.88

2.10 You just deposited $3000 in an investment account and will deposit $5000 more 4 years
from now. How much will be in the account 12 years from now if the rate of return is 10%
per year? Use (a) tabulated factor values, (b) TVM functions on a financial calculator, and
(c) built-in functions on a spreadsheet.

(a) F = 3000(F/P,10%,12) + 5000(F/P,10%,8)


= 3000(3.1384) + 5000(2.1436)
= $20,133.20

(b) Sum two calculator functions


FV(10,12,–3000) + FV(10,8,–5000)
9,415.29 + 10,717.94 = $20,133.23

(c) If the spreadsheet function is = – FV(10%,12,3000) – FV(10%,8,5000), the


display is $20,133.23

Copyright 2021 © McGraw-Hill Education. All rights reserved. No reproduction or distribution without the prior
written consent of McGraw-Hill Education.
4
2.11 APR Performance Vehicles is considering the purchase of dual-axis inclinometers for
installation in a new line of tractors. The distributor of the inclinometers is temporarily
overstocked and is offering them at a 40% discount from the regular cost of $142. If the
purchaser gets them now instead of 2 years from now, which is when they will be needed,
what is the present worth of the savings per unit? The company would pay the regular
price, if purchased in 2 years. Assume the interest rate is 10% per year.

Cost now = 142(0.60) = $85.20


Present worth at regular cost = 142(P/F,10%,2)
= 142(0.8264)
= $117.35

Present worth of savings = 117.35 – 85.20


= $32.15

2.12 Nusile Tecnnology, Inc. manufactures high-accuracy liquid level transducers. It is


investigating whether it should update in-place equipment now or wait and do it later. If the
cost now is $200,000, what will be the equivalent amount 3 years from now at an interest rate
of 10% per year?

F = 200,000(F/P,10%,3)
= 200,000(1.3310)
= $266,200

2.13 A family that won a $100,000 prize in a state lottery decided to invest half of the money in
a college fund for their child. If the fund earned interest at 6% per year, how much was in
the account 14 years after it was started?

F = 50,000(F/P,6%,14)
= 50,000(2.2609)
= $113,045

2.14 How large of a repayment must Theresa make each year starting next year if she borrows
$60,000 now to start up a consulting office and if she promises to make equal annual
payments for 5 years. Assume the interest rate is 8% per year. Develop the answer using
(a) tabulated factor values, (b) a financial calculator, and (c) spreadsheet functions.

(a) A = 60,000(A/P,8%,5)
= 60,000(0.25046)
= $15,027.60

(b) If calculator function is PMT(8,5,–60000,0), the answer is $15,027.39

(c) A spreadsheet function of = – PMT(8%,5,60000) displays $15,027.39

Copyright 2021 © McGraw-Hill Education. All rights reserved. No reproduction or distribution without the prior
written consent of McGraw-Hill Education.
5
2.15 Ametek Technical & Industrial Products (ATIP) manufactures brushless blowers for
boilers, food service equipment, and fuel cells. The company borrowed $17,000,000 for a
plant expansion and repaid the loan in eight annual payments of $2,737,680, with the first
payment made 1 year after the company received the money. What annual interest rate did
ATIP pay? Develop the answer using (a) tabulated factor values, (b) a financial calculator,
and (c) spreadsheet functions.

(a) 17,000,000(A/P,i,8) = 2,737,680


(A/P,i,8) = 0.16104
From interest tables at n = 8, i = 6% per year

(b) Calculator function is i(8,–2737680,17000000,0) to obtain i = 6.00%

(c) The spreadsheet function = RATE(8,–2737680,17000000) displays 6.00%

2.16 A design/build engineering company that usually gives year-end bonuses in the amount of
$8000 to each of its engineers is having cash flow problems. The company said that
although it couldn’t give bonuses this year, it would give each engineer two bonuses next
year: the regular one of $8000 plus an amount equivalent to the $8000 that each engineer
should have gotten this year. If the interest rate is 8% per year, what will be the total
amount of bonus money the engineer gets next year?

Total bonus next year = 8,000 (F/P,8%,1) + 8,000


= 8000(1.0800) + 8000
= $16,640

2.17 Assume the cost of a homeland security border fence is $3 million per mile. If the life of
such a fence is 10 years, what is the equivalent annual cost of a 10-mile-long fence at an
interest rate of 8% per year? Develop the answer using (a) tabulated factor values, (b) a
financial calculator, and (c) spreadsheet functions.

(a) A = 3,000,000(10)(A/P,8%,10)
= 30,000,000(0.14903)
= $4,470,900

(b) If calculator function is PMT(8,10,–30000000,0), the answer is $4,470,884.66

(c) The spreadsheet function = – PMT(8%,10,30000000) displays an A of $4,470,884.66

2.18 One of the biggest vulnerabilities in a control system is network devices, such as Ethernet-
based network switches that are located in unsecured locations and accessible to everyone.
DeltaX switches, manufactured by Dahne Security, allow the user to automatically lock
and unlock the port access to all switches in the network. The company is considering
expanding its manufacturing lines now or doing it in three years. If the cost now would be

Copyright 2021 © McGraw-Hill Education. All rights reserved. No reproduction or distribution without the prior
written consent of McGraw-Hill Education.
6
$1.9 million, what equivalent amount could the company afford to spend in 3 years? The
interest rate is 15% per year.

F = 1,900,000(F/P,15%,3)
F = 1,900,000(1.5209)
= $2,889,710

2.19 Turik Electronics manufactures microprocessorbased soft starters that use thyristors for
controlled reduced voltage during starting and stopping. The company is planning a
production-line expansion that will cost $1.3 million. If the company uses a minimum
attractive rate of return of 15% per year, what is the equivalent annual cost in years 1
through 5 of the investment?

A = 1,300,000(A/P,15%,5)
= 1,300,000(0.29832)
= $387,816

2.20 At 30 years old, Sally decided she wants to retire at 60. Determine the size of her
investment package 30 years from now if she deposits $12,000 each year, beginning
1 year from now, and the account earns interest at a rate of 10% per year. Solve using
(a) tabulated factors, and (b) a spreadsheet function.

Factor solution
(a) F = 12,000(F/A,10%,30)
= 12,000(164.4940)
= $1,973,928

(b) The function = – FV(10%,30,12000) to display $1,973,928.27

2.21 Loadstar Sensors is a company that makes load/force sensors based on capacitive sensing
technology. For a major plant expansion project, the company wants to have $30 million
5 years from now. If the company already has $15 million in an investment account for the
expansion, how much more must the company add to the account now so that it will have
the $30 million 5 years from now? The funds earn interest at the rate of 10% per year.
Solve using (a) tabulated factors, and (b) a spreadsheet. Compare answers.

(a) P = 30,000,000(P/F,10%,5) – 15,000,000


= 30,000,000(0.6209) – 15,000,000
= $3,627,000

(b) If the spreadsheet function is = –PV(10%,5,30000000) – 15,000000, the


display is $3,627,640

The increased decimal accuracy of a spreadsheet function indicates an increased


required amount of $640.

Copyright 2021 © McGraw-Hill Education. All rights reserved. No reproduction or distribution without the prior
written consent of McGraw-Hill Education.
7
2.22 If GHD Plastics purchases a new building now for $1.3 million for a regional corporate
headquarters, what must the building be worth in 10 years if the company expects all
expenditures to earn a rate of return of at least 18% per year?

In $1 million terms,
F = 1.3(F/P,18%,10)
= 1.3(5.2338)
= 6.80394 ($6,803,940)

2.23 Thompson Mechanical Products is planning to set aside $150,000 now for possible
replacement of large synchronous refiner motors when it becomes necessary. If the
replacement isn’t needed for 5 years, how much will the company have in its investment
set-aside account? Assume a rate of return of 8% per year. Write the single-cell spreadsheet
function.

F = 150,000(F/P,8%,5)
= 150,000(1.4693)
= $220,395

Spreadsheet function: = – FV(8%,5,150000) displays a F of $220,399

2.24 CGK Rheosystems makes high-performance rotational viscometers capable of steady shear
and yield stress testing in a rugged, compact footprint. How much could the company
afford to spend now on new equipment in lieu of spending $200,000 one year from now
and $300,000 three years from now, if the company uses an interest rate of 15% per year?

P = 200,000(P/F,15%,1) + 300,000(P/F,15%3)
= 200,000(0.8696) + 300,000(0.6575)
= $371,170

2.25 Labco Scientific sells high-purity chemicals to universities, research laboratories, and
pharmaceutical companies. The company wants to invest in new equipment that will reduce
shipping costs by better matching the size of the completed products with the size of the
shipping container. The new equipment is estimated to cost $450,000 to purchase and
install. How much must Labco save each year for 3 years in order to justify the investment
at an interest rate of 10% per year?

A = 450,000(A/P,10%,3)
= 450,000(0.40211)
= $180,950

2.26 How much must Animatics Sensors and Controls, Inc. invest each year, beginning 1 year
from now, to have $2.7 million dollars for new-product development 5 years from now if
the company investments earn a rate of return of (a) 18% per year, and (b) 8% per year?
What is the percentage increase necessary to cover the reduction in return?

Copyright 2021 © McGraw-Hill Education. All rights reserved. No reproduction or distribution without the prior
written consent of McGraw-Hill Education.
8
(a) 18% per year
A = 2,700,000(A/F,18%,5)
= 2,700,000(0.13978)
= $377,406

(b) 8% per year


A = 2,700,000(A/F,8%,5)
= 2,700,000(0.17046)
= $460,242
Increase = [(460,242 – 377,406)/377,406]×100% = 21.9% per year

2.27 Metso Automation, which manufactures addressable quarter-turn electric actuators, is


planning to set aside $100,000 now and $150,000 one year from now for possible
replacement of the heating and cooling systems in three of its larger manufacturing plants.
If the replacement won’t be needed for 4 years, how much will the company have in the
account if it earns interest at a rate of 8% per year? Use (a) tabulated factors, and (b) a
spreadsheet function to answer.

(a) Factors:
F = 100,000(F/P,8%,4) + 150,000(F/P,8%,3)
= 100,000(1.3605) + 150,000(1.2597)
= $325,005

(b) Spreadsheet:
= – FV(8%,4,100000) – FV(8%,3,150000) displays an F value of $325,005.70

2.28 For a profession, Henry Mueller has successfully written short articles for several monthly
parks and wildlife magazines. His steady annual cash flows for an 8-year period are shown
in the table. He received a good bonus in a single year. Determine the future worth (now,
year 8) of the net cash flows at an interest rate of 10% per year. Use (a) tabulated factors,
and (b) a single-cell spreadsheet function to answer.
Year 1 2 3 4 5 6 7 8
Revenues, $1000 200 200 200 200 200 200 400 200
Expenses, $1000 90 90 90 90 90 90 90 90

(a) Factors (In $1000 units)


F = (200 – 90)(F/A,10%,8) + 200(F/P,10%,1)
= 110(11.4359) + 200(1.1000)
= $1,477,949

(b) Spreadsheet
= – FV(10%,8,110000) – FV(10%,1,200000) displays the amount $1,477,947.69

Copyright 2021 © McGraw-Hill Education. All rights reserved. No reproduction or distribution without the prior
written consent of McGraw-Hill Education.
9
2.29 China spends an estimated $100,000 per year on cloud seeding efforts, which includes using
antiaircraft guns and rocket launchers to fill the sky with silver iodide. In the United States,
utilities that run hydroelectric dams are among the most active cloud seeders, because they
believe it is a cost-effective way to increase limited water supplies by 10% or more. If the
yields of cash crops will increase by 4% each year for the next 3 years because of extra
irrigation water captured behind dams during cloud seeding, what is the maximum amount the
farmers should spend now on the cloud seeding activity? The value of the cash crops without
the extra irrigation water would be $600,000 per year. Use an interest rate of 10% per year.

P = 600,000(0.04)(P/A,10%,3)
= 24,000(2.4869)
= $59,686

2.30 In an effort to reduce childhood and teenage obesity by decreasing the consumption of sugared
beverages, some states have imposed taxes on soda and other soft drinks. A survey by Roland
Sturm of 7300 fifth-graders revealed if taxes averaged 4 cents on each dollar’s worth of soda,
no real difference in overall consumption was noticed. However, if taxes were increased to 18
cents on the dollar, Sturm calculated they would make a significant difference. For a student
who consumes 100 sodas per year, what is the future worth of the extra cost to the individual
for an increase from 4 cents to 18 cents per soda? Assume the student consumes sodas from the
5th grade through graduation in grade 12. Use an interest rate of 6% per year.

F = (0.18 – 0.04)(100)(F/A,6%,8)
= 14(9.8975)
= $138.57

2.31 Major automobile insurance companies are now offering telematics, technology that
collects information about an individual’s driving behavior. In exchange, the companies
reward good driving behavior with policy discounts or cash back awards. If, on average,
good driving reduces the likelihood of an accident that would cost the company $1700
eight years from now, how much of an annual discount could the insurance company give
to just break even at an interest rate of 10% per year?

A = 1700(A/F,10%,8)
= 1700(0.08744)
= $148.65

2.32 To improve crack detection in aircraft, the U.S. Air Force combined ultrasonic inspection
procedures with laser heating to identify fatigue cracks. Early detection of cracks may
reduce repair costs by as much as $200,000 per year. What is the present worth of these
projected savings over the next 5 years at an interest rate of 10% per year?

P = 200,000((P/A,10%,5)
= 200,000(3.7908)
= $758,160

Copyright 2021 © McGraw-Hill Education. All rights reserved. No reproduction or distribution without the prior
written consent of McGraw-Hill Education.
10
2.33 As a principal in the consulting firm where you have worked for 20 years, you have
accumulated 5000 shares of company stock. One year ago, each share of stock was worth
$40. The company has offered to buy back your shares for $225,000. At what interest rate
would the firm’s offer be equivalent to the worth of the stock last year? Solve using hand
calculation first, then write the spreadsheet function to display the rate.

5000(40 )(1 + i) = 225,000


1 + i = 1.125
i = 0.125
= 12.5% per year

The spreadsheet function = RATE(1,–200000,225000) will display 12.5%.

2.34 What compound interest rate per year is equivalent to a 12% per year simple interest rate
over a 15-year period?

Simple: Total interest = (0.12)(15) × 100 = 180%

Compound: 1.8 = (1 + i)15


i = 4.0%

2.35 To make CDs look more attractive than they really are, some banks advertise that their
rates are higher than their competitors’, but the fine print says that the rate is a simple
interest rate. If a person deposits $10,000 at 10% per year simple interest, what compound
interest rate would yield the same amount of money in 3 years?

Simple: F = 10,000 + 10,000(3)(0.10)


= $13,000

Compound: 13,000 = 10,000(F/P,i,3)


(F/P,i,3) = 1.3000

Use equation to find i


(1 + i)3 = 1.3000
1 + i = (1.3000)0.333
1 + i = 1.0912
i = 9.1% per year

2.36 Assume you plan to retire early in 30 years with $1 million in your IRA (individual
retirement account). You want to know the interest rate required to reach your goal if the
account currently has $19,627 in it, and (a) no further deposits are made, and (b) you plan
to deposit $5000 annually for each of the 30 years. (Hint: It is easier to solve part (b) using
a spreadsheet function.)

Copyright 2021 © McGraw-Hill Education. All rights reserved. No reproduction or distribution without the prior
written consent of McGraw-Hill Education.
11
(a) 1,000,000 = 19,627(F/P,i,30)
(F/P,i,30) = 50.95022

Look for 50.95022 in interest tables under F/P column at n = 30


i = 14% per year

(b) The function = RATE(30,–5000,–19627,1000000) displays an i of 9.28% per year.

2.37 What interest rate is required for uniform deposits of $1453.26 in years 1 through 15 to
accumulate to $50,000?

50,000 = 1453.26(F/A,i,15)
(F/A,i,15) = 34.4054

Look for 34.4054 in interest tables under F/A column at n = 15


i = 11% per year

2.38 Sterling Pumps de Mexico has a fund for equipment replacement that contains $500,000. If
the company spends $75,000 per year on new equipment, how many years will it take to
reduce the fund to less than $75,000 at an interest rate of 10% per year? Solve using factors
and the NPER function.

Hand
500,000 = 75,000(P/A,10%,n)
(P/A,10%,n) = 6.6667

From 10% factor table, n is between 11 and 12 years; therefore, n = 11 years.

Spreadsheet
= NPER(10%,–75000,500000) displays 11.5 years to reach $75,000

2.39 Ophra invested well and plans to retire now because she has $2,300,000 in her tax-deferred
IRA account. How long will she be able to withdraw $125,000 per year (beginning 1 year
from now) if her investments earn at a rate of 4% per year? Solve using factors and the
NPER function.

Hand
2,300,000 = 125,000(P/A,4%,n)
(P/A,4%,n) = 18.4000

From 4% table, n is between 33 and 34 years, close to 34 years

Spreadsheet
= NPER(4%,–125000,2300000) displays 33.95 years.

Copyright 2021 © McGraw-Hill Education. All rights reserved. No reproduction or distribution without the prior
written consent of McGraw-Hill Education.
12
2.40 How many years will it take Rexchem, Inc. to accumulate $400,000 for a chemical feeder,
if the company deposits $50,000 each year starting 1 year from now, into an account that
earns interest at 12% per year?

400,000 = 50,000(F/A,12%,n)
(F/A,12%,n) = 8.0000

From 12% interest table, n is between 5 and 6 years. Therefore, n = 6

2.41 How many years will it take for money to increase to at least three times the initial amount,
if money earns at 10% per year?

F = P(F/P,10%,n)
3P = P(F/P,10%,n)
(F/P,10%,n) = 3.000

From 10% interest tables, n is between 11 and 12 years; therefore, n = 12 years

2.42 Acme Bricks, a masonry products company, wants to have $600,000 on hand before it
invests in new conveyors, trucks, and other equipment. If the company sets aside $80,000
per year in an account that increases in value at a rate of 15% per year, how many years
will it be before Acme can purchase the equipment?

600,000 = 80,000(F/A,15%,n)
(F/A,15%,n) = 7.50

From 15% interest table, n is between 5 and 6 years; therefore, n = 6 years


Use a spreadsheet function to get n = 5.4 years

2.43 You own a small engineering consulting company. If you invest $200,000 of the
company’s money in a natural gas well that is expected to provide income of $29,000 per
year, how long must the well produce at this income level to get the money back plus a rate
of return of 10% per year?

200,000 = 29,000(P/A,10%,n)
(P/A,10%,n) = 6.8966

From 10% interest table, n is between 12 and 13 years; therefore, n = 13 years


Use a spreadsheet function to display an n of 12.3 years

2.44 Demco Products, a company that manufactures stainless-steel control valves, has a fund for
equipment replacement that contains $500,000. The company plans to spend $85,000 each
year on new equipment. (a) Estimate the number of years it will take to reduce the fund to
no more than $85,000 at an interest rate of 10% per year. (b) Use the NPER function to
determine the exact number of years.

Copyright 2021 © McGraw-Hill Education. All rights reserved. No reproduction or distribution without the prior
written consent of McGraw-Hill Education.
13
(a) 500,000 = 85,000(P/A,10%,n)
(P/A,10%,n) = 5.8824

From 10% table, n is between 9 and 10 years.

(b) Using the function = NPER(10%,–85000,500000), the displayed n = 9.3 years.

2.45 An engineer who was contemplating retirement had $1.6 million in his investment
portfolio. However, a severe recession caused his portfolio to decrease to only 55% of the
original amount, so he kept working. If he was able to invest his money at a rate of return
of 9% per year after the recession ended, how many years did it take for his account to get
back to the $1.6 million value if no additional funds were invested? Solve by factors and
using a single-cell function.

Starting amount = 1,600,000(0.55) = $880,000

Hand solution: 1,600,000 = 880,000(F/P,9%,n)


(F/P,9%,n) = 1.8182

From 9% interest table, n is between 6 and 7 years; therefore, n = 7 years

Spreadsheet solution
= NPER(9%,–880000,1600000) displays an n of 6.94 or approximately 7 years

2.46 A cash flow sequence starts in year 1 at $3000 and decreases by $200 each year through
year 10. (a) Determine the value of the gradient G; (b) determine the amount of cash flow
in year 8; and (c) determine the value of n for the gradient.

(a) G = $–200 (b) CF8 = 3000 – (8–1)200 = $1600 (c) n = 10

2.47 Cisco Systems expects sales to be described by the cash flow sequence (6000 + 5k), where
k is in years and cash flow is in millions. In actual dollar amounts, determine (a) the value
of the gradient G, (b) the amount of cash flow in year 6, and (c) the value of n for the
gradient if the cash flow ends in year 12.

(a) G = $5 million (b) CF6 = $6000 + 5(6) = 6030 million (c) n = 12

2.48 For a cash flow sequence that starts in year 1 and is described by (900 – 100k), where k
represents years 1–5, determine (a) the value of the gradient G, and (b) the cash flow in
year 5.

(a) G = $–100 (b) CF5 = 900 – 100(5) = $400

2.49 Profits from recycling paper, cardboard, aluminum, and glass at a liberal arts college have
increased at a constant rate of $1100 in each of the last 3 years. This year’s profit (end of
year 1) is expected to be $6000, and the profit trend is expected to continue through year 5.

Copyright 2021 © McGraw-Hill Education. All rights reserved. No reproduction or distribution without the prior
written consent of McGraw-Hill Education.
14
(a) What will the profit be at the end of year 5? (b) What is the present worth of the profit
at an interest rate of 8% per year? Solve using factors and a spreadsheet.

Factor solution:
(a) Profit in year 5 = 6000 + 1100(4) = $10,400

(b) P = 6000(P/A,8%,5) + 1100(P/G,8%,5)


= 6000(3.9927) + 1100(7.3724)
= $32,066
Spreadsheet solution:
Enter the increasing arithmetic series $6000 through $10,400 into cells A2 through A6.
The function = NPV(8%,A2:A6) displays $32,066

2.50 An arithmetic cash flow gradient series equals $500 in year 1, $600 in year 2, and amounts
increasing by $100 per year through year 9. At i = 10% per year, determine the present
worth of the cash flow series in year 0. Solve using factors and a spreadsheet.

Factor solution
P0 = 500(P/A,10%,9) + 100(P/G,10%,9)
= 500(5.7590) + 100(19.4215)
= 2879.50 + 1942.15
= $4821.65

Spreadsheet solution
Enter the series in cells A2 through A10. The function = NPV(10%,A2:A10) displays
$4821.66

2.51 Solar Hydro manufactures a revolutionary aeration system that combines coarse and fine
bubble aeration components. This year (year 1) the cost for check valve components is
$9000. Based on closure of a new contract with a distributor in Malaysia and volume
discounts, the company expects this cost to decrease. The cost in year 2 and each year
thereafter should decrease by $560. (a) Using factors, what is the equivalent annual cost for
a five-year period at an interest rate of 10% per year? (b) Solve using a spreadsheet and
compare the two methods to determine the A value.

(a) A = 9000 – 560(A/G,10%,5)


= 9000 – 560(1.8101)
= $7986

(b) Two functions are necessary to find A. Enter the series into cells A2 through A6. Enter
the function = NPV(10%,A2:A6) into cell A7, followed by = – PMT(10%,5,A7) to
display an A of $7986.33.

In this case, the spreadsheet solution takes more time since there is no direct function for
gradient factors.

Copyright 2021 © McGraw-Hill Education. All rights reserved. No reproduction or distribution without the prior
written consent of McGraw-Hill Education.
15
2.52 A report by the Government Accountability Office (GAO) shows that the GAO expects the
U.S. Postal Service to lose a record $7 billion at the end of this year, and, if the business
model is not changed, the losses will total $241 billion by the end of year 10. If the losses
increase uniformly over the 10-year period, determine the following:
a. The expected increase in losses each year.
b. The loss 5 years from now.
c. The equivalent annual cost of the losses at an interest rate of 8% per year.

In $ billion units,
(a) G = (241 – 7)/9 = $26 billion per year

(b) Loss in year 5: 7 +4(26) = $111 billion

(c) A = 7 + 26(A/G,8%,10)
= 7 + 26(3.8713)
= $107.7 billion

2.53 Omega Instruments budgeted $300,000 per year to pay for special-order ceramic parts over
the next 5 years. If the company expects the cost of the parts to increase uniformly
according to an arithmetic gradient of $10,000 per year, what is the cost estimated to be in
year 1 at an interest rate of 10% per year?

300,000 = A + 10,000(A/G,10%,5)
300,000 = A + 10,000(1.8101)
A = $281,899

2.54 As shown in the table below, Charlotte’s mother and father have experienced decreasing
revenues from their business, Thompson’s Print and Gift Shop, over the last 8 years due to
the increased use of digital media and e-commerce purchases. Determine the following for
Charlotte’s parents at i = 8% per year using the method(s) identified in each part.
a. Equivalent annual revenues for each of the 8 years by factor and spreadsheet.
b. The equivalent current value (year 8) of the recorded revenues by factor and spreadsheet.
c. (Optional question) The current purchasing power of the 8 years of revenue if inflation
has averaged 3% per year using the purchasing power reductions listed in Figure 1.3 by
spreadsheet only.

Year 1 2 3 4 5 6 7 8
Revenue, $1000 200 195 190 185 180 175 170 165

(a) A = 200,000 – 5,000(A/G,8%,8)


= 200,000 – 5,000(3.0985)
= $184,508

(b) F = 184,508(F/A,8%,8)
= 184,508(10.6366)
= $1,962,532

Copyright 2021 © McGraw-Hill Education. All rights reserved. No reproduction or distribution without the prior
written consent of McGraw-Hill Education.
16
(c) Initially, determine P using NPV function in cell B12. Then PMT and FV determine (a)
and (b) answers, respectively. Find current purchasing power of each revenue using the
reduction percentage. Again, the NPV and FV functions are applied to determine F with
3% inflation removed. Purchasing power is F = $1,745.235.

2.55 HGT Oil expects receipts from a fracked well to decline according to an arithmetic gradient
of $50,000 per year. If this year’s receipts are expected to be $280,000 (i.e., end of year 1)
and the company expects the useful life of the well to be 5 years, (a) what is the amount of
the cash flow in year 3, and (b) what is the equivalent uniform annual worth in years 1
through 5 of the income from the well at an interest rate of 12% per year?

(a) CF3 = 280,000 – 2(50,000)


= $180,000

(b) A = 280,000 – 50,000(A/G,12%,5)


= 280,000 – 50,000(1.7746)
= $191,270

2.56 For the cash flow revenues shown below, find the value of G that makes the equivalent
annual worth in years 1 through 7 equal to $500. The interest rate is 10% per year.
Yea Cash Flow, $ Year Cash Flow, $
r
0 4 200 + 3G
1 200 5 200 + 4G
2 200 + G 6 200 + 5G
3 200 + 2G 7 200 + 6G

500 = 200 + G(A/G,10%.7)


500 = 200 + G(2.6216)
G = $114.43

Copyright 2021 © McGraw-Hill Education. All rights reserved. No reproduction or distribution without the prior
written consent of McGraw-Hill Education.
17
2.57 For the cash flows shown, determine the value of G that makes the present worth in year 0
equal to $14,000. The interest rate is 10% per year.
Year 0 1 2 3 4
Cash flow, $ per year — 8000 8000−G 8000−2G 8000−3G

14,000 = 8000(P/A,10%,4) – G(P/G,10%,4)


14,000 = 8000(3.1699) – G(4.3781)
G = $2594.55

2.58 GKX Industries expects sales of its hydraulic seals (in inch and metric sizes) to increase
according to the cash flow sequence ($70 + 4k), where k is in years and cash flow is in
$1000.
a. What is the amount of the cash flow in year 3?
b. What is the future worth of the entire cash flow series in year 10 at i = 10% per year?

(a) CF3 = 70 + 3(4) = $82 ($82,000)

(b) P = 74(P/A,10%,10) + 4(P/G,10%,10)


= 74(6.1446) + 4(22.8913)
= $546.266 ($546,266)

F = 546.266(F/P,10%,10)
= 546,266(2.5937)
= $1416.850 ($1,416,850)

2.59 Amazon plans to install a computer system to “cube” an item’s dimensions—measure its
height, length, and width so that the proper box size will be used for shipment. This will
save packing material, cardboard, and labor. If the savings will be $150,000 the first year,
$160,000 the second, and amounts increasing by $10,000 each year for 8 years, what is the
present worth of the system at an interest rate of 15% per year?

P = 150,000(P/A,15%,8) + 10,000(P/G,15%,8)
= 150,000(4.4873) + 10,000(12.4807)
= $797,902

2.60 Ford Motor Company was able to reduce by 80% the cost required for installing data
acquisition instrumentation on test vehicles by using MTSdeveloped spinning wheel force
transducers. The cost this year (i.e., end of year 1) is expected to be $2000. (a) What was
the cost the year before installation of the transducers? (b) If the costs are expected to
increase by $250 each year for the next 4 years, what is the equivalent annual worth of the
costs (years 1–5) at an interest rate of 7% per year?

(a) Cost = 2000/0.2


= $10,000

Copyright 2021 © McGraw-Hill Education. All rights reserved. No reproduction or distribution without the prior
written consent of McGraw-Hill Education.
18
(b) A = 2000 + 250(A/G,7%,5)
= 2000 + 250(1.8650)
= $2466.25 per year

2.61 For the cash flow series shown below, determine the value of G that will make the future
worth in year 4 equal to $6000 at an interest rate of 15% per year.
Year 0 1 2 3 4
Cash flow 0 2000 2000−G 2000−2G 2000−3G

Convert future to present and then solve for G using P/G factor:
6000(P/F,15%,4) = 2000(P/A,15%,4) – G(P/G,15%,4)
6000(0.5718) = 2000(2.8550) – G(3.7864)
G = $601.94

2.62 For the cash flows below, determine the amount in year 1 if the annual worth in years 1
through 9 is $601.17 and the interest rate is 10% per year.
Year 1 2 3 4 5 6 7 8 9
Cost, $ A A+30 A+60 A+90 A+120 A+15 A+18 A+21 A+24
0 0 0 0

601.17 = A + 30(A/G,10%,9)
601.17 = A + 30(3.3724)
A = $500

2.63 A major drug company anticipates that in future years it could be involved in litigation
regarding perceived side effects of one of its antidepressant drugs. In order to prepare a
“war chest,” the company wants to have money available 6 years from now that has a
present worth today of $50 million. The company expects to set aside $6 million the first
year and uniformly increase amounts in each of the next 5 years (through year 6). (a) If the
company can earn 12% per year on the money it sets aside, by how much must it increase
the amount set aside each year to achieve its goal? (b) What is the projected purchasing
power of the $50 million present worth equivalent 6 years hence, if inflation prevails at 5%
per year?

(a) In $1 million units,


50 = 6(P/A,12%,6) + G(P/G,12%,6)
50 = 6(4.1114) + G(8.9302)
G = $ 2.83662 ($2,836,620)

(b) Purchasing power in 6 years: 50,000,000/(1.05)6 = $37,310,770

2.64 The future worth in year 10 of an arithmetic gradient cash flow series for years 1 through
10 is $500,000. (a) If the arithmetic gradient, G, is +$3000 per year, determine the cash
flow, A, in year 1 at an interest rate of 10% per year. (b) (Advanced exercise) Use the

Copyright 2021 © McGraw-Hill Education. All rights reserved. No reproduction or distribution without the prior
written consent of McGraw-Hill Education.
19
GOAL SEEK tool to determine the base amount, A, of the series. (Hint: Refer to Appendix
A to learn how to use this spreadsheet tool and make a guess that A = $10,000 initially.)

Copyright 2021 © McGraw-Hill Education. All rights reserved. No reproduction or distribution without the prior
written consent of McGraw-Hill Education.
20
(a) P in year 0 = 500,000(P/F,10%,10)
= 500,000(0.3855)
= $192,750

192,750 = A(P/A,10%.10) + 3000(P/G,10%,10)


192,750 = A(6.1446) + 3000(22.8913)
A = $20,193

(b) Enter a cash flow of $10,000 in cell B5 and increase by $3000 each year. Use the NPV
and FV functons, respectively. Use the GOAL SEEK template to set cell B17 to 500,000
by changing cell B5. Display shows that A = $20,196.32 (cell E5) and F = $500,000.00.

2.65 Assume you work at Taco Cabaña part time during college. Your federal income taxes are
$3500 this year (year 1) and are expected to follow a geometric gradient series with
increases of 5% each year through year 6, when you plan to complete graduate studies.
Calculate the present worth of the tax series at i = 10% per year.

Pg = 3500{1 – [(1 + 0.05)/(1 + 0.10)]6}/(0.10 – 0.05)


= $17,049

2.66 If you were told to prepare a table of factor values (like those in the back of this book) for
calculating the present worth of a geometric gradient series, determine the first three values

(n = 1, 2, and 3) for an interest rate of 10% per year and a rate of change g of 4% per year.

Copyright 2021 © McGraw-Hill Education. All rights reserved. No reproduction or distribution without the prior
written consent of McGraw-Hill Education.
21
Copyright 2021 © McGraw-Hill Education. All rights reserved. No reproduction or distribution without the prior
written consent of McGraw-Hill Education.
22
For n = 1: {1 – [(1+0.04)1/(1+0.10)1}]}/(0.10 –0.04) = 0.9091
For n = 2: {1 – [(1+0.04)2/(1+0.10)2}]}/(0.10 –0.04) = 1.7686
For n = 3: {1 – [(1+0.04)3/(1+0.10)3}]}/(0.10 –0.04) = 2.5812

2.67 Charlotte, a newly graduated chemical engineer is preparing long term for her retirement
by depositing 10% of her salary each year into a hightechnology stock fund. If her salary
this year (end of year 1) is $60,000, and she expects it to increase by 4% each year,
determine the future worth of the investment fund after 15 years if it earns 4% per year.

Here g = i = 0.04. Determine P, then F.

P = 60,000(0.1)[15/(1 + 0.04)]
= $86,538

F = 86,538(F/P,4%,15)
= 86,538(1.8009)
= $155,847

2.68 Determine the difference in the present worth values of the following two commodity
contracts at an interest rate of 8% per year.
Contract 1: costs $10,000 in year 1; costs will escalate at a rate of 4% per year for 10 years.
Contract 2: same cost in year 1, but costs will escalate at 6% per year for 11 years.

Pg1 = 10,000{1 – [(1 + 0.04)/(1 + 0.08)]10}/(0.08 – 0.04)


= $78,590

Pg2 = 10,000{1 – [(1 + 0.06)/(1 + 0.08)]11}/(0.08 – 0.06)


= $92,926

Difference: Pg2 is larger by $14,336

2.69 A 50-year-old engineer planning for retirement places 10% of his salary each year into a
highyield bond fund. If his salary this year (end of year 1) is $100,000 and he expects his
salary to increase by 3% each year, determine the future worth of the retirement fund after
15 years provided it earns 7% per year.

First find Pg and then convert to F in year 15


Pg = (0.10)(100,000){1 – [(1 + 0.03)/(1 + 0.07)]15}/(0.07 – 0.03)}
= 10,000(10.883) = $108,830

F = 108,830(F/P,7%,15)
= 108,830 (2.7590)
= $300,262

Copyright 2021 © McGraw-Hill Education. All rights reserved. No reproduction or distribution without the prior
written consent of McGraw-Hill Education.
23
2.70 The future worth in year 10 of a geometric gradient series of cash flows was found to be
$80,000. If the interest rate was 15% per year and the annual rate of increase was 9% per
year, what was the cash flow amount in year 1?

First convert future worth to P, then use Pg equation to find A1

P = 80,000(P/F,15%,10)
= 80,000(0.2472)
= $19,776

Pg = 19,776 = A1{1 – [(1 + 0.09)/(1 + 0.15)]10}/(0.15 – 0.09)

A1 = 19,776/6.91373
= $2860.40

2.71 A company that manufactures purgable hydrogen sulfide monitors is planning to make
deposits such that each one is 5% larger than the preceding one. (a) How large must the
first amount be (at the end of year 1) if the deposits extend through year 10 and the fourth
deposit is $12,500? Use an interest rate of 10% per year. (b) What is the 10-year equivalent
annual worth of the deposits?

(a) Decrease deposit in year 4 by 5% per year for three years to get back to year 1.
First deposit = 12,500/(1 + 0.05)3
= $10,798.00

(b) Determine Pg, then convert to an A value

Pg = 10,798{1 – [(1 + 0.05)/(1 + 0.10)]10}/(0.10 – 0.05)


= 10,798(7.43981)
= $80,335

A = 80,335(A/P,10%,10)
= 80,335(0.16275)
= $13,074

2.72 Find the 10-year equivalent annual worth of an investment that starts at $8000 in year 1 and
increases by 10% each year. The rate of return is 10% per year.

First find P and then convert to A with g = i = 0.10

P = 8000[10/(1 + 0.10)]
= $72,727

A = 72,727(A/P,10%,10)
= 72,727(0.16275)
= $11,836

Copyright 2021 © McGraw-Hill Education. All rights reserved. No reproduction or distribution without the prior
written consent of McGraw-Hill Education.
24
2.73 Verizon Communications said it plans to spend $22.9 billion to expand its fiber-optic
Internet and television network so that it can compete with cable TV providers like
Comcast Corp. The company expects to attract 950,000 customers in year 1 and grow its
customer base by a constant amount of 142,500 customers per year. Estimates indicate that
income will average $800 per customer per year. What is the future worth of the total
subscription income in year 5 assuming Verizon uses a MARR of 10% per year?

P = 950,000(800)(P/A,10%,5) + (142,500)(800)(P/G,10%,5)
= 760,000,000(3.7908) + 114,000,000(6.8618)
= $3,663,253,200

F = 3,663,253,200 (F/P,10%,5)
= 3,663,253,200 (1.6105)
= $5,899,669,279

2.74 Altmax LLC, a company that manufactures automobile wiring harnesses, has budgeted
P = $400,000 now to pay for a specially designed wire clip over the next 5 years. If the
company expects the cost of the clips to increase by 4% each year, what is the expected
cost in year 3 if the company uses an interest rate of 10% per year?

Solve for A1 in geometric gradient equation and then find cost in year 3

400,000 = A1[1 – (1.04/1.10)5]/(0.10 – 0.04)


4.0759 A1 = 400,000
A1 = $98,138

Cost in year 3 = 98,138(1.04)2


= $106,146

2.75 A small northern California consulting firm wants to start a recapitalization pool for
replacement of network servers. If the company invests $5000 at the end of year 1 but
decreases the amount invested by 5% each year, how much will be in the account 5 years
from now? Interest is earned at a rate of 8% per year.

First find P and then convert to F

P = 5000[1 – (0.95/1.08)5]/(0.08 + 0.05)


= $18,207

F = 18,207(F/P,8%,5)
= 18,207(1.4693)
= $26,751

Copyright 2021 © McGraw-Hill Education. All rights reserved. No reproduction or distribution without the prior
written consent of McGraw-Hill Education.
25
2.76 Hughes Cable Systems plans to offer its employees a salary enhancement package that has
revenue sharing as its main component. Specifically, the company will set aside 1% of total
sales for year-end bonuses for all of its employees. The sales are expected to be $5 million
the first year, $5.5 million the second year, and amounts increasing by 10% each year for
the next 5 years. At an interest rate of 8% per year, what is the equivalent annual worth in
years 1 through 5 of the bonus package?

Solve for P in geometric gradient equation and then convert to A


A1 = 5,000,000(0.01) = 50,000

P = 50,000[1 – (1.10/1.08)5]/(0.08 – 0.10)


= $240,215

A = 240,215(A/P,8%,5)
= 240,215(0.25046)
= $60,164

2.77 Bart is a junior in college. He plans to invest equal, annual amounts starting 3 years from
now for 5 consecutive years to accumulate a total of $50,000 at the end of 15 years. If the
funds earn 8% per year, what is the required amount of each deposit?

A(F/A,8%,5)(F/P,8%,8) = 50,000
A(5.8666)(1,8509) = 50,000
A = $4604.70 per year

2.78 Assume you start an investment account by purchasing international corporate stocks with
$1000 now, $1500 at the end of year 3, and $500 at the end of each year in years 4 through
10. How much money would you have in the account immediately after the last deposit in
year 10 if the interest rate is 6% per year?

Factor:
F= 1000(F/P,6%,10) + 1500(F/P,6%,7) + 500(F/A,6%,7)
= 1000(1.7908) + 1500(1.5036) + 500(8.3938)
= $8243

Spreadsheet:
For example, if the amounts are entered into cells F4 through F14, use the NPV and FV
functions in order.
In cell F16: = NPV(6%,F5:F14) + F4 displays a P of ,$4602.97
In cell F17: = – FV(6%,10,F16) displays a F of $8243.21

2.79 Find the present worth of cash flows of $1000 that start now (time 0) and continue through
year 8, provided the interest rate is 10% per year.

P = 1000 + 1000(P/A,10%,8) = $6334.90

Copyright 2021 © McGraw-Hill Education. All rights reserved. No reproduction or distribution without the prior
written consent of McGraw-Hill Education.
26
2.80 Calculate the equivalent annual worth for years 1 through 7 of Merchant Trucking
Company’s net cash flows shown below. Use an interest rate of 10% per year.
Year 0 1 2 3 4 5 6 7
Cash flow, $ million −200 200 200 200 200 200 200 200

A = –200(A/P,10%,7) + 200
= –200(0.20541) + 200
= $158.918 ($158,918,000)

2.81 Use (a) factors, and (b) a spreadsheet to determine the amount of money that you could
accumulate by the end of year 12 from deposits of $1000 per year in years 4 through 12 if
investments earn at the rate of 5.5% per year.

(a) Factor solution, using the F/A formula at i = 0.055:


F = 1000(F/A,5.5%,9)
= 1000(11.2563)
= $11,256.30

(b) Spreadsheet solution:


If zeroes and 1000 entries are correctly placed in cells A2 through A14, the NPV and
FV functions at 5.5% over the 12-year period display a P of $5920.58 and a F value of
$11,256.26, respectively.

2.82 For the cash flow diagram shown below, find the future worth in year 10 at an interest rate
of 10% per year.

F = 100(F/A,10%,3)(F/P,10%,7) + 100(F/A,10%,4)(F/P,10%,2) + 100


= 100(3.3100)(1.9487) + 100(4.6410)(1.2100) + 100
= $1306.58

2.83 How much money was deposited each year for 5 years if it now amounts to $100,000 and
the last deposit was made 10 years ago? Assume the account earned at 7% per year.

100,000 = A(F/A,7%,5)(F/P,7%,10)
100,000 = A(5.7507)(1.9672)
A = $8839.56

Copyright 2021 © McGraw-Hill Education. All rights reserved. No reproduction or distribution without the prior
written consent of McGraw-Hill Education.
27
2.84 Cindy’s father is now 50 years old. Twenty-one and twenty years ago, respectively, he
placed two equal amounts into a balanced stock and bond mutual fund. Due to the growth
of the fund, he could withdraw $10,000 now (year 0) and $10,000 per year for the next 14
years.
a. If the account earned interest at 10% per year, how large was the amount of each deposit
that Cindy’s father made? Are you surprised at how low the deposits were?
b. (Advanced exercise) Use a spreadsheet and the GOAL SEEK tool to find the deposits.
(Hint: Refer to Appendix A to learn how to use this spreadsheet tool and make an initial
guess of some amount for the two deposits.)

(a) Move the two unknown deposits, X, to current year –1, use the A/P factor and set equal
to $10,000.

X(F/A,10%,2)(F/P,10%,19)(A/P,10%,15) = 10,000
X(2.1000)(6.1159)(0.13147) = 10,000
X = $5922.34

(b) For a spradsheet solution, use two FV functions to find the value in year –1, followed by
the PMT function for 15 years. Guess at a deposit amount of, say, $20,000, in years –21
and –20. Set GOAL SEEK to fix the PMT amount at $10,000 by changing the cells with
the $20,000 deposit entries. The result of GOAL SEEK will be $5922.17.

2.85 By spending $10,000 now, and $25,000 three years from now, a plating company can
increase its income in years 4 through 10. How much extra income per year is needed in
years 4 through 10 to recover the original expenditures plus a return of 12% per year?

A = [10,000(F/P,12%,3) + 25,000](A/P,12%,7)
= [10,000(1.4049) + 25,000](0.21912)
= $8556.42

2.86 Assume you borrow $10,000 today and promise to repay the loan in two payments, one in
year 2 and the other in year 5, with the one in year 5 being only half as large as the one in
year 2. At an interest rate of 10% per year, what is the size of the payment in year 5?

Let X = payment in year 5


10,000 = 2X(P/F,10%,2) + X(P/F,10%,5)
= 2X(0.8264) + X(0.6209)
= X(2.2737)

X = $4398.12

2.87 You hope to make two equal-amount deposits for future savings, one now and the other 3
years from now in order to accumulate $300,000 ten years in the future. If the interest rate
is 14% per year, what is the size of each deposit?

Copyright 2021 © McGraw-Hill Education. All rights reserved. No reproduction or distribution without the prior
written consent of McGraw-Hill Education.
28
x(F/P,14%,10) + x(F/P,14%,7) = 300,000
x(3.7072) + x(2.5023) = 300,000
6.2095x = 300,000
x = $48,313.06

2.88 For the cash flow diagram below, what value of x will make the total equivalent worth in
year 7 equal to $8000 at i = 10% per year?

8,000 = 700(F/A,10%,7) + x(P/F,10%,1)


8,000 = 700(9.4872) + x(0.9091)
1358.96 = x(0.9091)
x = $1494.84

2.89 If a company makes a deposit of $1000 now, in what year must it make another deposit of
$1000 for the account to have a value as close to $8870 as possible in year 20? Assume the
account earns interest at 10% per year.

1000(F/P,10%,20) + 1000(F/P,10%,n) = 8870


1000(6.7275) + 1000(F/P,10%,n) = 8870
1000(F/P,10%,n) = 2142.5
(F/P,10%,n) = 2.1425

From 10% factor tables, n is very close to 8 years where (F/P,10%,8) = 2.1436
Payment year = 20 – 8
= 12

2.90 You plan to pay $38,000 cash for the new truck you want to buy 5 years from now. If you
have already saved $12,500, how much will your wellto- do aunt have to give you 2 years
from now (as a graduation present) for you to have the total amount of $38,000? Assume
all funds are invested and return 8% per year.

12,500(F/P,8%,5) + x(F/P,8%,3) = 38,000


12,500(1.4693) + x(1.2597) = 38,000
x = $15,586

Copyright 2021 © McGraw-Hill Education. All rights reserved. No reproduction or distribution without the prior
written consent of McGraw-Hill Education.
29
2.91 Loadstar Sensors is a company that makes load/force sensors based on capacitive sensing
technology. The company wants to have $30 million for a plant expansion five years from
now. If the company currently has a total of $15 million in an investment account set aside
for the expansion, how much must the company add to the account next year (i.e., 1 year
from now) to accumulate the $30 million 5 years from now? The account earns interest at
10% per year.

P = 30,000,000(P/F,10%,4) – 15,000,000(F/P,10%,1)
= 30,000,000(0.6830) – 15,000,000(1.10)
= $3,990,000

2.92 Amalgamated Iron and Steel purchased a new machine for ram cambering large I-beams.
The company expects to bend 80 beams at $2000 per beam in each of the first 3 years, after
which the company expects to bend 100 beams per year at $2500 per beam through year 8.
If the company’s minimum attractive rate of return is 18% per year, what is the present
worth of the expected income? Solve using factors and a spreadsheet.

(a) Factor
P = 80(2000)(P/A,18%,3) + 100(2500)(P/A,18%,5)(P/F,18%,3)
= 80(2000)(2.1743) + 100(2500)(3.1272)(0.6086)
= $823,691

(b) Spreadsheet
For entries in A2 through A4 of 160000 and A5 through A9 of 250000, the NPV
function displays a P value of $823,706.88.

2.93 Sierra Electric Company is considering the purchase of a hillside ranch for possible use as a
wind energy farm sometime in the future. The owner of the 500-acre ranch will sell for
$3000 per acre if the company will pay her in two payments—one payment now and
another that is twice as large three years from now. If the transaction interest rate is 8% per
year, what is the amount of the first payment?

Cost of the ranch is P = 500(3000) = $1,500,000

1,500,000 = x + 2x(P/F,8%,3)
1,500,000 = x + 2x(0.7938)
x = $579,688

2.94 The cost of energy for operating high-lift pumps in a water distribution system was $1.4
million for the first 4 years. Beginning in year 5, the cost decreased by $30,000 each year
through the end of year 12. What is the equivalent annual cost of the energy in years 1
through 12 if the interest rate is 8% per year?

First find P in year 0 and then convert to A in years 1 through 12

Copyright 2021 © McGraw-Hill Education. All rights reserved. No reproduction or distribution without the prior
written consent of McGraw-Hill Education.
30
P0 = 1,400,000(P/A,8%,3) + [1,400,000(P/A,8%,9) – 30,000(P/G,8%,9)](P/F,8%,3)
= 1,400,000(2.5771) + [1,400,000(6.2469) – 30,000(21.8081)](0.7938)
= $10,030,907

A = 10,030,907(A/P,8%,12)
= 10,030,907(0.13270)
= $1,331,101

2.95 An arithmetic gradient has cash flow of $500 in year 2, $600 in year 3, and amounts
increasing by $100 per year through year 10. At i = 10% per year, what is the present worth
of the cash flow series in year 0?

P1 = 500(P/A,10%,9) + 100(P/G,10%,9)
= 500(5.7590) + 100(19.4215)
= 2879.50 + 1942.15
= $4821.65

P0 = P1(P/F,10%,1)
= 4821.65(0.9091)
= $4383.36

2.96 An industrial engineering consulting firm signed a lease agreement for simulation software.
Calculate the present worth in year 0 if the lease requires a payment of $30,000 now and
amounts increasing by 5% per year through year 7. Use an interest rate of 10% per year.

Let Pg represent P in year –1; then find P in year 0.

Pg = (30,000) [1 – (1.05/1.10)8]
(0.10 – 0.05)
= $186,453.68

P0 = 186,453.68(F/P,10%,1)
= 186,453.68(1.10)
= $205,099

2.97 In planning for your retirement, you know that you won’t be able to save any money for the
first 3 years after you start working, but you expect to save $5000 in year 4, $5150 in year
5, and amounts increasing by 3% each year through year 25. (a) If your investments earn
10% per year, what amount will you have at the end of year 25? (b) If you had the total
future equivalent amount calculated above in hand today, and future inflation averaged 3%
per year (the same as the percentage increase you expect to add annually to your savings),
what is the remaining purchasing power 25 years in the future?

(a) P3 = {5000[1 – (1.03/1.10) 22]/(0.10 – 0.03)}


= 54,615.32

Copyright 2021 © McGraw-Hill Education. All rights reserved. No reproduction or distribution without the prior
written consent of McGraw-Hill Education.
31
F25 = 54,615.32 (F/P,10%,22)
= 54,615.32 (8.1403)
= $444,585

(b) Using the 52% decrease amount found in Table 1.3, 48% remains
Purchasing power in year 25 is 444,585(0.48) = $212,401

By Equation [1.8], the more accurate decrease is 0.5224; therefore 0.4776 remains.
Purchasing power in year 25 is 444,585(0.4776) = $212,334

2.98 Levi Strauss has some of its jeans stonewashed under a contract with independent U.S.
Garment Corp. If U.S. Garment’s operating cost per machine is $22,000 per year for years
1 and 2 and then it increases by $1000 per year through year 5, what is the equivalent
uniform annual cost per machine (years 1–5) at an interest rate of 12% per year?

Find P at t = 0, then convert to A


P = [22,000(P/A,12%,4) + 1000(P/G,12%,4) + 22,000](P/F,12%,1)
= [22,000(3.0373) + 1000(4.1273) + 22,000](0.8929)
= $82,993

A = 82,993(A/P,12%,5)
= 82,993(0.27741)
= $23,023

2.99 San Antonio is considering various options for providing water in their 50-year plan,
including desalting. One brackish aquifer is expected to yield desalted water that will
generate revenue of $4.1 million per year for the first 4 years, after which less production
will decrease revenue each year by $50,000 per year. If the aquifer will be totally depleted
in 25 years, what is the present worth of the desalting option at an interest rate of 6% per
year? Solve using factors and a spreadsheet with a single NPV function. What is the
numerical difference in the two results?

Factor
P = [4,100,000(P/A,6%,22) – 50,000(P/G,6%,22)](P/F,6%,3)
+ 4,100,000(P/A,6%,3)
= [4,100,000(12.0416) – 50,000(98.9412](0.8396)
+ 4,100,000(2.6730)
= $48,257,271

Spreadsheet
Sample solution: Enter 4,100,000 in cells B2-B5 followed by a decreasing series in cells
B6-B26. The function = NPV(6%,B2:B26) displays a P value of $48,258,115 (rounded).

Difference: Spreadsheet answer is larger by $844 due to accuracy of the computations.

Copyright 2021 © McGraw-Hill Education. All rights reserved. No reproduction or distribution without the prior
written consent of McGraw-Hill Education.
32
2.100 A low-cost noncontact temperature measuring tool may be able to identify railroad car
wheels that are in need of repair long before a costly structural failure occurs. If the
BNSF railroad saves $100,000 in years 1 through 5, $110,000 in year 6, and constant
amounts increasing by $10,000 each year through year 20, what is the equivalent annual
worth over the 20 years of the savings? The interest rate is 10% per year. Solve using (a)
factors, and (b) a spreadsheet.

Factor
First find P in year 0 and then convert to A

P0 = 100,000(P/A,10%,4) + [100,000(P/A,10%,16) + 10,000(P/G,10%,16)](P/F,10%,4)


= 100,000(3.1699) + [100,000(7.8237) + 10,000(43.4164)](0.6830)
= $1,147,883

A = 1,147,883(A/P,10%,20)
= 1,147,883(0.11746)
= $134,830

Spreadsheet
Sample solution: Enter 100,000 in cells B2-B6 followed by the increasing series in cells
B7-B21. The function = NPV(10%,B2:B21) displays a P value of $1,147,896,39.
Convert to A using a PMT function to display $134,831.48

2.101 A start-up company selling color-keyed carnauba car wax borrows $40,000 at an interest
rate of 10% per year. In planning for the financial future of the company, the owner
wishes to repay the loan over a 5-year period with annual payments such that the third
through fifth payments are $2000 greater than the first two. Determine the size of the first
two payments.

Let x = size of first two payments

40,000 = x(P/A,10%,2) + (x + 2000)(P/A,10%,3)(P/F,10%,2)


40,000 = x(1.7355) + (x + 2000)(2.4869)(0.8264)
3.79067x = 35,889.65
x = $9467.89

2.102 Silastic-LC-50 is a liquid silicon rubber designed to provide high clarity, superior
mechanical properties, and short cycle time for high-speed manufacturers. One high-
volume manufacturer used it to achieve smooth release from molds. The company’s
projected growth will result in silicon costs of $26,000 in years 1 and 2, with costs
increasing by $2000 per year in years 3 through 5. At an interest rate of 10% per year,
what is the present worth, annual worth, and future worth of these costs? Solve using (a)
factors, and (b) a spreadsheet.

Copyright 2021 © McGraw-Hill Education. All rights reserved. No reproduction or distribution without the prior
written consent of McGraw-Hill Education.
33
Factors
P = 26,000(P/F,10%.1) + [26,000(P/A,10%,4) + 2000(P/G,10%,4)](P/F,10%,1)
= 26,000(0.9091) + [26,000(3.1699) + 2000(4.3781)](0.9091)
= $106,523

A = 106,523(A/P,10%,5)
= 106,523(0.2638)
= $28,101

F = 106,523(F/P,10%,5)
= 106,523(1.6105)
= $171,555

Spreadsheet solution

2.103 The Pedernales Electric Cooperative estimates that the present worth now of income from
an investment in renewable energy sources is $12,475,000. There will be no income in
years 1 and 2, but in year 3 income will be $250,000, and thereafter it will increase
according to an arithmetic gradient through year 15. What is the required gradient, if the
interest rate is 15% per year? Solve using (a) factors, and (b) a spreadsheet with the
GOAL SEEK tool.

(a) Factor solution


12,475,000(F/P,15%,2) = 250,000(P/A,15%,13) + G(P/G,15%,13)
12,475,000(1.3225) = 250,000(5.5831) + G(23.1352)
16,498,188 –1,395,775 = 23.1352G
23.1352G = 15,102,413
G = $652,789

Copyright 2021 © McGraw-Hill Education. All rights reserved. No reproduction or distribution without the prior
written consent of McGraw-Hill Education.
34
(b) Spreadsheet solution and GOAL SEEK

2.104 John and Mariam have owned a house in Virginia for the last 9 years. The series of their
annual property tax expenses is shown below. If the same funds had been invested in the
stock market, they would likely have earned at a rate of 10% per year. Calculate the
equivalent annual amount in years 1 through 9 if they had been able to invest the same
amount of money each year in the stock market instead of paying property taxes.

Property Property
Year Taxes, $ Year Taxes, $
0 5000 5 7500
1 5500 6 8000
2 6000 7 8500
3 6500 8 9000
4 7000 9 9500

A = 5000(A/P,10%,9) + 5500 + 500(A/G,10%,9)


= 5000(0.17364) + 5500 + 500(3.3724)
= $8054

2.105 A build-to-operate (BTO) company signed a contract to operate Alamosa County


industrial wastewater treatments plants for the next 20 years. The contract will pay the
company $2.5 million now and amounts increasing by $200,000 each year through year
20. At an interest rate of 10% per year, what is the present worth now? Solve using (a)
tabulated factors, and (b) a spreadsheet.

(a) Factors: Find P in year –1 using gradient factor and then move forward 1 year

P–1 = 2,500,000(P/A,10%,21) + 200,000(P/G,10%,21)


= 2,500,000(8.6487) + 200,000(58.1095)
= $33,243,650

Copyright 2021 © McGraw-Hill Education. All rights reserved. No reproduction or distribution without the prior
written consent of McGraw-Hill Education.
35
F = P0 = 33,243,650(F/P,10%,1)
= 33,243,650 (1.1000)
= $36,568,015

(b) Spreadsheet: If entries are in cells B2 through B22, the function


= NPV(10%,B3:B22)+B2 displays $36,568,004, which is the
present worth in year 0.

2.106 Find the future worth in year 10 of $50,000 in year 0 and amounts increasing by 15% per
year through year 10 at an interest rate of 10% per year.

Pg-1 = 50,000{1 – [(1 + 0.15)/(1 + 0.10)]11}/(0.10 – 0.15)


= 50,000{–0.63063}/–0.05
= $630,635

F = 630,635(F/P,10%,11)
= 630,635(2.8531)
= $1,799,265

2.107 Calculate the present worth of all costs for a newly acquired 3D printer with an initial
cost of $29,000, a trade-in value of 8% of initial cost at the end of its 10-year life, and an
annual operating cost of $13,000 for the first 4 years, increasing by 10% per year
thereafter. Use an interest rate of 10% per year.

P = 29,000 + 13,000(P/A,10%,3) + 13,000[7/(1 + 0.10)](P/F,10%,3)


– 29,000(0.08)(P/F,10%,10)
= 29,000 + 13,000(2.4869) + 82,727(0.7513) – 29,000(0.08)(0.3855)
= $122,589

2.108 Union Pacific is considering the elimination of a railroad grade crossing by constructing a
dualtrack overpass. The railroad subcontracts for maintenance of its crossing gates at
$11,500 per year, starting next year (year 1). However, beginning 4 years from now the
costs are expected to increase by 10% per year into the foreseeable future (that is, $12,650
in year 4, $13,915 in year 5, etc.). If the railroad uses a 10-year study period and an interest
rate of 15% per year, how much could the railroad afford to spend now on the overpass in
lieu of the maintenance contracts? Solve using (a) factors, and (b) a spreadsheet.

(a) Factors (b) Spreadsheet


Find P in year 2, then move back to year 0

P2 = 11,500[1 – (1.10/1.15)8]/(0.15 – 0.10)


= $68,829

P0 = 11,500(P/A,15%,2) + P2(P/F,15%,2)
= 11,500(1.6257) + 68,829(0.7561)
= $70,737

Copyright 2021 © McGraw-Hill Education. All rights reserved. No reproduction or distribution without the prior
written consent of McGraw-Hill Education.
36
2.109 McCarthy Construction is trying to bring the company-funded portion of its employee
retirement fund into compliance with HB-301. The company has already deposited
$500,000 in each of the last 5 years. Beginning in year 6, McCarthy will increase its
deposits by 15% per year through year 20. How much will be in the fund immediately
after the last deposit, if the fund grows at a rate of 12% per year? Solve using
(a) tabulated factors, and (b) a spreadsheet.

(a) Factors:
Find P in year 4 for the geometric gradient, (b) Spreadsheet:
then move all cash flows to the future of year 20.

P4 = 500,000[1 – (1.15/1.12)16]/(0.12 – 0.15)


= $8,773,844

F = 500,000(F/A,12%,4)(F/P,12%,16) + P4(F/P,12%,16)
= 500,000(4.7793)(6.1304) + 8,773,844(6.1304)
= $68,436,684

ADDITIONAL PROBLEMS AND FE EXAM REVIEW QUESTIONS

2.110 A single deposit made 30 years ago by your grandmother is worth $45,000 today. If the
account earned interest at 10% per year, the amount she deposited was closest to:
a. $2579
b. $3285
c. $4565
d. $8505

P = 45,000(P/F,10%,30)
= $2578.50
Answer is (a)

2.111 An arithmetic gradient has cash flow of $1000 in year 4, $1200 in year 5, and amounts
increasing by $200 per year through year 10. If you use the factor 200(P∕G,10%,?) to find
P in year 3, the value of n to use in the P∕G factor is
a. 6
b. 7
c. 8
d. 9

Answer is (b)

Copyright 2021 © McGraw-Hill Education. All rights reserved. No reproduction or distribution without the prior
written consent of McGraw-Hill Education.
37
2.112 Summit Metals is planning to expand a local manufacturing operation 5 years from now at
a cost of $100,000. If the company plans to deposit money into an account each year for
4 years beginning 2 years from now (i.e., first deposit is in year 2) to pay for the expansion,
the equation that represents the amount of the deposit at 9% per year interest is
a. A = 100,000(A∕F,9%,5)
b. A = 100,000(A∕F,9%,4)
c. A = 100,000(A∕P,9%,4)
d. A = 100,000(A∕F,9%,4)(P∕F,9%,1)

Answer is (b)

2.113 For the diagram shown, the respective values of n to calculate the present worth in year 0
by the equation P0 = 100(P∕A,10%,n1)(P∕F,10%,n2) are
a. n1 = 6 and n2 = 1
b. n1 = 6 and n2 = 2
c. n1 = 7 and n2 = 1
d. n1 = 7 and n2 = 2

Answer is (a)

2.114 The amount of money that you can spend now for a much safer car in lieu of spending
$30,000 three years from now at an interest rate of 12% per year is closest to:
a. $15,710
b. $17,805
c. $19,300
d. $21,355

P = 30,000(P/F,12%,3)
= 30,000(0.7118)
= $21,354
Answer is (d)

2.115 A manufacturing company spent $30,000 on a new conveyor belt. If the conveyor belt
resulted in cost savings of $4200 per year, the length of time it would take for the
company to recover its investment at 8% per year is closest to:
a. 7 to 8 years

Copyright 2021 © McGraw-Hill Education. All rights reserved. No reproduction or distribution without the prior
written consent of McGraw-Hill Education.
38
b. 9 to 10 years
c. 11 to 12 years
d. 13 to 14 years

30,000 = 4200(P/A,8%,n)
(P/A,8%,n) = 7.14286
n is between 11 and 12 years
Answer is (c)

2.116 The cost of lighting and maintaining a transmission and receiving antenna on top of a
mountain in El Paso, Texas, is $90,000 per year. At an interest rate of 10% per year, the
present worth of maintaining the antenna for 10 years is closest to:
a. $1,015,000
b. $894,000
c. $712,000
d. $553,000

P = 90,000(P/A,10%,10)
= 90,000(6.1446)
= $553,014
Answer is (d)

2.117 An enthusiastic new engineering graduate plans to start a consulting firm by borrowing
$100,000 at 10% per year interest. The loan payment each year to pay off the loan in 7
years is closest to:
a. $18,745
b. $20,540
c. $22,960
d. $23,450

A = 100,000(A/P,10%,7)
= 100,000(0.20541)
= $20,541
Answer is (b)

2.118 Charley, an engineer who believes in the “save now, play later” philosophy wanted to
retire in 20 years with $1.5 million. At a 10% per year return on his investments, the
amount he must invest each year (starting in year 1) to reach the $1.5 million goal is
closest to:
a. $26,190
b. $28,190
c. $49,350
d. $89,680

Copyright 2021 © McGraw-Hill Education. All rights reserved. No reproduction or distribution without the prior
written consent of McGraw-Hill Education.
39
A = 1,500,000(A/F,10%,20)
= 1,500,000(0.01746)
= $26,190
Answer is (a)

2.119 An investment of $75,000 in equipment that will reduce the time for machining and
packaging self-locking fasteners will save $20,000 per year. At an interest rate of 10%
per year, the number of years required to recover the initial investment is closest to:
a. 9 years
b. 7 years
c. 5 years
d. 3 years

75,000 = 20,000(P/A,10%,n)
(P/A,10%,n) = 3.75
By interpolation or NPER function, n = 4.9 years
Answer is (c)

2.120 Sheryl is planning for her retirement now. She expects to save $5000 in year 1, $6000 in
year 2, and amounts increasing by $1000 each year through year 20. If the investments
earn 10% per year, the amount Sheryl will have at the end of year 20 is closest to:
a. $242,568
b. $355,407
c. $597,975
d. $659,125

F = [5000(P/A,10%,20) + 1000(P/G,10%,20)](F/P,10%,20)
= [5000(8.5136) + 1000(55.4069)](6.7275)
= $659,126
Answer is (d)

2.121 Income from a precious metals mining operation has been decreasing uniformly for 5
years. If income in year 1 was $300,000 and it decreased by $30,000 per year through
year 4, the equivalent annual worth of the income at 10% per year is closest to:
a. $310,500
b. $258,600
c. $203,900
d. $164,800

A = 300,000 – 30,000(A/G,10%,4)
= 300,000 – 30,000(1.3812)
= $258,564
Answer = (b)

Copyright 2021 © McGraw-Hill Education. All rights reserved. No reproduction or distribution without the prior
written consent of McGraw-Hill Education.
40
2.122 If you are able to save $5000 in year 1, $5150 in year 2, and amounts increasing by 3%
each year through year 20, the amount you will have at the end of year 20 at a 10% per
year return is closest to:
a. $60,810
b. $102,250
c. $351,500
d. $410,000

F = {5000[1 – (1.03/1.10)20]/(0.10 – 0.03)}(F/P,10%,20)


= {5000[1 – (1.03/1.10)20]/(0.10 – 0.03)}(6.7275)
= $351,528
Answer is (c)

2.123 The present worth in year 0 of a lease that requires a payment of $9000 now and amounts
increasing by 5% per year through year 10 at 8% per year interest is closest to:
a. $73,652
b. $79,939
c. $86,330
d. $87,454

P–1 = 9000[1– (1.05/1.08)11]/(0.08–0.05) = $79,939


P0 = 79,939(F/P,8%,1) = $86,335
Answer is (c)

2.124 At an interest rate of 8% per year, the future worth in year 15 of an investment plan that
requires a deposit of $9000 now and amounts increasing by 8% per year through year 7,
and nothing further, is closest to:
a. $282,303
b. $465,376
c. $228,395
d. $609,328

P–1 = A1(n/1+i)
= 9000[8/(1.08)]
= $66,667

P0 = P–1(F/P,8%,1)
= 66,667(1.0800)
= $72,000

F15 = P0(F/P,8%,15)
= 72,000(3.1722)
= $228,398
Answer is (c)

Copyright 2021 © McGraw-Hill Education. All rights reserved. No reproduction or distribution without the prior
written consent of McGraw-Hill Education.
41
Solutions to end-of-chapter problems
Basics of Engineering Economy, 3rd edition
Leland Blank and Anthony Tarquin

Chapter 3
Nominal and Effective Interest Rates

3.1 For an interest rate of 2% per quarter, determine the nominal interest rate per (a) semiannual
period; (b) year; and (c) 2 years.

(a) Nom i/6 mths = 0.02*2 = 4%


(b) Nom i/year = 0.02*4 = 8%
(c) Nom i/2 years = 0.02*8 = 16%

3.2 Identify the following interest rate statements as either nominal or effective: (a) 1.3% per
month; (b) 1% per week compounded weekly; (c) APR of 15% per year compounded
monthly; (d) effective 1.5% per month compounded daily; (e) 15% per year compounded
semiannually; ( f ) 6.2% APY; (g) 3% per quarter; (h) 3% per quarter compounded monthly.

(a) effective (b) effective (c) nominal (d) effective

(e) nominal (f) effective (g) effective (h) nominal

3.3 Identify the compounding period in months associated with each of the following interest
statements:
a. 1% per month
b. 2.5% per quarter
c. 9.3% per year compounded semiannually
d. Nominal 7% per year compounded quarterly
e. Effective 6.8% per year compounded monthly
f. Effective 3.4% per quarter compounded weekly

(a) Month (b) 3 months (c) 6 months (d) 3 months (e) Month (f) ¼ month or 1 week

3.4 Determine the number of times interest is compounded in 1 year for the following interest
statements: (a) 1% per month; (b) 2% per quarter; (c) 8% per year compounded
semiannually; and (d) 6% per year compounded continuously.

(a) 12 (b) 4 (c) 2 (d) Infinite

3.5 For an interest rate of 0.5% per 2 months, determine the number of times interest is
compounded in (a) 2 months; (b) 2 semiannual periods; and (c) 3 years.

Copyright 2021 © McGraw-Hill Education. All rights reserved. No reproduction or distribution without the prior
written consent of McGraw-Hill Education.
1
(a) one time (b) six times (c) 18 times

Copyright 2021 © McGraw-Hill Education. All rights reserved. No reproduction or distribution without the prior
written consent of McGraw-Hill Education.
2
3.6 Identify the compounding period for the following interest statements: (a) 2% per quarter;
(b) 8% per year compounded semiannually; (c) 4.5% per year compounded monthly;
(d) effective 3.5% per quarter compounded weekly; and (e) 1% per month compounded
continuously.

(a) Quarter (b) Semiannual (c) Month (d) Week (e) Continuous

3.7 Identify the following interest rate statements as either nominal or effective: (a) APR of
1.5% per month compounded daily; (b) 17% per year compounded quarterly; (c) effective
15% per year compounded monthly; (d) nominal 0.6% per month compounded weekly;
(e) 0.3% per week compounded weekly; and ( f ) 8% per year.

(a) Nominal; (b) Nominal; (c) Effective; (d) Nominal; (e) Effective; (f) Effective

3.8 You were just informed of the interest rate on your student loan. It is stated as 0.50% per
quarter. Determine the nominal interest rate r (a) per month; (b) per semiannual period;
(c) per year; and (d) over a 2-year period.

(a) r/month = 0.5/3 = 0.167%


(b) r/6-months = 0.5(2) = 1%
(c) r/year = 0.5(4) = 2%
(d) r/2-years = 0.5(8) = 4%

3.9 Convert the given interest rates in the left-hand column into the APR rates listed in the right-
hand column. Assume 4 weeks/month.

Given Interest Rate Requested Interest Rate


1% per month Nominal rate per year
3% per quarter Nominal rate per 6 months
2% per quarter Nominal rate per year
0.28% per week Nominal rate per quarter
6.1% per 6 months Nominal rate per 2 years

1% per month = nominal 12% per year


3% per quarter = nominal 6% per six months
2% per quarter = nominal 8% per year
0.28% per week is 0.28%(12) or nominal 3.36% per quarter
6.1% per six months = nominal 24.4% per two years

3.10 The Second National Bank of Fullerton advertises an APR of 14%, but it indicates in the
“small print” that compounding occurs on a monthly basis for personal loans with no
collateral required. Determine the APY you would pay using (a) an equation, and (b) a
spreadsheet function. (Note: Check Appendix A if you need help using a spreadsheet
function.)

Copyright 2021 © McGraw-Hill Education. All rights reserved. No reproduction or distribution without the prior
written consent of McGraw-Hill Education.
3
(a) Equation: i = (1 + 0.14/12)12 –1
= 14.93% per year

(b) Spreadsheet: = EFFECT(14%,12) displays 0.1493, which is 14.93%

3.11 The owner of DIY Truck Accessories provides loans at purchase time with payments every
3 months. When pressed for the effective annual rate, the number 15.865% compounded
quarterly was given. (a) Verify that the nominal annual rate is 15% as shown in Table 3.3.
(b) Determine the effective quarterly rate.

(a) Use Equation [3.2] and solve for r, the annual nominal rate.

0.15865 = (1 + r/4)4 – 1
(1.15865)0.25 = (1 + r/4)
0.0375 = r/4
r = 0.15 (15% per year)

(b) Effective i per quarter = r/4 = 15%/4 = 3.75% per quarter

3.12 An interest rate of 21% per year compounded every 4 months is equivalent to what
effective rate per year? Also, write the spreadsheet function.

i = (1 + 0.21/3)3 – 1
= 0.225 or 22.5% per year

Spreadsheet solution: = EFFECT(21%,3) displays 22.5%

3.13 Western Energy makes quarterly deposits into an account reserved for purchasing new
equipment 2 years from now. The rate of return on the deposits is 12% per year
compounded monthly. Calculate the effective annual rate of return, i.e., the APY.

i = (1 + r/m)m – 1
= (1 + 0.12/12)12 – 1
= 0.12683 (12.683%)

3.14 The Premier Car Title Loan Company makes emergency loans of up to $500 for 1 month
for a fee of 4% of the loan amount. Your friend, Clare, is in a tight spot and borrows $500.
What is (a) the amount of interest paid in 1 month; (b) the nominal interest rate per year;
and (c) the effective rate per year?

(a) Interest paid = 500(0.04) = $20

(b) Interest rate per month = 4%

r = (4%)(12) = 48% per year

Copyright 2021 © McGraw-Hill Education. All rights reserved. No reproduction or distribution without the prior
written consent of McGraw-Hill Education.
4
(c) i = (1 + 0.48/12)12 – 1
= 60.1% per year

3.15 Eckelberger Products, Inc. makes high-speed recorders with high-speed scanning. The
small company has been growing at an average rate of 75% per year for the past 4 years.
The CEO asked you to convert the past growth rate into a monthly rate for its annual
report. If the past growth rate was an effective rate, what was the effective growth rate per
month? Solve by equation and spreadsheet function, if instructed to do so.

Equation: 0.75 = (1 + r/12)12 – 1


r/12 = (1 + 0.75)1/12 – 1
= 0.0477 (4.77% per month)

Spreadsheet: Nominal function displays the nominal annual rate; divide by 12 for monthly.

= NOMINAL(75%,12)/12 displays a monthly rate 0.0477 or 4.77%, which


is the effective monthly rate.

3.16 Beth has a no-annual-fee Visa credit card that charges an APR of 12.5% per year. Interest
is compounded monthly. If a payment is missed by even 1 day, the rate jumps to an APR of
24.5% for the remainder of the time she keeps the card or until the total debt is paid off.
(This is not an uncommon practice for many banks that issue credit cards when you read
the fine print.) The current balance is $2000 and there is a minimum payment of $20.83 per
month at the 12.5% rate. Beth decided “enough already”; she decided to stop using the card
immediately and pay the balance off as soon as her budget allowed. She realizes that if she
misses a payment due date, the minimum will increase to $40.83 per month until the total
balance is paid at this higher APR.
a. Verify the monthly minimums for both nominal (APR) rates for a balance of $2000.
b. If she pays well over the minimum at $40 per month on time, how many months and
years will it take to become debt-free of this card? Use an equation and a spreadsheet
function to determine the answers.
c. If Beth misses the due date on the first payment of $40 and now increases the monthly
amount to $41, determine the months and years to be free of the debt.
d. After seeing the time required at $41 per month, she increases the payments to $50 and
stays on time every month. Now determine the months and years to be free of this credit
card debt.
e. Compare and comment on the three time periods determined in parts b, c, and d, and the
total amounts paid. What did you learn from this exercise?

(a) At 12.5%: Minimum = 2000(0.125/12) = $20.83 per month

At 24.5%: Minimum = 2000(0.245/12) = $40.83 per month

Copyright 2021 © McGraw-Hill Education. All rights reserved. No reproduction or distribution without the prior
written consent of McGraw-Hill Education.
5
(b) Determine n (in months) at the rate of 12.5%/12 = 1.0417% per month.

Equation: 2000 = 40(P/A,1.0417%,n)


(P/A,1.0417%,n) = 50.0

From the factor tables, n ≈ 70 months or about 6 years

Spreadsheet: = NPER(12.5%/12,–40,2000) displays an n of 71 months or 5.9 years.

(c) Determine n (in months) at the rate of 24.5%/12 = 2.0417% per month.

Equation: 2000 = 41(P/A,2.0417%,n)


(P/A,2.0417%,n) = 48.7805

From factor tables, n is hard to determine. In general, n > 240 months

Spreadsheet: = NPER(24.5%/12, –41,2000) displays an n of 272 months or 22.7 years.

(d) Equation: 2000 = 50(P/A,2.0417%,n)


(P/A,2.0417%,n) = 40.0

From factor tables, n ≈ 84 months or 7 years

Spreadsheet: = NPER(24.5%/12,–50,2000) displays an n of 83.9 months or 7 years.

(e) APR, % Payment, $ Time, months Total paid, $


12.5 40 71 2,840
24.5 41 272 11,152
24.5 50 84 4,200

The effect of increasing the payments by $9 from $41 to $50 at the higher rate provides
a huge reduction in time and total amount paid to become debt free.

Lesson learned: Sample answer - Don’t get into debt with a credit card.

3.17 Two brothers each deposited $5000 per year for 10 years into different annuity plans.
Abraham received an APY of 5%, while Mel, according to him, got a much higher rate at
5% due to continuous compounding. After 10 years, how much more did Mel have because
of continuous compounding? Round to the nearest dollar amounts.

Abraham: F = 5000(F/A,5%,10)
= $62,889

Mel: i = e0.05 – 1 = 1.05127 – 1


= 0.05127

Copyright 2021 © McGraw-Hill Education. All rights reserved. No reproduction or distribution without the prior
written consent of McGraw-Hill Education.
6
F = 5000(F/A,5.127%,10)
= 5000(12.652)
= $63,264

Difference = 63,264 – 62,889 = $375 more for Mel

3.18 Identify the payment period and compounding period for the following situations:
a. Deposits are made each quarter into an account reserved for purchasing new equipment 2
years from now. The interest rate on the deposits is 12% per year compounded monthly.
b. Cash from sales at a small hardware store is deposited daily into an account that pays
interest of 8% per year compounded semiannually.
c. Payroll deductions of $100 per week are made into an investment account that pays
interest of 6% per year compounded quarterly.

(a) PP = quarter CP = month


(b) PP = daily CP = 6 months
(c) PP = weekly CP = quarterly

3.19 How much money could Midland Refining afford to spend 4 years from now in lieu of
spending $400,000 now for upgrading its tank farm if the interest rate is 10% per year
compounded semiannually? Use factors and a spreadsheet to find the F value.

F = 400,000 (F/P,5%,8)
= 400,000(1.4775)
= $591,000

Spreadsheet function = FV(5%,8,–400000) displays an F of $590,982.18

3.20 A present sum of $5000 at an interest rate of 8% per year compounded semiannually is
equivalent to how much money 8 years ago?

P = 5000(P/F,4%,16)
= 5000(0.5339)
= $2669.50

3.21 Techten, Inc., which specializes in online security software development, wants to have
$85 million available in 3 years to expand its East Coast campus. How much money must
the company set aside now in an account that earns interest at a rate of 8% per year
compounded quarterly?

P = (85 million)(P/F,2%,12) = (85 million)(0.7885)


= $67.02 million

Copyright 2021 © McGraw-Hill Education. All rights reserved. No reproduction or distribution without the prior
written consent of McGraw-Hill Education.
7
3.22 An investment banker recommends that you place deposits of $5000 now and $7000 five
years from now into an aggressive growth fund of international IT corporate stocks that is
expected to return 8% per year compounded quarterly. If you do and it performs as
expected, how much will you have accumulated after 12 years? Solve using (a) factors and
(b) spreadsheet functions, as instructed.

(a) Factors: F = 5000(F/P,2%,48) + 7000(F/P,2%,28)


= 5000(2.5871) + 7000(1.7410)
= $25,123

(b) Spreadsheet: The sum of two FV functions displays F to be $25,122.52.

= FV(2%,48,–5000) + FV(2%,28,–7000)

3.23 Soil Mediators, Inc., plans to finance a site reclamation project using xeriscape landscaping
(meaning no irrigation; only rainfall) that will require a 4-year cleanup and planting period.
If the company borrows $4.5 million now and expects a MARR of 16% per year
compounded semiannually on its investment, how much will the company have to receive
in a lump sum payment when the project is over?

F = (4.5 million)(F/P,8%,8)
= (4.5 million)(1.8509)
= $8,329,050

3.24 Pollution control equipment for a pulverized coal cyclone furnace is expected to cost
$190,000 two years from now and another $120,000 four years from now. If Monongahela
Power wants to set aside enough money now to cover these costs, how much must be
invested at an interest rate of 9% per year compounded monthly? Solve using (a) factors
and (b) spreadsheet functions, as instructed.

(a) Factors: P = 190,000(P/F,0.75%,24) + 120,000(P/F,0.75%,48)


= 190,000(0.8358) + 120,000(0.6986)
= $242,634

(b) Spreadsheet: Summing two PV functions displays the investment now to be a P of


$–242,641.66

= PV(0.75%,24,,190000) + PV(0.75%,48,,120000)

3.25 Deployment of hypersonic missiles is the U.S. strategy to counter advanced missile
development by China and Russia. The development project was originally expected to
have an equivalent present worth of $3.9 billion, but due to extra time needed to write
computer code, delayed tests, and implementation problems the actual cost was much
higher. (a) Assume the total project development time was 10 years and costs increased at a
rate of 0.5% per month. What was the final projected future cost of the project? (b) If the

Copyright 2021 © McGraw-Hill Education. All rights reserved. No reproduction or distribution without the prior
written consent of McGraw-Hill Education.
8
project actually cost $12 billion over a 12-year period, use factors and the RATE function
to determine the rate of increase in cost per month from the original estimate.

In $ billion units,
(a) F = 3.9(F/P,0.5%,120)
= 3.9(1.8194)
= $7.09566 ($7,095,660,000)

(b) 12 = 3.9(F/P,i,144)
(F/P,i,144) = 3.077

Factor tables indicate that i is between 0.75% and 1.0% per month.

The spreadsheet function = RATE(144,–3.9,12) displays an i of 0.78% per month.

3.26 Ellis Autotrol, Inc. plans to install an ecofriendly energy recovery system that will cost an
estimated $86,900 if it is installed 5 years from now. The company uses an interest rate of
10% per year compounded continuously. (a) How much could it spend now? (b) Solve this
problem using either the EXP or EFFECT function on a spreadsheet.

(a) i/year = e0.10 –1


= 10.517%

P = 86,900{1/[(1+ 0.10517)5]}
= $52,708

(b) The relation = EXP(0.1) – 1 or the relation = EFFECT(10%,10000) both display the
i/year of 0.10517. Then the function = PV(10.517%,5,–86900) displays the P value of
$52,708.

3.27 A friend tells you she invested $10,000 in a cousin’s Harley-Davidson dealership in a
Wyoming resort district that promises a return of 20% per year compounded continuously.
(a) If the return is realized as promised, how much will her investment be worth 10 years
from now? (b) Solve this problem using either the EXP or EFFECT function on a
spreadsheet.

(a) i/year = e0.20 – 1


= 22.14%

F = 10,000(1 + 0.2214)10 [also F = 10,000(F/P,22.14%,10)]


= $73,889

(b) The relation = EXP(0.2) – 1 or the relation = EFFECT(20%,10000) both display the
i/year of 0.2214. Then the function = FV(22.14%,10,–10000) displays the F value of
$73,889.

Copyright 2021 © McGraw-Hill Education. All rights reserved. No reproduction or distribution without the prior
written consent of McGraw-Hill Education.
9
3.28 How much will ASW Hydrotherm have to deposit into an investment account each month
in order to accumulate $980,000 for production line expansion 3 years from now? The
company’s MARR is 18% per year compounded monthly.

A = 980,000(A/F,1.5%,36)
= 980,000(0.02115)
= $20,727

3.29 The West Gulf Maritime Association and Port Houston spent $500,000 on two simulators
to give new crane operators experience in the cab of ship-to-shore cranes that unload
massive containers. If the simulator gets new operators to more quickly meet the expected
standard of moving 30 to 35 containers each hour, what must be the savings in operator
training and accident reduction costs each year in order to recover the investment in 5 years
at an interest rate of 10% per year compounded quarterly? Use a formula or spreadsheet
function to solve, as instructed.

i/year = (1 + 0.10/4)4 – 1)
= 10.381% per year

A/P formula: A = 500,000{0.10381(1 + 0.10381)5/[(1 + 0.10381)5 – 1]}


= 500,000(0.26637)
= $133,186

Spreadsheet: = PMT(10.381%,5,–500000) displays an A of $133,185.

3.30 Fort Bliss, a U.S. Army military base, contributed $3.3 million of the $87 million capital
cost for a desalting plant constructed and operated by El Paso Water (EPW). In return,
EPW agreed to sell water to Fort Bliss at $0.85 per thousand gallons for 20 years. If the
Army base uses 200 million gallons of water per month, what is the Army’s total cost per
month for water (i.e., water cost and amortization) if the capital cost is amortized at 6% per
year compounded monthly?

A = 3,300,000(A/P,0.5%,240) + (200,000,000/1000)0.85
= 3,300,000(0.00716) + (200,000,000/1000)0.85
= 23,628 + 170,000
= $193,628 per month

3.31 Radio Frequency Identification (RFID) is technology used by drivers with “speed passes”
at toll booths and ranchers who track livestock from “farm-to-fork.” Walmart uses the
technology to track products within its stores. If RFID-tagged products will result in better
inventory control that will save your company $1.3 million per month beginning 3 months
from now, how much could you afford to spend to implement the technology at an interest
rate of 12% per year compounded monthly? You hope to recover the investment in 2½
years.

Copyright 2021 © McGraw-Hill Education. All rights reserved. No reproduction or distribution without the prior
written consent of McGraw-Hill Education.
10
P = 1,300,000(P/A,1%,28)(P/F,1%,2)
= 1,300,000(24.3164)(0.9803)
= $30,988,577

3.32 Assume that Austin Water purchases surface water from the Lower Colorado River
Authority at a cost of $120,000 per month in the months of February through September.
Instead of paying monthly, the utility makes a single payment of $800,000 at the end of the
year (i.e., end of December) for the water it used. The delayed payment essentially
represents a subsidy by the Authority to the water utility. At an interest rate of 3% per year
compounded monthly, what is the amount of the subsidy?

F = 120,000(F/A,0.25%,8)(F/P,0.25%,3)
= 120,000(8.0704)(1.0075)
= $975,711

Subsidy = 975,711 – 800,000


= $175,711

3.33 How much money must be invested each quarter if Adkef Technologies wants to
accumulate $825,000 in 5 years in a fund that grows at a rate of 16% per year compounded
continuously? Solve this problem using the factor formula and either the EXP or EFFECT
function on a spreadsheet, or as instucted.

Factor formula:
i/quarter = e0.04 – 1 = 4.08%

A = F(A/F,4.08%,20)
= 825,000{0.0408/[1 + 0.0408)20 – 1]}
= $27,476

Spreadsheet:
The functions = EXP(0.04) – 1 and = EFFECT(4%,10000) both display the
quarterly rate of 0.0408 (4.08% per quarter). Use the function
= PMT(4.08%,20,–825000) to determine an A of $27,476.

3.34 Corrosion problems and manufacturing defects in a Kinder-Morgan pipeline caused


longitudinal weld seam failures, resulting in a pressure reduction to 80% of the design
value. If the reduced pressure results in delivery of less product valued at $100,000 per
month, what will be the future value of the lost revenue after a 2-year period at an interest
rate of 15% per year compounded continuously?

Rate per month = 0.15/12 = 0.0125


i/month = e0.0125 – 1 = 0.0126 (1.26%)

Copyright 2021 © McGraw-Hill Education. All rights reserved. No reproduction or distribution without the prior
written consent of McGraw-Hill Education.
11
F = 100,000(F/A,1.26%,24)
= 100,000{[(1 + 0.0126)24 –1]/0.0126}
= 100,000(27.8213)
= $2,782,130

Note: Spreadsheet function is = FV(1.26%,24,–100000)

3.35 You have a very good salary and a smart investment manager. She tells you that in only 6
years you can accumulate $688,530 from quarterly deposits of $20,000 into an account that
has a historical return of 12% per year compounded quarterly. It seems like a “stiff”
commitment, but one worth trying as you prepare for an early retirement from your current
defense contractor engineering job. (a) Verify the 6-year time commitment if all other
estimates are correct. (b) What is the total amount that you will deposit? (c) You are a
cautious investor. Suppose the market does not perform as your investment manager
predicts and the return is only 50% of what she predicted. What is the future value of your
quarterly deposits?

(a) 688,530 = 20,000(F/A,3%,n)


(F/A,3%,n) = 34.4265
n = 24 quarters or 6 years

Another solution:
F = 20,000(F/A,3%,24)
= 20,000(34.4265)
= $688,530

(b) Amount invested = 20,000(24) = $480,000

(c) F = 20,000(F/A,1.5%,24)
= 20,000(28.6335)
= $572,670

3.36 McMillan Company manufactures electronic flow sensors that are designed as an
alternative to ball-and-tube rotometers. The company recently spent $3 million to increase
the capacity of an existing production line. If the extra revenue generated by the expansion
amounts to $200,000 per month, how long will it take to recover the investment at a
MARR of 12% per year compounded monthly? Solve using (a) tabulated factors and (b) a
spreadsheet.

(a) 3,000,000 = 200,000(P/A,1%,n)


(P/A,1%,n) = 15.000

From 1% table, n is between 16 and 17; therefore, n = 17 months

(b) Spreadsheet function = NPER(1%,200000,–3000000) displays an n of 16.3 months.

Copyright 2021 © McGraw-Hill Education. All rights reserved. No reproduction or distribution without the prior
written consent of McGraw-Hill Education.
12
3.37 Metalfab Pump and Filter expects the cost of steel bodies for 6-inch valves to increase by $2
every 3 months. If the cost for the first quarter is expected to be $80, what is the present worth
of the costs for a 3-year time period at a MARR of 12% per year compounded quarterly?

P = 80(P/A,3%,12) + 2(P/G,3%,12)
= 80(9.9540) + 2(51.2482)
= $898.82

3.38 Revenue from sales of hardened steel connectors was $50,000 in the first quarter, $51,000 in the
second, and amounts increasing by $1000 per quarter through year 4. What is the equivalent
uniform amount per quarter if the interest rate is 12% per year compounded quarterly?

A = 50,000 + 1000(A/G,3%,16)
= 50,000 + 1000(6.8742)
= $56,874

3.39 Atlas Moving and Storage wants to have enough money to purchase a new tractor trailer in
4 years at a cost of $290,000. The company plans to set aside $4000 in month 1 and
increase its set aside by a uniform amount each month. (a) Using factors, determine how
much the monthly increase must be, provided the funds earn 6% per year compounded
monthly. (b) (Spreadsheet exercise) Find the gradient using a spreadsheet and the GOAL
SEEK tool.

(a) 290,000(P/F,0.5%,48) = 4000(P/A,0.5%,48) + G(P/G,0.5%,48)


290,000(0.7871) = 4000(42.5803) + G(959.9188)
959.9188G = 57,938
G = $60.36 per month

(b) Enter the amounts of $4000 with some (guessed) arithmetic gradient, say, $100 per
month. Develop functions for the P and F values. (Note: Only 20 months shown.) Use
GOAL SEEK with the set cell (D6 here) to be $290,000 and the changing cell to be
determined (cell D1 here). After GOAL SEEK solution, the answer is $60.36, the same
as that determined using the P/G factor.

Copyright 2021 © McGraw-Hill Education. All rights reserved. No reproduction or distribution without the prior
written consent of McGraw-Hill Education.
13
3.40 Fieldsaver Technologies, a manufacturer of precision laboratory equipment, borrowed $2
million to renovate one of its testing labs. The loan was repaid in 2 years through quarterly
payments that increased by $50,000 each time. (a) At an interest rate of 12% per year
compounded quarterly, what was the size of the first payment? (b) (Spreadsheet exercise)
Find the first payment using a spreadsheet and the GOAL SEEK tool.

(a) 2,000,000 = A(P/A,3%,8) + 50,000(P/G,3%,8)


2,000,000 = A(7.0197) + 50,000(23.4806)
A = $117,665

(b) Enter the payments increasing by $50,000 with some (guessed) initial amount, say,
$100,000 per quarter. Develop the NPV function. Use GOAL SEEK with the set cell
(C11 here) to be P = $2,000,000 and the changing cell to be determined (cell D2 here).
After GOAL SEEK solution, the answer is $117,665, the same as that determined using
factors.

3.41 The cost to manufacture a firing system component used in a rapid deployment missile
defense system was $23,000 the first year; however, the company expects the cost to
increase by 2% each year. Calculate the present worth of this cost over a 5-year period at
an interest rate of 10% per year compounded semiannually.

PP = year; CP = 6 months; use effective annual i

i/year = (1 + 0.10/2)2 – 1
= 0.1025 (i.e., 10.25%), or from Table 3.3

Using Equation [2.10],

Pg = (23,000) [1 – (1.02/1.1025)5]
(0.1025 – 0.02)
= $89,823

Copyright 2021 © McGraw-Hill Education. All rights reserved. No reproduction or distribution without the prior
written consent of McGraw-Hill Education.
14
3.42 Equipment maintenance costs for manufacturing explosion-proof pressure switches are
projected to be $125,000 in year 1 and increase by 3% each year through year 5. What is
the equivalent annual worth of the maintenance costs at an interest rate of 10% per year
compounded semiannually? Solve using (a) the factor formula, and (b) a spreadsheet.

PP = year; CP = 6 months; use effective annual i

(a) i/year = (1 + 0.10/2)2 – 1 = 0.1025 (10.25% per year)

Pg = (125,000)[1 – (1.03/1.1025)5]/(0.1025 – 0.03)


= 125,000(3,9766)
= $497,080

A = 497,080(A/P,10.25%,5)
= 497,080(0.26548)
= $131,965

(b) Use the NPV and PMT functions at 10.25%.

3.43 Julia will be setting aside 10% of her monthly paycheck for automatic investment in a stock
mutual fund that pays dividends at 8% per year compounded semiannually. If the deposit is
$1200 per month beginning 1 month from now, how much will she have after 25 years?
Assume there is no interperiod compounding.

PP < CP. Move monthly deposits to end of 6-month compounding period; find F

F = 1200(6)(F/A,4%,50)
= 7200(152.6671)
= $1,099,203

3.44 Income from recycling paper and cardboard at the U.S. Army’s Fort Benning Maneuver
Center has averaged $3000 per month for 2½ years. What is the future worth of the income
(after the 2½ years) at an interest rate of 6% per year compounded quarterly? Assume there
is no interperiod compounding.

A = 3000(3) = $9000 at the end of each quarter. PP < CP.

Copyright 2021 © McGraw-Hill Education. All rights reserved. No reproduction or distribution without the prior
written consent of McGraw-Hill Education.
15
F = 9000(F/A,1.5%,10)
= 9000(10.7027)
= $96,324

3.45 Today, an engineer deposited $10,000 into an account that pays interest at 8% per year
compounded semiannually. If there is no interperiod compounding and withdrawals of
$1000 in months 2, 11, and 23 are already planned, what will be the future value at the end
of 3 years?

Move withdrawals to beginning of periods and then find F.


F = (10,000 – 1000)(F/P,4%,6) – 1000(F/P,4%,5) – 1000(F/P,4%,3)
= 9000(1.2653) – 1000(1.2167) – 1000(1.1249)
= $9046

3.46 Gentech Digital wants to have enough money in its capital investment account to replace a
robot that is involved in the manufacture of circuit boards. How much would the company
have to deposit every month in order to accumulate $280,000 in 5 years if the rate of return
on the account is 8% per year compounded semiannually? Assume no interperiod
compounding.

Calculate A per six months and then divide by 6 because of no interperiod interest.

A/6 months = 280,000(A/F,4%,10)


= 280,000(0.08329)
= $23,321.20

A/month = 23,321.20/6
= $3886.87

3.47 (a) How many monthly deposits of $750 would a company have to make to accumulate
$150,000 if the interest rate is 10% per year compounded semiannually? Assume no
interperiod compounding. (b) Is the answer the same if a spreadsheet is used instead of
factors?

(a) With no interperiod interest, semiannual deposit is 6(750) = $4500

150,000 = 4500(F/A,5%,n)
(A/F,5%,n) = 33.3333

From 5% factor table, n is very close to 20 semiannual periods

Number of monthly deposits = 20(6) = 120

(b) Spreadsheet: The function = NPER(5%,–4500,150000) displays an n of 20.1


semiannual periods. Now, monthly periods = 20.1(6) = 121 (rounded)

Copyright 2021 © McGraw-Hill Education. All rights reserved. No reproduction or distribution without the prior
written consent of McGraw-Hill Education.
16
3.48 Julie has a low credit rating, plus she was furloughed from her job 2 months ago. She has a
new job starting next week and expects a salary to start again in a couple of weeks. Since
she is a little short on money to pay her rent, she decided to borrow $100 from a loan
company, which will charge her only $10 interest if the $110 is paid no more than 1 week
after the loan is made. What is the (a) nominal annual, and (b) effective annual interest rate
that she will pay on this loan?

(a) Interest rate per week = (10/100)(100%) = 10%

r = (10%)(52) = 520% per year

(b) i = (1 + 5.20/52)52 – 1
= 141.04 (14,104% per year)

3.49 A wise mechanical engineering graduate began saving money for early retirement by
depositing $1500 per month into a fixed rate account that pays 6% per year compounded
semiannually. If she started saving 1 month after she started working, what is the expected
value of the account at the end of 20 years?

PP < CP. Move monthly deposits to end of 6-month compounding period; find F

F = 1500(6)(F/A,3%,40)
= 9000(75.4013)
= $678,612

3.50 Many college students have Visa credit cards that carry an interest rate of “Simple 24%
per year” (i.e., 2% per month). When the balance on such a card is $5000, the minimum
payment is $110.25.
a. What is the amount of interest in the first payment?
b. How long will it take in months to pay off the $5000 balance, if the cardholder continues
to make payments of $110.25 per month and adds no other charges to the card?

(a) Interest in payment = 5000(0.02) = $100

(b) 5000 = 110.25(P/A,2%,n)


(P/A,2%,n) = 45.3515

From 2% interest table, n ≈ 120 months or 10 years.

3.51 Treasury securities are issued and backed by the U.S. government and, therefore, are
considered to be the lowest-risk securities on the market. As an investor looking for
protection against inflation, you are considering the purchase of inflation-adjusted bonds
known as U.S. Treasury Inflation-Protected Securities (TIPS). With these securities, the
face value (which is paid at maturity) is regularly adjusted to account for inflation;
however, the semiannual interest payment (called the bond dividend) remains the same.

Copyright 2021 © McGraw-Hill Education. All rights reserved. No reproduction or distribution without the prior
written consent of McGraw-Hill Education.
17
You purchased a 10-year $10,000 TIPS bond with dividend of 4% per year payable
semiannually (i.e., $200 every 6 months). Assume there is no inflation adjustment for the
first 5 years, but in years 6 through 10, the bond face value increases by $1000 each year.
You use an expected investment return of 6% per year compounded semiannually.
(a) What is the total net amount of money that you will have received during the 10-year
life of the bond without any consideration of dividend reinvestment? (b) What will be the
equivalent future worth of all the income received with dividend reinvestment included?

(a) Total net = – 10,000 + 15,000 + 200(20) = $9000

(b) Future worth of interest income is calculated at 3% per 6-month period for 20 periods.

F = 200(F/A,3%,20)
= 200(26.8704)
= $5374.08

Face value of bond at maturity = $15,000

Total future worth = 5374.08 + 15,000 = $20,374.08

3.52 If you deposit $1000 per month into an investment account that pays interest at a rate of
6% per year compounded quarterly, how much will be in your account at the end of
5 years? Assume no interperiod compounding.

A per quarter = 3(1000) = $3000

F = 3000(F/A,1.5%,20)
= 3000(23.1237)
= $69,371

3.53 Two young couples (all about to turn 25 years old) are discussing how to enjoy life.
Couple X believes the best way is to sacrifice early in life and retire at a young age.
Couple Y believes that you can have fun early in life and still retire at a relatively young
age. Both couples plan to invest in the stock market, because they know that the average
return over long periods of time is 10% per year without inflation considerations.
Couple X plans to invest $2500 per month (from one of their two salaries) in stocks
for 15 years, and then stop investing, but let the account grow for the next 15 years, at
which time they will be 55 years old. Couple Y plans to invest no money for the first
15 years, but will begin investing $2500 per month thereafter.
a. How much money will couple X have when they turn 55, if they did start investing 1
month after age 25?
b. How much money will couple Y have when they turn 55, if they started investing 1
month after they turned 40 years old?
c. If couple Y continues to invest $2500 per month beyond the age of 55, at what age will
they be able to retire with the same amount of money as couple X?

Copyright 2021 © McGraw-Hill Education. All rights reserved. No reproduction or distribution without the prior
written consent of McGraw-Hill Education.
18
d. How many extra years did couple Y have to invest $2500 per month to accumulate the
same amount of money as couple X?

(a) i = 10%/12 = 0.833% per month

Amounts couple X will have after investing for 15 years (180 months) (age 40) and 30
years (age 55). The FV functions at 10%/12 display the results below.

F15 = 2500(F/A,0.833%,180) or = – FV(10%/12,180,2500)


= $1,036,176

F30 = 1,036,176(F/P,0.833%,180) or = – FV(10%/12,180,,1036176)


= $4,615,045

(b) Amount couple Y will have after investing for 15 years (age 55) is

F15 = 2500(F/A,0.833%,180) or = – FV(10%/12,180,2500)


= $1,036,176

(c) Time to accumulate same amount as couple X is found by solving for n or using the
NPER function.

4,615,045 = 2500(F/A,0.833%,n) or = NPER(10%/12,–2500,4615045)


n = 336.95 months
= 28 years

Age = 40 + 28 = 68

(d) Total for Y is 28; total for X is 15, since nothing was invested the second 15 years.

Extra years for couple Y = 28 – 15 = 13 years

3.54 A stock option is a device by which the buyer can buy or sell a stock at an agreed-upon
price within a stated period of time. The time can be as short as 1 week and as long as 3
years. There is an upfront cost to purchase the option.
When the stock for AT&T was selling for $32.50 per share, an option that would
allow you to purchase AT&T for $34 at any time within the next 3-month period could be
purchased for $270. If AT&T stock were to rise to $33 one month after you purchased the
option, you would likely be able to sell that option for $300. If there is a $7 commission
for buying the option and another $7 commission for selling it, what would be the nominal
rate of return per year on the option transaction?

Profit/month = 300 – 270 – 14


= $16

Amount invested = 270 + 14


= 284

Copyright 2021 © McGraw-Hill Education. All rights reserved. No reproduction or distribution without the prior
written consent of McGraw-Hill Education.
19
Rate of return/month = 16/284
= 5.63%

Nominal rate/year = 5.63(12)


= 67.6%

3.55 An interest rate of 12% per year compounded monthly is nearest to:
a. 12.08% per year
b. 12.28% per year
c. 12.48% per year
d. 12.68% per year

i/yr = (1 + 0.01)12 –1 = 0.1268 = 12.68%


Answer is (d)

3.56 An interest rate of 1.5% per month compounded continuously is closest to an effective
rate of:
a. 1.51% per quarter
b. 4.5% per quarter
c. 4.6% per quarter
d. 9% per 6 months

i/quarter = e0.045 –1 = 0.0460 (4.60%)


Answer is (c)

3.57 For an interest rate of 1% per quarter compounded continuously, the effective semiannual
interest rate is closest to:
a. 1.005%
b. 2.0%
c. 2.02%
d. 2.20%

i/semi-annual = e0.02 –1= 0.0202 (2.02%)


Answer is (c)

3.58 The multistate Powerball Lottery, worth $182 million, was won by a single individual who
had purchased five tickets at $1 each. The individual was given two choices:
Option 1: Receive 26 payments of $7 million each, with the first payment to be made now,
with the rest to be made at the end of each of the next 25 years.
Option 2: Receive a single lump sum payment now that would be equivalent to the 26
payments of $7 million each. If the state uses an interest rate of 4% per year compounded
annually, the amount of the lump-sum payment is closest to:
a. Less than $109 million
b. $109,355,000
c. $116,355,000
d. Over $120 million

Copyright 2021 © McGraw-Hill Education. All rights reserved. No reproduction or distribution without the prior
written consent of McGraw-Hill Education.
20
P = 7 + 7(P/A,4%,25)
= $116.3547 million
Answer is (c)

3.59 Encon Environmental Testing needs to purchase $40,000 worth of equipment 2 years from
now. At an interest rate of 12% per year compounded quarterly, the uniform quarterly
worth of the equipment is closest to:
a. $3958
b. $4189
c. $4341
d. $4500

A = 40,000(A/F,3%,8)
= 40,000(0.11246)
= $4498.40
Answer is (d)

3.60 Nippon Iron & Steel invested $950,000 in a new shearing unit. At an interest rate of 12%
per year compounded monthly, the monthly income required to recover the investment in 3
years is closest to:
a. $31,550
b. $35,880
c. $39,370
d. $43,550

A = 950,000(A/P,1%,36)
= 950,000(0.03321)
= $31,550
Answer is (a)

3.61 Royalties paid to holders of mineral rights tend to decrease with time as resources become
depleted. In one particular case, the rights holder received a royalty check of $18,000 six
months after the lease was signed. She continued to receive checks at 6-month intervals,
but the amount decreased by $2000 each time. At an interest rate of 10% per year
compounded semiannually, the equivalent uniform semiannual worth of the royalty
payments through the first 4 years is closest to:
a. $11,511
b. $15,352
c. $19,236
d. $24,489

AW = 18,000 – 2000 (A/G, 5%, 8)


= 18,000 – 2000 (3.2445)
= $11,511
Answer is (a)

Copyright 2021 © McGraw-Hill Education. All rights reserved. No reproduction or distribution without the prior
written consent of McGraw-Hill Education.
21
3.62 Attendance at the annual Northwest Livestock Show and Rodeo declined for the 5-year
period from 2016 through 2020. The attendance was 25,880 in 2016, 22,000 in 2017, and
13,500 in 2020, a 15% per year decrease. The average ticket price was $10 per person over
the 5-year period. The show management company president wants to know the equivalent
worth in 2022, the next show year, at its MARR of 16% per year compounded
semiannually. The equivalent worth in 2022 is closest to:
a. $1,322,123
b. $672,260
c. $785,425
d. $1,346,070

P2015 = 258,800{1–[(1–0.15)5/(1+0.08)5]}/(0.08+0.15)
= 258,800(3.03488)
= $785,426

F2022 = 785,426(F/P,8%,7)
= $1,346,063
Answer is (d)

3.63 You have made quarterly deposits for 3 years into a REIT (Real Estate Investment Trust)
fund that compounds interest at 1% per month. The value of n in the F∕A factor that will
determine F at the end of the 3-year period is:
a. 3
b. 12
c. 36
d. None of these

Answer is (b)

3.64 An engineer who is saving for retirement plans to deposit $500 every quarter into an
investment portfolio, starting one quarter from now. The portfolio should return 6% per
year compounded semiannually with no interperiod interest. The total at the end of 25 years
is closest to:
a. $119,400
b. $112,800
c. $108,700
d. $102,300

PP < CP with no interperiod compounding

F = 1000(F/A,3%,50)
= $112,796.90
Answer is (b)

Copyright 2021 © McGraw-Hill Education. All rights reserved. No reproduction or distribution without the prior
written consent of McGraw-Hill Education.
22
3.65 A company that makes flange mount, motorized rotary potentiometers expects to spend
$50,000 for a replacement machine 4 years from now. At an interest rate of 12% per year
compounded quarterly, the present worth of the machine’s cost is closest to:
a. $21,480
b. $28,820
c. $31,160
d. $35,930

P = 50,000(P/F, 3%, 16)


= 50,000(0.6232)
= $31,160
Answer is (c)

3.66 The cost of replacing part of a cell phone video-chip production line in 6 years is estimated
to be $500,000. At an interest rate of 14% per year compounded semiannually, the uniform
amount that must be deposited into a sinking fund every 6 months is closest to:
a. $21,335
b. $24,825
c. $27,950
d. $97,995

A = 500,000(A/F,7%,12)
= 500,000(0.05590)
= $27,950 per 6 months
Answer is (c)

3.67 Michael is short of money to make his car loan payment this month. He borrows $300 from
his brother-in-law and promises to repay $315 when he is paid by his employer at the end
of 1 week. He did repay the $315 in exactly 1 week. The effective annual interest rate
Michael paid is closest to:
a. 80%
b. 260%
c. 1160%
d. 8750%

Interest rate per week = (15/300)(100%) = 5%

r = (5%)(52) = 260% per year

i = (1 + 2.60/52)52 – 1
= 11.64 (1164% per year)
Answer is (c)

Copyright 2021 © McGraw-Hill Education. All rights reserved. No reproduction or distribution without the prior
written consent of McGraw-Hill Education.
23
Solutions to end-of-chapter problems
Basics of Engineering Economy, 3rd edition
Leland Blank and Anthony Tarquin

Chapter 4
Present Worth Analysis

4.1 What is the main difference between mutually exclusive and independent
alternatives?

When evaluating mutually exclusive alternatives, only one can be accepted. On the
other hand, when evaluating independent alternatives, more than one can be
accepted, subject to budget limitations.

4.2 What is the difference in cash flows between revenue and cost alternatives?

Revenue alternatives have revenue, cost, and possibly savings cash flow estimates.
Cost alternatives assume the revenue is the same for all alternatives; therefore, they
have only cost cash flows. A cost alternative can have a savings cash flow estimate.

4.3 What is meant by do-nothing alternative?

The do-nothing alternative means that the status-quo should be maintained, that is, if
none of the alternatives under consideration are economically attractive, all of them
should be rejected.

4.4 State two conditions under which the do-nothing alternative is not an option.

The do-nothing alternative is not an option (1) when it is absolutely required that
one of the defined alternatives be selected (e.g., legal purposes), and (2) when each
alternative has only cost cash flow estimates.

4.5 You are conducting an evaluation of three mutually exclusive alternatives for a long-
term manufacturing process. What time period is required for conducting a present
worth analysis if the estimated lives are 3, 4, and 6 years, respectively?

The analysis must be conducted for the least common multiple of years, which is 12.

4.6 Two methods can be used for producing expansion anchors. Method A costs $80,000
initially and will have a $15,000 salvage value after 3 years. The operating cost with
this method will be $30,000 per year. Method B will have a first cost of $120,000, an
operating cost of $8000 per year, and a $40,000 salvage value after its 3-year life. At

Copyright 2021 © McGraw-Hill Education. All rights reserved. No reproduction or distribution without the
prior written consent of McGraw-Hill Education.
1
the MARR of 12% per year, which method should be used on the basis of a present
worth analysis?

PWA = –80,000 – 30,000(P/A,12%,3) + 15,000(P/F,12%,3)


= –80,000 – 30,000(2.4018) + 15,000(0.7118)
= $–141,377

PWB = –120,000 – 8,000(P/A,12%,3) + 40,000(P/F,12%,3)


= –120,000 – 8,000(2.4018) + 40,000(0.7118)
= $–110,742

Select Method B

4.7 The manager of a canned-food processing plant is trying to decide between two
different labeling machines. The photocell unit will have a first cost of $26,000, an
AOC of $4800, and a service life of 4 years. The capacitive sensor unit will cost
$32,000 to buy and install and an estimated AOC of $2100 during its 4-year life. At an
interest rate of 9% per year, which is economically better on the basis of PW values?

PWPhoto = –26,000 – 4,800(P/A,9%,4)


= –26,000 – 4,800(3.2397)
= $–41,551

PWCap = –32,000 – 2100(P/A, 9%, 4)


= –32,000 – 2100(3.2397)
= $–38,803

Select Capacitive

4.8 A software package created by Navarro & Associates can be used for analyzing and
designing three-sided guyed towers and three- and four-sided self-supporting towers.
A single-user license will cost $4000 per year. A site license has a one-time cost of
$15,000. A structural engineering consulting company is trying to decide between
two 5-year strategies: first, to buy one single-user license now and one each year for
the next four years, or second, to buy a site license now. Determine which strategy
should be adopted at a MARR of 12% per year for a 5-year planning period using
the present worth method.

PWsingle = –4000 – 4000(P/A,12%,4)


= –4000 – 4000(3.0373)
= $–16,149

PWsite = $–15,000

Buy the site license

Copyright 2021 © McGraw-Hill Education. All rights reserved. No reproduction or distribution without the
prior written consent of McGraw-Hill Education.
2
4.9 You and your sister just inherited from your grandparents a commercial rental
building on the northwest business corridor of a major city. Fortunately, it is fully
leased, but it badly needs a new roof. Two types of material can be used on the 1500
square meters of roof surface. Asphalt shingles will cost $14 per square meter
installed and are guaranteed for 10 years. Fiberglass shingles will cost $17 per
square meter installed and are guaranteed for 20 years. If the fiberglass shingles are
selected, the owner will be able to sell the building for an estimated $4500 more than
if the asphalt shingles are used. Which shingles should be installed if you use a
MARR of 16% per year and you two have agreed to sell the building in 10 years? (a)
Solve using tabulated factors. (b) Write the spreadsheet relations and PV function
that will find the PW values.

(a) PWasphalt = – (1,500)(14)


= $–21,000

PWfibcrglass = – (1,500)(17) + 4,500(P/F,16%,10)


= –25,500 + 4,500(0.2267)
= $–24,480
Use asphalt

(b) For asphalt relation: = –1500*14


For fiberglass, add relation and PV function: = – 1500*17 – PV(16%,10,4500)

4.10 The CFO of Marta Aaraña Cement Industries knew 10 years ago that many of the
diesel-fueled systems in its quarries would need replacement at an estimated cost
of $18 million now. There were two alternative ways to prepare financially for the
replacement cost. Alternative A was to build a fund with an initial amount of $1
million starting 1 year later, i.e., 9 years ago, and the commitment of 10% increases
through the 10th year. The fund was estimated to earn at 5.25% per year. Alterna-
tive B was to do nothing then and borrow the estimated $18 million at an interest
rate of 4% per year for 5 years when the replacements were purchased, i.e., now.
The CFO chose alternative A, build the fund.
The replacements and the invoice arrived today with a price tag of $18
million. Was selecting alternative A 10 years ago the financially better decision
based on the equivalent worth of the two plans today?

First, determine if today’s value of deposits over 10 years equals the $18 million
target. Use Pg equation to find P with g = 0.1 and i = 0.0525, then find F. Monetary
terms are in $ million.

PWA = 1{[1– (1.1/1.0525)10]/–0.0475}


= 1{–0.5549/–0.0475}
= 11.68236 ($11,682,360)

Copyright 2021 © McGraw-Hill Education. All rights reserved. No reproduction or distribution without the
prior written consent of McGraw-Hill Education.
3
FWA = 11.68236(F/P,5.25%,10)
= 11,68236(1.66810)
= $19.4873 ($19,487,300)

Alternative B: PWB = $18,000,000

Today’s worth is $1,487,300 over the target of $18 million, plus the loan payments
for 5 years have been avoided. The decision for A was financially correct, given the
facts and no inflation considered.

4.11 The Bureau of Indian Affairs provides various services to American Indians and
Alaska Natives. The Director of Indian Health Services is working with chief
physicians at some of the 230 clinics nationwide to select the better of two medical
X-ray system alternatives to be located at secondary-level clinics. At 5% per year,
select the more economical system. Solve using (a) tabulated factors, and (b)
single-cell spreadsheet functions, as instructed.

X-ray System Del Medical Siemens


First cost, $ −250,000 −224,000
Annual operating cost, $/year −231,000 −235,000
Overhaul in year 3, $ — −26,000
Overhaul in year 4, $ −140,000 —
Salvage value, $ 50,000 10,000
Expected life, years 6 6

(a) Monetary units are in $1000. Calculate PW values to select Siemens.

PWDel = –250 – 231(P/A,5%,6) –140(P/F,5%,4) + 50(P/F,5%,6)


= –250 –231(5.0757) – 140(0.8227) + 50(0.7462)
= $–1500.355 ($–1,500,355)

PWSie = –224 – 235(P/A,5%,6) – 26(P/F,5%,3) + 10(P/F,5%,6)


= –224 –235(5.0757) – 26(0.8638) + 10(0.7462)
= $–1431.786 ($–1,431,786)

(b) By spreadsheet, enter the following into single cells to display the PW values.

PWDel : = – PV(5%,6,–231000,50000) –250000 – PV(5%,4,–140000)

PWSie : = – PV(5%,6,–235000,10000) –224000 – PV(5%,3,–26000)

4.12 The Briggs and Stratton Commercial Division designs and manufactures small
engines for golf turf maintenance equipment. A robotics-based testing system will
ensure that their new signature guarantee program entitled “Always Insta-Start”
does indeed work for every engine produced. Compare the two systems at a MARR

Copyright 2021 © McGraw-Hill Education. All rights reserved. No reproduction or distribution without the
prior written consent of McGraw-Hill Education.
4
of 10% per year. Solve using (a) tabulated factors, and (b) single-cell spreadsheet
functions, as instructed.

System Pull Push


First cost, $ −1,500,000 −2,250,000
Annual M&O cost, $/year −700,000 −600,000
Rebuild cost in year 3, $ 0 −500,000
Salvage value, $ 100,000 50,000
Estimated life, years 8 8

(a) Monetary units are in $1000. Calculate PW values to select the pull system.

PWpull = –1500 – 700(P/A,10%,8) + 100(P/F,10%,8)


= –1500 – 700(5.3349) + 100(0.4665)
= $–5187.780 ($–5,187,780)

PWpush = –2250 – 600(P/A,10%,8) + 50(P/F,10%,8) – 500(P/F,10%,3)


= –2250 – 600(5.3349) + 50(0.4665) – 500(0.7513)
= $–5803.265 ($–5,803,265)

(b) Enter these single-cell functions to display PW values.

PWpull : = – PV(10%,8,–700000,100000)–1500000
PWpush : = – PV(10%,8,–600000,50000)–2250000–PV(10%,3,–500000)

4.13 PEMEX, Mexico’s petroleum corporation, has an estimated budget for oil and gas
exploration that includes equipment for three offshore platforms as shown. Use PW
analysis to select the best alternative at a MARR of 15% per year. Select platform
X, Y, or Z using (a) tabulated factors, and (b) single-cell spreadsheet functions, as
instructed.

Platform X Y Z
First cost, $ million −300 −450 −510
M&O, $ million per year −320 −290 −230
Salvage value, $ million 75 50 90
Estimated life, years 20 20 20

(a) Monetary units are in $ million. Calculate PW values to select platform Z.

PWX = –300 – 320(P/A,15%,20) + 75(P/F,15%,20)


= –300 – 320(6.2593) + 75(0.0611)
= $–2298.3935 ($–2.298 billion)
PWY = –450 – 290(P/A,15%,20) + 50(P/F,15%,20)
= –450 – 290(6.2593) + 50(0.0611)
= $–2262.142 ($–2.262 billion)

Copyright 2021 © McGraw-Hill Education. All rights reserved. No reproduction or distribution without the
prior written consent of McGraw-Hill Education.
5
PWZ = –510 – 230(P/A,15%,20) + 90(P/F,15%,20)
= –510 – 230(6.2593) + 90(0.0611)
= $–1944.14 ($–1.944 billion)

(b) Enter the PV functions to display PW values in $ million units.

PWX: = – PV(15%,20,–320,75) –300


PWY: = – PV(15%,20,–290,50) –450
PWZ: = – PV(15%,20,–230,90) –510

4.14 A company that manufactures and markets integrated systems for analysis of
genetic variation and function is considering the mutually exclusive projects
shown, all of which can be considered to be viable for only 10 years. If the
company’s MARR is 15% per year, determine which should be selected on the
basis of a present worth analysis. Financial values are in $1000 units.

Project W X Y Z
First cost, $ −1400 −2000 −4200 −6300
Annual net income, $/year 320 510 1000 1300
Salvage value, $ 9 5 7 7

Monetary units are $1000. Calculate PW values; select largest PW; Project Y.
      
 PWW = –1400 + 320(P/A,15%,10) + 9(P/F,15%,10)
= –1400 + 320(5.0188) + 9(0.2472)
= $208.2408

PWX = –2000 + 510(P/A,15%,10) + 5(P/F,15%,10)


= –2000 + 510(5.0188) + 5(0.2472)
= $560.824

PWY = –4200 + 1000(P/A,15%,10) + 7(P/F,15%,10)


= –4200 + 1000(5.0188) + 7(0.2472)
= $820.5304

PWZ = –6300 + 1300(P/A,15%,10) + 7(P/F,15%,10)


= –6300 + 1300(5.0188) + 7(0.2472)
= $226.1704

Select Project Y

4.15 A new family of small robots are available that have a compact footprint and a
large work envelope that provides ease of use and short cycle times. Two are under
consideration as part of a process to manufacture blowdown control valves for
cooling towers. Process X costs $80,000 initially and will have a $15,000 salvage
value after 3 years. The operating cost with this robot will be $30,000 in year 1,

Copyright 2021 © McGraw-Hill Education. All rights reserved. No reproduction or distribution without the
prior written consent of McGraw-Hill Education.
6
increasing by $4000 each year. Process Y will have a first cost of $120,000, an
operating cost of $8000 in year 1, increasing by $6500 each year, and a $40,000
salvage value after its 3-year life. At an interest rate of 12% per year, which
process should be used on the basis of a present worth analysis?

PWX = –80,000 – [30,000(P/A,12%,3) + 4000(P/G,12%,3)] + 15,000(P/F,12%,3)


= –80,000 – [30,000(2.4018) + 4000(2.2208)] + 15,000(0.7118)
= $–150,260

PWY = –120,000 – [8,000(P/A,12%,3) + 6500(P/G,12%,3)] + 40,000(P/F,12%,3)


= –120,000 – [8,000(2.4018) + 6500(2.2208)] + 40,000(0.7118)
= $–125,178

Select Process Y

4.16 A public water utility is trying to decide between two different sizes of pipe for a
new water main. A 250-mm line will have an initial cost of $155,000, whereas a
300-mm line will cost $210,000. Since there is more head loss through the 250-mm
pipe, the pumping cost is expected to be $3000 more per year than for the 300-mm
line, and the extra cost is expected to increase by $15 each year. If the lines are
expected to last for 30 years, which size should be selected on the basis of a present
worth analysis using a MARR of 10% per year?

PW250 = –155,000 – [3000(P/A,10%,30) + 15(P/G,10%30)]


= –155,000 – [3000(9.4269) + 15(77.0766)]
= $–184,437

PW300 = $–210,000

Install the 250 mm pipe

4.17 A metallurgical engineer is considering two materials as the additive to a


manufacturing process as shown. Which material should be selected on the basis of
a PW comparison at an interest rate of 12% per year?

Material Nickel-Cadmium Aluminum-Beryllium


First cost, $ −15,000 −35,000
M&O costs, $/year −9,000 −7,000
Increase in cost each year, $/year 730 325
Salvage value, $ 2,000 10,000
Life, years 5 5

(a) PWNC = –15,000 – [9000(P/A,12%,5) + 730(P/G,12%,5)] + 2000(P/F,12%,5)


= –15,000 – [9000(3.6048) + 730(6.3970)] + 2000(0.5674)
= $–50,978

Copyright 2021 © McGraw-Hill Education. All rights reserved. No reproduction or distribution without the
prior written consent of McGraw-Hill Education.
7
PWAB = –35,000 – [7000(P/A,12%,5) + 325(P/G,12%,5)] + 10,000(P/F,12%,5)
= –35,000 – [7000(3.6048) + 325(6.3970)] + 10,000(0.5674)
= $–56,639

Select Nickel-Cadmium (NC)

(b) Spreadsheet solution; select NC alternative

4.18 The TechEdge Corporation offers two forms of 4-year service contracts on its
closed-loop water purification system used in the manufacture of semiconductor
packages for microwave and high-speed digital devices. The Professional Plan has
an initial fee of $52,000 with annual fees starting at $1000 in contract year 1 and
increasing by $500 each year. Alternatively, the Executive Plan costs $62,000 up
front with annual fees starting at $5000 in contract year 1 and decreasing by $500
each year. The initial charge is considered a setup cost for which there is no
salvage value expected. Evaluate the plans at a MARR of 9% per year. Solve using
(a) factors, and (b) a spreadsheet.

(a) PWProf = –52,000 – 1000(P/A,9%,4) – 500(P/G,9%,4)


= –52,000 – 1000(3.2397) – 500(4.5113)
= $–57,495

PWExec = –62,000 – 5000(P/A,9%,4) + 500(P/G,9%,4)


= –62,000 – 5000(3.2397) + 500(4.5113)
= $–75,943

Select the Professional Plan

Copyright 2021 © McGraw-Hill Education. All rights reserved. No reproduction or distribution without the
prior written consent of McGraw-Hill Education.
8
(b) A spreadsheet solution follows; select the Professional plan.

4.19 In order to retain high-performing engineers, a large semiconductor company


provides corporate stock as part of the compensation package. In one particular
year, the company offered 1000 shares of either class A or class B stock. The class
A stock was selling for $30 per share at the time, and stock market analysts
predicted that it would increase at a rate of 6% per year for the next 5 years. Class
B stock was selling for $20 per share, but its price was expected to increase by
12% per year. At a return rate of 8% per year, which stock should the engineers
select on the basis of a present worth analysis and a 5-year planning horizon?

Find Pg for each stock and select higher value

PgA = 30,000{1 – [(1 + 0.06)/(1 + 0.08)]5}/(0.08 – 0.06)


= $133,839

PgB = 20,000{1 – [(1 + 0.12)/(1 + 0.08)]5}/(0.08 – 0.12)


= $99,710

Select class A stock

4.20 An investment broker that Ava trusts recommended that she purchase a $50,000,
15-year municipal bond that generates a dividend of 4% per year payable quarterly.
She will pay a discounted amount of $45,000 now for the bond. In general, Ava
hopes to make 8% per year compounded quarterly on her investments. Using the
PW value, determine if this is a financially advantageous investment for her. Solve
with (a) factors, and (b) one single-cell spreadsheet function.

I = (50,000)(0.04)/4
= $500 every 3 months

(a) PW = 500(P/A,2%,60) + 50,000(P/F,2%,60) – 45,000


= 500(34.7609) + 50,000(0.3048) – 45,000
= $–12,380

(b) Function is = – PV(2%,60,500,50000) – 45000

Conclusion: Not a financially sound investment since PW < 0 at 2% per quarter.

Copyright 2021 © McGraw-Hill Education. All rights reserved. No reproduction or distribution without the
prior written consent of McGraw-Hill Education.
9
4.21 Navistar Electric issued 1000 debenture bonds 3 years ago with a face value of
$5000 each and a bond interest rate of 8% per year payable semiannually. The
bonds have a maturity date of 20 years from the date they were issued. If the
interest rate in the marketplace is 10% per year compounded semiannually,
determine the present worth today of one bond.

There are 17 years or 34 semiannual periods remaining in the life of the bond.

I = 5000(0.08)/2
= $200 every 6 months

PW = 200(P/A,5%,34) + 5000(P/F,5%,34)
= 200(16.1929) + 5000(0.1904)
= $4190.58

4.22 An engineer planning for his retirement thinks that the rates of return in the
marketplace will decrease before he retires. Therefore, he plans to invest in
corporate bonds. He plans to buy a $50,000 bond that has a coupon rate of 12% per
year payable quarterly with a maturity date 20 years from now.
a. How much should he be able to sell the bond for in 5 years if the market rate of
return is 8% per year compounded quarterly?
b. If he invests all the dividends he receives at a rate of return of 12% per year
compounded quarterly, what is the total amount he will have immediately after
he sells the bond 5 years from now?

(a) I = (50,000)(0.12)/4
= $1500 per quarter

Five years from now there will be 15(4) = 60 payments left. Find PW5.

PW5 = 1500(P/A,2%,60) + 50,000(P/F,2%,60)


= 1500(34.7609) + 50,000(0.3048)
= $67,381

(b) Total = 1500(F/A,3%,20) + 67,381


= 1500(26.8704) + 67,381
= $107,687

4.23 Jamal bought a 5%, $1000, 20-year bond for $925. He received semiannual interest
for 8 years, then sold it immediately after the 16th interest payment for $800. Did
Jamal make the return of 5% per year compounded semiannually that he wanted?
Solve using (a) factors, and (b) a spreadsheet.

(a) Semiannual bond payment is 1000(0.05)/2 = $25 per 6 months. Semiannual


interest rate is 5%/2 = 2.5%.

Copyright 2021 © McGraw-Hill Education. All rights reserved. No reproduction or distribution without the
prior written consent of McGraw-Hill Education.
10
PW = –925 + 25(P/A,2.5%,16) + 800(P/F,2.5%,16)
= –925 + 25(13.0550) + 800(0.6736)
= $–59.74

No, the bond investment did not make the target rate since PW < 0.

(b) Spreadsheet solution follows. Also, the single-cell function


= – 925 – PV(2.5%,16,25,800) will display the same result.

4.24 An investor believes that future stock market returns are going to decline.
Therefore, he decided to invest in some fixed-income securities. He paid $19,000
for a corporate bond with a face value of $20,000. The bond has an interest rate of
10% per year payable yearly. If the investor plans to sell the bond immediately
after receiving the fourth dividend payment, what is the minimum he will have to
receive in order to make a return of 14% per year? Solve using (a) tabulated
factors, and (b) the GOAL SEEK tool on a spreadsheet.

(a) Let R = amount received

I = 20,000(0.10) = $2000 per year

19,000 = 2000(P/A,14%,4) + R(P/F,14%,4)


R(0.5921) = 19,000 – 2000(2.9137)
R = $22,248

The investor would have to sell the bond for $22,247 to make 14% per year.

(b) Final GOAL SEEK template is shown with B7 the target cell for PW = 0 and
C2 the changing cell that displays $22,248.

Copyright 2021 © McGraw-Hill Education. All rights reserved. No reproduction or distribution without the
prior written consent of McGraw-Hill Education.
11
4.25 Two processes can be used for producing a polymer that reduces friction loss in
engines. Process T will have a first cost of $700,000, an operating cost of $60,000
per year, and a salvage value of $80,000 after its 2-year life. Process W will have a
first cost of $1,350,000, an operating cost of $25,000 per year, and a $120,000
salvage value after its 4-year life. Process W will also require updating at the end
of year 2 at a cost of $90,000. Which process should be selected on the basis of a
present worth analysis at a MARR of 12% per year?

PWT = –700,000 – 60,000(P/A,12%,4) – 620,000(P/F,12%,2) + 80,000(P/F,12%,4)


= –700,000 – 60,000(3.0373) – 620,000(0.7972) + 80,000(0.6355)
= $–1,325,622

PWW = –1,350,000 – 25,000(P/A,12%,4) – 90,000(P/F,12%,2)


+ 120,000(P/F,12%,4)
= –1,350,000 – 25,000(3.0373) – 90,000(0.7972) + 120,000(0.6355)
= $–1,421,421

Select process T

4.26 Virgin Galactic is considering two materials for certain parts in a reusable space
vehicle: carbon fiber reinforced plastic (CFRP) and fiber reinforced ceramic
(FRC). The costs are shown below. Which should be selected on the basis of a
present worth comparison if MARR = 10% per year? (a) Solve using tabulated
factors. (b) (Spreadsheet exercises) Solve using single-cell functions. (c) Using
your spreadsheet functions, will the selection change if the estimated life of the
CFRP material is 4, rather than 2, years?

Material CFRP FRC


First cost, $ −205,000 −235,000
Maintenance cost, $/year −29,000 −27,000
Salvage value, $ 2,000 20,000
Life, years 2 4

Copyright 2021 © McGraw-Hill Education. All rights reserved. No reproduction or distribution without the
prior written consent of McGraw-Hill Education.
12
(a) PWCFRP = –205,000 – 29,000(P/A,10%,4) – 203,000(P/F,10%,2)
+ 2000(P/F,10%,4)
= –205,000 – 29,000(3.1699) – 203,000(0.8264) + 2000(0.6830)
= $–463,320

PWFRC = –235,000 – 27,000(P/A,10%,4) + 20,000(P/F,10%,4)


= –235,000 – 27,000(3.1699) + 20,000(0.6830)
= $–306,927

Select material FRC

(b) PWCFRP function: = – 205000 – PV(10%,4,–29000,2000) + PV(10%,2,203000)

PWFRC function: = – 235000 – PV(10%,4,–27000,20000)

(c) PWCFRP function: = – 205000 – PV(10%,4,–29000,2000), which


displays a PWCFRP of $–295,560.

Select CFRP. The selection does change.

4.27 An engineer is considering two different liners for an evaporation pond that will
receive salty concentrate from a brackish water desalting plant. A plastic liner will
cost $0.90 per square foot initially and require replacement in 15 years when
precipitated solids will have to be removed from the pond using heavy equipment.
This removal will cost $500,000. A rubberized elastomeric liner is tougher and,
therefore, is expected to last 30 years, but it will cost $2.20 per square foot. If the
size of the pond is 110 acres (1 acre = 43,560 square feet), which liner is more cost
effective on the basis of a present worth comparison at a MARR of 8% per year?

Compare PW of costs over 30 years.

PWPlastic = – (0.90)(110)(43,560) – [(0.90)(110)(43,560) + 500,000](P/F,8%,15)


= – 4,312,440 – [4,312,440 + 500,000](0.3152)
= $–5,829,321

PWRubberized = – (2.20)(110)(43,560)
= $–10,541,520

Select plastic liner

4.28 Machines that have the following costs are under consideration for a robotized
welding process. Use an interest rate of 10% per year and PW analysis to
determine which machine should be selected. Show (a) factor, and (b) spreadsheet
solutions, as instructed. (c) Use your spreadsheet to determine if the choice is

Copyright 2021 © McGraw-Hill Education. All rights reserved. No reproduction or distribution without the
prior written consent of McGraw-Hill Education.
13
different when management restricts all evaluations to a 3-year study period.
Assume the market value of Y remains at $95,000.

Machine X Machine Y
First cost, $ −250,000 −430,000
AOC, $ per year −60,000 −40,000
Salvage value, $ 70,000 95,000
Life, years 3 6

(a) PWX = –250,000 – 60,000(P/A,10%,6) – 180,000(P/F,10%,3) +


70,000(P/F,10%,6)
= –250,000 – 60,000(4.3553) – 180,000(0.7513) + 70,000(0.5645)
= $–607,037

PWY = –430,000 – 40,000(P/A,10%,6) + 95,000(P/F,10%,6)


= –430,000 – 40,000(4.3553) + 95,000(0.5645)
= $–550,585

Select machine Y

(b) Spreadsheet solution to select machine Y.

(c) PWX function: = – PV(10%,3,–60000,70000) – 250000, which


displays $–346,619
PWY function: = – PV(10%,3,–40000,95000) – 430000, which
displays $–458,099

Select machine X; the decision does change.

4.29 A sports mortgage is the brainchild of Stadium Capital Financing Group, a


company headquartered in Chicago, Illinois. It is an innovative way to finance
cash-strapped sports programs by allowing fans to sign up to pay a “mortgage”
over a certain number of years for the right to buy good seats at football games for
several decades with season ticket prices locked in. The locked-in price period is
50 years in California. Assume you and your brother went to UCLA.

Copyright 2021 © McGraw-Hill Education. All rights reserved. No reproduction or distribution without the
prior written consent of McGraw-Hill Education.
14
Your brother, Harold, purchases a $50,000 mortgage and pays for it now to
get season tickets for $290 each for 50 years, while you, being a three-time
alumnus of the same university, are able to buy season tickets at $390 in year 1,
with prices increasing by $20 per year for 50 years. (a) Which of you made the
better deal at an interest rate of 8% per year? (b) What should Harold have been
willing to pay UCLA up front for the mortgage to make the two plans exactly
equivalent economically? (Assume Harold has no reason to give extra money to
UCLA at this point and that the seats are the same level and next to each other.)

(a) PWHarold = –50,000 – 290(P/A,8%,50)


= –50,000 – 290(12.2335)
= $–53,548

PWYou = –390(P/A,8%,50) – 20(P/G,8%,50)


= –390(12.2335) – 20(139.5928)
= $–7563

You made a far better choice, economically.

(b) Let MX = ‘mortgage’ cost for Harold for equivalence of plans

–7,563 = –MX – 290(P/A,8%,50)


Mx = 7563 – 290(12.2335)
= $4015

Harold should pay only $4015, not $50,000, economically speaking.

4.30 An electric switch manufacturing company has to choose one of three different
assembly methods. Method A will have a first cost of $40,000, an annual operating
cost of $9000, and a service life of 2 years. Method B will cost $80,000 to buy and
will have an annual operating cost of $6000 over its 4-year service life. Method C
will cost $130,000 initially with an annual operating cost of $4000 over its 8-year
life. Methods A and B will have no salvage value, but Method C will have some
equipment worth an estimated $12,000. Which method should be selected using
present worth analysis at a MARR of 10% per year?

PWA = –40,000[1+ (P/F,10%,2) + (P/F,10%,4) + (P/F,10%,6)] – 9000(P/A,10%,8)


= –40,000 [1 + 0.8264 + 0.6830 + 0.5645] – 9000(5.3349)
= $–170,970

PWB = –80,000[1 + (P/F,10%,4)] – 6000(P/A,10%,8)


= –80,000[1 + 0.6830] – 6000(5.3349)
= $–166,649

Copyright 2021 © McGraw-Hill Education. All rights reserved. No reproduction or distribution without the
prior written consent of McGraw-Hill Education.
15
PWC = –130,000 – 4000(P/A,10%,8) + 12,000(P/F,10%,8)
= –130,000 – 4000(5.3349) + 12,000(0.4665)
= $–145,742

Select method C

4.31 Compare the following machines on the basis of their present worth values. Use i =
12% per year. (a) Make the computations using factors. (b) (Spreadsheet exercise)
Develop the single-cell functions that display the PW values for the New and Used
alternatives.

Machine New Used


First cost, $ −44,000 −23,000
Annual operating cost, $ −7,000 −9,000
Annual repair cost, $ −210 −350
Overhaul after 10 years, $ −1,900 —
Salvage value, $ 4,000 3,000
Life, years 14 7

(a) PWnew = –44,000 – 7,210(P/A,12%,14) – 1900(P/F,12%,10) + 4,000(P/F,12%,14)


= –44,000 – 7,210(6.6282) – 1900 (0.3220) + 4,000(0.2046)
= $–91,583

PWused = –23,000 – 9350(P/A,12%,14) – 23,000(P/F,12%,7) + 3000(P/F,12%,7)


+ 3000(P/F,12%,14)
= –23,000 – 9350(6.6282) – 23,000(0.4523) + 3000(0.4523)
+ 3000(0.2046)
= $–93,406

Buy the new machine

(b) New machine: = – 44000 – PV(12%,14,–7210,4000) – PV(12%,10,–1900)


Used machine: = – 23000 – PV(12%,14,–9350,3000) – PV(12%,7,–20000)

4.32 Carl and Susan Jacks have invested their savings for some years earning at a rate of
6% per year to purchase, for cash, a mid-sized SUV for their family. After much
searching online and several showroom visits, two options are the finalists. The
estimates below include all first cost elements (purchase, extended warranty, taxes,
license, etc.) and AOC (insurance, fuel, regular maintenance, etc.). Perform a
spreadsheet-based FW evaluation at 6% per year on two time bases and select the
more economical vehicle. Carl wants to plan using a 10-year horizon and Susan
believes they will get tired of a SUV after 5 years. She wants a shorter planning
horizon.
a. Ten-year LCM with the repurchase of another Cadillac in year 5 for $65,000,
which is $10,000 more than the first purchase.

Copyright 2021 © McGraw-Hill Education. All rights reserved. No reproduction or distribution without the
prior written consent of McGraw-Hill Education.
16
b. Five-year study period (SP) assuming the resale value is 20% of the first cost for
both SUVs.
c. Is the selection different between the LCM- and SP-based analyses?

SUV Toyota Lexus GM Cadillac


First cost, $ −65,000 −55,000
AOC, year 1, $ −8,500 −9,750
Increase in AOC, %/year 2 3
Salvage value, end of life, $ 10,000 11,000
Replacement in year 5, $ — −65,000
Life, years 10 5

Spreadsheet follows with analysis for 10-year and 5-year evaluations.

(a) LCM of 10 years. Select Toyota Lexus since FW of costs is smaller.

(b) SP of 5 years. Select GM Cadillac since FW of costs is smaller.

(c) Selections are different, because there is a lower FW of costs over 5 years for
GM Cadillac due, in part, to no re-purchase at a higher price in year 5.

4.33 Akash Uni-Safe in Chennai, India, makes Terminator fire extinguishers. The
company needs replacement equipment to form the neck at the top of each
extinguisher during production. Select between two metal-constricting machines.
Use the corporate MARR of 15% per year with (a) present worth analysis using
tabulated factors, (b) future worth analysis using tabulated factors, and (c) a
spreadsheet for PW and FW as requested above.

Copyright 2021 © McGraw-Hill Education. All rights reserved. No reproduction or distribution without the
prior written consent of McGraw-Hill Education.
17
Machine D E
First cost, $ −62,000 −77,000
AOC, $ per year −15,000 −21,000
Salvage value, $ 8,000 10,000
Life, years 4 6

(a) PW analysis requires an LCM of 12 years to select machine D.

PWD = –62,000[1 + (P/F,15%,4)+(P/F,15%,8)] – 15,000(P/A,15%,12)


+ 8,000[(P/F,15%,4)+(P/F,15%,8) + (P/F,15%,12)]
= –62,000[1 + 0.5718 + 0.3269] – 15,000 (5.4206)
+ 8,000[0.5718 + 0.3269 + 0.1869]
= $–190,344

PWE = –77,000[1 + (P/F,15%,6)] – 21,000(P/A,15%,12)


+ 10,000[(P/F,15%,6) + (P/F,15%,12)]
= –77,000[1 + 0.4323] – 21,000(5.4206) + 10,000[0.4323 + 0.1869]
= $–217,928

(b) Calculate the FW from PW values over the 12-year LCM, or set up FW
relations directly from cash flow estimates. Select machine D.

FWD = PWD(F/P,15%,12)
= –190,344(5.3503)
= $–1,018,398

FWE = PWE(F/P,15%,12)
= –217,928(5.3503)
= $–1,165,980

(c) A spreadsheet solution for parts (a) and (b) follows.

4.34 HJ Heinz Corporation is constructing a distribution facility in Italy for products


such as Heinz Ketchup, Jack Daniel’s sauces, HP steak sauce, and Lea & Perrins
Worcestershire sauce. A 15-year life is expected for the structure. The exterior of
the building is not yet selected. One alternative is to use concrete walls as the

Copyright 2021 © McGraw-Hill Education. All rights reserved. No reproduction or distribution without the
prior written consent of McGraw-Hill Education.
18
façade and paint them. This will require painting now and every 5 years at a cost of
$80,000 each time. Another alternative is an anodized metal exterior attached to the
concrete wall. This will cost $200,000 now and require only minimal maintenance
of $500 every 3 years. A metal exterior is more attractive and will have a resale
value of an estimated $25,000 more than concrete 15 years from now. Painting (for
concrete) or maintenance (for metal) will be performed in the last year of
ownership to promote selling the property. Use future worth analysis and MARR =
12% per year to select the exterior finish (a) using tabulated factors, and (b) a
spreadsheet.

(a) Use C for painted concrete and M for metal exterior.


FWC = –80,000[(F/P,12%,15)+(F/P,12%,10)+(F/P,12%,5)+1]
= –80,000[5.4736 + 3.1058 + 1.7623 + 1]
= $–907,336

FWM = –200,000(F/P,12%,15) – 500[(F/P,12%,12)+(F/P,12%,9)+(F/P,12%,6)


+(F/P,12%,3)+1] + 25,000
= –200,000(5.4736) – 500[3.8969 + 2.7731 + 1.9738 + 1.4049 + 1]
+ 25,000
= $–1,075,244

Select the painted concrete exterior.

(b) Spreadsheet follows to select the painted concrete exterior.

4.35 Raytheon, a defense contractor, has been asked by the police department of
Middletown to estimate and analyze the life-cycle costs for a proposed drone
surveillance system to monitor traffic patterns and congestion within the central
thoroughfares of the city. The list includes the following categories: R&D costs
(R&D), nonrecurring investment costs (NRI), recurring investment costs (RI),
scheduled and unscheduled maintenance costs (Maint), equipment usage costs
(Equip), and disposal costs (Disp). The costs (in $ million units) for the 20-year life
cycle have been estimated. Calculate the present worth at a MARR of 7% per year.

Copyright 2021 © McGraw-Hill Education. All rights reserved. No reproduction or distribution without the
prior written consent of McGraw-Hill Education.
19
Year R&D NRI RI Maint Equip Disp
0 5.5 1.1
1 3.5
2 2.5
3 0.5 5.2 1.3 0.6 1.5
4 10.5 3.1 1.4 3.6
5 10.5 4.2 1.6 5.3
6–10 6.5 2.7 7.8
11 on 2.2 3.5 8.5
18–20 2.7

PW of LCC = – 6.6 – 3.5(P/F,7%,1) – 2.5(P/F,7%,2) – 9.1(P/F,7%,3)


– 18.6(P/F,7%,4) – 21.6(P/F,7%,5) – 17(P/A,7%,5)(P/F,7%,5)
– 14.2(P/A,7%,10)(P/F,7%,10) – 2.7(P/A,7%,3)(P/F,7%,17)
= – 6.6 – 3.5(0.9346) – 2.5(0.8734) – 9.1(0.8163) – 18.6(0.7629)
– 21.6(0.7130) – 17(4.1002)(0.7130) – 14.2(7.0236)(0.5083)
– 2.7(2.6243)(0.3166)
= $–151,710,860

4.36 Three different plans were presented to the Alphabet Corporation for operating an
identity-theft scanning system. Plan A involves renewable 1-year contracts with
payments of $1 million at the beginning of each year. Plan B is a 2-year contract
that requires four payments of $600,000 each, with the first one made now and the
other three at 6-month intervals. Plan C is a 3-year contract that entails a payment
of $1.5 million now and a second payment of $0.5 million 2 years from now.
Assuming that the company could renew any of the plans under the same payment
conditions, determine which plan is best on the basis of a PW analysis at a MARR
of 6% per year compounded semiannually.

i/year = (1 + 0.03)2 – 1 = 6.09%; use LCM = 6 years or 12 6-month periods.

PWA = –1,000,000 – 1,000,000(P/A,6.09%,5)


= –1,000,000 – 1,000,000(4.2021)
= $–5,202,100

PWB = –600,000 – 600,000(P/A,3%,11)


= –600,000 – 600,000(9.2526)
= $–6,151,560

PWC = –1,500,000 – 500,000(P/F,3%,4) – 1,500,000(P/F,3%,6)


– 500,000(P/F,3%,10)
= –1,500,000 – 500,000(0.8885) – 1,500,000(0.8375) – 500,000(0.7441)
= $–3,572,550

Select plan C

Copyright 2021 © McGraw-Hill Education. All rights reserved. No reproduction or distribution without the
prior written consent of McGraw-Hill Education.
20
4.37 A military unit received two proposals for a project to design, build, and maintain
barracks for soldiers in training. Proposal A involves an off-the-shelf “bare-bones”
design and standard-grade construction of walls, windows, doors, and other features.
With this option, heating and cooling costs will be greater, maintenance costs will be
higher, and replacement will occur earlier than proposal B. The initial cost for A will
be $750,000. Heating and cooling costs will average $6000 per month with
maintenance costs averaging $2000 per month. Minor remodeling will be required in
years 5, 10, and 15 at a cost of $150,000 each time in order to render the units usable
for 20 years. They will have no salvage value. Proposal B will include tailored design
and construction costs of $1.3 million initially with estimated heating and cooling costs
of $3000 per month and maintenance costs of $1000 per month. There will be no
salvage value at the end of the 20-year life. Which proposal should be accepted on the
basis of a life-cycle cost analysis at a MARR of 0.5% per month?

LCCA = –750,000 – (6000 + 2000)(P/A,0.5%,240) – 150,000[(P/F,0.5%,60)


+ (P/F,0.5%,120) + (P/F,0.5%,180)]
= –750,000 – (8000)(139.5808) – 150,000[(0.7414) + (0.5496) + (0.4075)]
= $–2,121,421

LCCB = –1,300,000 – (3000 + 1000)(P/A,0.5%,240)


= –1,300,000 – (4000)(139.5808)
= $–1,858,323

Select proposal B

4.38 The Health Department of Winter Park plans to develop an upgraded software
system to assist medical personnel in the treatment of opioid-addicted patients. A
program cost estimation approach (discussed earlier in section 1.8) has been used
to make estimates for a 10-year progam with categories of development,
programming, operating, and support costs. There are three alternatives under
consideration, identified as A (tailored system), B (adapted system), and C (current
system). The cost estimates are summarized below. Perform a life-cycle cost
analysis to identify the best alternative at 8% per year using (a) tabulated factors
first, then (b) a spreadseet to verify your selection.

Alternative Cost Component Estimated Cost and Time Frame


A Development $250,000 now, $150,000 years 1–4
Programming $45,000 now, $35,000 years 1–2
Operation $50,000 years 1–10
Support $30,000 years 1–5
B Development $10,000 now
Programming $45,000 year 0, $30,000 years 1–3
Operation $80,000 years 1–10
Support $40,000 years 1–10
C Operation $175,000 years 1–10

Copyright 2021 © McGraw-Hill Education. All rights reserved. No reproduction or distribution without the
prior written consent of McGraw-Hill Education.
21
(a) LCCA = –250,000 – 150,000(P/A,8%,4) – 45,000 – 35,000(P/A,8%,2)
–50,000(P/A,8%,10) – 30,000(P/A,8%,5)
= –250,000 – 150,000(3.3121) – 45,000 – 35,000(1.7833)
–50,000(6.7101) – 30,000(3.9927)
= $–1,309,517

LCCB = –10,000 – 45,000 – 30,000(P/A,8%,3) – 80,000(P/A,8%,10)


– 40,000(P/A,8%,10)
= –10,000 – 45,000 – 30,000(2.5771) – 80,000(6.7101
–40,000(6.7101)
= $–937,525

LCCC = –175,000(P/A,8%,10)
= –175,000(6.7101)
= $–1,174,268

Select alternative B

(b) Spreadsheet follows to verify selection of B.

4.39 Robert became quite well-to-do as founder and president of Carlson Auto Paint and
Supply, Inc. (CAPS). Now, he wants to start a permanent fund to support research
directed toward improved sustainability of painted surfaces at his alma mater. He
plans to contribute money now so that $100,000 per year can be withdrawn each
year forever, beginning in year 6. If the fund earns interest at a rate of 8% per year,
how much money must be donated now?

CC = [(100,000/0.08)](P/F,8%,5)
= 1,250,000(0.6806)
= $850,750

Copyright 2021 © McGraw-Hill Education. All rights reserved. No reproduction or distribution without the
prior written consent of McGraw-Hill Education.
22
4.40 Determine the capitalized cost of $100,000 now and $50,000 per year in years 1
through infinity at (a) an interest rate of 10% per year, and (b) an interest rate of
10% per year compounded continuously.

(a) CC = 100,000 + 50,000/0.10


= $600,000

(b) i per year = e0.10 – 1 = 10.517%

CC = 100,000 + 50,000/0.10517
= $575,421

4.41 Joe and Marylyn Hardwick, both graduates of a small, private university in
Vermont, have decided they can contribute $50,000 every 5 years forever to the
university, initially during their lives and thereafter through a charitable foundation
they have already established. Their contribution will begin 5 years from now.
Determine the capitalized cost of the infinite series of contributions at an interest
rate of 8% per year.

Convert F in year 5, 10, … to a 5-year A series; then divide by i.

A = 50,000(A/F,8%,5)
= 50,000(0.17046)
= $8523

CC = 8523/0.08 = $106,538

4.42 Compare the alternatives shown on the basis of their capitalized costs using a
MARR of 10% per year.

Alternative M N
First cost, $ −150,000 −800,000
Annual operating cost, $ per year −50,000 −12,000
Salvage value, $ 8,000 1,000,000
Life, years 5 ∞

For M, first find AW and then divide by i to find CC.

AWM = –150,000(A/P,10%,5) – 50,000 + 8000(A/F,10%,5)


= –150,000(0.26380) – 50,000 + 8000(0.16380)
= $–88,260

CCM = –88,260/0.10
= $–882,600

Copyright 2021 © McGraw-Hill Education. All rights reserved. No reproduction or distribution without the
prior written consent of McGraw-Hill Education.
23
CCN = – 800,000 – 12,000/0.10
= $–920,000

Select alternative M

4.43 A patriotic group of firefighters is raising money to erect a permanent (i.e., infinite life)
monument in New York City to honor those killed in the line of duty. The initial cost
of the monument will be $1,500,000 and the annual maintenance will cost $5000.
There will be an additional one-time cost of $20,000 in 2 years to add names of those
who were missed initially. At an interest rate of 6% per year, how much money must
they raise now in order to construct and maintain the monument forever?

CC = –1,500,000 – 5000/0.06 – 20,000(P/F,6%,2)


= –1,500,000 – 5000/0.06 – 20,000(0.8900)
= $–1,601,133

4.44 The president of Biomed Products is considering a long-term contract to outsource


maintenance and operations that will significantly improve the energy efficiency of
their imaging systems. The payment schedule has two large payments in the first
years with continuing payments thereafter. The proposed schedule is $200,000
now, $300,000 four years from now, $50,000 every 5 years, and an annual amount
of $8000 beginning 15 years from now and continuing indefinitely. Determine the
capitalized cost at 8% per year.

Monetary terms are in $1000 units.

CC = –200 – 300(P/F,8%,4) – 50(A/F,8%,5)/0.08 – (8/0.08)(P/F,8%,14)


= –200 – 300(0.7350) – 50(0.17046)/0.08 – (8/0.08)(0.3405)
= $–561.088 ($–561,088)

4.45 Yvonne’s father was a true believer in “giving back.” He endowed a program 35
years ago to help students receive degrees when they are short on funds. (a) How
much money was contributed 35 years ago if it earned at a rate of 10% per year
(with no withdrawals) and is now sufficient to provide a perpetual income of
$10,000 annually beginning this year, year 35? (b) If Yvonne wants to start her
own scholarship fund that generates $10,000 annually starting next year, what is
the amount she must contribute if earnings remain at 10% per year?

(a) Determine PW in year -1; then find PW 34 years earlier.

PW–1 = CC–1 = 10,000/0.10 = 100,000

PW–35 = 100,000(P/F,10%,34)
= 100,000(0.0391)
= $3910

(b) CCnow = 10,000/0.10 = $100,000

Copyright 2021 © McGraw-Hill Education. All rights reserved. No reproduction or distribution without the
prior written consent of McGraw-Hill Education.
24
4.46 Assume that 25 years ago your dad invested $200,000, plus $25,000 in years 2
through 5, and $40,000 per year from year 6 until now. At a very good interest rate
of 12% per year, determine (a) the present worth in year 0 (i.e., 25 years ago), and
(b) the annual amount that can be withdrawn forever starting next year (year 26), if
no additional investments are made.

(a) PW–25 = 200,000 + 25,000(P/A,12%,4)(P/F,12%,1)


+ 40,000(P/A,12%,20)(P/F,12%,5)
= 200,000 + 25,000(3.0373)(0.8929) + 40,000(7.4694)(0.5674)
= $437,326

(b) Find FW value in year 0 (25 years after PW–25), which is the CC base, and
multiply by i.

A = FW0 × i
= [200,000(F/P,12%,25) + 25,000(F/A,12%,4)(F/P,12%,20)
+ 40,000(F/A,12%,20)](0.12)
= [200,000(17.0001) + 25,000(4.7793)(9.6463) + 40,000(72.0524)](0.12)
= 7,434,680(0.12)
= $892,161 per year, forever

4.47 An aggressive stockbroker claims an ability to consistently earn 12% per year on
an investor’s money. Believing the broker, you invest $10,000 now, $30,000 three
years from now, and $8000 per year for 5 years starting 4 years from now. (a) How
much money can you withdraw every year forever, beginning 20 years from now?
(b) What is the capitalized cost of your investments if the $8000 per year
investment is expected to continue for an unspecified time into the future instead of
just 5 years?

(a) Find future value in year 19, which is the CC base; then multiply by i.

F19 = 10,000(F/P,12%,19) + 30,000(F/P,12%,16) + 8000(F/A,12%,5)(F/P,12%,11)


= 10,000(8.6128) + 30,000(6.1304) + 8000(6.3528)(3.4785)
= $446,826

A = 446,826(0.12)
= $53,619 per year, forever

(b) CC = 10,000 + 30,000(P/F,12%,3) + (8000/0.12)(P/F,12%,3)


= 10,000 + 30,000(0.7118) + 66,667(0.7118)
= $78,807

4.48 A company that makes food-friendly silicone (for use in cooking and baking pan
coatings) is considering four independent projects shown, all of which can be
considered to be viable for only 10 years. The company’s MARR is 15% per year.
(a) Determine which projects to implement. Financial values are in $1000 units. (b)

Copyright 2021 © McGraw-Hill Education. All rights reserved. No reproduction or distribution without the
prior written consent of McGraw-Hill Education.
25
(Spreadsheet exercise) Use GOAL SEEK to determine the required first cost of
project D to just accept it. Is this a maximum or minimum required first cost?

Project A B C D
First cost, $ −1200 −2000 −5000 −7000
Annual net income, $/year 200 400 1100 1300
Salvage value, $ 5 6 8 7

(a) Compare PW values against DN PW = $0; select all with PW ≥ 0. Monetary


units are $1000.
      
 PWA = –1200 + 200(P/A,15%,10) + 5(P/F,15%,10)
= –1200 + 200(5.0188) + 5(0.2472)
= $–195.004 (Reject)

PWB = –2000 + 400(P/A,15%,10) + 6(P/F,15%,10)


= –2000 + 400(5.0188) + 6(0.2472)
= $9.003 (Accept)

PWC = –5000 + 1100(P/A,15%,10) + 8(P/F,15%,10)


= –5000 + 1100(5.0188) + 8(0.2472)
= $522.658 (Accept)

PWD = –7000 + 1300(P/A,15%,10) + 7(P/F,15%,10)


= –7000 + 1300(5.0188) + 7(0.2472)
= $–473.830 (Reject)

(b) Sample solution: Enter net income values into cells B2-B12 (in $1000 units).
Use NPV function in cell B14 to display PW = –$473.870. GOAL SEEK
template changes cell B2 to force cell B14 to equal 0.00.

Result is a maximum first cost of $–6526.13 ($–6,526,130)

4.49 An engineer calculated the PW values for four alternatives to develop a remotely
controlled vibrations control system for offshore platform application. The results
in the table use a MARR of 14% per year. Determine which alternative(s) should
be selected if (a) the alternatives are mutually exclusive, and (b) if the projects are
independent.

Alternative I J K L
Life, n, years 3 4 12 6
PW over n years, $ 16.08 31.12 −257.46 140.46
PW over 6 years, $ 26.94 15.78 −653.29 140.46
PW over 12 years, $ 39.21 60.45 −257.46 204.46

Copyright 2021 © McGraw-Hill Education. All rights reserved. No reproduction or distribution without the
prior written consent of McGraw-Hill Education.
26
(a) Use LCM of 12 years; select L with PW = $204.46

(b) Use PW over life of each alternative; select I, J and L with PW > 0

4.50 The product development group of a high-tech electronics company developed five
proposals for new products. The company wants to expand its product offerings, so
it will undertake all projects that are economically attractive at the company’s
MARR of 20% per year. The cash flows (in $1000 units) associated with each
project are estimated. Which projects, if any, should the company accept on the
basis of a present worth analysis, under the following conditions?
a. There is no budget restriction.
b. (Spreadsheet exercise) No more than $1.2 million (i.e., $1200 in $1000 units)
can be invested and the initial investment of project 2 was corrected to be $
−400, with other estimates remaining as shown. (Hint: Use PV function results
to obtain bundle PW values.)

Project 1 2 3 4 5
Initial investment, $ −400 −510 −660 −820 −900
M&O cost, $/year −100 −140 −280 −315 −450
Revenue, $/year 360 235 400 605 790
Salvage value, $ — 22 — 8 95
Life, years 3 10 5 8 4

(a) PW1 = –400 + (360 – 100)(P/A,20%,3)


    = –400 + 260(2.1065)
= $147.69 ($147,690) Accept

PW2 = –510 + (235 – 140)(P/A,20%,10) + 22(P/F,20%,10)


= –510 + 95(4.1925) + 22(0.1615)
= $–108.160 ($–108,160) Reject

PW3 = –660 + (400 – 280)(P/A,20%,5)


= –660 + 120(2.9906)
= $–301.128 ($–301,128) Reject

PW4 = –820 + (605 – 315)(P/A,20%,8) + 8(P/F,20%,8)


= –820 + 290(3.8372) + 8(0.2326)
= $294.649 ($294,649) Accept

PW5 = –900 + (790 – 450)(P/A,20%,4) + 95(P/F,20%,4)


= –900 + 340(2.5887) + 95(0.4823)
= $25.977 ($25,977) Accept

Projects 1, 4 and 5 are acceptable with PW > 0

Copyright 2021 © McGraw-Hill Education. All rights reserved. No reproduction or distribution without the
prior written consent of McGraw-Hill Education.
27
(b) With an initial investment for project 2 of $–400, PW2 = $1.84; it is now
acceptable. Project 3 is eliminated from further consideration. Functions are of
the format = NPV(20%,first_year:last_year) + investment
Of the 24 = 16 bundles, only 5 plus the DN option qualify; all others have an
investment > $1200. For the PW of a bundle, add the PW values for projects in
the bundle.

Select project 4 only with the largest PW.

4.51 Analon Health, a midsized health care company in Toronto, is considering the
addition of one or more of four new services in hospitals. The total amount of
investment capital available for new ventures is $800,000. Analon uses a 5-year
project recovery period and a MARR of 20% per year. All cash flows are in $1000
units. (a) Using tabulated factors, determine which one(s) should be undertaken on
the basis of a PW analysis. (b) Make the selection using a spreadsheet and PV
functions.

Services R1 S2 T3 U4
Initial development, $ −200 −400 −500 −700
M&O cost, $/year −50 −200 −300 −400
Revenue, $/year 150 450 520 770

(a) Of the 24 = 16 possible bundles, there are 7 within the $800,000 budget limit, as
follows: DN; R1; S2; T3; U4; as well as combination bundles of R1,S2, and
R1,T3. Money values are in $1000 units.

Copyright 2021 © McGraw-Hill Education. All rights reserved. No reproduction or distribution without the
prior written consent of McGraw-Hill Education.
28
PWDN = $0
PWR1 = –200 + (150 – 50)(P/A,20%,5)
= –200 + 100(2.9906)
= $99.06 ($99,060)
PWS2 = –400 + (450 – 200)(P/A,20%,5)
= –400 + 250(2.9906)
= $347.650 ($347,650)
PWT3 = –500 + (520 – 300)(P/A,20%,5)
= –500 + 220(2.9906)
= $157.932 ($157,932)
PWU4 = –700 + (770 – 400)(P/A,20%,5)
= –700 + 370(2.9906)
= $406.522 ($406,522)
PWR1,S2 = 99.060 + 347.650
= $446.710 ($446,710)
PWR1,T3 = 99.060 + 157.932
= $256.992 ($256,922)
Select product lines R1 and S2 with the highest PW of $446.710
(b) Spreadsheet verifies selection of R1 and S2.

4.52 Based on PW values, determine which of the following independent projects


should be selected for investment if $240,000 is available and the MARR is 10%
per year. (NCF = net cash flow)

Project Initial Investment, $ NCF, $/Year Life, Years


A −100,000 50,000 8
B −125,000 24,000 8
C −120,000 75,000 8
D −220,000 39,000 8
E −200,000 82,000 8

Copyright 2021 © McGraw-Hill Education. All rights reserved. No reproduction or distribution without the
prior written consent of McGraw-Hill Education.
29
Develop the bundles with less than $240,000 investment, and select the one with
the largest PW value.

Bundle Projects Initial investment, $ NCF, $/year PW at 10%, $


1 A –100,000 50,000 166,746
2 B –125,000 24,000 3,038
3 C –120,000 75,000 280,118
4 D –220,000 39,000 –11,938
5 E –200,000 82,000 237,464
6 AB –225,000 74,000 169,784
7 AC –220,000 125,000 446,864
8 DN 0 0 0

PW1 = –100,000 + 50,000(P/A,10%,8)


= –100,000 + 50,000(5.3349)
= $166,746

PW2 = –125,000 + 24,000(P/A,10%,8)


= –125,000 + 24,000(5.3349)
= $3038

PW3 = –120,000 + 75,000(P/A,10%,8)


= –120,000 + 75,000(5.3349)
= $280,118

PW4 = –220,000 + 39,000(P/A,10%,8)


= –220,000 + 39,000(5.3349)
= $–11,939

PW5 = –200,000 + 82,000(P/A,10%,8)


= –200,000 + 82,000(5.3349)
= $237,462

All other PW values are obtained by adding the respective PW for bundles 1-5.

Conclusion: Select PW = $446,864, which is bundle 7 (projects A and C) with


$220,000 total investment.

4.53 Dwayne has four independent vendor proposals to contract the nationwide oil
recycling services for the Ford Corporation manufacturing plants. All combinations
are acceptable, except that vendors B and C cannot both be chosen. Revenue
sharing of recycled oil sales with Ford is a part of the requirement. Develop all
possible mutually exclusive bundles under the additional following restrictions and
select the best projects. The corporate MARR is 10% per year.

Copyright 2021 © McGraw-Hill Education. All rights reserved. No reproduction or distribution without the
prior written consent of McGraw-Hill Education.
30
a. Initially determine PW values for each vendor using tabulated factors and
spreadsheet functions.
b. A maximum of $4 million can be spent.
c. A larger budget of $5.5 million is allowed, but no more than two vendors can be
selected.
d. There is no limit on investment.

Life, Annual Net Revenue,


Vendor Initial Investment, $ Years $/Year
A −1.5 million 8 360,000
B −3.0 million 10 600,000
C −1.8 million 5 620,000
D −2.0 million 4 630,000

(a) PW for each project using factors.

PWA = –1,500,000 + 360,000(P/A,10%,8) = $420,564


PWB = –3,000,000 + 600,000(P/A,10%,10) = $686,760
PWC = –1,800,000 + 620,000(P/A,10%,5) = $550,296
PWD = –2,000,000 + 630,000(P/A,10%,4) = $–2,963 (not acceptable)

Spreadsheet: Enter the following to display the project PW values.

A: = – PV(10%,8,360000) –1500000 Display: $420,573


B: = – PV(10%,10,600000) –3000000 Display: $686,740
C: = – PV(10%,5,620000) –1800000 Display: $550,288
D: = – PV(10%,4,630000) –2000000 Display; $–2,985 (not acceptable)

Formulate acceptable bundles from the 24 = 16 possibilities, without both B and C


and select projects with largest total PW of a bundle.

(b) With, b = $4 million, select projects A and C with PW = $970,860.


(Note: These PW values use the factor results above.)

Investment
Bundle , PW, $
$ million
DN 0 0
A –1.5 420,564
B –3.0 686,760
C –1.8 550,296
AC –3.3 970,860

Copyright 2021 © McGraw-Hill Education. All rights reserved. No reproduction or distribution without the
prior written consent of McGraw-Hill Education.
31
(c) For b = $5.5 million, select projects A and B with PW = $1,107,313.

Investment,
Bundle $ million PW, $
DN 0 0
A −1.5 420,564
B −3.0 686,760
C −1.8 550,296
AB −4.5 1,107,324
AC −3.3 970,860

(d) With no-limit, select all with PW > 0. Select projects A, B and C.

4.54 From the PW values shown for four mutually exclusive alternatives, the one(s) to
select is:

Alternative PW at 8%, $
A −25,000
B −12,000
C 10,000
D 15,000

a. Only A
b. Only D
c. Only A and B
d. Only C and D

Answer is (b)

4.55 The present worth of $50,000 now, $10,000 per year in years 1 through 15, and
$20,000 per year in years 16 through infinity at 10% per year is closest to:
a. Less than $169,000
b. $169,580
c. $173,940
d. $195,730

PW = 50,000 + 10,000(P/A,10%,15) + [20,000/0.10](P/F,10%,15)


= $173,941
Answer is (c)

Copyright 2021 © McGraw-Hill Education. All rights reserved. No reproduction or distribution without the
prior written consent of McGraw-Hill Education.
32
4.56 At a return of 10% per year, the amount you must deposit in your retirement
account each year for years 0 through 9 (i.e., 10 deposits) if you want to withdraw
$50,000 per year forever beginning 30 years from now is closest to:
a. $4239
b. $4660
c. $4974
d. $5471

CC = [50,000/0.10](P/F,10%,20)(A/F,10%,10)
= $4662.33
Answer is (b)

Problems 4.57 through 4.59 are based on the following estimates for two mutually
exclusive alternatives. The cost of money is 10% per year.

Machine X Y
Initial cost, $ −66,000 −46,000
Annual cost, $/year −10,000 −15,000
Salvage value, $ 10,000 24,000
Life, years 6 3

4.57 The present worth of machine X is closest to:


a. $−65,270
b. $−87,840
c. $−103,910
d. $−114,310

PWX = –66,000 –10,000(P/A,10%,6) + 10,000(P/F,10%,6)


= –66,000 –10,000(4.3553) + 10,000(0.5645)
= $–103,908
Answer is (c)

4.58 In comparing the machines on a present worth basis, the present worth of machine
Y is closest to:
a. $−65,270
b. $−97,840
c. $−103,910
d. $−114,310

LCM is 6 years.
PWY = –46,000 –15,000(P/A,10%,6) – 22,000(P/F,10%,3) + 24,000(P/F,10%,6)
= –46,000 –15,000(4.3553) – 22,000(0.7513) + 24,000(0.5645)
= $–114,310
Answer is (d)

Copyright 2021 © McGraw-Hill Education. All rights reserved. No reproduction or distribution without the
prior written consent of McGraw-Hill Education.
33
4.59 The capitalized cost of machine X is closest to:
a. $−103,910
b. $−114,310
c. $−1,039,080
d. $−238,580

CCX = [–66,000(A/P,10%,6) – 10,000 + 10,000(A/F,10%,6)]/0.10


= [–66,000(0.22961) – 10,000 + 10,000(0.12961)]/0.10
= $–238,582
Answer is (d)

Problems 4.60 through 4.62 are based on the following estimates for four independent
projects. The cost of money is 8% per year.

Project Initial Investment, $ PW, $


1 −20,000 2,400
2 −35,000 9,200
3 −40,000 −7,300
4 −55,000 11,400

4.60 If the investment budget is $75,000, all of the following mutually exclusive
bundles should be evaluated, except:
a. DN
b. 2,4
c. 2,3
d. 1,2

Answer is (b)

4.61 If only two projects may be selected with no more than $100,000 invested, the
projects selected are:
a. 1 and 2
b. 3 and 4
c. 2 and 3
d. 2 and 4

Bundle 2,4 invests $90,000 with the largest PW = $20,600.


Answer is (d)

4.62 You learned of a serious error in estimation for project 3. Its PW is actually $2500,
not $−7300. If only two projects may be selected with no more than $100,000
invested, the projects selected are now:
a. 2 and 3
b. 3 and 4
c. 2 and 4
d. None of these

Copyright 2021 © McGraw-Hill Education. All rights reserved. No reproduction or distribution without the
prior written consent of McGraw-Hill Education.
34
The bundle of 2 and 4 still has the largest PW at $20,600.
Answer is (c)

4.63 A $20,000 collateral bond has a coupon rate of 7% per year payable quarterly. The
bond matures 30 years from now. At a market interest rate of 7% per year
compounded semiannually, the amount and frequency of the bond interest
payments is:
a. $1400 per year
b. $1400 per quarter
c. $350 per year
d. $350 per quarter

I/quarter = 20,000(0.07)/4
= $350
Answer is (d)

4.64 A $25,000 mortgage bond with an interest rate of 6% per year payable
semiannually is due 12 years from now. The present worth of the bond at an
interest rate of 10% per year payable semiannually is closest to:
a. $18,100
b. $18,925
c. $19,330
d. $22,155

I = 25,000(0.06)/2
= $750 every six months

PW = 750(P/A,5%,24) + 25,000(P/F,5%,24)
= 750(13.7986) + 25,000(0.3101)
= $18,101
Answer is (a)

4.65 As part of your inheritance, you received a bond that will pay interest of $700
every 6 months for 15 years. If the coupon rate is 7% per year, the face value of the
bond is:
a. $10,000
b. $20,000
c. $30,000
d. $40,000

700 = V(0.07)/2
V = $20,000
Answer is (b)

Copyright 2021 © McGraw-Hill Education. All rights reserved. No reproduction or distribution without the
prior written consent of McGraw-Hill Education.
35
4.66 You own a $50,000 corporate bond with a bond interest rate of 6% payable
monthly that is due 10 years from now. If you want to sell it today and realize a
rate of return of 12% per year compounded monthly, the amount you must receive
is closest to:
a. $32,575
b. $33,125
c. $35,250
d. $38,550

I = 50,000(0.06)/12
= $250 per month

PW = 250(P/A,1%,120) + 50,000(P/F,1%,120)
= 250(69.7005) + 50,000(0.3030)
= $32,575
Answer is (a)

Copyright 2021 © McGraw-Hill Education. All rights reserved. No reproduction or distribution without the
prior written consent of McGraw-Hill Education.
36
Solutions to end-of-chapter problems
Basics of Engineering Economy, 3rd edition
Leland Blank and Anthony Tarquin

Chapter 5
Annual Worth Analysis

5.1 What is the major advantage that the annual worth method has over the present
worth method?

The equal service requirement is met without using the least common multiple
(LCM) of alternative lives.

5.2 Assume that an alternative has a 3-year life and that you calculated its annual worth
over its 3-year life cycle. If you were told to provide the annual worth of that
alternative for a 4-year study period, would the annual worth value you calculated
from the alternative’s 3-year life cycle be a valid estimate of the annual worth over
the 4-year study period? Why or why not?

The estimate obtained from the three-year AW would not be valid, because the AW
calculated over one life cycle is valid only for the entire cycle, not part of the cycle.
Here the asset would be used for only a part of its second 3-year life cycle.

5.3 A theft-avoidance locking system has a first cost of $10,000, an AOC of $7000, and
no salvage value after its 3-year life. Assume that you were told the service provided
by this asset would be needed for only 5 years. This means that the asset will have to
be repurchased and kept for only 2 years. What would its market value, call it M,
have to be after 2 years in order to make its annual worth the same as it is for its
3-year life cycle at an interest rate of 10% per year? Determine the market value M
using (a) factors, and (b) a spreadsheet with PMT functions and GOAL SEEK.

(a) Let M = market value after 2 years

–10,000(A/P,10%,3) – 7000 = –10,000(A/P,10%,2) – 7000 + M(A/F,10%,2)


–10,000(0.40211) – 7000 = –10,000(0.57619) – 7000 + M(0.47619)
M = $3656

(b) Spreadsheet uses PMT functions to find AW values. For 3-year life AW =
$11,021. In this solution, GOAL SEEK sets cell D3 to –11021 and changes cell
E1 to display an M of $3656, the same as the factor-based solution.

Copyright 2021 © McGraw-Hill Education. All rights reserved. No reproduction or distribution without the
prior written consent of McGraw-Hill Education.
5.4 An asset with a first cost of $20,000 has an annual operating cost of $12,000 and a
$4000 salvage value after its 4-year life. The project will be needed for 6 years. This
means that the asset will have to be repurchased and kept for only 2 years.
a. Determine the capital recovery amount over the 4-year life if a MARR of 10% per
year is required.
b. What spreadsheet function displays a CR? Write the function for this situation.
c. What must be the market value, M, of the repurchased 2-year-old asset for the
annual worth to be the same as it is for one complete life cycle of the asset?
Again, MARR is 10% per year.

(a) CR = –20,000(A/P,10%,4) + 4000(A/F,10%,4)


= –20,000(0.31547) + 4000(0.21547)
= $–5,448

(b) The PMT function will display the CR value.

Function: = – PMT(10%,4,–20000,4000) displays a CR of $–5447.53

(c) Let M = market value after 2 years. Set 2-year AW relation equal to the AW for
4 years; solve for M.

AW4 = –20,000(A/P,10%,4) – 12,000 + 4000(A/F,10%,4)


= –20,000(0.31547) – 12,000 + 4000(0.21547)
= $–17,448

–17,448 = –20,000(A/P,10%,6) – 12,000 – (20,000 – 4000)(P/F,10%,4)(A/P,10%,6)


+ M(A/F,10%,6)

= –20,000(0.22961) – 12,000 – (20,000 – 4000)(0.6830)(0.22961)


+ M(0.12961)

(0.12961)M = 1,653.38
M = $12,756

Copyright 2021 © McGraw-Hill Education. All rights reserved. No reproduction or distribution without the
prior written consent of McGraw-Hill Education.
5.5 You are about to start a side business while in college. To do so, you just spent
$30,000 for a used, fully equipped taco truck. You plan to provide quality food at
lunch to construction workers at sites where there is no visible food outlet that is
fast, affordable, and offers tasty, zesty lunches. The total AOC is estimated to be
$12,000 and gross revenue is expected to be in the range of $20,000 to $30,000 per
year. If the truck’s salvage value after 5 years is estimated to be $14,000 and you
want a return of 10% per year on your investment, what is the minimum net annual
income required to meet your expectation? Is the estimated gross revenue range
sufficient?

CR = –30,000(A/P,10%,5) + 14,000(A/F,10%,5)
= –30,000(0.26380) + 14,000(0.16380)
= $–5,621

Net income required = CR + AOC


= 12,000 + 5,621 = $17,621

Conclusion: Revenue range of $20,000 to $30,000 is quite sufficient, as the entire


range allows recovery of CR and AOC.

5.6 Eight years ago, Ohio Valley Trucking purchased a large-capacity dump truck for
$115,000 to provide short-haul earth-moving services. The company sold it today for
$45,000. Operating and maintenance costs averaged $10,500 per year. A complete
overhaul at the end of year 4 cost an extra $3600. (a) Calculate the annual cost of the
truck at i = 8% per year. (b) (Spreadsheet exercise) Write a single-cell function to
display the annual cost.

(a) AW = –115,000(A/P,8%,8) – 10,500 – 3600(P/F,8%,4)(A/P,8%,8)


+ 45,000(A/F,8%,8)
= –115,000(0.17401) – 10,500 – 3600(0.7350)(0.17401) + 45,000(0.09401)
= $–26,741 per year

(b) Spreadsheet: PMT for purchase and salvage + AOC + PMT for overhaul with
embedded PV function. Same Aw is displayed.

= – PMT(8%,8,–115000,45000) – 10500 – PMT(8%,8,–PV(8%,4,–3600))

5.7 You just paid $2000 for a major repair on the suspension system of a car that you
purchased 5 years ago. The cost of periodic maintenance has been $800 every 2
years. If you donate the car to charity after 8 years of ownership, what is the
equivalent annual cost of maintenance and repairs for the 8-year period? (a) Use an
interest rate of 8% per year and assume that you paid the $800 maintenance cost
immediately before donating the car in year 8. (b) (Spreadsheet exercise) If you enter
year numbers from −5 to 3, what are the cash flow entries for each year to display
the correct AW value?

Copyright 2021 © McGraw-Hill Education. All rights reserved. No reproduction or distribution without the
prior written consent of McGraw-Hill Education.
(a) AW = –2000(P/F,8%,5)(A/P,8%,8) – 800(A/F,8%,2) -5 0
= –2000(0.6806)(0.17401) – 800(0.48077) -4 0
= $–621 per year -3 -800
-2 0
(b) Remember to enter zeros for the non-maintenance cost years. -1 -800
0 -2000
5.8 White Oaks Properties builds strip shopping centers and small malls. 1 -800
The company plans to replace its refrigeration, cooking, HVAC, and 2 0
other equipment with newer models in the entire center built 9 years 3 -800
ago. The original purchase price of the equipment was $638,000 nine
years ago and the operating cost has averaged $240,000 per year. Determine the
equivalent annual cost of the installed equipment, if the company can now sell it for
$184,000. The company’s MARR is 25% per year.

AW = –638,000(A/P,25%,9) – 240,000 + 184,000(A/F,25%,9)


= –638,000(0.28876) – 240,000 + 184,000(0.03876)
= $–417,097

5.9 U.S. Steel is considering a plant expansion to produce austenitic, precipitation


hardened, duplex, and martensitic stainless-steel round bars that is expected to cost
$13 million now and another $10 million 1 year from now. If total operating costs
will be $1.2 million per year starting 1 year from now, and the estimated salvage
value of the plant is virtually zero, how much must the company make annually in
years 1 through 10 to recover its investment plus a return of 15% per year?

Find the CR. In $1 million units

CR = [–13 – 10(P/F,15%,1)](A/P,15%,10)
= [–13 – 10(0.8696)](0.19925)
= $–4.3229

Revenue required is $4,322,900 per year

5.10 A company that manufactures magnetic flow meters expects to undertake a project
that will have the cash flows estimated. At an interest rate of 10% per year, what is
the equivalent annual cost of the project? Find the AW value using (a) tabulated
factors, and (b) a spreadsheet.

First cost, $ −800,000


Equipment replacement cost in year 2, $ −300,000
Annual operating cost, $/year −950,000
Salvage value, $ 250,000
Life, years 4

Copyright 2021 © McGraw-Hill Education. All rights reserved. No reproduction or distribution without the
prior written consent of McGraw-Hill Education.
(a) Factor:
AW = –800,000(A/P,10%,4) – 300,000(P/F,10%,2)(A/P/10%,4) – 950,000
+ 250,000(A/F,10%,4)
= –800,000(0.31547) – 300,000(0.8264)(0.31547) – 950,000
+ 250,000(0.21547)
= $–1,226,720

(b) Spreadsheet:

5.11 A 600-ton press used to produce composite-material fuel cell components for
automobiles using proton exchange membrane (PEM) technology can reduce the
weight of enclosure parts up to 75%. At MARR = 12% per year, use tabulated
factors to calculate the (a) capital recovery, and (b) total annual revenue required.
(c) Solve using a spreadsheet.

Installed cost = $−3.8 million


n = 12 years
Salvage value = $250,000
Annual operating costs = $−350,000 in year 1, increasing by $25,000 per year

(a) Use Equation [5.3] for CR per year.

CR = –3,800,000(A/P,12%,12) + 250,000(A/F,12%,12)
= –3,800,000(0.16144) + 250,000(0.04144)
= $–603,112

(b) AW = CR + A of AOC
= –603,112 – 350,000 – 25,000(A/G,12%,12)
= –603,112 – 350,000 – 25,000(4.1897)
= $–1,057,855

Revenue required is $1,057,855 per year

(c) One way to use a spreadsheet follows:


CR: single-cell entry = – PMT(12%,12,–3800000,250000)
AW: Enter increasing gradient into B3:B14 and add PMT functions
= – PMT(12%,12,–3800000,250000) – PMT(12%,12,NPV(12%,B3:B14))

Copyright 2021 © McGraw-Hill Education. All rights reserved. No reproduction or distribution without the
prior written consent of McGraw-Hill Education.
5.12 A large textile company is trying to decide which sludge dewatering process it
should use ahead of its sludge drying operation. The costs associated with
centrifuge and belt press systems are shown. Compare them on the basis of their
annual worths using an interest rate of 10% per year.

System Centrifuge Belt Press


AOC, $/year −31,000 −35,000
Overhaul in year 2, $ — −26,000
Salvage value, $ 40,000 10,000
Life, years 6 4

AWcentrifuge = –250,000(A/P,10%,6) – 31,000 + 40,000(A/F,1%,6)


= –250,000(0.22961) – 31,000 + 40,000(0.12961)
= $–83,218

AWbelt = –170,000(A/P,10%,4) – 35,000 – 26,000(P/F,10%,2)(A/P,10%,4)


+ 10,000(A/F,10%,4)
= –170,000(0.31547) – 35,000 – 26,000(0.8264)(0.31547)
+ 10,000(0.21547)
= $–93,254

Select centrifuge with lower AW of costs

5.13 A materials engineer is considering two compounds for the outer shells of optical
fibers. The germanium tetrachloride process (G) has the following estimates: first
cost of $85,000; annual M&O costs of $32,000; a useful life of 8 years; and a
$20,000 salvage value. The phosphorus oxychloride process (P) will have a first
cost of $97,000, annual M&O costs of $27,000, a useful life of 3 years, and a
$48,000 salvage value. The MARR is 12% per year.
a. Which process should be selected on the basis of an annual worth comparison?
Use a 3-year study period. Assume the G process will have a market value of
$40,000 after 3 years.
b. Is the selection different if the full 8-year life is allowed for procss G? Why?

(a) AWG = –85,000(A/P,12%,3) – 32,000 + 40,000(A/F,12%,3)


= –85,000(0.41635) – 32,000 + 40,000(0.29635)
= $–55,536

AWP = –97,000(A/P,12%,3) – 27,000 + 48,000(A/F,12%,3)


= –97,000(0.41635) – 27,000 + 48,000(0.29635)
= $–53,161

Select P, the phosphorus oxychloride process

Copyright 2021 © McGraw-Hill Education. All rights reserved. No reproduction or distribution without the
prior written consent of McGraw-Hill Education.
(b) AWG = –85,000(A/P,12%,8) – 32,000 + 20,000(A/F,12%,8)
= –85,000(0.20130) – 32,000 + 20,000(0.08130)
= $–47,485

Now select G, the germanium tetrachloride process,

Yes, the selection changes from P to G. Why? One prime reason is that process
G now has 5 more years to recover the $85,000 first cost plus the MARR.

5.14 Accurate air flow measurement requires straight unobstructed pipe for a minimum
of 10 diameters upstream and 5 diameters downstream of the measuring device. In
one particular application, physical constraints compromised the pipe layout, so the
engineer was considering installing the air flow probes in an elbow, knowing that
flow measurement would be less accurate but good enough for process control. This
is plan A, which would be acceptable for only 2 years. It will have a first cost of
$25,000 with annual maintenance estimated at $4000. Alternatively, a recently
designed submersible air flow probe could be installed in a drop pipe with the
transmitter located in a waterproof enclosure on the handrail. This plan, B, would
cost $88,000, but because it is accurate, it would not have to be replaced for at least
6 years. Its maintenance cost is estimated to be $1400 per year. Neither system will
have a salvage value. At an interest rate of 12% per year, which one should be
selected using AW values?

AWA = –25,000(A/P,12%,2) – 4000


= –25,000(0.59170) – 4,000
= $–18,793

AWB = –88,000(A/P,12%,6) – 1400


= –88,000(0.24323) – 1400
= $–22,804

Select plan A

5.15 Your friend is considering a new exhaust system for his Lamborghini Diablo. The
estimates from two shops are shown below. Besides installation, shop Y includes a
full replacement warranty for 4 years in the quote. Since money does not appear to
be a problem for your friend, and he asked you about the economics of the two
quotes, which should he choose based on annual worth values and an interest rate of
12% per year?

Shop X Y
Installed cost, $ −1765 −2900
Total AOC, $/year −1200 −900
Salvage value, $ 0 800
Life, years 2 4
AWX = –1765(A/P,12%,2) – 1200
Copyright 2021 © McGraw-Hill Education. All rights reserved. No reproduction or distribution without the
prior written consent of McGraw-Hill Education.
= –1765(0.59170) – 1200
= $–2244.35

AWY = –2900(A/P,12%,4) – 900 + 800(A/F,12%,4)


= –2900(0.32923) – 900 + 800(0.20923)
= $–1687.38

Select shop Y

5.16 The City Council in a southwestern city is considering the construction of


permanent restrooms in 22 of its smaller parks (i.e., parks less than 12 acres) or pay
for portable toilets on a year-round basis. The cost of constructing the 22 permanent
restrooms will be $3.8 million. The 22 portable restrooms can be rented for $7500
each the first year with rental cost increases of $25 per portable per year for a
maximum contract period of 10 years. The service life of a permanent restroom is
20 years. Using an interest rate of 6% per year and an annual worth analysis,
determine if the city should build the permanent restrooms or lease the portable
ones.

AWpermanent = –3,800,000(A/P,6%,20)
= –3,800,000(0.08718)
= $–331,284

AWportable = –22(7500) – 22(25)(A/G,6%,10)


= –165,000 – 550(4.0220)
= $167,212

The city should lease the portables.

5.17 A remotely located air sampling station can be powered by solar cells or by running
an aboveground electric line to the site and using conventional power. Solar cells
will cost $16,600 to install and will have a useful life of 5 years with no salvage
value. Annual costs for inspection, cleaning, and other maintainance issues are
expected to be $2400. A new power line will cost $31,000 to install, with power
costs expected to be $1000 per year. Since the air sampling project will end in
5 years, the salvage value of the line is considered to be zero. At an interest rate of
10% per year and using an AW analysis (a) which alternative should be selected,
and (b) what must be the first cost of the aboveground line to make the two
alternatives equally attractive economically? (c) (Spreadsheet exercise) Solve both
parts above using a spreadsheet and the GOAL SEEK tool.

(a) AWSolar = –16,600(A/P,10%,5) – 2400


= –16,600(0.26380) – 2400
= $–6779

Copyright 2021 © McGraw-Hill Education. All rights reserved. No reproduction or distribution without the
prior written consent of McGraw-Hill Education.
AWLine = –31,000(A/P,10%,5) – 1000
= –31,000(0.26380) – 1000
= $–9178

Use the solar cells

(b) Set AWLine = –6779 and solve for first cost Pline

–6779 = PLine(A/P,10%,5) – 1000


= PLine(0.26380) – 1000

PLine = $21,906

(c) Spreadsheet solution indicates solar as the choice. GOAL SEEK uses cell G4 as
the changing and cell G12 to be forced to equal AWSolar = –6779. Result is:

AWLine must be ≤ $21,907

5.18 A chemical engineer is considering two styles of pipes for moving distillate from a
refinery to the tank farm. A small pipeline will cost less to purchase (including valves
and other appurtenances) but will have a high head loss and, therefore, a higher
pumping cost. The small pipeline will cost $1.7 million installed and will have
an operating cost of $12,000 per month. A larger-diameter pipeline will cost
$2.1 million installed, but its operating cost will be only $8000 per month. Which
pipe size is more economical at a MARR of 12% per year compounded monthly on
the basis of an evaluation using the annual worth method? Assume the salvage value
is 10% of the first cost for each pipeline at the end of the 10-year study period.

AWsmall = –1,700,000(A/P,1%,120) – 12,000 + 170,000(A/F,1%,120)


= –1,700,000(0.01435) – 12,000 + 170,000(0.00435)
= $–35,656 per month

AWlarge = –2,100,000(A/P,1%,120) – 8,000 + 210,000(A/F,1%,120)


= –2,100,000(0.01435) – 8,000 + 210,000(0.00435)
= $–37,222 per month

Select small pipeline

Copyright 2021 © McGraw-Hill Education. All rights reserved. No reproduction or distribution without the
prior written consent of McGraw-Hill Education.
5.19 Equipment needed at a Valero Corporation refinery for the conversion of corn stock
to ethanol, a cleaner burning gasoline additive, will cost $175,000 and have a net
revenue of $35,000 the first year, increasing by $10,000 per year over the life of
5 years. (a) Use a spreadsheet (and tabulated factors, if instructed to do so) to
calculate the AW amounts at different MARR values to determine when the project
switches from financially justified to unjustified. (b) Develop a spreadsheet chart
that plots AW versus the MARR.

(a) AW = –175,000(A/P,i%,5) + 35,000 + 10,000(A/G,i%,5) indicates the turn to


unjustified just above 15% per year.

@12%: AW = –175,000(0.27741) + 35,000 + 10,000(1.7746) = $+4199


@14%: AW = –175,000(0.29128) + 35,000 + 10,000(1.7399) = $+1425
@15%: AW = –175,000(0.29832) + 35,000 + 10,000(1.7228) = $+22
@16%: AW = –175,000(0.30541) + 35,000 + 10,000(1.7060) = $–1387

(b) Spreadsheet indicates just above 15% at the point where AW = 0.

5.20 The TT Racing and Performance Motor Corporation wishes to evaluate two
alternative machines for NASCAR motor tune-ups. (a) Use the AW method at 9%
per year to select the better alternative. (b) Use spreadsheet single-cell functions to
find the better alternative.

Machine R S
First cost, $ −250,000 −370,500
Annual operating cost, $ per year −40,000 −50,000
Life, years 3 5
Salvage value, $ 20,000 20,000

(a) Factors: Calculate AW values to select machine R.

AWR = –250,000(A/P,9%,3) + 20,000(A/F,9%,3) – 40,000


= –250,000(0.39505) + 20,000(0.30505) – 40,000
= $–132,662

Copyright 2021 © McGraw-Hill Education. All rights reserved. No reproduction or distribution without the
prior written consent of McGraw-Hill Education.
AWS = –370,500(A/P,9%,5) + 20,000(A/F,9%,5) – 50,000
= –370,500(0.25709) + 20,000(0.16709) – 50,000
= $–141,910

(b) Spreadsheet: Enter single-cell functions.


R: = –PMT(9%,3,–250000,20000) – 40000 Display: $–132,663
S: = –PMT(9%,5,–370500,20000) – 50000 Display: $–141,911

5.21 Some estimates have been presented to Charlene, the Director of Operations (DO)
at Holly Farms, which is considering alternative environmental chambers for a
project that will detail laboratory confirmations of online bacteria tests in chicken
meat for the presence of E. coli 0157:H7 and Listeria monocytogenes. Answer the
following questions for Charlene.
a. If the project study period is 6 years and i = 10% per year, perform an AW
evaluation to determine which chamber is more economical.
b. Chamber D103 can be purchased with different options and, therefore, at a
different installed cost and salvage value. They range from $300,000 to
$500,000. Will the selection between alternatives change if one of these other
models is installed?
c. Use single-cell spreadsheet functions to solve part b.

Chamber D103 490G


Installed cost, $ −400,000 −250,000
AOC, $ per year −4,000 −3,000
Salvage value, 10% of P, $ 40,000 25,000
Life, years 3 2

(a) AWD103 = –400,000(A/P,10%,3) + 40,000(A/F,10%,3) – 4000


= –400,000(0.40211) + 40,000(0.30211) – 4000
= $–152,760

AW490G = –250,000(A/P,10%,2) + 25,000(A/F,10%,2) – 3000


= –250,000(0.57619) + 25,000(0.47619) – 3000
= $–135,143

Select chamber 490G

(b) Determine AWD103 values at the bounds of P and S.

For P = $–300,000 and S = $30,000:


AWD103 = –300,000(A/P,10%,3) + 30,000(A/F,10%,3) – 4000
= –300,000(0.40211) + 30,000(0.30211) – 4000
= $–115,570

Copyright 2021 © McGraw-Hill Education. All rights reserved. No reproduction or distribution without the
prior written consent of McGraw-Hill Education.
For P = $–500,000 and S = $50,000:
AWD103 = –500,000(A/P,10%,3) + 50,000(A/F,10%,3) – 4000
= –500,000(0.40211) + 50,000(0.30211) – 4000
= $–189,950
Cheaper model D103 with P = $–300,000 will change the decision.
(c) Spreadsheet for part (b): Use PMT functions at different PD103 values.

490G: = –PMT(10%,2,–250000,25000)–3000 Display: $–135,143


PD103 = –300,000: = –PMT(10%,3,–300000,30000)–4000 Display: $–115,571
PD103 = –500,000: = –PMT(10%,3,–500000,50000)–4000 Display: $–189,952
Again, cheapest model with P D103 = $–300,000 will change the decision to D103

5.22 Blue Whale Moving and Storage recently purchased a warehouse building in
Santiago. The manager has two good options for moving pallets of stored goods in
and around the facility. Alternative 1 includes a 4000-pound capacity, electric
forklift (P = $−30,000; n = 12 years; AOC = $−1000 per year; S = $8000), and 500
new pallets at $10 each. The forklift operator’s annual salary and indirect benefits
are estimated at $82,000.
Alternative 2 uses two electric pallet movers (“walkies”) each with a 3000-
pound capacity (for each mover, P = $−2,000; n = 4 years; AOC = $−150 per year;
no salvage) and 800 pallets at $10 each. The two operators’ salaries and benefits
will total $105,000 per year. For both options, new pallets are purchased now and
every two years that the equipment is in use.
a. If the MARR is 8% per year, use tabulated factors to determine which alternative
is better economically.
b. Develop your own spreadsheet to select the forklift or the walkies.
c. Use your spreadsheet to determine the maximum first cost of the forklift before
the walkies become economically better. Comment on your results.
(a) Factors:
AWforklift = CR – AOC – salary – AW of pallets
= –30,000(A/P,8%,12) + 8000(A/F,8%,12) – 1000 – 82,000
–500(10)[1+(P/F,8%,2) + (P/F,8%,4)+(P/F,8%,6) + (P/F,8%,8)
+ (P/F,8%,10)](A/P,8%,12)
= –30,000(0.13270) + 8000(0.05270) – 83.000
–5000[1+ (0.8573) + (0.7350) + (0.6302) + (0.5403)
+ (0.4632)](0.13270)
= $– 89,363
AWwalkies = – (2)2,000(A/P,8%,4) – 2(150) – 105,000
– 800(10)[1+(P/F,8%,2)](A/P,8%,4)
= – 4000(0.30192) – 105,300 – 8000[1+(0.0.8573)](0.30192)
= $–110,994
Select forklift (Alternative #1)

Copyright 2021 © McGraw-Hill Education. All rights reserved. No reproduction or distribution without the
prior written consent of McGraw-Hill Education.
(b) Spreadsheet: Displays the same AW values to select the forklift.

(c) Use GOAL SEEK to determine the changing cell (F4) value when AWForklift =
AWWalkies = $–110,994 in cell G18. Result is $–193,012.

Comment: It is a huge range for the forklift first cost from $30,000 to $193,012
before the walkies are a better economic choice.

5.23 The State of Chiapas, Mexico, decided to fund a program for literacy. The first cost
of $200,000 now and an update budget of $100,000 every 7 years forever is
requested. Determine the perpetual equivalent annual cost at an interest rate of 10%
per year.

AW = –200,000(0.10) – 100,000(A/F,10%,7)
= –20,000 – 100,000(0.10541)
= $–30,541 per year

Copyright 2021 © McGraw-Hill Education. All rights reserved. No reproduction or distribution without the
prior written consent of McGraw-Hill Education.
5.24 Assume you won a worldwide lottery that pays $5 million in year 0, $5 million in
year 1, and $200,000 in years 5 through 100. Assuming that 100 years is as “long”
as infinity, calculate the perpetual equivalent annual worth for years 1 through
infinity at an interest rate of 10% per year.

AW = 5M(0.10) + 5M(P/F,10%,1)(0.10) + [(200,000/0.10)(P/F,10%,4)](0.10)


= 5M(0.10) + 5M(0.9091)(0.10) + [(200,000/0.10)(0.6830)](0.10)
= $1,091,150 per year

5.25 Cisco, Inc. has a proposal from the Engineering Planning Division to invest Cisco
retained earnings in the design, testing, and development of the next generation of
smart grids useful in the Internet of Things (IoT) environment. The initial
investment projection is $5,000,000 in year 0, $2,000,000 in year 10, and $100,000
in years 11 and beyond. At i = 10% per year, calculate the infinite-life equivalent
annual cost of the proposal in years 0 through infinity.

Find PW in year -1; multiply by i

PW–1 = – 5,000,000(P/F,10%,1) – 2,000,000(P/F,10%,11)


– (100,000/0.10)(P/F,10%,11]
= – 5,000,000(0.9091) – 2,000,000(0.3505) – 1,000,000(0.3505)
= $–5,597,000

AW = PW(i)
= –5,597,000(0.10)
= $–559,700 per year

5.26 Compare two alternatives for a physical security system surrounding a power
distribution substation using annual worth analysis and a MARR of 10% per year.

System Condi Torro


First cost, $ −25,000 −130,000
M&O cost, $ per year −9,000 −2,500
Salvage value, $ 3,000 100,000
Life, years 3 ∞

AWCondi = –25,000(A/P,10%,3) – 9000 + 3000(A/F,10%,3)


= –25,000(0.40211) – 9000 + 3000(0.30211)
= $–18,146 per year

AWTorro = –130,000(0.10) – 2500


= $–15,500 per year

Select the Torro system

Copyright 2021 © McGraw-Hill Education. All rights reserved. No reproduction or distribution without the
prior written consent of McGraw-Hill Education.
5.27 A new bridge across the Allegheny River in Pittsburgh is expected to be permanent
and will have an initial cost of $30 million. This bridge must be resurfaced every
five years at a cost of $1 million. The annual inspection and operating costs are
estimated to be $50,000. Determine its equivalent annual cost of this long-life
construction at i = 10% per year.

First cost in year 0 is nonrecurring; multiply by 10%. All other costs are recurring;
find AW directly.

AW = –30,000,000(0.10) – 50,000 – 1,000,000(A/F,10%,5)


= –30,000,000(0.10) – 50,000 – 1,000,000(0.16380)
= $–3,213,800

5.28 A major repair on the transmission of Judy’s 3-year old car cost her $2000 because
the warranty expired after 2 years of ownership. Based on this experience, she will
plan on additional $2000 expenses every 3 years henceforth. Also, she spends $800
every 2 years for regular maintenance. This is in years 2, 4, 6, 8, and 10 when she
plans to donate the car to charity. Use these costs to determine the equivalent
annual amount for years 1 through infinity that Judy should put aside. Assume cars
she owns in the future will have the same cost pattern and let i = 5% per year. Solve
using (a) tabulated factors, and (b) a spreadsheet.

(a) Factors: Find equivalent AW over one life cycle of the two recurring series
($–800 and $–2000). These will continue forever.

AW800 = {–800[(P/F,5%,2)+(P/F,5%,4)+(P/F,5%,6)+(P/F,5%,8)
+(P/F,5%,10)]}(A/P,5%,10)
= {–800[(0.9070)+(0.8227)+(0.7462)+(0.6768)+(0.6139)]}(0.12950)
= $–390

AW2000 = {–2000[(P/F,5%,3)+(P/F,5%,6)+(P/F,5%,9)]}(A/P,5%,10)
= {–2000[(0.8638)+(0.7462)+(0.6446)]}(0.12950)
= $–584

Total AW = –390 – 584


= $–974 per year

Copyright 2021 © McGraw-Hill Education. All rights reserved. No reproduction or distribution without the
prior written consent of McGraw-Hill Education.
(b) Spreadsheet solution: Enter costs into cells with zeros in non-cost years. Use
PMT function to find AW over 10 years.

5.29 A West Virginia coal mining operation has installed an in-shaft monitoring system
for oxygen tank and gear readiness for emergencies. Based on maintenance patterns
for previous systems, costs are minimal for the first few years, then they increase
for a time period, and then level off. Maintenance costs are expected to be $150,000
in year 3, $175,000 in year 4, and amounts increasing by $25,000 per year through
year 6 and remain constant thereafter for the expected 10-year life of this system. If
similar systems will replace the current one, determine the perpetual equivalent
annual maintenance cost at i = 10% per year. (a) Solve using tabulated factors.
(b) (Spreadsheet exercises) Solve using a spreadsheet, and (c) write a single-cell
PMT function to find AW using the cell entries.

(a) Factors: Perpetual AW is equal to AW over one life cycle.

AW = {[–150,000(P/A,10%,4) – 25,000(P/G,10%,4)](P/F,10%,2)
– [225,000(P/A,10%,4)(P/F,10%,6)]}(A/P,10%,10)
= {[–150,000(3.1699) – 25,000(4.3781)](0.8264)
– [225,000(3.1699)(0.5645)]}(0.16275)
= $–144,198

(b) Spreadsheet: AW displayed is $–144,194

(c) Single-cell PMT function is in column C above. AW is $–144,194

Copyright 2021 © McGraw-Hill Education. All rights reserved. No reproduction or distribution without the
prior written consent of McGraw-Hill Education.
5.30 For the cash flows shown, use an annual worth comparison and a MARR of 10%
per year to determine the following:
a. The alternative that is economically best.
b. (Spreadsheet exercise) The first cost required for each of the two alternatives not
selected in (a) so that all alternatives are equally acceptable.

X Y Z
First cost, $ −90,000 −400,000 −650,000
Annual cost, $ per year −40,000 −20,000 −13,000
Overhaul every 10 years, $ — — −80,000
Salvage value, $ 7,000 25,000 200,000
Life, years 3 10 ∞

(a) AWX = –90,000(A/P,10%,3) – 40,000 + 7000(A/F,10%,3)


= –90,000(0.40211) – 40,000 + 7000(0.30211)
= $–74,075

AWY = –400,000(A/P,10%,10) – 20,000 + 25,000(A/F,10%,10)


= –400,000(0.16275) – 20,000 + 25,000(0.06275)
= $–83,531

AWZ = –650,000(0.10) – 13,000 – 80,000(A/F,10%,10)


= –650,000(0.10) – 13,000 – 80,000(0.06275)
= $–83,020

Select alternative X

(b) GOAL SEEK (right figure, row 2) finds the required first costs for
Y = $–341,912 and Z = $–560,564 by setting both AW values to
AWx = $–74,076 and solving.

Copyright 2021 © McGraw-Hill Education. All rights reserved. No reproduction or distribution without the
prior written consent of McGraw-Hill Education.
5.31 The cost associated with maintaining rural highways follows a predictable pattern.
There are usually no costs for the first 3 years, but thereafter maintenance is
required for re-striping, weed control, light replacement, shoulder repairs, and
more. For one section of state highway S102, these costs are projected to be $6000
in year 3, $7000 in year 4, and amounts increasing by $1000 per year through the
highway’s expected 30-year life. Assuming it is replaced with a similar roadway,
determine the perpetual equivalent annual cost (years 1 to ∞) at an interest rate of
8% per year (a) using factors. (b) (Spreadsheet exercise) Verify your factor-based
answer on a spreadsheet in the most efficient way you know.

(a) Perpetual AW is equal to AW over one life cycle

AW = –[6000(P/A,8%,28) + 1000(P/G,8%,28)](P/F,8%,2)(A/P,8%,30)
= –[6000(11.0511) + 1000(97.5687)](0.8573)(0.08883)
= $–12,480

(b) With no function to directly accommodate gradients, list costs in cells B2


through B32 and use PMT function to verify AW. (Note: Costs through year 11
only are shown.)

5.32 At 20 years old, Josh is an avid saver. He wants to invest an equal amount each year
from age 21 to 50 (30 years) such that starting at age 65 he can make a guaranteed
annual withdrawal of $50,000 forever without touching the corpus (principal),
which will be the inheritance money for his family. He will make no deposits
during the years of age 51 through 65. At a conservative return of 6% per year for
all the years, what amount must he invest each year from age 21 through 50?

Determine the corpus needed at age 64, find the PW value 15 years earlier, then use
the A/F factor to determine the annual amount for 30 years.

AW = [50,000/0.06](P/F,6%,15)(A/F,6%,30)
= [50,000/0.06](0.4173)(0.01265)
= $4399 per year for 30 years

Copyright 2021 © McGraw-Hill Education. All rights reserved. No reproduction or distribution without the
prior written consent of McGraw-Hill Education.
5.33 A systems engineer is trying to determine which components to include in the
sidewalk design of a smart city project that will have robots that roam underground
doing routine chores like delivering mail. Design X will have a first cost of
$48,350, M&O costs of $19,500 per year, and an expected useful life of 4 years
with no salvage value. Design Y will be more flexible, so its useful life will be 6
years with M&O costs of $28,600 and a salvage value of $4000. The first cost of
Design Y such that its AW is the same as the AW for Design X, at an interest rate
of 10% per year, is closest to:
a. $26,484
b. $29,050
c. $33,394
d. $38,575

Set AW values equal; solve for PY

–48,350(A/P,10%,4) –19,500 = –PY(A/P,10%,6) – 28,600 + 4000(A/F,10%,6)


–48,350(0.31547) –19,500 = –PY(0.22961) – 28,600 + 4000(0.12961)
PY(0.22961) = 6671.41
PY = $29,055
Answer is (b)

5.34 The annual worth of costs for four independent projects is shown below. On the
basis of the results, the project(s) to select are:

Project A B C D
AW, $ 500 −2,000 −3,000 7,500

a. Only D
b. Only A
c. A and D
d. None of them

Answer is (c)

Problems 5.35 through 5.38 are based on the following cash flows and a MARR of
10% per year.

Alternative X Y Z
First cost, $ −200,000 −450,000 −800,000
Annual cost, $/year −60,000 −30,000 −10,000
Salvage value, $ 20,000 8000 150,000
Life, years 5 10 ∞

Copyright 2021 © McGraw-Hill Education. All rights reserved. No reproduction or distribution without the
prior written consent of McGraw-Hill Education.
5.35 In comparing the alternatives by the annual worth method, the annual worth of
alternative X is closest to:
a. $−109,484
b. $−116,958
c. $−121,394
d. $−129,573

AWX = –200,000(A/P,10%,5) – 60,000 + 20,000(A/F,10%,5)


= –200,000(0.26380) – 60,000 + 20,000(0.16380)
= $–109,484
Answer is (a)

5.36 The annual worth of perpetual service for alternative X is closest to:
a. $−76,724
b. $−78,000
c. $−109,484
d. $−129,573

AWPerpetual = –200,000(A/P,10.%,5) – 60,000 + 20,000(A/F,10%,5)


= –200,000(0.26380) – 60,000 + 20,000(0.16380)
= $–109,484
Answer is (c)

5.37 The annual worth of Alternative Y is closest to:


a. $−102,736
b. $−109,484
c. $−116,958
d. $−121,394

AWY = –450,000(A/P,10%,10) – 30,000 + 8,000(A/F,10%,10)


= –450,000(0.16275) – 30,000 + 8,000(0.06275)
= $–102,736
Answer is (a)

5.38 The annual worth of Alternative Z is closest to:


a. $−78,000
b. $−90,000
c. $−98,000
d. $−116,394

AWZ = –800,000(0.10) – 10,000 = $–90,000


Answer is (b)

5.39 The annual worth equivalent of a long-life project which has estimated revenues of
$50,000 now, $10,000 per year in years 1 to 15, and $20,000 per year in years 16
through infinity at 10% per year is closest to:

Copyright 2021 © McGraw-Hill Education. All rights reserved. No reproduction or distribution without the
prior written consent of McGraw-Hill Education.
a. Less than $16,900
b. $16,950
c. $17,390
d. $19,570

Find PW in year 0 and then multiply by i.

PW0 = 50,000 + 10,000(P/A,10%,15) + (20,000/0.10)(P/F,10%,15)


= 50,000 + 10,000(7.6061) + (20,000/0.10)(0.2394)
= $173,941

AW = 173,941(0.10)
= $17,394
Answer is (c)

5.40 An alumnus of West Virginia University wishes to start an endowment that will
provide scholarship money of $40,000 per year beginning in year 5 and continuing
indefinitely. The donor plans to give money now and for each of the next 2 years. If
the size of each donation is exactly the same, the amount that must be donated each
year at a return of 8% per year is closest to:
a. $190,820
b. $122,280
c. $127,460
d. $132,040

AW = [40,000/0.08](P/F,8%,2)(A/F,8%,3)
= [40,000/0.08](0.8573)(0.30803)
= $132,037
Answer is (d)

5.41 At a return of 10% per year, the amount you must deposit into your retirement
account each year for 10 years, starting now (years 0−9), if you want to withdraw
$50,000 per year forever beginning 30 years from now is closest to:
a. $4239
b. $4660
c. $4974
d. $5471

AW = [50,000/0.10](P/F,10%,20)(A/F,10%,10)
= [50,000/0.10](0.1486)(0.06275)
= $4662
Answer is (b)

Copyright 2021 © McGraw-Hill Education. All rights reserved. No reproduction or distribution without the
prior written consent of McGraw-Hill Education.
Solutions to end-of-chapter problems
Basics of Engineering Economy, 3rd edition
Leland Blank and Anthony Tarquin

Chapter 6
Rate of Return Analysis

6.1 What does a rate of return of −100% mean?

A rate of return of −100% means that the entire investment is lost.

6.2 A $10,000 loan amortized over 5 years at an interest rate of 10% per year would
require payments of $2638 to completely extinguish the loan when interest is
charged on the unrecovered balance. If interest is charged on the principal instead of
the unrecovered balance, what would be the balance after 5 years if the same $2638
payments are made each year?

Total interest paid = 10,000(0.10)(5)


= $5,000
Total due = 10,000 + 5,000
= $15,000
Total payments = 5(2638)
= $13,190
Balance = 15,000 – 13,190
= $1810

6.3 A-1 Mortgage makes loans with the interest charged on the loan principal rather than
on the unpaid balance. For a 4-year loan of $10,000 at 10% per year, what equal
annual payment would be required to complete repayment of the loan in 4 years, if
interest is charged on (a) the principal and (b) the unrecovered balance?

(a) Annual payment = [10,000/4 + 10,000(0.10)]


= $3500

(b) A = 10,000(A/P,10%,4)
= 10,000(0.31547)
= $3154.70

6.4 Spectra Scientific of Santa Clara, California, manufactures Q-switched solid-state


industrial lasers for LED substrate scribing and silicon wafer dicing. The company
got a $60 million loan, amortized over a 5-year period at 8% per year interest. What
is the amount of the unrecovered balance (a) immediately before the payment is
made at the end of year 1, and (b) immediately after the first payment?

Copyright 2021 © McGraw-Hill Education. All rights reserved. No reproduction or distribution without the
prior written consent of McGraw-Hill Education.
1
(a) Balance before payment = 60,000,000(1.08)
= $64,800,000

Annual payment = 60,000,000(A/P,8%,5)


= 60,000,000(0.25046)
= $15,027,600

Balance after payment = 64,800,000 – 15,027,600


= $49,772,400

6.5 You and three college buddies have formed a company, Right For You, Inc., that
will customize off-the-shelf marketing software locally rather than contracting with
the original development company or a large, nationally based customizer. The
target clients are bricks-and-mortar merchants and smaller e-commerce businesses.
Because of your unique approach to this type of software tailoring, a venture
capitalist group (individuals or companies that invest in innovative start-ups at low
rates and a percentage of the business for the future) has agreed to invest in your
start-up with $1 million to be recovered from business proceeds at a low interest rate
of 3% per year over a 4-year period, and a 15% share in Right For You. During the
negotiations with the venture capitalist, you did not ask about the basis of the 3%
interest. To help your partners understand the finances, determine the following:
a. The minimum revenue needed to recover the $1 million investment and earn the
3% per year during the life of the loan.
b. The difference in the equal annual payments if loan interest is charged on the
principal rather than on the unpaid balance.
c. The difference in the unpaid balance immediately after the first payment, i.e., end
of year 1, if loan interest is charged on the principal rather than on the unpaid
balance.

(a) This the capital recovery amount at 3%.

CR = 1,000,000(A/P,3%,4) = $269,030

(b) For interest on principal, interest and payment are:

Interest each year: 0.03(1,000,000) = $30,000


Payment each year: 1,000,000/4 + 30,000 = $280,000

For unrecovered balance, annual payment is:


Payment each year = 1,000,000(A/P,3%,4)
= 1,000,000(0.26903)
= $269,030

This is the same as the CR calculated above.

Copyright 2021 © McGraw-Hill Education. All rights reserved. No reproduction or distribution without the
prior written consent of McGraw-Hill Education.
2
Difference = 280,000 – 269,030
= $10,970 per year more for principal-based interest

(c) For interest on principal:

Balance after first payment = 1,000,000 + 1,000,000(0.03) – 280,000


= $750,000

For unrecovered balance:

Balance after first payment = 1,000,000 + 1,000,000(0.03) – 269,030


= $760,970

Difference = 769,970 – 750,000


= $19,970 higher on unrecovered balance-based interest

6.6 At the International Conference on Robotics and Automation, a robot was unveiled
by a team of young engineers that would help people who needed daily physical
assistance getting dressed. The team used haptics as the sensorial method for training
the robot. In order to improve its performance, the team was seeking a loan of
$1,000,000 that they would repay in 5 years. A wealthy entrepreneur offered two
options for the $1 million loan. Plan A was a 10% loan with interest charged on the
$1 million principal of the loan. Plan B was a 14% loan with interest charged on the
unpaid balance. Which plan would result in higher total interest paid and by how
much?

For interest on principal:

Interest each year is 0.10(1,000,000) = $100,000


Total interest paid = 100,000(5) = $500,000

For unrecovered balance:

Payment each year = 1,000,000(A/P,14%,5)


= 1,000,000(0.29128)
= $291,280

Total interest paid = 291,280(5) – 1,000,000


= $456,400

Difference = 500,000 – 456,400


= $43,600 more interest on 10% principal-based loan

Copyright 2021 © McGraw-Hill Education. All rights reserved. No reproduction or distribution without the
prior written consent of McGraw-Hill Education.
3
6.7 A powerful set of “glasses,” called eSight, can give sight to the legally blind by
using high-definition video, magnification, contrast, and propriety algorithms to
enhance imagery into something they can see. The cost of the device is $9995. If the
company that makes the device spent $1.9 million to develop it, what rate of return
will the company make over a 5-year planning period if it sells 200 devices each
year at a profit of $2500 per device?

Factors: 2500(200) = 1,900,000(A/P,i*,5)


(A/P,i*,5) = 0.26316

From tables, i* is between 9% and 10%; close to 10% per year

Spreadsheet: Using = RATE(5,2500*200,−1900000) display an i* of 9.91% per year

6.8 (a) Use tabulated factors and a spreadsheet function to determine the interest rate per
period for the following rate of return equation. (b) Is it possible to use the RATE
function to find i*? Why?

0 = −35,000 + 9000(P∕A,i*,5) + 8000(P∕F,i*,8).

(a) Factors: PW relation 0 = − 35,000 + 9000(P/A,i*,5) + 8000(P/F,i*,8)

Try 12%: − 35,000 + 9000(3.6048) + 8000(0.4039) = $674.40 > 0 too low


Try 14%: − 35,000 + 9000(3.4331) + 8000(0.3506) = $−1,297.30 < 0 too high

Interpolation yields i* = 12.68% per period

Spreadsheet: Enter cash flows in cells B1:B9, with zeros in B7 and B8


Function = IRR(B1:B9) display an i* of 12.66%

(b) No, because the $8000 future value is in year 8, not year 5. This does not fit the
format required for the RATE function. Must use the IRR function.

6.9 Xavier looked at his company’s cash flows for the recent past on a newly offered
home delivery service of pet food and supplies. He wants to know the rate of return
per month if $149,333 was invested in a project with monthly costs of $28,500 and
income of $34,500 over a 2½-year period. (a) Determine the monthly i* for him. (b)
Determine his effective annual rate if monthly compounding is assumed.

(a) 0 = −149,333 + (34,500 – 28,500)(P/A,i*,30)


(P/A,i*,30) = 24.8888
i* = 1.25% per month

(b) Nominal per year = 1.25(12) = 15%


Using Equation [3.2] or Table 3.3

Effective i* = (1 + 0.15/12)12 – 1 = 0.1608 (16.08% per year)

Copyright 2021 © McGraw-Hill Education. All rights reserved. No reproduction or distribution without the
prior written consent of McGraw-Hill Education.
4
6.10 Determine the rate of return per year for the cash flows shown below. Use (a)
tabulated factors, and (b) a spreadsheet.

Year 0 1 2 3
Cash Flow, $ −125,000 −7,500 84,000 78,000

(a) Factors: Move all cash flows to year 0. In $1000 units,

0 = −125 – 7.5(P/F,i*,1) + 84(P/F,i*,2) + 78(P/F,i*,3)

Try 8%: −125 – 7.5(0.9259) + 84(0.8573) + 78(0.7938) = $1985.35 > 0 too low
Try 9%: −125 – 7.5(0.9174) + 84(0.8417) + 78(0.7722) = $−946.10 < 0 too high

Interpolation yields i* = 8.68% per year

(b) Spreadsheet: Enter cash flows for years 0 to 3 in cells B1:B4

Function: = IRR(B1:B4) displays an i* of 8.67%

6.11 Chemco Enterprises is the manufacturer of Ultra-Dry, a hydrophobic coating that


will waterproof anything. Over a 5-year period, the costs associated with the pilot
test product line were as follows: first cost of $30,000 and annual costs of $18,000.
Annual revenue was $27,000 and used equipment was salvaged for $4000. What
rate of return did the company make on this product?

0 = −30,000 + (27,000 – 18,000)(P/A,i%,5) + 4000(P/F,i%,5)

By trial and error in factor tables, interpolate between 16% and 20%.
By the function = RATE(5,27−18,−30,4), i* = 17.85 %

6.12 A graduate of NMSU who started a successful business wanted to start an


endowment in her name that would provide scholarships to students with
entrepreneurial interests. She wanted the scholarships to amount to $10,000 per
year and she wanted the first one to be given on the day she made the donation. If
she planned to donate $100,000, what rate of return will the university have to make
to award the $10,000 per year scholarships forever?

(100,000 – 10,000)i* = 10,000


i* = 11.1%

6.13 PPG manufactures an epoxy amine that is used to protect the contents of
polyethylene terephthalate (PET) containers from reacting with oxygen. The cash
flows (in $ millions) associated with the process are shown below. Determine the
rate of return using (a) tabulated factors, and (b) a spreadsheet. (c) (Spreadsheet
exercise) Demonstrate the use of GOAL SEEK to find i* if the IRR function were
not available.

Copyright 2021 © McGraw-Hill Education. All rights reserved. No reproduction or distribution without the
prior written consent of McGraw-Hill Education.
5
Year Cost, $ Revenue, $
0 −10 —
1 −4 2
2 −4 3
3 −4 9
4 −3 9
5 −3 9
6 −3 9

(a) Factors: PW relation


0 = −10 – 4(P/A,i*,3) − 3(P/A,i*,3)(P/F,i*,3) + 2(P/F,i*,1)
+ 3(P/F,i*,2) + 9(P/A,i*,4)(P/F,i*,2)

Try 14%: $0.25 > 0 too low


Try 16%: $−0.63 < 0 too high

By interpolation, i* = 14.57%

(b) Spreadsheet: Using = IRR(B2:B8) the display is an i* of 14.55 %

(c) Develop NPV function with trail i* in cell C12. Use GOAL SEEK to change C12
such that cell D12 is 0.00. Display is an i* of 14.55%.

6.14 A 473-foot, 7000-ton World War II troop carrier (once commissioned as the SS
Excambion) was sunk in the Gulf of Mexico to serve as an underwater habitat and
diving destination. The project took 10 years of planning and cost $4 million, which
was spent equally at $400,000 in years 1 through 10. Fishing and recreation
activities, estimated at $270,000 per year, will begin in year 11 and are expected to
continue in perpetuity. Determine the rate of return on the venture using
(a) tabulated factors, and (b) the GOAL SEEK tool.

(a) Tabulated factors; Move all amounts to year 10.

0 = −400,000(F/A,i*,10) + 270,000/i*

Copyright 2021 © McGraw-Hill Education. All rights reserved. No reproduction or distribution without the
prior written consent of McGraw-Hill Education.
6
Try 5%: −400,000(12.5779) + 270,000/0.05 = $368,840 > 0 too low
Try 6%: −400,000(13.1808) + 270,000/0.06 = $−772,320 < 0 too high
Interpolation yields i* = 5.32%
(b) Solve for i* by spreadsheet using the FV function and GOAL SEEK. For
example, place a guess i* in cell B1and set up the single-cell function. GOAL
SEEK forces the FV function to 0.00 by changing cell B1.
= −FV($B$1,10,−400000) + 270000/$B$1
Display is i* = 5.29% per year

6.15 Steel cable barriers in highway medians are a low-cost way to improve traffic safety
without busting state department of transportation budgets. Cable barriers cost
$44,000 per mile, compared with $72,000 per mile for guardrail and $419,000 per
mile for concrete barriers. Furthermore, cable barriers tend to snag tractor-trailer
rigs, keeping them from ricocheting back into same-direction traffic. The state of
Ohio spent $4.97 million installing 113 miles of cable barriers.
a. If the cables prevent accidents totaling $1.3 million per year, determine the rate
of return that this represents over a 10-year study period. Use tabulated factors, a
calculator, and a spreadsheet.
b. Determine the rate of return for 113 miles of guardrail if accident prevention is
$1.1 million per year over a 10-year study period. To do so, first write the ROR
relation and then find i* using a single-cell spreadsheet function.
(a) All monetary values in $ million units
Factors: 0 = −4.97 + 1.3(P/A,i*,10)
(P/A,i*,10) = 3.8231
Interpolation between 22% and 24% yields i* = 22.83%
Calculator: Function i(10,1.3,−4.97,0) displays an i* of 22.80%
Spreadsheet: Function = RATE(10,1.3,−4.97) displays an i* of 22.80%
(b) Cost of guardrail = 72,000(113) = $8.136 million
ROR relation: 0 = −8.136 + 1.1(P/A,i*,10)
(P/A,i*,10) = 7.39636
Spreadsheet function = RATE(10,1100000,−72000*113) displays an i* of 5.9%

6.16 Techstreet.com is a small web design business that provides services for two main
types of websites: brochure sites and e-commerce sites. One package involves an
up-front payment of $90,000 and monthly payments of 1.4¢ per “hit.” Kathy Cutler
has a new eBay franchise and is considering the e-commerce package. She expects
to have at least 6000 hits per month, and hopes that 1.5% of the hits will result in a

Copyright 2021 © McGraw-Hill Education. All rights reserved. No reproduction or distribution without the
prior written consent of McGraw-Hill Education.
7
sale. If the average income from sales (after fees and expenses) is $150, what rate
of return per month will Kathy realize if she uses the website for 2 years?

0 = −90,000(A/P,i*,24) – 0.014(6000) + 0.015(6000)(150)


0 = −90,000(A/P,i*,24) + 13,416
(A/P,i*,24) = 0.15093

Factors: Solve by trial and error; interpolate between, say, 12% and 15%

Spreadsheet: = RATE(24,13416,−90000) displays an i* of 14.30% per month

6.17 On the advice of your uncle, you purchased 10 shares of a well-established U.S.-
based corporate stock for $20.50 per share. After 1 quarter, you received $0.25 per
share dividends each quarter for 2 years. At that point, the stock price had gone
down in a short-term recession, so you purchased 10 more shares at $14.00 per
share. The stock continued to pay 25¢ per share on all 20 shares. After 3 years (12
quarters) you decided to sell the stock since it had increased in market value to
$22.00 per share. Make the following assumptions: (a) no commissions for the
purchase or sale of the stock, (b) no government taxes on the dividends, and (c)
quarterly compounding of the rate of return. Did you realize the anticipated 7% per
year that the stock market historically returns?

Enter cash flows for purchases at $20.50(10) in quarter 0, and $14.00(10) in quarter
8. Dividends are entered each quarter at $0.25 per share. Sale in quarter 12 is
$22.00(20). Use the IRR function to display i* per quarter and EFFECT function to
display the effective i* per year as 16.58%.

Conclusion: You made well above the historic 7% per year return.

Copyright 2021 © McGraw-Hill Education. All rights reserved. No reproduction or distribution without the
prior written consent of McGraw-Hill Education.
8
6.18 EV Box is a manufacturer of electric vehicle charging stations and charging
software. The initial cost of one part of their manufacturing process was $130,000
with annual costs of $49,000. Revenues were $78,000 in year 1, increasing by
$1000 per year. A salvage value of $23,000 was realized when the process was
discontinued after 8 years. Using a spreadsheet (or factors, as instructed) determine
the rate of return the company made on the process.

0 = −130,000 + (78,000 – 49,000)(P/A,i*,8) + 1000(P/G,i*,8) + 23,000(P/F,i*,8)

Factors: Try at 15%: $20,132 > 0 too low


Try at 20%: $−3,490 < 0 too high

Interpolation results in i* ≈ 19.3%

Spreadsheet: Use the IRR function to display an i* of 19.17%

6.19 When evaluating two alternatives by the rate of return method, if alternative A has a
rate of return of 10% and alternative B has a rate of return of 18%, what is known
about the rate of return on the increment between A and B if the investment
required in B is (a) larger than that required for A, and (b) smaller than that
required for A?

(a) The rate of return on the increment has to be larger than 18%.
(b) The rate of return on the increment has to be smaller than 10%.

6.20 Determine the overall rate of return on a $150,000 investment that returns 15% on
the first $50,000 and 25% on the remaining $100,000.

Overall ROR = [50,000(0.15) + 100,000(0.25)]/150,000


= 1/3(0.15) + 2/3(0.25)
= 0.2167 (21.67%)

6.21 For each of the following scenarios, state whether an incremental investment
analysis is required to select alternative X or Y, and state why or why not. Assume
that alternative Y requires a higher initial investment than X and that the MARR is
19% per year.
a. X has an ROR of 28% per year and Y has an ROR of 19% per year.
b. X has an ROR of 18% per year and Y has an ROR of 23% per year.
c. X has an ROR of 16% per year and Y has an ROR of 19% per year.
d. X has an ROR of 30% per year and Y has an ROR of 23% per year.
e. X has an ROR of 21% per year and Y has an ROR of 22% per year.
f. X has an ROR of 18% per year and Y has an ROR of 17% per year.

Copyright 2021 © McGraw-Hill Education. All rights reserved. No reproduction or distribution without the
prior written consent of McGraw-Hill Education.
9
Answers are in table below.

Part Incremental analysis Reason


(a) No Select X; ∆i* < 19%
(b) No Select Y; ∆i* > MARR for Y only
(c) No Select Y; only i* for Y > MARR
(d) Yes Must determine if ∆i* ≥ MARR
(e) No Select Y; ∆i* > 22%
(f) No Neither i* ≥ MARR

6.22 Assume you are the CIO (chief investment officer) for Dragon Industries, LLP
(LLP stands for Limited Liability Partnership). There is $1,000,000 set aside in a
reserve fund to purchase new equipment. If $300,000 is invested at 30%, $200,000
at 25%, and the remaining $500,000 at 20% per year, what is the overall rate of
return on the entire $1,000,000?

Overall ROR = [300,000(0.30) + 200,000(0.25) + 500,000(0.20)]/1,000,000


= 0.24 (24%)

6.23 A total of $50,000 was invested in projects to reduce insider theft in an appliance
warehouse. The two projects, identified as Y and Z, were implemented for 1 year
each. Based on the significantly reduced losses, the overall rate of return on the
$50,000 was determined to be 40%, with the ROR on the $20,000 invested in Y at
15%. (a) What was the size of the investment in Z, and (b) what was the rate of
return on Z?

(a) Size of investment in Z = 50,000 – 20,000


= $30,000

(b) 30,000(RORZ) + 20,000(0.15) = 50,000(0.40)


RORZ = 0.567 (56.7%)

6.24 For the alternatives shown, determine the sum of the incremental cash flows for
Q − P for the LCM of years.

Alternative
P Q
First cost, $ −50,000 −85,000
AOC, $ per year −8,600 −2,000
Annual revenue, $ per year 22,000 45,000
Salvage value, $ 3,000 8,000
Life, years 3 6

Copyright 2021 © McGraw-Hill Education. All rights reserved. No reproduction or distribution without the
prior written consent of McGraw-Hill Education.
10
Year Alternative Q Alternative P Q−P
0 −85,000 −50,000 −35,000
1 43,000 13,400 29,600
2 43,000 13,400 29,600
3 43,000 13,400−50,000+3000 76,600
4 43,000 13,400 29,600
5 43,000 13,400 29,600
6 43,000+8,000 13,400+3,000 34,600
Sum = +194,600

6.25 Two options are available for setting up a wireless meter scanner and controller. A
simple setup is good for 2 years and has an initial cost of $12,000, no salvage value,
and an AOC of $27,000 per year. A more permanent system has a higher first cost
of $73,000, but it has an estimated life of 6 years and a salvage value of $15,000. It
costs only $14,000 per year to operate and maintain. If the two options are
compared using an incremental rate of return, what are the incremental cash flows
in (a) year 0, (b) year 2, and (c) year 6?

(a) Year 0: Incremental CF0 = −73,000 − (−12,000)


= $−61,000

(b) Year 2: Incremental AOC = −14,000 − (−27,000) = $13,000


Incremental re-purchase cost = 0 − (−12,000) = 12,000

Incremental CF2 = 13,000 + 12,000


= $25,000

(c) Year 6: Incremental salvage = 15,000 − 0 = $15,000

Incremental AOC = −14,000 − (−27,000) = $13,000

Incremental CF6 = 15,000 + 13,000


= $28,000

6.26 The tabulation of the incremental cash flows between alternatives A and B is
shown. Alternative A has a 3-year life and alternative B a 6-year life. If neither
alternative has a salvage value, what is the (a) first cost of alternative A and (b) first
cost of alternative B?

Copyright 2021 © McGraw-Hill Education. All rights reserved. No reproduction or distribution without the
prior written consent of McGraw-Hill Education.
11
Incremental Cash Flow
Year (B − A), $

0 −20,000
1 5,000
2 5,000
3 12,000
4 5,000
5 5,000
6 5,000

(a) Year 3 incremental CF represents the first cost of A plus the incremental
annual costs of B and A. Let PA be the first cost of A.

First cost of A: 5000 + (0 − PA) = 12,000


PA = $−7000

(b) First cost of B: −20,000 = PB − (−7000)


PB = $−27,000

6.27 Specialty Gases & Chemicals manufactures nitrogen trifluoride, a highly


specialized gas used as an industrial cleansing agent for flat panels installed in
laptop computers, televisions, and desktop monitors. The incremental cash flow
associated with two alternatives for chemical storage and handling systems
(identified as P3 and X3) has been calculated in $1000 units. (a) Determine the rate
of return on the incremental cash flows. (b) If the MARR is 25% per year and X3
requires the larger initial investment, determine if it is justified. Select X3 or P3.
Solve by spreadsheet or factors, as instructed.

Incremental Cash Flow


Year (X3 – P3), $1000
0 –4600
1–9 1100
10 2000

(a) Factors: 0 = −4600 + 1100(P/A,∆i*,9) +2000(P/F,∆i*,10)

By trial and error, ∆i* = 20.9% per year

Spreadsheet: = RATE(10,1100,−4600,900) displays ∆i* = 20.96%

Copyright 2021 © McGraw-Hill Education. All rights reserved. No reproduction or distribution without the
prior written consent of McGraw-Hill Education.
12
(b) Since ∆i* = 20.9% < MARR = 25%, the extra investment in X3 is not justified.
Select alternative P3.

6.28 As groundwater wells age, they sometimes begin to pump sand (and they become
known as “sanders”) and this can cause damage to downstream desalting
equipment. This situation can be dealt with by drilling a new well at a cost of
$1,000,000 or by installing a tank and self-cleaning screen ahead of the desalting
equipment. The tank and screen will cost $230,000 to install and $61,000 per year
to operate and maintain. A new well will have a pump that is more efficient than the
old one and it will require almost no maintenance, so its operating cost will be only
$18,000 per year. The salvage values are estimated to be 10% of the first cost.
Using a present worth ROR relation, a MARR of 6% per year, and a 20-year study
period (a) calculate the incremental rate of return, and (b) determine which
alternative is economically better. (c) (Spreadsheet exercise) Verify your selection
using single-cell spreadsheet functions for an AW analysis of each alternative
separately.

(a) Factors: 0 = −770,000 + 43,000(P/A,∆i*,20) + 77,000(P/F,∆i*,20)

By trial and error, ∆i* = 1.8% per year

Spreadsheet: = RATE(20,43000,−770000,77000) displays a ∆i* of 1.81%

(b) Install the tank and screen, since 1.8% < MARR = 6%

(c) New well: = − PMT(6%,20,−PV(6%,20,−18000,100000) − 1000000)


AW = $−102,466

Tank and screen: = − PMT(6%,20,−PV(6%,20,−61000,23000) − 230000)


AW = $−80,427

Select tank and screen with a smaller AW of costs.

6.29 Harold and Mavone plan to purchase furniture, appliances, some heirloom artifacts,
as well as new woodworking and pottery-making equipment to furnish a renovated
heritage home in Brazos de Dios, Texas, that they have recently purchased. The
hobby equipment is a questionable purchase economically, since the couple plans to
sell their artifacts online for a secondary retirement income. Estimates have been
developed using two vendors for hobby enthusiasts that provide equipment and
marketing services on contract. Note that the vendors’ contract periods vary. If the
hoped-for MARR is 20% per year, determine which vendor, or neither, should be
selected using an incremental ROR analysis. Solve using spreadsheet functions.

Copyright 2021 © McGraw-Hill Education. All rights reserved. No reproduction or distribution without the
prior written consent of McGraw-Hill Education.
13
Vendor
Hobby-Tru (H) Knack’s (K)

Initial cost, $ −8,000 −20,500


M&O costs, $/year −10,000 −8,500
Revenue, $/year 13,000   15,500
Resale value, $ 500    1,000
Contract life, years 3 6

Add DN alternative since these are revenue alternatives; determine i* for H and K.
From IRR function, i*H = 8.9% < MARR = 20%. Eliminate H (Hobby-Tru).

Select K (Knack’s) since i*K = 25.9% > MARR and K has a higher initial cost.

If i*H is not determined initially, the incremental cash flows are determined and the
IRR function displays ∆i* = 35.7%; select K.

6.30 The Texas Department of Transportation (TxDOT) is considering two designs for
crash barriers along a reconstructed portion of I-10. Design 2B will cost $3 million
to install and $135,000 per year to maintain. Design 4R will cost $3.7 million to
install and $70,000 per year to maintain. Determine which design should be
selected based on a rate of return analysis if TxDOT uses a MARR of 6% per year
and a 20-year project period.

0 = −700,000 + 65,000(P/A,∆i*,20)
(P/A,∆i*,20) = 10.7692

Factor: Solve for ∆i* using tables at n = 20 and interpolate; ∆i* = 6.8% per year

Spreadsheet: = RATE(20,65000,−700000) yields ∆i* = 6.79% per year

∆i* > MARR of 6% per year; select design 4R, the more expensive one.

Copyright 2021 © McGraw-Hill Education. All rights reserved. No reproduction or distribution without the
prior written consent of McGraw-Hill Education.
14
6.31 The manager of Liquid Sleeve, Inc., a company that makes a sealing solution for
machine shaft surfaces that have been compromised by abrasion, high pressures, or
inadequate lubrication, is considering adding Al or Fe nanoparticles to its solution
to increase the product’s performance at high temperatures. The costs associated
with each are shown below. The company’s MARR is 20% per year.
a. Determine which nanoparticle type the company should select using an
incremental rate of return analysis.
b. Solve part (a) using a spreadsheet and, on the same worksheet, plot the PW
versus different i values for each alternative. Indicate the breakeven i* value and
the MARR value on the plot.
c. Plot PW versus Δi values and use it to select the better alternative with
MARR = 20% per year. Is the answer the same as in part (a)?

Type Fe Type Al
First cost, $ −150,000 −280,000
Annual operating cost, $ per year −92,000 −74,000
Salvage value, $   30,000   70,000
Life, years 2 4

(a) Develop cash flow series to get incremental cash flows.

Incremental
Cash flows, $1000 cash flow, $1000
Year Type Fe Type Al (Al − Fe)___
0 −150 −280 −130
1 −92 −74 18
2 −92 + 30 − 150 −74 138
  3 −92 −74 18
4 −92 + 30 −74 + 70 58

Factor: Solve by trial and error between i values of, say, 25% and 30%;
interpolate to get ∆i* = 27.3% per year.

Spreadsheet: Enter incremental cash flows; IRR function displays ∆i* = 27.35%

Since 27.3% > MARR = 20%; select type Al

(b) and (c) plots are developed using i and ∆i values. Decision: select Al.

Copyright 2021 © McGraw-Hill Education. All rights reserved. No reproduction or distribution without the
prior written consent of McGraw-Hill Education.
15
6.32 A biotech company is considering two processes for isolating DNA material. The
incremental cash flows between the two alternatives, J and S, have an incremental
rate of return that is less than 40%, which is the MARR of the company. However,
the company CEO prefers the more expensive process S. She believes the company
can implement cost controls to reduce the annual operating cost of the more
expensive process, alternative S. How much of a reduction in AOC in $ per year is
necessary for Δi* to equal the MARR? Solve using factors first, then a single-cell
function coupled with the GOAL SEEK tool.

Incremental Cash Flow


Year (S – J), $

0 –900,000
1 400,000
2 400,000
3 400,000

Factor: 0 = −900,000 + AOC(P/A,40%,3)


= −900,000 + AOC(1.5889)
AOC = $566,430

Copyright 2021 © McGraw-Hill Education. All rights reserved. No reproduction or distribution without the
prior written consent of McGraw-Hill Education.
16
Required reduction = 566,430 – 400,000
= $166,430 per year

Spreadsheet: Use a PV function and GOAL SEEK to change the cell with
the current ∆AOC = $400,000 while forcing the PV function
value to equal zero. Result is $566,422.

Required reduction = 566,422 – 400,000 = $166,422 per year

6.33 As project manager, you received from the purchasing department a listing of
estimated annual cash flow differences between two electrode setups you want to
install. The MARR is 12% per year and alternative Dryloc requires the larger initial
investment compared to NPT. Only one can be selected.
a. Determine which should be selected using an AW-based ROR analysis.
b. Use a graph of the incremental values to determine the largest MARR value that
justifies the NPT alternative.

Incremental Cash Flow


Year (Dryloc – NPT), $

0 –56,000
1–8 8,900
9 12,000

(a) 0 = −56,000(A/P,∆i*,9) + 8900 + (12,000 – 8900)(A/F,∆i*,9)

Solve for ∆i* by trial and error or spreadsheet.

Spreadsheet: = RATE(9,8900,−56000,3100) displays a ∆i* of 8.48%, which is


less than MARR = 12%

Select Dryloc

(b) By graph, to select NPT the maximum MARR ≈ 8%. (Actually, it is 8.48%,
as determined by the RATE function.)

Any MARR > 8.48% indicates selection of Dryloc.

Copyright 2021 © McGraw-Hill Education. All rights reserved. No reproduction or distribution without the
prior written consent of McGraw-Hill Education.
17
6.34 Poly-Chem Plastics is considering two types of injection molding machines:
hydraulic and electric. The hydraulic press (HP) will have a first cost of $600,000,
annual costs of $200,000, and a salvage value of $70,000 after 5 years. Electric
machine technology (EMT) will have a first cost of $800,000, annual costs of
$150,000, and a salvage value of $130,000 after 5 years.
a. Use an AW-based equation to determine the ROR on the extra investment
required for the EMT alternative.
b. Determine the machine to select, if the MARR is 16% per year.
c. Plot the AW versus i graph for each alternative’s cash flows and utilize it to
determine the largest MARR which will justify the extra investment of $200,000
in EMT.

(a) EMT has a larger initial investment than HP.

0 = −200,000(A/P,∆i*,5) + 50,000 + 60,000(A/F,∆i*,5)

Solve for ∆i* by trial and error or spreadsheet

Spreadsheet: ∆i* = 14.5%

(b) From part (a), ∆i* < MARR = 16%; select HP.

(c) Graph indicates a crossing of AW lines at ≈ 14.5%. Any MARR below this
value will justify the extra investment in EMT.

Copyright 2021 © McGraw-Hill Education. All rights reserved. No reproduction or distribution without the
prior written consent of McGraw-Hill Education.
18
6.35 A metal plating company on government contract is considering four different
methods for recovering by-product heavy metals from a processing site’s liquid
waste. The investment costs and annual net incomes associated with each method
have been estimated. All methods have an 8-year life; the MARR is 11% per year;
and an AW-based ROR analysis is required by the government agency prior to final
selection. (a) If the methods are independent, because they can be implemented at
different plants, which ones are acceptable? (b) If the methods are mutually
exclusive, select the economically best one.

First Salvage Annual Net


Method Cost, $ Value, $ Income, $/year

A −30,000 +1,000 +4,000


B −36,000 +2,000 +5,000
C −41,000 +500 +8,000
D −53,000 −2,000 +10,500

(a) Calculate i* by trial and error or RATE function; compare to MARR = 11%.
RATE is used here in all cases to determine i* values. DN is an option.

A: 0 = −30,000(A/P,i*,8) + 4000 + 1000(A/F,i*,8)


i* = 2.1%
REJECT

B: 0 = −36,000(A/P,i*,8) + 5000 + 2000(A/F,i*,8)


i* = 3.4%
REJECT

Copyright 2021 © McGraw-Hill Education. All rights reserved. No reproduction or distribution without the
prior written consent of McGraw-Hill Education.
19
C: 0 = −41,000(A/P,i*,8) + 8000 + 500(A/F,i*,8)
i* = 11.3%
ACCEPT

D: 0 = −53,000(A/P,i*,8) + 10,500 − 2000(A/F,i*,8)


i* = 11.1%
ACCEPT (marginal)

(b) Revenue alternatives; compare to DN initially.

From (a) above, eliminate A and B since i* < MARR = 11%. Determine ∆i*
between D and C.

D vs C: 0 = −12,000(A/P,∆i*,8) + 2,500 − 2500(A/F,∆i*,8)


∆i* = 10.4%
Eliminate D

Select method C

6.36 Old Southwest Canning Co. has determined that any one of four sterilizing
machines can be used in its chili-canning operation. The costs of the machines are
estimated; all machines have a 5-year useful life. (a) If the MARR is 25% per year,
determine which machine should be selected on the basis of an ROR analysis. (b)
(Spreadsheet exercises) Develop a spreadsheet similar to Figure 6.11 to select one
alternative. (c) There is a controversy about the MARR in this selection between
yourself and the capital investment officer. You want 15%, not 25%, since the
equipment’s sterilization ability selected at MARR of 25% does not have excellent
quality, as tests have shown. Use your spreadsheet to determine if the selection will
change at 15% from the previous choice. (Note: If your instructor asks for it, apply
the logical IF function to make the determination if a machine is incrementally
justified for the two MARR values. Refer to Appendix A for help on developing the
IF function.)

Machine First Cost, $ AOC, $

1 −28,000 −20,000
2 −51,000 −12,000
3 −32,000 −19,000
4 −33,000 −18,000

Copyright 2021 © McGraw-Hill Education. All rights reserved. No reproduction or distribution without the
prior written consent of McGraw-Hill Education.
20
(a) Factors: Rank cost alternatives by increasing initial investment: 1, 3, 4, 2

3 to 1: 0 = −4000 + 1000(P/A,∆i*,5)
∆i* = 7.93% < MARR = 25% Eliminate 3

4 to 1: 0 = −5000 + 2000(P/A,∆i*,5)
∆i* = 28.65% > MARR = 25% Eliminate 1

2 to 4: 0 = −18,000 + 6000(P/A,∆i*,5)
∆i* = 19.86% < MARR = 25% Eliminate 2

Select machine 4

(b) Spreadsheet (below, top table) is set up with the ordered alternatives 1, 3, 4, 2.
Costs only are involved. Select machine 4, as in the factor-based solution.

(c) Spreadsheet (below, bottom table) has the MARR of 15% entered into cell H30
and IF functions in row 30.

Selection does change to machine 2 at MARR = 15%.

6.37 Terry, an engineering technology graduate who is very entrepreneurial, wants to


start an excavation and foundation business in Orlando by investing his own
savings to fund part of the start-up. A primary decision is the size of a used dump
truck to purchase. He knows that as the bed size increases, the net income increases,
but he is uncertain whether the incremental expenditures for the larger sizes are
justified. The estimated cash flows are listed below; all trucks have a 5-year useful
life. Terry expects a return of at least 18% per year on this investment. (a)
Determine which size truck he should purchase. (b) If two trucks are to be

Copyright 2021 © McGraw-Hill Education. All rights reserved. No reproduction or distribution without the
prior written consent of McGraw-Hill Education.
21
purchased, select the size of the second truck. (c) Make both selections using a
spreadsheet. (Using single-cell functions for each incremental analysis is
acceptable.)

Annual
Truck Bed Initial M&O, Revenue, Salvage
Size, m3 Cost, $ $/year $/year Value, $

8 −30,000 −14,000 26,500 2,000


10 −34,000 −15,500 30,000 2,500
15 −38,000 −18,000 33,500 3,000
20 −48,000 −21,000 40,500 3,500
25 −57,000 −26,000 49,000 4,600

These are revenue alternatives; add DN initially and compare incrementally. All ∆i*
values can be determined by trial and error or spreadsheet function. The functions
are used here.

(a) 8 vs. DN: 0 = −30,000(A/P,∆i*,5) + (26,500 – 14,000) + 2000(A/F,∆i*,5)


∆i* = 31.7%
Eliminate DN

10 vs. 8: 0 = −4000(A/P,∆i*,5) + (14,500 – 12,500) + 500(A/F,∆i*,5)


∆i* = 42.4%
Eliminate 8

15 vs. 10: 0 = −4000(A/P,∆i*,5) + (15,500 – 14,500) + 500(A/F,∆i*,5)


∆i* = 10.9%
Eliminate 15

20 vs. 10: 0 = −14,000(A/P,∆i*,5) + (19,500 – 14,500) + 1000(A/F,∆i*,5)


∆i* = 24.2%
Eliminate 10

25 vs. 20: 0 = −9000(A/P,∆i*,5) + (23,000 – 19,500) + 1100(A/F,∆i*,5)


∆i* = 29.0%
Eliminate 20

Purchase 25 m3 bed size

(b) For second truck, purchase truck that was eliminated next to last: 20 m3

Copyright 2021 © McGraw-Hill Education. All rights reserved. No reproduction or distribution without the
prior written consent of McGraw-Hill Education.
22
(c) Spreadsheet analysis results in selection of 25 m3 truck bed size. For part (b),
purchase size that was eliminated next to last, the 20 m3 bed size.

If single-cell RATE functions are used, a solution sample follows.

6.38 You are considering five projects, all of which can be considered to last
indefinitely. If the MARR is 15% per year, determine which should be selected if
they are (a) independent projects, and (b) mutually exclusive alternatives.

First Net Annual ROR,


Alternative Cost, $ Income, $ %

A −20,000 +3,000 15.0


B −10,000 +2,000 20.0
C −15,000 +2,800 18.7
D −70,000 +10,000 14.3
E −50,000 +6,000 12.0

(a) Select all projects with a ROR ≥ MARR of 15%. Select A, B, and C

Copyright 2021 © McGraw-Hill Education. All rights reserved. No reproduction or distribution without the
prior written consent of McGraw-Hill Education.
23
(b) Eliminate all alternatives with ROR < MARR; compare others incrementally:
Eliminate D and E. Rank remaining revenue alternatives according to
increasing first cost: B, C, A. Add DN initially.

B vs DN: ∆i* = 2000/10,000


= 0.20 20% > MARR Eliminate DN

C vs B: ∆i* = 800/5000
= 0.16 16% > MARR Eliminate B

A vs C: ∆i* = 200/5000
= 0.04 4% < MARR Eliminate A

Select alternative C

6.39 The four proposals described below are being evaluated to serve as patients’ records
security systems in North American, European, and Asian hospitals operated by
AMD Health Corp.
a. If the proposals are independent, which one(s) should be selected at a MARR of
17% per year?
b. If the proposals are mutually exclusive, which one should be selected at a
MARR of 14.5% per year?
c. If the proposals are mutually exclusive, which one should be selected at a
MARR of 9% per year?

Δi*, %, When
Overall Compared with
Initial Rate of Proposal
Investment, Return,
Proposal $ % A B C

A −600,000 11.7
B −900,000 22.2 43.3
C −1,400,000 17.9 22.5 10.0
D −1,900,000 15.8 17.8 10.0 10.0

(a) Select all proposals with an overall ROR ≥ 17% per year.

Select proposals B and C

Copyright 2021 © McGraw-Hill Education. All rights reserved. No reproduction or distribution without the
prior written consent of McGraw-Hill Education.
24
(b) Compare revenue alternatives (proposals) incrementally after ranking: DN, A,
B, C, D.

A to DN: ∆i* = 11.7% < 14.5% Eliminate A


B to DN: ∆i* = 22.2% > 14.5% Eliminate DN
C to B: ∆i* = 10.0% < 14.5% Eliminate C
D to B: ∆i* = 10.0% < 14.5% Eliminate D

Select proposal B

(c) Compare proposals incrementally after ranking: DN, A, B, C, D

A to DN: ∆i* = 11.7% > 9% Eliminate DN


B to A: ∆i* = 43.3% > 9% Eliminate A
C to B: ∆i* = 10.0% > 9% Eliminate B
D to C: ∆i* = 10.0% > 9% Eliminate C

Select proposal D

6.40 Leenflow, Inc., a company that sells a mobile marketing platform that helps
businesses improve engagement with customers in a personalized way, wants to
expand its operation by adding new products. Any or all of the products shown
below can be added. If the company uses a MARR of 15% per year and a 5-year
study period, which products, if any, should the company introduce? Write a PW-
based ROR equation and a single-cell spreadsheet function for each analysis. Use
the spreadsheet display for your decisions.

1 2 3 4

Initial cost, $ −340,000 −500,000 −570,000 −620,000


Annual cost, $/year −70,000 −64,000 −48,000 −40,000
Annual revenue, $/year 180,000   190,000 220,000   205,000

Proposals are independent; compare each against DN. Monetary units in $1000.

Product 1: Relation: 0 = −340 + (180 – 70)(P/A,i*,5)


Spreadsheet: = RATE(5,180−70,−340) displays 18.52%

i* = 18.52% > MARR = 15% ACCEPT

Product 2: Relation: 0 = −500 + (190 – 64)(P/A,i*,5)


Spreadsheet: = RATE(5,190−64,−500) displays 8.23%

i* = 8.23% < MARR = 15% REJECT

Copyright 2021 © McGraw-Hill Education. All rights reserved. No reproduction or distribution without the
prior written consent of McGraw-Hill Education.
25
Product 3: Relation: 0 = −570 + (220 –48)(P/A,i*,5)
Spreadsheet: = RATE(5,220−48,−570) displays 15.49%

i* = 15.49% > MARR = 15% ACCEPT

Product 4: Relation: 0 = −620 + (205 – 40)(P/A,i*,5)


Spreadsheet: = RATE(5,205−40,−620) displays 10.35%

i* = 10.35% < MARR = 15% REJECT

Leenflow should introduce products 1 and 3.

6.41 Spectrum Imaging Systems is considering the purchase of a new printer based on
recent contracts it received for printing weekly magazines and mail-out advertising
materials. The operating costs and revenues generated are related to a large extent
to the speed and other capabilities of the printer. Spectrum uses a 3-year planning
period and a MARR of 15% per year. Which of the four printers detailed below
should the company acquire on the basis of an incremental ROR analysis?

First AOC, Revenue, Salvage


Printer Cost, $ $ per year $ per year Value, $

Nx-1 −500,000 −350,000 450,000 70,000


Nx-2 −600,000 −300,000 460,000 85,000
Nx-3 −650,000 −275,000 480,000 95,000
Nx-4 −750,000 −200,000 510,000 120,000

Revenue alternatives; add DN; calculate i*; eliminate if i* < 15%; order remaining
alternatives; determine ∆i* using trial & error or RATE function; compare to 15%.
RATE functions applied here. Monetary units are $1000.
Nx-1: = RATE(3,100,−500,70) i* = −12.60% Eliminate
Nx-2: = RATE(3,160,−600,85) i* = −2.74% Eliminate
Nx-3: = RATE(3,205,−650,95) i* = 4.25% Eliminate
Nx-4: = RATE(3,310,−750,120) i* = 17.77% Retain

With only Nx-4 remaining, make the conclusion.

∆i* =17.77% > MARR = 15% Select Nx-4

Copyright 2021 © McGraw-Hill Education. All rights reserved. No reproduction or distribution without the
prior written consent of McGraw-Hill Education.
26
6.42 Allstate Insurance Company is considering the installation of one of five fraud
detection systems, all of which can be considered to last indefinitely. If the
company’s MARR is 14% per year, determine which one should be selected on the
basis of a rate of return analysis using the estimates shown.

A B C D E

First cost, $ −10,000 −25,000 −15,000 −70,000 −50,000


Annual net income, $/year 2,000     4,000   2,900   10,000   6,000
Overall ROR, % 20.0  20.0 19.3    14.3 12.0

Order revenue alternatives: DN, A, C, B, E, D; MARR = 14%

A to DN: ∆i* = (2000/10,000)(100) = 20% > MARR Eliminate DN


C to A: ∆i* = (900/5000)(100) = 18% > MARR Eliminate A
B to C: ∆i* = (1100/10,000)(100) = 11% < MARR Eliminate B
E to C: ∆i* = (3100/35,000)(100) = 8.9% < MARR Eliminate E
D to C: ∆i* = (7100/55,000)(100) = 12.9% < MARR Eliminate D

Select alternative C

6.43 Caroline received the analysis below from an employee concerning four revenue
proposals.
a. If the proposals are independent, which one(s) should she select at MARR =
15.5% per year?
b. If the proposals are mutually exclusive, which one should she select at MARR =
10% per year?
c. If the proposals are mutually exclusive, which one should she select at MARR =
14% per year?

Δi*, When
Compared with
Proposal
Initial Overall
Proposal Investment, $ i*, % A B C

A −40,000 29
B −75,000 15 9
C −100,000 16 7 20
D −200,000 14 10 13 12

Copyright 2021 © McGraw-Hill Education. All rights reserved. No reproduction or distribution without the
prior written consent of McGraw-Hill Education.
27
(a) Select A and C with ROR > MARR = 15.5%

(b) Order proposals: DN, A, B, C, D; MARR = 10%


A to DN: ∆i* = 29% > MARR Eliminate DN
B to A: ∆i* = 9% < MARR Eliminate B
C to A: ∆i* = 7% < MARR Eliminate C
D to A: ∆i* = 10% = MARR Eliminate A

Select D, which has a ∆i* exactly equal to MARR

(c) Same as part (b), except with MARR = 14%, the last comparison changes.

D to A: ∆i* = 10% < MARR Eliminate D

Select A

6.44 According to Descartes’ rule of signs, how many possible i* values are there for net
cash flow series that have the following signs:
a. −+++−+++−
b. −−++
c. +−−−−−−+−+−−−
d. −−−−+++−−−−−+

(a) four; (b) one; (c) five; (d) three

6.45 Charles owns a home with solar panels on the roof. His utility company has a “buy-
back” program that charges homeowners for the net energy used each quarter. He
listed the quarterly charges and buy-back dollars for the 2 years he has been in the
program. (a) Use Descartes’ rule to determine the maximum number of possible i*
values. (b) Use Norstrom’s criterion to determine if there is only one positive i*
value. (c) Determine the i* value(s) for Charles.

Quarter Charges, $ Buy-back, $

1 −200 50
2 −100 100
3 −100 250
4 −100 260
5 −100 200
6 −150 170
7 −120 150
8 −150 20

Copyright 2021 © McGraw-Hill Education. All rights reserved. No reproduction or distribution without the
prior written consent of McGraw-Hill Education.
28
NCF = net cash flow; S1 = NCF1 < 0

Quarter Charges Buy-back NCF Cumulative NCF


1 −200 50 −150 −150
2 −100 100 0 −150
3 −100 250 +150 0
4 −100 260 +160 +160
5 −100 200 +100 +260
6 −150 170 +20 +280
7 −120 150 +30 +310
8 −150 20 −130 +180

(a) From net cash flows, there are two sign changes, so, two possible i* values

(b) From cumulative cash flow, sign starts negative and changes once. Norstrom’s
criterion is satisfied; therefore, there is a unique nonnegative i* value.

(c) By spreadsheet, enter the NCF values; use IRR function to determine i* =
42.3% per quarter.

6.46 A manufacturer of heavy-tow carbon fibers (used for sporting goods, thermoplastic
compounds, windmill blades, etc.) reported the net cash flows shown.
a. Determine the number of possible i* values.
b. (Spreadsheet exercises) Apply Norstrom’s criterion, then write the PW-based
ROR relation for trial-and-error determination of the i* value(s), and finally
determine all rate of return values between −50% and 120% using the IRR
function with “guess” values entered.
c. Determine the PW values at different rates of return: −50%, 0%, 120%, plus the
i* value(s) determined in part b.
d. What conclusion can you make about the i* values of this cash flow series?

Year Net Cash Flow, $

0 −17,000
1   20,000
2  −5,000
3  8,000

(a) There are three sign changes in NCF series, therefore, three possible i* values.

Copyright 2021 © McGraw-Hill Education. All rights reserved. No reproduction or distribution without the
prior written consent of McGraw-Hill Education.
29
(b) Cumulative NCF starts < 0, but has more than one sign change; no unique
i* > 0 is indicated. The PW-based ROR relation is

0 = −17,000 + 20,000(P/F,i*,1) − 5000(P/F,i*,2) + 8000(P/F,i*,3)

Spreadsheet: IRR function at different guesses displays an i* of 24.41%


for all values.

(c) PW values decrease for all ROR values from −50% to 120%.

(d) The additional two possible i* values are not in the range identified (and may
not be real numbers).

6.47 Hitec Home Services manufactures a ventilation controller designed for monitoring
and controlling carbon monoxide in parking garages, boiler rooms, tunnels, etc. The
net cash flows associated with one phase of the operation are shown below.
a. How many possible rate of return values are there for this cash flow series?
b. Find all the rate of return values between 0 and 100% using tabulated factors and
a spreadsheet.
c. (Added spreadsheet exercise) Resolve to find any i* values between −99% and
0%. Does the result agree with the conclusion of the rule of signs (Descartes’
rule) and cumulative cash flow (Norstrom’s criterion) tests?

Year Net Cash Flow, $

0 −30,000
1   20,000
2   15,000
3 −2,000

Copyright 2021 © McGraw-Hill Education. All rights reserved. No reproduction or distribution without the
prior written consent of McGraw-Hill Education.
30
(a) By cash flow rule of signs, there are two sign changes; may be two i* values.

(b) 0 = −30,000 + 20,000(P/F,i*,1) + 15,000(P/F,i*,2) − 2000(P/F,i*,3)

Factors:
Try 6%: $538.8 > 0 too low
Try 7%: $160.4 > 0 too low
Try 8%: $−210.1< 0 too high

Interpolation yields i* = 7.43%

Spreadsheet: Function = IRR(B1:B4,guess) displays an i* of 7.43% for any


guess value from 0% to 100%

(c) Cumulative NCF starts negative and has one sign change. There is one unique
positive i*. The IRR function with guesses in the range 0% to −99% finds
another i* at −88.1956%. This does agree with the two tests, as −88.1956% <
0, thus not positive.

6.48 Arc-bot Technologies, manufacturers of six-axis, electric servo-driven robots, has


experienced annual expenses and savings in its shipping department through
improved supply-chain software applications. (a) Determine the number of possible
rate of return values. (b) Find all i* values between 0 and 100%.

Year Expenses, $ Savings, $

0 −33,000 0
1 −15,000 18,000
2 −40,000 38,000
3 −20,000 55,000
4 −13,000 12,000

The NCF and cumulative NCF are shown below.

Year Expenses, $ Savings, $ NCF, $ Cumulative NCF, $


0 −33,000   0 −33,000 −33,000
1 −15,000 18,000 +3,000 −30,000
2 −40,000 38,000 −2000 −32,000
3 −20,000 55,000 +35,000 +3000
4 −13,000 12,000 −1000 +2000

(a) There are four sign changes in NCF series; four possible i* values.

Copyright 2021 © McGraw-Hill Education. All rights reserved. No reproduction or distribution without the
prior written consent of McGraw-Hill Education.
31
(b) Cumulative cash flow starts negative and changes sign only once. Therefore,
there is only one positive, real-number i* value.

0 = −33,000 + 3000(P/F,i*,1) − 2000(P/F,i*,2) + 35,000(P/F,i*,3)


−1000(P/F,i*,4)

Solve by trial and error or spreadsheet

Spreadsheet: i* = 2.1% per year

6.49 Five years ago, Logocom made a $5 million investment in a new high-temperature
material. The product was not well accepted after the first year on the market.
However, when it was reintroduced 4 years later, it did sell well during the year.
Major research funding to broaden the applications has cost $15 million in year 5.
Determine the rate of return for these net cash flows (in $1000 units).

Year NCF, $1000


0 −5,000
1  4,000
2 0
3 0
4  20,000
5 −15,000

Cash flow signs change two times, but cumulative cash flow starts negative and
changes sign only once. (St series is: −5000; −1000; −1000; −1000; +19,000; and
+4000.) There is only one positive, real-number i* value.

0 = −5000 + 4000(P/F,i*,1) + 20,000(P/F,i*,4) − 15,000(P/F,i*,5)

Solve by trial and error or spreadsheet.

Spreadsheet: i* = 44.1% per year

6.50 State a fundamental reason why the internal rate of return (IRR) and the external
rate of return (ERR) will have different values when the cash flow series is
nonconventional and multiple i* values are indicated.

In the case of the IRR, all excess funds are assumed to be retained within the project
and earn at any one of the multiple i* value. For the ERR, the external rates for
reinvestment of excess funds and borrowing in negative net cash flow years will
alter the i* value, except when the reinvestment rate is exactly equal to the i* value.

Copyright 2021 © McGraw-Hill Education. All rights reserved. No reproduction or distribution without the
prior written consent of McGraw-Hill Education.
32
6.51 Explain why it is not a good practice to set the reinvestment rate on excess funds
and the borrowing rate for negative NCF years equal to each other when the
external rate of return (ERR) is to be determined.

Setting the reinvestment and borrowing rates equal to each other implies that no
profit margin is anticipated over time. A company cannot continue in business in
such circumstances.

6.52 For the cash flow series shown (a) determine the number and type of possible i*
values using the two sign tests, (b) find the ERR using the MIRR method and
Equation [6.4] with a reinvestment rate of 20% per year and a borrowing rate of
10% per year, and (c) use the MIRR function to find the ERR.
Year 1 2 3 4 5 6
Cash Flow, $ +4100 −2000 −7000 +12,000 −700 +800

(a) Rule of signs test: up to 4 i* values


Cumulative CF sign test: inconclusive, since S0 > 0

(b) PW0 = −2000(P/F,10%,2) − 7000(P/F,10%,3) − 700(P/F,10%,5)


= −2000(0.8264) − 7000(0.7513) − 700(0.6209)
= $−7347

FW6 = 4100(F/P,20%,5) + 12,000(F/P,20%,2) + 800


= 4100(2.4883) + 12,000(1.4400) + 800
= $28,282

−7347(F/P,i′,6) + 28,282 = 0
−7347(1 + i′)6 + 28,282 = 0

ERR = i' = (28,282/7347)1/6 – 1


= 0.252 (25.2%)
(c) Enter the CF values in cells B1 (as a 0) through B7. The function
= MIRR(B1:B7,10%,20%) displays an i′ of 25.2%.

6.53 For the net cash flows shown, find the external rate of return with a reinvestment
rate of 15% per year, using (a) the manual ROIC method and (b) a spreadsheet to
verify the answer.

Year Net Cash Flow, $


0 +48,000
1 +20,000
2 −90,000
3 +50,000
4 −52,000

Copyright 2021 © McGraw-Hill Education. All rights reserved. No reproduction or distribution without the
prior written consent of McGraw-Hill Education.
33
(a) Cash flow rule of signs indicates three possible rate of return values.
Cumulative cash flow test provides no information, since cumulative cash flow
series starts with a positive value and changes sign more than once. Use
Equation [6.5] with ir = 0.15.

F0 = 48,000 > 0, use ir


F1 = 48,000(1.15) + 20,000 = 75,200 > 0, use ir
F2 = 75,200(1.15) − 90,000 = −3520 < 0, use i″
F3 = −3520(1 + i″) + 50,000 = 46,480 −3520i″ > 0 for i″ < 1320%; use ir
F4 = (46,480 – 3520i″)(1.15) −52,000 = 1,452 – 4048i″

Set F4 = 0 and solve for i″

0 = 1452 – 4048i″
i″ = 0.359 (35.9%)

(b) Develop a spreadsheet similar to Figure 6.8 to determine i″ = 35.9% in cell F8.
GOAL SEEK forces cell C6 to 0.00 while changing cell F8.

6.54 The net cash flows series shown may have multiple i* values. If the reinvestment
rate is 18% per year and the borrowing rate is 10%, find the ERR using (a) the
manual MIRR approach coupled with the RATE function, and (b) the MIRR
spreadsheet function.
Year 0 1 2 3
Net cash flow, $ +16,000 −32,000 −25,000 +70,000

(a) Follow the steps of the modified ROR procedure.

PW0 = – 32,000(P/F,10%,1) – 25,000(P/F,10%,2)


= – 32,000(0.9091) – 25,000(0.8264)
= $−49,751

FW3 = 16,000(F/P,18%,3) + 70,000


= 16,000(1.6430) + 70,000
= $96,288

Copyright 2021 © McGraw-Hill Education. All rights reserved. No reproduction or distribution without the
prior written consent of McGraw-Hill Education.
34
96,288 = 49,751(F/P,i′,3)
(F/P,i′,3) = 1.9354

The function = RATE(3,49751,−96288) yields an i′ of 24.6% per year

(b) Enter NCF values in C2 through C5. The function = MIRR(C2:C5,10%,18%)


displays an i′ of 24.6%.

6.55 For the cash flows shown, determine:


a. the number of possible i* values.
b. the i* value displayed by the IRR function.
c. the external rate of return using the MIRR method if ir = 18% per year and
ib = 10% per year. Verify your answer with the spreadsheet function.
Year 0 1 2 3 4
Revenues, $ 0 25,000 19,000 4000 28,000
Costs, $ −6000 −30,000 −7000 −6000 −12,000

First find net cash flows (NCF)

         Year         0     1              2              3              4


        Revenue, $         0 25,000 19,000      4,000 28,000
       Costs, $        −6,000 −30,000 −7,000     −6,000 −12,000
NCF, $ −6000 −5000 12,000 −2000 16,000
Cumulative, $ −6000 −11,000 +1,000 −1,000 +15,000

(a) Rule of signs test: three sign changes; therefore, up to 3 i* values


Cumulative CF sign test: Starts with NCF0 < 0; 3 sign changes; no unique,
positive i*

(b) Enter NCF values. Function: = IRR(B1:B5) displays i* = 39.9% per year

(c) PW0 = − 6000 − 5000(P/F,10%,1) − 2000(P/F,10%,3)


= − 6000 − 5000(0.9091) − 2000(0.7513)
= $−12,048

FW4 = 12,000(F/P,18%,2) + 16,000


= 12,000(1.3924) + 16,000
= $32,709

−12,048(F/P,i',4) + 32,709 = 0
−12,048(1 + i')4 + 32,709 = 0
i' = (32,709/12,048)1/4 – 1
= 0.284 (28.4%)

Function = MIRR(B1:B5,10%,18%) verifies that i′ = 28.4% per year

Copyright 2021 © McGraw-Hill Education. All rights reserved. No reproduction or distribution without the
prior written consent of McGraw-Hill Education.
35
6.56 Harley worked for many years to save enough money to start his own residential
landscape design business. The net cash flows shown are those he recorded for the
first 6 years as his own boss. Find the external rate of return using the modified rate
of return approach with a reinvestment rate of 15% per year and a borrowing rate of
8%. Additionally, after using the procedure, use the MIRR function to confirm your
answer.
Year 0 1 2 3 4 5 6
NCF, $ −9000 +4100 −2000 −7000 +12,000 +700 +800

Manual: Follow the steps of the modified ROR procedure.

PW0 = −9000 − 2000(P/F,8%,2) − 7000(P/F,8%,3)


= −9000 − 2000(0.8573) − 7000(0.7938)
= $−16,271

FW6 = 4100(F/P,15%,5) + 12,000(F/P,15%,2) + 700(F/P,15%,1) + 800


= 4100(2.0114) + 12,000(1.3225) + 700(1.1500) + 800
= $25,722

25,722 = 16,271(F/P,i′,6)
(F/P,i′,6) = 1.5808

Use interpolation in factor tables or spreadsheet to find i′.

Spreadsheet: = RATE(6,−16271,25722) displays an i′ of 7.9% per year

Spreadsheet verification: Enter NCF in B2:B8; enter = MIRR(B2:B8,8%,15%) to


display an i′ of 7.9% per year.

6.57 Your friend owns a company that makes clutch disks for race cars. He had the net
cash flows shown for one department over a 4-year period after making an
improvement investment of $65,000.
a. Apply the two sign rules first, then calculate the internal rate of return.
b. Calculate the external rate of return using the ROIC method manually with a
reinvestment rate of 15% per year.
c. Calculate the external rate of return using the MIRR method manually with a
reinvestment rate of 15% per year and a borrowing rate of 8% per year.
d. Rework parts (b) and (c) using spreadsheet functions.
Year 0 1 2 3 4
NCF, $1000 −65 30 84 −10 −12

Copyright 2021 © McGraw-Hill Education. All rights reserved. No reproduction or distribution without the
prior written consent of McGraw-Hill Education.
36
First, calculate the cumulative net cash flow (NCF) series.

Cumulative
Year NCF, $1000 NCF, $1000
0 −65 −65
1 30 −35
2 84 +49
3 −10 +39
4 −12 +27

(a) The NCF series indicates up to two i* values; cumulative NCF starts out
negative and changes sign only once, indicating there is only one, real-number
positive i* value.

0 = −65 + 30(P/F,i*,1) + 84(P/F,i*,2) – 10(P/F,i*,3) – 12(P/F,i*,4)

Spreadsheet: Use IRR function to solve for i* = 28.64% per year

(b) Apply net-investment procedure for the ROIC method because reinvestment
rate ir is not equal to the i* rate of 28.64% per year

F0 = −65 F0 < 0; use i″

F1 = −65(1 + i″) + 30 F1 < 0; use i″

F2 = F1(1 + i″) + 84 F2 > 0; use ir (F2 must be > 0 because last two terms
are negative)

F3 = F2(1 + 0.15) −10 F3 > 0; use ir (F3 must be > 0 because last term is
negative)

F4 = F3(1 + 0.15) –12

Set F4 = 0 and solve for i″ by trial and error

F1 = −65 – 65 i″ + 30
F2 = (−65 – 65 i″ + 30)(1 + i″) + 84
= −65 − 65i″ + 30 –65i″ – 65 i″2 + 30i″ +84
= −65 i″2 –100 i″ + 49

F3 = (−65 i″2 –100 i″ + 49)(1.15) – 10


= −74.8 i″2 –115 i″ + 56.4 – 10
= −74.8 i″2 –115 i″ + 46.4
F4 = (−74.8 i″2 –115i″ + 46.4)(1.15) –12
= −86i″2 –132.3 i″ + 53.3 – 12
= −86 i″2 –132.3i″ + 41.3

Copyright 2021 © McGraw-Hill Education. All rights reserved. No reproduction or distribution without the
prior written consent of McGraw-Hill Education.
37
Set F4 = 0 and solve by quadratic equation, trial and error, or spreadsheet.

i″ = 0.26.61 (26.61% per year)

(c) PW0 = −65 −10(P/F,ib,3) − 12(P/F,ib,4)


= −65 −10(P/F,8%,3) − 12(P/F,8%,4)
= −65 − 10(0.7938) − 12(0.7350)
= $−81.76

FW4 = 30(F/P,ir,3) + 84(F/P,ir,2)


= 30(F/P,15%,3) + 84(F/P,15%,2)
= 30(1.5209) + 84(1.3225)
= $156.72

PW0(F/P,i′,4) + FW4 = 0
−81.76(1 + i′)4 + 156.72 = 0
(1 + i′)4 = 1.9168
i′ = 0.1766 (17.66%)

(d) MIRR: Function: = MIRR(B2:B6,8%,15%) displays an i′ of 17.66%, which


verifies the manual solution.

ROIC: Spreadsheet displays an i″ of 26.62% using GOAL SEEK, which


verifies the manual solution.

6.58 According to Descartes’ rule of signs, the possible number of i* values for the
following net cash flow series is: ++++−−−−−−+−+−−−++
a. 2
b. 4
c. 6
d. 8

Answer is (c)

Copyright 2021 © McGraw-Hill Education. All rights reserved. No reproduction or distribution without the
prior written consent of McGraw-Hill Education.
38
6.59 When using the modified ROR method to remove multiple ROR values, an
additional estimate needed besides the cash flows and their timings is the:
a. ROIC value
b. external rate of return
c. reinvestment rate
d. internal rate of return

Answer is (c)

6.60 For the net cash flows and cumulative net cash flows shown below, the value of x is
nearest:
a. $−8,000
b. $−16,000
c. $16,000
d. $41,000
Year 1 2 3 4 5
Net cash flow, $ +13,000 −29,000 −25,000 +50,000 x
Cumulative NCF, $ +13,000 −16,000 −41,000 +9,000 +1,000

9000 + x = 1000
x = −8000
Answer is (a)

6.61 Scientific Instruments, Inc. uses a MARR of 8% per year. The company is
evaluating a new process to reduce water effluents from its manufacturing
processes. The estimates associated with the process follow. In evaluating the
process on the basis of a rate of return analysis, the correct equation to use is

Alternative I

First cost, $ −40,000


NCF, $ per year  13,000
Salvage value, $  5,000
Life, years  3

a. 0 = −40,000 + 13,000(P/A,i*,3) + 5000(P/F,i*,3)


b. 0 = −40,000(A/P,i*,3) + 13,000 + 5000(A/F,i*,3)
c. 0 = −40,000(F/P,i*,3) + 13,000(F/A,i*,3) + 5000
d. Any of the above

Answer is (d)

Copyright 2021 © McGraw-Hill Education. All rights reserved. No reproduction or distribution without the
prior written consent of McGraw-Hill Education.
39
6.62 For the cash flows shown, the sum of the incremental cash flows (B − A) is
a. $2500
b. $3500
c. $6000
d. $8000

Year A B

0 −10,000 −14,000
1 +2,500 +4,000
2 +2,500 +4,000
3 +2,500 +4,000
4 +2,500 +4,000
5 +2,500 +4,000

Find sum from incremental cash flows column.

Year A B B−A
0 −10,000 −14,000 −4000
1 +2500 +4000 +1500
2 +2500 +4000 +1500
3 +2500 +4000 +1500
4 +2500 +4000 +1500
5 +2500 +4000 +1500
+3500
Answer is (b)

6.63 A small engineering firm borrows $300,000 at 0.7% per month interest. If the firm
makes a payment of $50,000 at the end of the first month, the unrecovered loan
balance immediately after the payment is made is closest to:
a. $187,900
b. $198,300
c. $224,600
d. $252,100

Amount due after 1 month = 300,000(1.007)


= $302,100
Balance after $50,000 payment = 302,100 – 50,000
= $252,100

Answer is (d)

Copyright 2021 © McGraw-Hill Education. All rights reserved. No reproduction or distribution without the
prior written consent of McGraw-Hill Education.
40
6.64 If a 6-year study period is used for an ROR evaluation, the incremental cash flow
between machines B and A in year 3 is closest to:
a. $1200
b. $4200
c. $12,000
d. $13,200

Machine
A B

First cost, $ −15,000 −25,000


Annual operating cost, $ −1,600 −400
Salvage value, $ 3,000 6,000
Life, years 3 6

Incremental CF, year 3 = − 400 − (−1600 – 15,000 + 3000)


= +$13,200
Answer is (d)

6.65 A total of $200,000 was invested in two projects identified as X1 and X2. If the
overall rate of return on the $200,000 was 26% and the overall rate of return on the
$40,000 invested in X2 was 14%, the overall rate of return on X1 was closest to:
a. >30%
b. 29%
c. 27%
d. 25%

40,000(0.14) + (200,000 – 40,000)(RORX1) = 200,000(0.26)


RORX1 = 0.29 (29%)
Answer is (b)

6.66 The four alternatives described below are being evaluated by the rate of return
method. If the alternatives are mutually exclusive, the selected alternative at a
MARR of 14.5% per year is:
a. A
b. B
c. C
d. D

Copyright 2021 © McGraw-Hill Education. All rights reserved. No reproduction or distribution without the
prior written consent of McGraw-Hill Education.
41
Δi*, %, When
Compared with
Overall Rate Alternative
Initial
Alternative Investment, $ of Return, % A B C

A −60,000 11.7 — — —
B −90,000 22.2 43.3 — —
C −140,000 17.9 22.5 10.0 —
D −190,000 15.8 17.8 10.0 10.0

Compare alternatives incrementally after ranking: DN, A, B, C, D

A to DN: ∆i* = 11.7% < 14.5% Eliminate A


B to DN: ∆i* = 22.2% > 14.5% Eliminate DN
C to B: ∆i* = 10.0% < 14.5% Eliminate C
D to B: ∆i* = 10.0% < 14.5% Eliminate D; Select B
Answer is (b)

6.67 Alternative G has a first cost of $250,000 and annual costs of $73,000. Alternative
H has a first cost of $350,000 and annual costs of $48,000. If the alternatives are
considered to last indefinitely, the rate of return on the increment of investment is
closest to:
a. 25%/year
b. 20%/year
c. 21%/year
d. 12%/year

n is infinity, use i = ∆A/∆P

i* = 25,000/100,000
= 0.25 (25% per year)
Answer is (a)

6.68 EPC, Inc. uses a MARR of 12% per year compounded semiannually. The company
is evaluating two new processes for expanding its epoxy resin operations. The cash
flows associated with each process are shown below. In evaluating the processes on
the basis of an ROR analysis, the incremental investment rate of return equation to
use is:

Copyright 2021 © McGraw-Hill Education. All rights reserved. No reproduction or distribution without the
prior written consent of McGraw-Hill Education.
42
Alternative
I J
First cost, $ −420,000 −520,000
AOC, $/year −15,000 −12,000
Salvage value, $ 5,000 6,000
Life, years 3 3

a. 0 = −100,000 + 3000(P/A,Δi*,3) + 1000(P/F,Δi*,3)


b. 0 = −420,000 − 15,000(P/A,Δi*,3) + 5000(P/F,Δi*,3)
c. 0 = −520,000 − 12,000(P/A,Δi*,3) + 6000(P/F,Δi*,3)
d. 0 = −100,000 − 3000(P/A,Δi*,3) + 1000(P/F,Δi*,3)

Answer is (a)

6.69 Jewel-Osco is evaluating three biometric systems that identify customers by a


finger scan and immediately deduct the amount of the bill directly from their
checking accounts. The alternatives were ranked according to increasing initial
investment and identified as alternatives A, B, and C. Based on the incremental
rates of return shown and the MARR of 16% per year, the alternative to select is:
a. A
b. B
c. C
d. DN

Comparison Δi*, %

A vs. DN 23.4
B vs. DN   8.1
C vs. DN 16.6
B vs. A −5.1
C vs. A 12.0
C vs. B 83.9

Only A and C have overall ROR > MARR of 16%

C to A: ∆i* = 12.0% ∆i* < MARR Eliminate C; select A

Answer is (a)

Copyright 2021 © McGraw-Hill Education. All rights reserved. No reproduction or distribution without the
prior written consent of McGraw-Hill Education.
43
6.70 For the cash flows shown, the correct equation for F2 using the ROIC method at the
reinvestment rate of 20% per year is:
a. F2 = [10,000(1 + i″) + 6000](1.20) − 8000
b. F2 = [10,000(1.20) + 6000(1 + i″)](1.20) − 8000
c. F2 = [10,000(1.20) + 6000](1.20) − 8000
d. F2 = [10,000(1.20) + 6000](1 + i″) – 8000

Year Cash Flow, $

0   10,000
1   6,000
2 −8,000
3 −19,000

Answer is (c)

6.71 The net cash flows associated with the manufacture and sale of stabilizer parts for
Stratolaunch, the world’s largest airplane with a 385-foot wingspan (longer than an
American football field), are shown. The NCF series indicates the possibility of
more than one i* value. If the borrowing rate is 10% per year and the reinvestment
rate is 15% per year, the external rate of return per year using the MIRR approach is
closest to:
a. 14.8%
b. 15.1%
c. 16.6%
d. 20.3%

Year NCF, $1000

0 −50
1 +20
2 +40
3 −15
4 +30

PW0 = −50 −15(P/F,10%,3)


= −50 −15(0.7513)
= $−61.27

Copyright 2021 © McGraw-Hill Education. All rights reserved. No reproduction or distribution without the
prior written consent of McGraw-Hill Education.
44
FW4 = 20(F/P,15%,3) + 40(F/P,15%,2) + 30
= 20(1.5209) + 40(1.3225) + 30
= $113.32

113.32 = 61.27(F/P,i′,4)
(F/P,i′,4) = 1.8495
i′ = 16.6%

Answer is (c)

6.72 If during an ROR evaluation of two mutually exclusive alternatives, multiple i*


values are indicated by the rule of signs and the determination of the exact ROR
value is not necessary, a way to avoid extra ROR computations is to use the MARR
and calculate the:
a. PW
b. AW
c. FW
d. Any of these

Answer is (d)

6.73 Assuming there are two i* values determined when an ROR analysis of a
nonconventional cash flow series is performed, one guideline for retaining and
discarding i* values is:
a. Take the average of the two i* values as the ROR
b. Retain the i* > 0, if one is positive and one is negative
c. Retain the larger i* value, if both are positive
d. Retain the smaller i* value, if both are positive

Answer is (b)

Copyright 2021 © McGraw-Hill Education. All rights reserved. No reproduction or distribution without the
prior written consent of McGraw-Hill Education.
45
Solutions to end-of-chapter problems
Basics of Engineering Economy, 3rd edition
Leland Blank and Anthony Tarquin

Chapter 7
Benefit/Cost Analysis and Public Sector Projects

7.1 Indicate whether the following characteristics are primarily associated with public
sector or private sector projects:
a. Profits
b. Income taxes
c. Disbenefits
d. Infinite life
e. Highway toll fees
f. Corporate bonds
(a) Private (b) Private (c) Public
(d) Public (e) Public (f)  Private

7.2 Identify the following items as a benefit, disbenefit, or cost:


a. Expenditure of $20 million for construction of a highway.
b. $200,000 annual income to local businesses because of tourism created by water
reservoir.
c. $300,000 per year upkeep costs for irrigation canals.
d. $100,000 per year loss of revenue by farmers because of expanded highway right-
of-way.
e. Fifteen percent fewer highway accidents because of improved lighting.
f. Twenty percent lower negative environmental impact because of electric cars.

(a) Cost (b) Benefit (c) Cost (d) Disbenefit (e) Benefit (f) Benefit

7.3 State whether a reduction in automobile accidents due to construction of a new state
highway is primarily a benefit, disbenefit, or cost from the listed viewpoints. Also
give a reason why you classified as you did.
a. Local resident driving to/from work
b. Automobile body repair shop
c. State highway/transportation construction budget
d. Personal injury/liability lawyer
e. For-profit emergency care facility/hospital
f. Amusement park one hour away

(a) Benefit due to safety


(b) Disbenefit – less business due to fewer repairs

Copyright 2021 © McGraw-Hill Education. All rights reserved. No reproduction or distribution without the
prior written consent of McGraw-Hill Education.
1
(c) Cost – construction/M&O
(d) Disbenefit – less business due to fewer clients
(e) Disbenefit – less business due to fewer patients
(f) Benefit – more visitors due to safer travel

7.4 Identify the following funding sources as primarily public or private.


a. Municipal bonds
b. Retained earnings
c. Sales taxes
d. Automobile license fees
e. Bank loans
f. Savings accounts
g. Engineer’s retirement plan
h. State fishing license revenues
i. Entrance fees to Disney World®
j. State park entrance fees

(a) Municipal bonds – public


(b) Retained earnings – private
(c) Sales taxes – public
(d) Automobile license fees - public
(e) Bank loans – private
(f) Savings accounts - private
(g) Engineer’s retirement plan - private
(h) State fishing license revenues - public
(i) Entrance fees to Disney World® – private
(j) State park entrance fees - public

7.5 Bethany has been out of work for several months because the plant where she
worked has closed due to international trade barriers. She has not told any of her
siblings or parents, and she is out of money. Her main reason for silence is shame,
though it is not her fault for being unemployed and short of money. She has always
had a very strong moral code of honesty and a strong desire to “go it on her own.”
Bethany is at her parents’ home and takes the opportunity to remove a $100 bill from
her dad’s billfold with the rationale that “what is theirs, is hers” because she is their
child. Besides, she knows that he would probably gladly give her some money if he
knew the situation. She leaves the house with the money, knowing she did something
that is against her own beliefs. Explain in your own words, (a) the temptation, (b) the
ethics dilemma, and (c) what Bethany should do at this point.

There are many answers possible. A sample response follows.

(a) Tempted to take money because she has a ‘rationale’ she believes and she is
not willing to face the truth and tell others of her dire circumstances

Copyright 2021 © McGraw-Hill Education. All rights reserved. No reproduction or distribution without the
prior written consent of McGraw-Hill Education.
2
(b) The dilemma is to violate her code of ethics versus telling others of her plight and
appear to be “begging” for help, which goes against her strong will to be
independent.

(c) One way out is to find a private moment with one or both parents, share the
situation, return the money to them with an apology, and provide an explanation
of why she acted as she did.

7.6 The cash flows associated with a public school building repair project are as follows:
costs $550,000 per year; benefits $600,000 per year; disbenefits $90,000 per year.
Determine the conventional B/C ratio.

(a) B/C = (600,000 – 90,000)/550,000 = 0.93

(b) Since B/C < 1.0, it is not economically acceptable

7.7 The cost of grading and spreading gravel on a curvy rural road is expected to be
$300,000. The road will have to be maintained at a cost of $25,000 per year. Even
though the new road is not very smooth, it allows access to an area that previously
could only be reached with off-road vehicles. The improved accessibility has led to a
150% increase in the property values along the road. If the previous market value of
the property was $900,000, calculate the conventional B/C ratio using an interest
rate of 6% per year and a 20-year study period.

B = 900,000(1.5) – 900,000
= $450,000

C = 300,000 + 25,000(P/A,6%,20)
= 300,000 + 25,000(11.4699)
= $586,748

B/C = 450,000/586,748
= 0.77

7.8 From the following data, use the conventional B/C ratio for a project which has a
20-year life to determine if it is economically justified. Use an interest rate of
8% per year.

Consequences
To the People To the Government
Annual benefits = $90,000 per year First cost = $750,000
Annual disbenefits = $10,000 per year Annual cost = $50,000 per year
Annual savings = $30,000 per year

Copyright 2021 © McGraw-Hill Education. All rights reserved. No reproduction or distribution without the
prior written consent of McGraw-Hill Education.
3
Base the B/C on AW values.

B = 90,000
D = 10,000
C = 750,000(A/P,8%,20) + 50,000
= 750,000(0.10185) + 50,000
= $126.388
S = 30,000

B/C = (90,000 – 10,000)/(126,388 – 30,000)


= 0.83

Since B/C < 1.0, the project is not justified.

7.9 A proposed regulation regarding the removal of arsenic from drinking water is
expected to have an annual cost of $200 per household per year. If it is assumed that
there are 126 million U.S. households, how many lives would have to be saved each
year for the B/C ratio to be equal to 1.0, provided the value of a single life is
$180,000 per year?

Let N = lives saved per year

Benefits = 180,000(N)
Cost = 200(126,000,000)
= 2.52 × 1010

Set B/C = 1.0


180,000N/2.52 × 1010 = 1.0
18N = 2.52 × 106
18N = 2,520,000
N = 140,000 persons per year

7.10 Calculate (a) the conventional B/C ratio, (b) the modified B/C ratio, and (c) the PI
for the following cash flow estimates at a discount rate of 10% per year. Compare
the resulting values.

Item Cash Flow

PW of benefits, $ 3,800,000
AW of disbenefits, $/year      45,000
First cost, $ 1,200,000
M&O costs, $/year   300,000
Life of project, years 20

Copyright 2021 © McGraw-Hill Education. All rights reserved. No reproduction or distribution without the
prior written consent of McGraw-Hill Education.
4
(a) Convert all cash flows to PW or AW. Using AW,

AWBenefits = 3,800,000(A/P,10%,20)
= 3,800,000(0.11746)
= $446,348

AWFirst cost = 1,200,000(A/P,10%,20)


= 1,200,000(0.11746)
= $140,952

Conventional B/C = (446,348 – 45,000)/(140,952 + 300,000)


= 0.91

(b) Modified B/C = (446,348 – 45,000 – 300,000)/140,952


= 101,348/140,952
= 0.72

(c) Disregard disbenefits when calculating the PI.

PI = (446,348 – 300,000)/140,952
= 1.04

Comparison: In this case, PI > 1.0, while the two B/C values are < 1.0, because
the disbenefits are neglected in the PI calculation.

7.11 The benefits associated with a nuclear power plant cooling water filtration project
located on the Ohio River are $10,000 per year forever, starting in year 1. The costs
are $50,000 in year 0 and $50,000 at the end of year 2. At i = 10% per year, use the
B/C ratio to determine if the project is justified economically.

Convert benefits to PW by dividing by i and then calculate B/C ratio.

B/C = [10,000/0.10]/[50,000 + 50,000(P/F,10%,2)]


= [100,000]/[50,000 + 50,000(0.8264)]
= 1.1

The project is justified.

7.12 The U.S. Army Corps of Engineers is considering the economic viability of
constructing a small flood control dam in an existing arroyo. The initial cost of the
project will be $4.2 million, with inspection and upkeep costs of $10,000 per year.
In addition, minor reconstruction will be required every 15 years at a cost of
$165,000. If flood damage will be reduced from the present cost of $330,000 per
year to $22,000 annually, calculate the conventional benefit to cost ratio to

Copyright 2021 © McGraw-Hill Education. All rights reserved. No reproduction or distribution without the
prior written consent of McGraw-Hill Education.
5
determine if the dam should be constructed. Assume that the dam will be permanent
and the discount rate is 8% per year.

Convert initial cost to AW and then calculate B/C ratio.

AWFirst cost = 4,200,000(0.08)


= $336,000

AWReconstruction = 165,000(A/F,8%,15)
= 165,000(0.03683)
= $6,077

AWCosts = 336,000 + 6077 + 10,000


= $352,077

AWBenefits = 330,000 – 22,000


= 308,000

B/C = 308,000/352,077
= 0.87

Do not build the dam

7.13 A government-funded wind-based electric power generation company in the


southern part of the country has developed the following estimates (in $1000) for a
new turbine farm. The MARR is 10% per year and the project life is 25 years.
Calculate the (a) conventional B/C ratio, (b) modified B/C ratio, and (c) PI value.

Benefits: $45,000 in year 0; $30,000 in year 5


Government savings: $2,000 in years 1 through 20
Cost: $50,000 in year 0
Disbenefits: $3000 in years 1 through 10

(a) PWB = 45,000 + 30,000(P/F,10%,5)


= 45,000 + 30,000(0.6209)
= $63,627

PWD = 3000(P/A,10%,10)
= 3000(6.1446)
= $18,433

PWC = $50,000

PWS = 2000(P/A,10%,20)
= 2000(8.5136)
= $17,027

Copyright 2021 © McGraw-Hill Education. All rights reserved. No reproduction or distribution without the
prior written consent of McGraw-Hill Education.
6
B/C = (B – D)/(C – S)
= (63,627 – 18,433)/(50,000 – 17,027)
= 45,194/32,973
= 1.37

(b) PWB = 45,000 + 30,000(P/F,10%,5)


= 45,000 + 30,000(0.6209)
= $63,627

PWD = 3000(P/A,10%,10)
= 3000(6.1446)
= $18,433

PWC = $50,000

PWS = 2000(P/A,10%,20)
= 2000(8.5136)
= $17,027

Modified B/C = (63,627 – 18,433 + 17,027)/50,000


= 62,221/50,000
= 1.24

(c) Disregard disbenefits.

PI = (B + S)/C
= (63,627 + 17,027)/50,000
= 1.61

7.14 TxDOT is considering the construction of a scenic byway through a rural area in
West Texas. The 12 km road is expected to cost $2.5 million per km, with annual
upkeep estimated at $75,000 per year. The improved accessibility is expected to
result in additional income to businesses from tourists of $4,400,000 per year, but
disbenefits of $130,000 per year have been identified. If the road is expected to
have a useful life of 40 years, calculate the conventional B/C ratio at an interest rate
of 8% per year to determine if the project is economically acceptable.

(a) AWFirst cost = 12(2,500,000)(A/P.8%,40)


= 12(2,500,000) (0.08386)
= $2,515,800
AWTotal Costs = 2,515,800 + 75,000
= $2,590,800

B/C = (4,400,000 – 130,000)/ 2,590,800


= 1.65

Project is economically acceptable.

Copyright 2021 © McGraw-Hill Education. All rights reserved. No reproduction or distribution without the
prior written consent of McGraw-Hill Education.
7
7.15 A project to extend canals for vegetable crop irrigation into an area that was
recently cleared of Mesquite trees (a nuisance tree in Texas) and large weeds is
projected to have a capital cost of $2,000,000. Annual maintenance and operation
costs will be $100,000 per year. Annual favorable consequences to the general
public of $820,000 per year will be offset to some extent by annual adverse
consequences of $400,000 to a portion of the general public. If the project is
assumed to have a 20-year life, what is the conventional B/C ratio at an interest rate
of 8% per year?

B = $820,000
D = $400,000

C = 2,000,000(A/P,8%,20) + 100,000
= 2,000,000(0.10185) + 100,000
= $303,700

B/C = (820,000 – 400,000)/303,700


= 1.38

7.16 The cash flows associated with a public works project in Buffalo, New York, are
shown. Use the modified B/C ratio at a discount rate of 5% per year to determine
the economic justification.

First cost, $ 50,000,000


AW of benefits, $/year 7,500,000
AW of disbenefits, $/year 1,700,000
M&O costs, $/year   900,000
Life of project, years 30

AW of first cost, C = 50,000,000(A/P,5%,30)


= 50,000,000(0.06505)
= $3,252,500

AW of B = AW of benefits – AW of disbenefits – AW of M&O costs


= 7,500,000 – 1,700,000 – 900,000
= $4,900,000

Modified B/C = 4,900,000/3,252,500


= 1.51

Project is justified.

Copyright 2021 © McGraw-Hill Education. All rights reserved. No reproduction or distribution without the
prior written consent of McGraw-Hill Education.
8
7.17 The conventional B/C ratio for the placement of a 24-hour public medical care
facility in a rural commuity was calculated to be 1.2. The benefits were $500,000
per year, disbenefits were $100,000 per year, and the maintenance costs were
$200,000 per year. What was the initial cost of the project, if a discount rate of 7%
per year was used and the facility was assumed to have a 50-year life?

Let P = initial cost. Use AW values.

1.2 = (500,000 – 100,000)/[P(A/P,7%,50) + 200,000]


1.2 = (500,000 – 100,000)/[P(0.07246) + 200,000]
400,000 = 1.2[P(0.07246) + 200,000]
160,000 = (0.086952)P
P = $1,840,096

7.18 The conventional B/C ratio estimate of 2.4 was reported to the County
Commissioners for a proposed mosquito control program. The person who prepared
the report stated that the health benefits were estimated to be $400,000 per year,
and that disbenefits of $25,000 per year were used in the calculation. She also stated
that the costs for chemicals, machinery, maintenance, and labor were estimated at
$150,000 per year. However, she forgot to list the cost for initiating the program
(trucks, pumps, tanks, etc.). If the project has a 10-year study period and an 8%-per-
year discount rate, determine the estimated initial cost.

Let P = initial cost. Use AW values.

2.4 = (400,000 – 25,000)/[P(A/P,8%,10) + 150,000]


2.4[P(0.14903) + 150,000] = 375,000
0.35767P = 15,000
P = $41,938

7.19 The modified B/C ratio for a city-owned hospital heliport project is 1.7. The initial
cost is $1 million, annual benefits are $150,000, and the estimated life is 30 years.
What is the amount of the annual M&O costs used in the calculation at a discount
rate of 6% per year?

1.7 = [150,000 – (M&O)]/1,000,000(A/P,6%,30)


150,000 – (M&O) = 1.7[1,000,000(0.07265)]
M&O = $26,495

7.20 Nick and Helen Adams have been offered a chance to invest in a Real Estate Trust
that pays annual benefits estimated at $12,000 for a lump sum investment now of
$70,000. The agent mentioned that there is a small capital improvement fund cost
of $2000 per year. He also mentioned that the projected profitability index of 1.20
is very favorable since for every $1 invested a return of $1.20 can be expected, over
time.

Copyright 2021 © McGraw-Hill Education. All rights reserved. No reproduction or distribution without the
prior written consent of McGraw-Hill Education.
9
Once the couple understood that the PI was calculated as the equivalent of
annual benefits ($12,000) minus costs ($2000) divided by the $70,000 up-front
investment amortized over 40 years at 10% per year, they quickly calculated, in
round numbers, the PI to be 1.40, not 1.20. It was clear to them that there is another
(hidden) annual cost if the PI of 1.20 is correct.

a. What is the size of this “unmentioned” cost?


b. (Spreadsheet exercise) Assume the agent agreed that there is an additional annual
charge equal to the amount calculated in part (a). Perform a spreadsheet analysis
similar to that in Example 7.3(c) to determine the actual ROR, assuming the
couple agreed to the investment and the extra annual cost.

(a) Let C2 = additional “unmentioned” annual cost. Use AW values.

1.2 = (12,000 – 2000 – C2)/70,000(A/P,10%,40)


= (10,000 – C2)/7158
C2 = 10,000 – 1.2(7158)
= $1410 per year

(b) Before and after GOAL SEEK images are shown. The AW values for the
numerator of PI is 12,000 – 2000 – 1410. Use the GOAL SEEK template to
force PI (cell C9) to equal 1.00 while changing cell B3. Result is i* = 12.1%
(cell G3).

7.21 Two alternatives, identified as X and Y, are evaluated using the B/C method.
Alternative Y has a higher total cost than X. If the B/C ratios are 1.2 and 1.0 for
alternatives X and Y, respectively, which alternative should be selected? Why?

Alternative X should be selected because the B/C ratio on the incremental cash
flows between the two alternatives has to be less than 1.0

7.22 A state agency is considering two mutually exclusive alternatives for upgrading the
skills of its technical staff. Alternative 1 involves purchasing software that will
reduce the time required to collect background information on each client. The total
cost for the purchase, installation, and training associated with the new software is
$840,900. The present worth of the benefits from increased efficiency is expected to
be $1,020,000.
Alternative 2 involves multimedia training to improve the performance of the
staff technicians. The total cost to develop, install, and train the technicians will be

Copyright 2021 © McGraw-Hill Education. All rights reserved. No reproduction or distribution without the
prior written consent of McGraw-Hill Education.
10
$1,580,000. The present worth of the benefits from increased performance due to
training is expected to be $1,850,000. Use a B/C analysis to determine which
alternative, if any, the agency should undertake.

Benefits are directly estimated; DN is first alternative. Compare against DN first,


then rank alternatives by increasing PW of total costs: DN, 1, 2

1 vs DN: B/C = 1,020,000/840,900


= 1.21 Eliminate DN

2 vs DN: B/C = 1,850,000/1,580,000


= 1.17 Eliminate DN

Order is: 1, 2; incrementally compare 2 vs 1

2 vs 1: ∆B = 1,850,000 – 1,020,000
= $830,000

∆C = 1,580,000 – 840,900
= $739,100

∆B/C = 830,000/739,100
= 1.12 Eliminate 1

Select alternative 2

7.23 The Logan Well Services Group is considering two sites for storage and recovery of
reclaimed water. The mountain site (MS) will use injection wells that cost $4.2
million to develop and $280,000 per year for M&O. This site will be able to
accommodate 150 million gallons per year. The valley site (VS) will involve
recharge basins that cost $11 million to construct and $400,000 to operate and
maintain. At this site, 890 million gallons can be injected each year. If the value of
the injected water is $3.00 per thousand gallons, which alternative, if either, should
be selected according to the B/C ratio method? Use an interest rate of 8% per year
and a 20-year study period.

Direct benefit alternatives; compare to DN first

MS vs DN: B = (150,000,000)(3.00/1000)
= $450,000

C = 4,200,000(A/P,8%,20) + 280,000
= 4,200,000(0.10185) + 280,000
= $707,770

B/C = 450,000/707,770 = 0.64 Eliminate MS

Copyright 2021 © McGraw-Hill Education. All rights reserved. No reproduction or distribution without the
prior written consent of McGraw-Hill Education.
11
VS vs DN: B = (890,000,000)(3.00/1000)
= $2,670,000

C = 11,000,000(A/P,8%,20) + 400,000
= 11,000,000(0.10185) + 400,000
= $1,520,350

B/C = 2,670,000/1,520,350 = 1.76

Select VS, the valley site, since B/C > 1.0

7.24 You work for the New Mexico Department of Transportation (NMDOT). Two
proposals to improve street safety and lighting in a colonia in south central New
Mexico have been proposed. (a) Use a B/C analysis and a discount rate of 8% per
year to evaluate the proposals. (b) (Here is a thought question.) If you reevaluate
proposal 1 benefits and find them to be understated by $100,000 per year, benefits
will increase from $530,000 to $630,000 per year. Now, the evaluation (1 vs. DN)
will have a B/C = 1.30. There are three questions: (1) What will be the time basis of
the incremental evaluation? (2) Using the comparison terminology “larger initial
cost” vs. “smaller initial cost,” what will be the order of incremental comparison
between the two proposals: (1 vs. 2) or (2 vs. 1)? (3) Which proposal will be
selected now? Why?

Proposal 1 Proposal 2

Initial cost, $ 900,000 1,700,000


Annual M&O costs, $ per year 120,000   60,000
Annual benefits, $ per year 530,000   650,000
Annual disbenefits, $ per year 300,000   195,000
Life, years   10 20

(a) Direct benefits proposals; compare against DN initially; use AW values

1 vs DN: B = 530,000
D = 300,000
C = 900,000(A/P,8%,10) + 120,000
= 900,000(0.14903) + 120,000
= 254,127

B/C = (530,000 – 300,000)/254,127


= 0.91 Eliminate proposal 1

Copyright 2021 © McGraw-Hill Education. All rights reserved. No reproduction or distribution without the
prior written consent of McGraw-Hill Education.
12
2 vs DN: B = 650,000
D = 195,000
C = 1,700,000(A/P,8%,20) + 60,000
= 1,700,000(0.10185) + 60,000
= 233,145

B/C = (650,000 – 195,000)/233,145


= 1.95 Eliminate DN

Since proposal 1 was eliminated, no incremental analysis is needed.

Select proposal 2

(b) Question 1: LCM of 20 years; repurchase proposal 1 in year 10.


Question 2: AW of costs for proposal 1 will remain at $254,127, which is larger
than the AW of costs for 2 ($233,145). Based on total costs, order is
2, 1. Comparison is 1 vs 2.
Question 3: Not necessary to do the incremental comparison 1 vs 2, because the
benefits for 1 are less than those for proposal 2 ($630,000 for 1 and
$650,000 for 2). Select proposal 2.

7.25 Conventional and solar alternatives are available for providing energy at a remote
radar site. Use the B/C ratio to determine which method should be selected at an
interest rate of 8% per year and a 5-year study period.

Method Conventional Solar

Initial cost, $ 300,000 2,500,000


Annual cost, $ per year 700,000    5,000

These are usage cost alternatives; DN is not considered and solar is the challenger.
Difference in annual cost is a benefit to solar.

ΔB = 700,000 – 5000 = 695,000

ΔC = (2,500,000 – 300,000)(A/P,8%,5)
= (2,200,000)(0.25046)
= 551,012

ΔB/C = 695,000/551,012
= 1.26 Eliminate conventional

Select solar

Copyright 2021 © McGraw-Hill Education. All rights reserved. No reproduction or distribution without the
prior written consent of McGraw-Hill Education.
13
7.26 The two alternatives shown are under consideration for improving security at a
county jail in Travis County, New York. Determine which one, if either, should be
selected based on a B/C analysis, an interest rate of 7% per year, and a 10-year
study period.

Extra Cameras New Sensors


(EC) (NS)

First cost, $ 38,000 87,000


Annual M&O, $ per year 49,000 84,000
Benefits, $ per year 110,000 87,000
Disbenefits, $ per year 16,000 —

Direct benefit alternatives; evaluate against DN first

EC vs DN: B = 110,000 per year


D = 16,000 per year
C = 38,000(A/P,7%,10) + 49,000
= 38,000(0.14238) + 49,000
= $54,410

(B – D)/C = (110,000 – 16,000)/54,410


= 1.73 Eliminate DN

NS vs DN: B = 130,000 per year


D=0
C = 87,000(A/P,7%,10) + 84,000
= 87,000(0.14238) + 84,000
= $96,387

(B – D)/C = (87,000 – 0)/96,387


= 0.90 Eliminate NS

Unnecessary to make the incremental comparison NS vs EC

Select EC, extra cameras

7.27 Two alternative projects to control flooding from rare, but sometimes heavy,
rainfalls in the arid southwest will have the cash flows shown. Determine which
alternative should be selected on the basis of a B/C analysis at i = 8% per year and a
20-year study period.

Copyright 2021 © McGraw-Hill Education. All rights reserved. No reproduction or distribution without the
prior written consent of McGraw-Hill Education.
14
Sanitary Open
Sewers (SS) Channels (OC)

First cost, $ 26 million 53 million


M&O cost, $ per year 1.4 million 30,000
Homeowner cleanup
costs, $ per year 950,000 0

Alternatives involve only usage costs; DN is not an option. Calculate AW of total


costs; comparison order: SS, OC.

CSS = 26,000,000(A/P,8%,20) + 1,400,000


= 26,000,000(0.10185) + 1,400,000
= $4,048,000

COC = 53,000,000(A/P,8%,20) + 30,000


= 53,000,000(0.10185) + 30,000
= $5,428,050

Cleanup costs are a benefit to OC

OC vs SS: ∆B = $950,000

∆B/C = (950,000 – 0)/(5,428,050 – 4,048,000)


= 0.69 Eliminate OC

Build SS, sanitary sewers

7.28 The four independent alternatives shown are being compared using the B/C method.
Which alternative(s), if any, should be selected?

B/C ΔB/C When Compared


Ratio with Alternative
Cost, $ for Alt.
Alternative millions vs. DN A B C D

A 20 1.1 —
B 25 0.96 0.40 —
C 33 1.22 1.42 2.14 —
D 45 0.89 0.72 0.80 0.08 —

Since alternatives are independent, only compare against DN. From ‘B/C ratio for
Alt. vs DN’ column, select Alternatives A and C with B/C > 1.0

Copyright 2021 © McGraw-Hill Education. All rights reserved. No reproduction or distribution without the
prior written consent of McGraw-Hill Education.
15
7.29 A group of engineers responsible for developing advanced missile detection and
tracking technologies such as shortwave infrared, thermal infrared detection, target
tracking radar, etc. recently came up with six proposals for consideration. The
present worth (in $ billions) of the capital requirements and benefits are shown.
Determine which one(s) should be undertaken, if they are (a) independent and (b)
mutually exclusive.

Alternative A B C D E F

PW of Capital, $ 80 50 72 43 89 81
PW of Benefits, $ 70 55 76 52 85 84

(a) B/CA = 70/80 = 0.88


B/CB = 55/50 = 1.10
B/CC = 76/72 = 1.06
B/CD = 52/43 = 1.21
B/CE = 85/89 = 0.95
B/CF = 84/81 = 1.04

Select all alternatives that have B/C ≥ 1.0. Select B, C, D, and F

(b) Rank acceptable alternatives (i.e., B/C ≥ 1.0) by increasing cost (D, B, C, F)
and perform incremental analysis.

B vs D: ΔB/C = (55 – 52)/(50 – 43)


= 0.43 Eliminate B

C vs D: ΔB/C = (76 – 52)/(72 – 43)


= 0.83 Eliminate C

F vs D: ΔB/C = (84 – 52)/(81 – 43)


= 0.84 Eliminate F

Select alternative D

7.30 Incremental comparisons between five mutually exclusive alternatives are shown.
The costs increase from A to E. If one must be accepted, according to the
incremental B/C ratios, which alternative should be selected?

Copyright 2021 © McGraw-Hill Education. All rights reserved. No reproduction or distribution without the
prior written consent of McGraw-Hill Education.
16
Comparison ΔB/C Ratio

B vs. A 0.8
C vs. B 1.4
D vs. C 1.3
C vs. A 1.1
D vs. A 0.2
D vs. B 1.9
E vs. C 1.2
E vs. D 0.9

B vs A < 1.0; eliminate B


C vs A > 1.0; eliminate A
D vs C > 1.0; eliminate C
E vs D < 1.0; eliminate E

Select D

7.31 From the data shown for six mutually exclusive alternatives, determine which one,
if any, should be selected.

Alternative A B C D E F

Annual cost, $ per year 8,000 25,000 15,000 32,000 17,000 20,000
Annual benefits, $ per year ? ? ? ? ? ?
B/C ratio (Alternative vs. DN) 1.23 1.12 0.87 0.97 0.71 1.10

Selected
Comparison ΔB/C Ratios

B vs. A 1.07
C vs. A 0.46
F vs. A 1.02
D vs. B 0.43
E vs. B 2.00
B vs. F 1.20
D vs. C 1.06
F vs. C 1.80

Copyright 2021 © McGraw-Hill Education. All rights reserved. No reproduction or distribution without the
prior written consent of McGraw-Hill Education.
17
Direct benefits alternatives; compare to DN initially. Eliminate C, D, and E because
overall B/C < 1.0; order is now A, F, B

F vs A: ∆B/C = 1.02 > 1.0 Eliminate A

B vs F: ∆B/C = 1.20 > 1.0 Eliminate F

Select B

7.32 The city of St. Louis, Missouri, is considering submitted proposals regarding the
disposal of used tires. All of the alternatives involve shredding, but the charges for
the service and handling of the tire shreds differ in each plan. An incremental B/C
analysis was initiated but never completed.
a. Fill in all the missing blanks in the table.
b. Determine which alternative should be selected.

ΔB/C When
Compared with
Alternative
PW of PW of PW of Overall
Alternative Costs, $ Benefits, $ Disbenefits, $ B/C Ratio J K L M

J 20 ? 1 1.05 —
K 23 28 ? 1.13 ? —
L 28 35 3 ? ? ? —
M ? 51 4 1.34 ? ? ? —

(a) First calculate PW of benefits, disbenefits, costs, and overall B/C ratio.

PW of benefits for J: 1.05 = (B – 1)/20


BJ = 22

PW of disbenefits for K: 1.13 = (28 – D)/23


DK = 2

Overall B/C for L: (B – D)/C = (35 – 3)/28


= 1.14

PW of costs for M: 1.34 = (51 – 4)/C


CM = 35

Direct benefits alternatives; all B/C > 0; perform incremental B/C calculations
using order J, K, L, M

Copyright 2021 © McGraw-Hill Education. All rights reserved. No reproduction or distribution without the
prior written consent of McGraw-Hill Education.
18
K vs J: ΔB/C = [(28 – 2) – (22 – 1)]/(23 – 20)
= 1.67

L vs J: ΔB/C = [(35 – 3) – (22 – 1)]/(28 – 20)


= 1.38

M vs J: ΔB/C = [(51 – 4) – (22 – 1)]/(35 – 20)


= 1.73

L vs K: ΔB/C = [(35 – 3) – (28 – 2)]/(28 – 23)


= 1.20

M vs K: ΔB/C = [(51 – 4) – (28 – 2)]/(35 – 23)


= 1.75

M vs L: ΔB/C = [(51 – 4) – (35 – 3)]/(35 – 28)


= 2.14

(b) Direct benefit alternatives; perform the incremental comparisons.

J vs DN: B/C = 1.05 Eliminate DN


K vs J: ∆B/C = 1.67 Eliminate J
L vs K: ∆B/C = 1.20 Eliminate K
M vs L: ∆B/C = 2.14 Eliminate L

Select M

7.33 (Spreadsheet exercise) The Victoria Water Cooperative must replace old water
distribution lines. Four sizes of pipe are available from vendors with installation
cost and annual maintenance/usage cost preliminary estimates provided in $ per
kilometer. Assuming a 15-year study period and an 8% per year discount rate,
which size pipe should be installed based on a B/C analysis? Use a spreadsheet to
perform the analysis.

Pipe Size, Centimeters

20 25 35 40

Initial cost, $ per km 9,800 11,300 14,600 17,300


Usage cost, $ per km 6,000 5,800 5,500 4,900

Spreadsheet modeled after Example 7.7 using AW values. Usage cost alternatives;
annual benefit is difference in annual usage costs calculated in row 9. Comparison
order: 20, 25, 35, 40. Select 40 cm pipe.

Copyright 2021 © McGraw-Hill Education. All rights reserved. No reproduction or distribution without the
prior written consent of McGraw-Hill Education.
19
7.34 All of the following are examples of public sector projects, except:
a. Bridges
b. Emergency relief
c. Prisons
d. Oil wells

Answer is (d)

7.35 If two mutually exclusive alternatives have B/C ratios of 1.4 and 1.5 for the lower
and higher cost alternatives, respectively, the following is correct:
a. The B/C ratio on the increment between them is equal to 1.5
b. The B/C ratio on the increment between them is between 1.4 and 1.5
c. The B/C ratio on the increment between them is less than 1.5
d. The higher cost alternative is the better one economically

The correct answer is (d) because the incremental B/C ratio has to be greater than
1.5 on the increment between the two.

7.36 An alternative has the following cash flows:

Benefits of $50,000 per year


Disbenefits of $27,000 per year
Initial cost of $250,000
M&O costs of $10,000 per year

If the alternative has an infinite life and the interest rate is 10% per year, the B/C
ratio is closest to:

a. 0.52
b. 0.66
c. 0.91
d. 1.16

B/C = (50,000 – 27,000)/[250,000(0.10) + 10,000]


= 0.66
Answer is (b)

Copyright 2021 © McGraw-Hill Education. All rights reserved. No reproduction or distribution without the
prior written consent of McGraw-Hill Education.
20
7.37 At the interest rate of 10% per year, an alternative with the following estimates has
a modified B/C ratio that is closest to:
a. 0.65
b. 0.72
c. 0.80
d. 1.04

Benefits of $60,000 per year


Disbenefits of $29,000 per year
Amortized first cost of $20,000 per year
M&O costs of $15,000 per year

B/C = (60,000 – 29,000 – 15,000)/20,000


= 0.8
Answer is (c)

7.38 The following are all correct about the profitability index (PI), except:
a. It is also called the “bang for the buck” measure
b. Disbenefits are placed in the denominator as a cost
c. It is similar to the modified B/C ratio
d. If PI > 1.0, the project is economically justified

Answer is (b)

7.39 An alternative with an infinite life has a B/C ratio of 1.5. If the alternative has
benefits of $50,000 per year and annual maintenance costs of $10,000 per year, the
first cost of the alternative at an interest rate of 10% per year is closest to:
a. $23,300
b. $85,400
c. $146,100
d. $233,000

1.5 = 50,000/(0.10P + 10,000)

P = $233,333
Answer is (d)

7.40 The following estimates (in $1000 units) have been developed for a new
cybersecurity system at Chicago’s O’Hare Airport. Calculate the conventional B/C
ratio at a discount rate of 10% per year.
a. < 1.15
b. 1.21
c. 1.52
d. 1.91

Copyright 2021 © McGraw-Hill Education. All rights reserved. No reproduction or distribution without the
prior written consent of McGraw-Hill Education.
21
First cost, $ 13,000
AW of benefits, $ per year 3,800
FW (in year 20) of disbenefits, $ 6,750
M&O costs, $ per year 400
Expected life, years 20

AW of C = 13,000(A/P,10%,20) + 400
= 13,000(0.11746) + 400
= $1927

AW of B – D = 3800 – 6750(A/F,10%,20)
= 3800 – 6750(0.01746)
= $3682

B/C = 3682/1927
= 1.91
Answer is (d)

7.41 The values shown are for a project that has an infinite life. If the conventional B/C
ratio is 1.5 and the interest rate is 10% per year, the disbenefits are closest to:
a. $12,000
b. $14,000
c. $16,000
d. $18,000

First cost, $ 840,000


M&O costs, $ per year 36,000
Benefits, $ per year 194,000
Disbenefits, $ per year ?

1.5 = (194,000 – D)/(840,000(0.10) + 36,000)


D = $14,000
Answer is (b)

7.42 If benefits are $10,000 per year forever starting in year 1, and the initial investment
is $50,000 at time zero, with a one-time M&O cost of $50,000 at the end of year 2,
the modified B/C ratio at i = 10% per year is closest to:
a. < 0.70
b. 0.93
c. 1.10
d. 1.20

Copyright 2021 © McGraw-Hill Education. All rights reserved. No reproduction or distribution without the
prior written consent of McGraw-Hill Education.
22
Modified B/C = [10,000/0.10 – 50,000(P/F,10%,2)]/50,000
= 58,680/50,000
= 1.17
Answer is (d)

7.43 Five mutually exclusive, direct benefit alternatives are compared using the B/C
method. The alternative to select is:
a. J
b. K
c. L
d. M

ΔB/C When Compared


Total Overall to Alternative
Investment, B/C
Alternative $ million Ratio J K L M N
J 20 1.02 —
K 25 1.18 1.20 —
L 33 1.12 1.42 0.98 —
M 45 0.89 0.72 0.80 0.08 —
N 49 0.94 0.84 0.89 0.76 1.32 —

Eliminate M and N because overall B/C < 1.0; compare J, K, and L incrementally.

K vs J: ∆B/C = 1.20 Eliminate J


L vs K: ∆B/C = 0.98 Eliminate L

Select K
Answer is (b)

7.44 For the six mutually exclusive alternatives shown, the incremental B/C ratio for the
comparison B vs. A is closest to:
a. 0.94
b. 1.07
c. 1.19
d. 1.28

Copyright 2021 © McGraw-Hill Education. All rights reserved. No reproduction or distribution without the
prior written consent of McGraw-Hill Education.
23
Alternative A B C D E F
Annual cost, $ per year 8,000 25,000 15,000 32,000 17,000 20,000
Annual benefits, $ per year ? ? ? ? ? ?
B/C ratio (Alternative vs. DN) 1.23 1.12 0.87 0.97 0.71 1.10

Benefits for A: B/8,000 = 1.23


B = $9,840

Benefits for B: B/25,000 = 1.12


B = $28,000

∆B/C = (28,000 – 9,840)/(25,000 – 8,000)


= 18,160/17,000
= 1.07
Answer is (b)

7.45 Herme owns and operates Dazzle Video Shop. He plans to purchase new VR
equipment and software for $10,000 and expects an annual net revenue boost of
$4000. However, one disbenefit is that he will lose income of $2500 per year on
other attractions where the VR will be located in the shop. If the MARR is 15% per
year, the PI for a planning period of 3 years is closest to:
a. 0.34
b. 0.58
c. 0.90
d. 1.10

Disregard disbenefit; use AW values; net annual revenue is (B – C) already stated.

PI = 4000/10,000(A/P,15%,3)
= 4000/4380
= 0.91
Answer is (c)

Copyright 2021 © McGraw-Hill Education. All rights reserved. No reproduction or distribution without the
prior written consent of McGraw-Hill Education.
24
Solutions to end-of-chapter problems
Basics of Engineering Economy, 3rd edition
Leland Blank and Anthony Tarquin

Chapter 8
Breakeven, Payback, Sensitivity, and Risk Analysis

8.1 Fixed costs for Universal Exports are $600,000 annually. Its main-line export item is
sold at a revenue of $2.10 per unit with variable costs of $1.50 per unit. (a) How
many units must be sold each year for breakeven? (b) What would the annual profit
be at sales of 1.3 million units?

(a) QBE = 600,000/(2.10 – 1.50)


= 1,000,000 units

(b) Profit = Revenue – Costs


= 1,300,000(2.10) – [600,000 + 1,300,000(1.50)]
= $180,000

8.2 A-Max Car Rental has a contract with a local car dealer to have major repairs done
for $450 per car. The rental agency estimates that for $280,000 they could develop
their own facility with a cost of $200 per car and have a $40,000 salvage value after
20 years. What is the minimum number of cars that must require major repairs every
year to make the change feasible, if the MARR is 10% per year?

–450x = –280,000(A/P,10%,20) + 40,000(A/F,10%,20) –200x


–450x = –280,000(0.11746) + 40,000(0.01746) –200x
–250x = –32,190.40
x = 129 cars per year

8.3 A call center for insurance-company inquiries has a capacity of 2 million calls per
year. The fixed cost of the center is $900,000 and the variable cost averages $1.90
per call. If the average revenue is $3.00 per call, determine the percentage of the
capacity that must be placed each year for the center to just break even.

QBE = 900,000/(3.00 – 1.90)


= 818,182 calls per year

This is 40.9% of the center’s 2 million capacity

8.4 Jake buys items using his eBay.com account for other people. Last year he purchased
310 items at an average cost of $35.00 per item; plus he charges a 15% fee for each
item he purchases for others. He figures his setup at home has a fixed cost of $50.00

Copyright 2021 © McGraw-Hill Education. All rights reserved. No reproduction or distribution without the
prior written consent of McGraw-Hill Education.
1
per day (220 work days per year) and a varible cost of $1.50 per item. (a) How many
items did he buy compared to his breakeven point? (b) What was his net profit or
loss for the year?

(a) QBE = 50(220)/(35(1.15) – 1.50)


= 11,000/38.75
= 284 items per year

He sold 310 – 284 = 26 more items than breakeven

(b) Profit = 35(1.15)(310) – 11000 – 1.50(310)


= $1012.50

8.5 A company that sells reverse-osmosis water purifiers has the following fixed and
variable cost components for its product over a 1-year period. Determine the revenue
per unit required to break even if the sales volume is estimated to be 5000 units.

Fixed Costs, $ Variable Costs, $ per Unit

Administrative 10,000 Materials 5


Lease cost 20,000 Labor 3
Insurance 7,000 Indirect labor 5
Utilities 3,000 Other overhead 20
Taxes 10,000
Other operations 50,000

Fixed Costs = 10,000 + 20,000 + 7,000 + 3,000 + 10,000 + 50,000


= $100,000

Variable Costs = 5 + 3 + 5 + 20
= $33 per unit

rQ = FC + vQ
r = [100,000 + 33(5,000)]/5,000
= $53 per unit

Alternate solution is:

r = FC/Q + v
= 100,000/5000 + 33
= $53 per unit

Copyright 2021 © McGraw-Hill Education. All rights reserved. No reproduction or distribution without the
prior written consent of McGraw-Hill Education.
2
Copyright 2021 © McGraw-Hill Education. All rights reserved. No reproduction or distribution without the
prior written consent of McGraw-Hill Education.
3
8.6 The manufacturer of a toy called SpinX has a capacity of 200,000 units annually. If
the fixed cost of the production line is $300,000 with a variable cost of $3 per unit
and revenue of $5 per unit, find the percent of present capacity that must be used to
break even.

QBE = FC/(r – v)
= 300,000/(5 – 3)
= 150,000 units

% capacity = (150,000/200,000)100%
= 75%

8.7 Almona, Inc. is your dad’s company. It manufactures electrostatic generators for
classroom demonstrations and has a capacity of 100,000 units per year. The fixed
cost of the process is $150,000. If the generators sell for $70 each, what is the
maximum variable cost per unit to break even when production is at 80% of
capacity?

R = 100,000(0.80)(70)
= $5,600,000

VC = 100,000(0.8)(v)

TC = 150,000 + 100,000(0.8)(v)

For breakeven v value, set TC equal to R

150,000 + 100,000(0.8)(v) = 5,600,000


v = $68.13 per unit

8.8 Brittany is co-oping this semester at Regency Aircraft, which customizes the
interiors of private and corporate jets. Her first assignment is to develop the
specifications for a new machine to cut, shape, and sew leather or vinyl covers and
trims. The first cost is not easy to estimate due to many options, but the annual
revenue minus M&O costs should net out at $15,000 per year over a 10-year life.
Salvage is expected to be 20% of the first cost. Determine what can be paid for the
machine now to recover the cost at a MARR of 8% per year under two scenarios:

I: No additional revenue will be received.


II:Outside contracting will generate additional revenue estimated at $10,000 the first
year, increasing by $5000 per year thereafter.

Solve using (a) tabulated factors, and (b) a spreadsheet with the GOAL SEEK tool.

Copyright 2021 © McGraw-Hill Education. All rights reserved. No reproduction or distribution without the
prior written consent of McGraw-Hill Education.
4
(a) Develop PW = 0 relation and solve for the breakeven first cost, P.

I: PW = –P + 0.2P(P/F,8%,10) + 15,000(P/A,8%,10)
0 = –P + 0.2P(0.4632) + 15,000(6.7101)
P = $110,928

II: PW = –P + 0.2P(P/F,8%,10) + 25,000(P/A,8%,10) + 5000(P/G,8%,10)


0 = –P + 0.2P(0.4632) + 25,000(6.7101) + 5000(25.9768)
P = $328,025

(b) Spreadsheet solution uses GOAL SEEK for each scenario to find P. Set PW
values (cells B13 and D13) to 0 and change values in cells B2 and D2 for
breakeven P values.

8.9 ABB purchased fieldbus communication equipment for a project in South Africa for
$3.15 million. If net cash flow is estimated at $500,000 per year, and a salvage value
of $400,000 is anticipated, determine how many years the equipment must be used
just to break even at interest rates ranging from 8% to 15% per year. Solve using (a)
tabulated factors, and (b) a spreadsheet.

(a) Using the relation PW = 0, solve for n at i values of 8% and 15%.

0 = –3,150,000 + 500,000(P/A,i%,n) + 400,000(P/F,i%,n)

i = 8%, n = 8: PW = –3,150,000 + 500,000(5.7466) + 400,000(0.5403)


= $–60,580
i = 8%, n = 9: PW = –3,150,000 + 500,000(6.2469) + 400,000(0.5002)
= $173,530
n = 8.2 years (by interpolation)

i = 15%, n = 19: PW = –3,150,000 + 500,000(6.1982) + 400,000(0.0703)


= $–22,780
i = 15%, n = 20: PW = –3,150,000 + 500,000(6.2593) + 400,000(0.0611)
= $4090
n = 19.8 years (by interpolation)

Copyright 2021 © McGraw-Hill Education. All rights reserved. No reproduction or distribution without the
prior written consent of McGraw-Hill Education.
5
(b) Breakeven retention ranges from 8 (at MARR = 8%) to 20 years (at MARR =
15%) for varying i values. This is a perfect example of where the spreadsheet is
very useful. Use the NPER function to determine n at each i value.

8.10 Because she lives in a large metropolitan city, Janice sold her car and received
approval to re-lease her reserved parking spot for the next four months so she can
make some extra money. The rental fee is $200 per month, and she expects to
charge $21 per day to one-day parkers. Transportation in a car pool will cost her
$8 per day. If there are a maximum of 20 work days per month for re-leasing the
spot, determine the following:
a. Total cost and revenue relations
b. Breakeven number of days per month
c. Her profit (or loss) if the number of re-leased days are 18, 19, 17, and 16 days,
respectively, over the 4-month period

(a) Let n = number of days per month to breakeven, where n ≤ 20. Since the car
pool charge is every day, the TC is a fixed cost. There is no variable cost.

TC = 200 + 8(20) = 360


R = 21n

(b) nBE = FC/r = 360/21


= 17.1 days per month

(c) There are 80 workdays in the 4 months. A total of 18+19+17+16 = 70 days are
re-leased.

Profit/Loss = R – TC
= 21(70) – 4(200) – 8(80)
= 1470 – 1440
= $30 (profit)

Copyright 2021 © McGraw-Hill Education. All rights reserved. No reproduction or distribution without the
prior written consent of McGraw-Hill Education.
6
8.11 An IE works in the automation department of a surgical equipment manufacturing
company that produces specially-ordered equipment for hospitals. To upgrade the
quality of the assembly process of the camera used in laparoscopic surgery probes,
two approaches are available: make and buy. The make alternative has an initial
equipment cost of $175,000, a life of 5 years, a $25,000 salvage value, a processing
cost of $3000 per camera, and an M&O cost of $60,000. The buy alternative
requires contracting the assembly operation externally at a cost of $8100 per
camera. If the MARR is 12% per year, how many cameras per year must be
assembled to justify the make alternative?

Let N = breakeven number of cameras per year

–8100N = –175,000(A/P,12%,5) + 25,000(A/F,12%,5) – 60,000 – 3000N


–8100N = –175,000(0.27741) + 25,000(0.15741) – 60,000 – 3000N
–5100N = –104,612
N = 20.5 cameras per year

At 21 or more cameras per year, the make alternative is justified.

8.12 You need to purchase new tires for the SUV that was willed to you by your favorite
aunt. The all-season light truck tires cost $90 each and are expected to last
30,000 miles. The premium brand on-off-road light truck tires cost $160 each.
Assuming you drive 10,000 miles per year, how many years will the premium tires
have to last for them to be as economically attractive as the all-season tires at an
interest rate of 10% per year? Use factors and a spreadsheet function to find the
breakeven point.

For all-season tires, n = 30,000/10,000 = 3 years. Equate the AW relations.

Factors: –90(A/P,10%,3) = –160(A/P,10%,n)


–90(0.40211) = –160(A/P,10%,n)
(A/P,10%,n) = 0.22619

From 10% table, n is between 6 and 7 years. By interpolation,

n = 6.1 years

Spreadsheet: Function = NPER(10%,.–90,160) displays an n of 6.04 years

8.13 A project manager received two proposals from a contractor to improve the staff
parking area at AGR Industries. Proposal A includes filling, grading, and paving at
an initial cost of $50,000. The life of the parking lot constructed in this manner is
expected to be 4 years, with an annual cost for maintenance and repainting of strips
at $3000. Proposal B provides a higher-quality pavement with an expected life of
12 years. The annual maintenance cost will be negligible, but the markings will

Copyright 2021 © McGraw-Hill Education. All rights reserved. No reproduction or distribution without the
prior written consent of McGraw-Hill Education.
7
have to be repainted in year 6 at a cost of $5000. If the MARR is 12% per year,
determine the amount AGR can spend on proposal B for the two to just break even.

The breakeven equation takes the form AWA = AWB, where PB = first cost of B.

–50,000(A/P,12%,4) – 3,000 = –PB(A/P,12%,12) –5,000(P/F,12%,6)(A/P,12%,12)


–50,000(0.32923) –3,000 = –PB(0.16144) –5,000(0.5066)(0.16144)
–19,461.50 = –PB(0.16144) – 408.93
19,052.57 = PB(0.16144)
PB = $118,016

8.14 Two small chemical feed pumps are under consideration for pumping a corrosive
liquid from a catalytic reforming process in a refinery. A pump with a brass
impeller costs $800 and is expected to last 3 years. A pump with a stainless-steel
impeller will cost $1900 and last 5 years. An overhaul costing $300 will be required
after 2000 operating hours for the brass impeller pump, while an overhaul costing
$700 will be required for the stainless-steel pump after 9000 hours. If the operating
cost of each pump is $0.50 per hour, how many hours per year must the pump be
operated to justify the purchase of the more expensive pump at an interest rate of
10% per year? Solve using (a) factors, and (b) the PMT spreadsheet function and
GOAL SEEK tool.

(a) Factors: Let H = breakeven hours per year; use AW values

800(A/P,10%,3) + (300/2000)H + 0.5H = 1,900(A/P,10%,5) + (700/9000)H + 0.5H


800(0.40211) + 0.15H + 0.5H = 1,900(0.2638) + 0.078H + 0.5H
0.072H = 179.532
H = 2,493.5 hours per year

(b) Spreadsheet: Develop two PMT functions; set initial hours per year (cell C3
before GOAL SEEK) at a number, say, 1000; use GOAL SEEK to
force the difference of the PMT results to 0.00 by changing I3
(after GOAL SEEK). Display is 2485.7 hours per year to
breakeven.

Copyright 2021 © McGraw-Hill Education. All rights reserved. No reproduction or distribution without the
prior written consent of McGraw-Hill Education.
8
8.15 Copper and nickel electroless plating processes are under consideration for printed
circuit boards. The copper process has fixed costs of $110,000 per year with a
variable cost of $50 per batch. The nickel process has a fixed cost of $85,000 per
year and a variable cost of $90 per batch. Determine the number of batches that
must be produced each year in order for the processes to break even.

Let x = number of batches

–110,000 – 50x = –85,000 – 90x


x = 625

8.16 The Rite family wishes to insulate the attic of their home to prevent heat loss. They
are considering R-l1 and R-19 insulation. They can install R-l1 for $1600 and R-19
for $2400. They expect to save $150 per year in heating and cooling expenses if
R-l1 is installed. How much extra must they save in utility expenses per year in
order to justify the R-19 insulation if they expect to recover the extra cost in 7 years
at an interest rate of 6% per year?

Let x = savings per year for R-19

–1,600(A/P,6%,7) + 150 = –2,400(A/P,6%,7) + x


–1,600(0.17914) + 150 = –2,400(0.17914) + x
–136.62 = –429.94 + x
x = $293.32 per year

Extra savings = 293.32 – 150 = $143.32 per year

8.17 Providing restrooms at parks, zoos, and other city-owned recreation facilities is a
considerable expense for municipal governments. City councils usually opt for
permanent restrooms in larger parks and portable restrooms in smaller ones. The
cost of renting and servicing a portable restroom is $7500 per year. In one
northeastern municipality, the parks director informed the city council that the cost
of constructing a permanent restroom is $218,000, and the annual cost of
maintaining it is $12,000. He remarked that the rather high cost is due to the
necessity to use expensive materials and construction techniques that are tailored to
minimize damage from vandalism that often occurs in unattended public facilities.
If the useful life of a permanent restroom is assumed to be 20 years, how many
portable restrooms could the city afford to rent each year and break even with the
cost of one permanent facility? Use an interest rate of 6% per year.

Let N = number of portables per year

–7500N = –218,000(A/P,6%,20) – 12,000


–7500N = –218,000(0.08718) – 12,000
–7500N = –31,005
N = 4.1

The city could afford four portables per year

Copyright 2021 © McGraw-Hill Education. All rights reserved. No reproduction or distribution without the
prior written consent of McGraw-Hill Education.
9
8.18 A waste-holding lagoon situated near the main plant of a chemical company
receives sludge on a daily basis. When the lagoon is full, it is necessary to remove
the sludge to a site located 4.95 kilometers from the main plant. Currently, when the
lagoon is full, the sludge is removed by pumping it into a tank truck and hauling it
away. This process requires the use of a portable pump that has an initial cost of
$800 and an 8-year life. The company can operate the pump at a cost of $25 per
day, but the truck and driver cost $110 per day.
Alternatively, the company has a proposal to install a pump and pipeline to the
remote site. The pump would have an initial cost of $600, a life of 10 years, and a
cost of $3 per day to operate. (a) If the MARR is 15% per year and the pipeline will
cost $3.52 per meter to construct, how many days per year must the lagoon require
pumping to justify construction of the pipeline? (b) The lagoon was pumped
40 times last year. Which alternative would have been cheaper? How do you know
this is the correct answer?

(a) Let x = days per year to pump the lagoon

AWtruck = AWpipeline
–800(A/P,15%,8) – 135x = –600(A/P,15%,10) – 3.52(4,950)(A/P,15%,10) – 3x
–800(0.22285) – 135x = –600(0.19925) – 17,424(0.19925) – 3x
–178.28 – 135x = –3,591.28 – 3x
x = 25.9 days per year

(b) The pipeline alternative, since it has a smaller variable cost slope (3 vs 135).

8.19 Brooke is evaluating two alternatives for improving (b) Spreadsheet and GOAL SEEK:
the exterior appearance of her Victorian-style house Difference of PW values set to
that she is remodeling inside. She plans to keep this 0.0 (C25) by changing C2.
as her home for 20 more years. The house can be Display is PS of $22,561.
completely painted at a cost of $12,800. The paint is
expected to remain attractive for 5 years, at which
time repainting will be necessary. Every time the
building is repainted (i.e., in years 5, 10, and 15), the
cost will increase by 20% over the previous time.
As an alternative, the exterior can be
covered with a vintage-appearing vinyl-coated
siding now and again 10 years from now at a cost
40% greater than the present cost of the siding.
At a MARR of 10% per year, what is the maximum
amount that Brooke should spend now on the
siding alternative so that the two alternatives will
just break even? Solve using (a) factors, and (b) a
spreadsheet.

(a) Factors: Use P for painting, S for siding; set PW

Copyright 2021 © McGraw-Hill Education. All rights reserved. No reproduction or distribution without the
prior written consent of McGraw-Hill Education.
10
relations equal; solve for PS.

Copyright 2021 © McGraw-Hill Education. All rights reserved. No reproduction or distribution without the
prior written consent of McGraw-Hill Education.
11
PWP = –12,800 – 12,800(1.20)(P/F,10%,5)
– 12,800(1.20)2 (P/F,10%,10)
– 12,800(1.20)3(P/F,10%,15)
= –12,800 – 15,360(0.6209) – 18,432(0.3855)
–22,118(0.2394)
= $–34,750

PWS = –PS – PS(1.40)(P/F,10%,10)


= –PS – PS(1.40)(0.3855)
= –1.5397PS

–34,750 = –1.5397PS
PS = $22,569

8.20 Samsung Electronics is trying to reduce supply chain risk by making more
responsible make/buy decisions through improved cost estimation. A high-use
component (expected usage is 5000 units per year) can be purchased for $25 per
unit with delivery promised within a week. Alternatively, Samsung can make the
component inhouse and have it readily available at a cost of $5 per unit, if
equipment costing $150,000 is purchased. Labor and other operating costs are
estimated to be $35,000 per year over the study period of 5 years. Salvage is
estimated at 10% of first cost and MARR is 12% per year. Neglect the element of
availability (a) to determine the breakeven quantity, and (b) to recommend making
or buying at the expected usage level.

(a) Solve the relation AWbuy = AWmake for Q = number of units per year.

–25Q = –150,000(A/P,12%,5) + 15,000(A/F,12%,5) – 35,000 – 5Q


–20Q = –150,000(0.27741) + 15,000(0.15741) – 35,000
Q = –74,250/–20
= 3713 units per year

(b) Since 5000 > 3713, select the make option. It has the smaller slope of 5 versus
25 for the buy option.

8.21 A rural subdivision has several miles of access road that needs a new surface
treatment. Alternative 1 provides a gravel base and pavement with an initial cost of
$500,000 that will last for 15 years and has an annual upkeep cost of $100 per mile.
Alternative 2 enhances the gravel base now at a cost of $50,000 and immediately
(i.e., time 0) coats the surface with a durable hot oil mix, which costs $130 per
barrel applied. A barrel covers 0.05 mile. Annual reapplication of the mix is
required. (a) If the discount rate is 6% per year, determine the number of miles at
which the two alternatives break even. (b) A drive in a pickup indicates a total of
12.5 miles of road. Which is the more economical alternative?

Copyright 2021 © McGraw-Hill Education. All rights reserved. No reproduction or distribution without the
prior written consent of McGraw-Hill Education.
12
(a) Solve the relation PW1 = PW2 for x miles

–500,000 –100x(P/A,6%,15) = –50,000 – [(130/0.05)x(1+ (P/A,6%,15)]


–100(9.7122)x + 2600(1 + 9.7122)x = – 50,000 + 500,000
x = 450,000/26,881
= 16.7 miles

(b) Since 12.5 < 16.7 miles, select alternative 2; it has the steeper slope of 2600 vs
100 times the P/A factor.

8.22 Three methods can be used to produce heat sensors for high-temperature furnaces.
Estimates are made for each method: A, B, and C. At an interest rate of 10% per
year, determine the breakeven annual production rate between the two lowest cost
methods.

Method A B C

First cost, $ −500,000


Fixed cost, $/year −140,000 −210,000
Variable cost, $/unit 62 28 53
Salvage value, $ 125,000
Life, years 5

Fixed cost, method C = –500,000(A/P,10%,5) + 125,000(A/F,10%,5)


= –500,000(0.2638) + 125,000(0.1638)
= $–111,425

Method A has a higher fixed cost and higher variable cost than Method C.
Eliminate A; determine breakeven between B and C.

Let x = number units per year to breakeven

–210,000 – 28x = –111,425 – 53x


x = 98,575/25
= 3943 sensors/year

8.23 In your junior year, you have been elected president of your student engineering
organization on campus. A computer and color printer are needed to support
activities planned for the fall and spring semesters. You can lease a fully equipped
system for $800 per month or purchase one for $8500 now and pay a $75 per month
maintenance fee. If the nominal interest rate is 15% per year, determine the months
of use necessary for the two options to break even using (a) tabulated factors, and

Copyright 2021 © McGraw-Hill Education. All rights reserved. No reproduction or distribution without the
prior written consent of McGraw-Hill Education.
13
(b) a single-cell spreadsheet function. (c) Not knowing what the next president will
want, you use a 9-month study period. Which option is economically better?

(a) Solve AWlease = AWbuy for n = number of months. Monthly i = 15/12 =1.25%

–800 = –8500(A/P,1.25%,n) – 75 = 0

For n = 12: –800 + 8500(0.09026) + 75 = $42.21 n is low


For n = 13: –800 + 8500(0.08382) + 75 = $–12.53 n is high

n = 12.8 months (by interpolation)

(b) The function = NPER(1.25%,–725,8500) displays an n of 12.8 months. The


–725 is the difference of the two monthly costs (–800 + 75 = –725).

(c) Since 9 < 12.8 months, select the lease option.

8.24 The Ecology Group wishes to purchase a piece of equipment for various metals
recycling. Machine 1 costs $123,000, has a life of 10 years, an AOC of $5000, and
requires one operator at a cost of $24 per hour to process 10 tons per hour. Machine
2 costs $70,000, has a life of 6 years, an AOC of $2500, and requires two operators
at a cost of $24 per hour for each operator to process 6 tons per hour.
a. Determine the breakeven tonnage of scrap metal at i = 7% per year and select the
better machine for a processing level of 1000 tons per year.
b. Calculate and plot the sensitivity of the breakeven tons per year to ± 15% change
in the hourly cost of an operator. Use 5% increments in the analysis.

(a) Let T = number of tons. Solve relation AW1 = AW2 for T.

Variable costs (VC) for each machine


VC1: –24T/10 = –2.4T VC2: –2(24)T/6 = –8T

–123,000(A/P,7%,10) – 5000 – 2.4T = –70,000(A/P,7%.6) – 2500 – 8T


(8–2.4)T = –70,000(0.20980) – 2500 + 123,000(0.14238) + 5000
5.6T = 5327
T = 951.2 tons

If tonnage is < breakeven, select machine 2; the slope is steeper (8 vs 2.4).

Since 1000 > 951.2, tons, select machine 1.

Copyright 2021 © McGraw-Hill Education. All rights reserved. No reproduction or distribution without the
prior written consent of McGraw-Hill Education.
14
(b) Set up the VC relation for each machine and solve for T in AW1 = AW2

Cost, AW1 = AW2 Breakeven


% change $/hour VC1 VC2 relation T value, tons
–15% 20.4 2.04 6.8T 4.76T = 5327 1119
T
–5% 22.8 2.28 7.6T 5.32T = 5327 1001
T
0% 24.0 2.40 8.0T 5.60T = 5327 951
T
Spreadsheet note: +5% 25.2 2.52 8.4T 5.88T = 5327 906
SOLVER can be T
used to find and +15% 27.6 2.76 9.2T 6.44T = 5327 827
plot breakeven T
with the
constraint AW1 = AW2.

8.25 Assume you started a sideline business in commercial photography last year using
your then-owned equipment. Due to excellent success, you plan to purchase new
equipment and upgrade your studio facility.
a. Determine the no-return payback period.
b. If you set a study period of 8 years and hope for a return of 15% per year, will
you meet your goal? First use factors, then the NPER function to answer this
question.

First cost of equipment, $ −130,000


Annual expenses, $ per year −45,000
Annual revenue, $ per year 75,000

(a) Let np = number of years until payback at 0%

Copyright 2021 © McGraw-Hill Education. All rights reserved. No reproduction or distribution without the
prior written consent of McGraw-Hill Education.
15
0 = –130,000 – 45,000np + 75,000np
30,000np = 130,000
np = 4.3 years

(b) Factor: 0 = –130,000 + 30,000(P/A,15%,np)


(P/A,15%,np) = 4.3333

np is between 7 and 8 years

Spreadsheet: Function = NPER( 15%,30000,–130000) displays 7.5 years

Yes, you will meet the goal within the 8 years.

8.26 Determine the payback period at an interest rate of 8% per year for an asset that
initially cost $28,000, has a salvage value of $1500 whenever it is sold, and
generates cash flow of $2900 per year. Use factors and a spreadsheet.

Factors: 0 = –28,000 + 2900(P/A,8%,np) + 1500(P/F,8%,np)

n = 15: 0 = –28,000 + 2900(8.5595) + 1500(0.3152) = $–2705 too low


n = 20: 0 = –28,000 + 2900(9.8181) + 1500(0.2145) = $+794 too high

np = 18.7 years

Spreadsheet: = NPER(8%,2900,–28000,1500)displays an np of 18.7 years

8.27 A vibrating sieve shaker in a soil analysis laboratory has a first cost of $8000 and an
expected salvage value of $500 when it is sold. A positive cash flow of only $900
per year can be attributed directly to the shaker. (a) Determine the payback period if
the required return is 8% per year. (b) If the asset will be in service for an estimated
12 years, should it be purchased?

(a) The payback period np is found by trial and error using PW

0 = –8,000 + 900(P/A,8%,np) + 500(P/F,8%,np)

n = 15: 0 = –8,000 + 900(8.5595) + 500(0.3152) = $–138.85 too low


n = 16: 0 = –8,000 + 900(8.8514) + 500(0.2919) = $+112.21 too high

np = 15.6 years

(b) Since 12 < np the shaker should not be purchased.

8.28 You work part time selling mobile phone ads on a commission basis. (a) Using
factors and a spreadsheet, determine the number of years you would have to work

Copyright 2021 © McGraw-Hill Education. All rights reserved. No reproduction or distribution without the
prior written consent of McGraw-Hill Education.
16
to break even, if income is estimated to be $5000 per year, expenses are $1500 per
year, your initial investment was $28,000, and your MARR is 10% per year. (b)
Determine the no-return payback period and then determine the minimum annual
income necessary to just break even over this no-return payback period, if you
imposed the MARR of 10% per year and expenses remain at $1500 per year.

(a) Factor: Develop the AW relation and solve for np

–28,000(A/P,10%,np) + 5,000 – 1,500 = 0


(A/P,10%,np) = 0.125

From 10% tables and interpolation, n = 16.9 years

Spreadsheet: = NPER(10%,3500,–28000) displays 16.9 years

(b) Spreadsheet function = NPER(0%,3500,–28000) displays 8 years.


Let R = annual income

–28,000(A/P,10%,8) + R – 1500 = 0
R = 28,000(0.18744) + 1500
= $6748 per year

8.29 How long would it take to recover an investment of $245,000 in enhanced CNC
controls that include axis control to eight axes on the milling model, if the
associated income is $92,000 per year, expenses are $38,000 per year, and the
salvage value is estimated to be 15% of the first cost? Use a MARR of 15% per
year. Solve using factors and write the spreadsheet function that determines np.

(a) Develop the PW relation and solve for np

–245,000 + (92,000 – 38,000)(P/A,15%,np) + 245,000(0.15)(P/F,15%,np) = 0

Solve by trial and error:


n = 7: –245,000 + (92,000 – 38,000)(4.1604) + 36,750(0.3759) = 0
–6524 < 0 low
n = 8: –245,000 + (92,000 – 38,000)(4.4873) + 36,750(0.3269) = 0
+9327 > 0 high
By interpolation, np = 7.4 years

(b) Spreadsheet: = NPER(15%,54000,–245000,0.15*245000) displays 7.4 years

8.30 The negotiated price of a car you want is $42,000 today. Its price is expected to
increase by $1000 each year. You now have $25,000 in an investment account that
is earning 10% per year. How many years will it be before you have enough to buy

Copyright 2021 © McGraw-Hill Education. All rights reserved. No reproduction or distribution without the
prior written consent of McGraw-Hill Education.
17
the car without borrowing any money? Solve by (a) factors and trial and error, and
(b) spreadsheet.

42,000 + 1000np = 25,000(F/P,10%,np)

Copyright 2021 © McGraw-Hill Education. All rights reserved. No reproduction or distribution without the
prior written consent of McGraw-Hill Education.
18
(a) Factors and trial and error:

Try n = 7: 49,000 > 48,718 too low


Try n = 8: 50,000 < 53,590 too high

By interpolation, np = 7.08 years

(b) Spreadsheet: Use GOAL SEEK to force the difference to be zero by changing
cell B3. Payback displayed is np = 7.08 years.

8.31 How long will Wall’s Pharmacy have to sell a new product that has an estimated
revenue of $5000 per month and costs of $1500 per month if the initial purchase
was $28,000 and the MARR is (a) 0% and (b) 3% per month? (c) Write the
spreadsheet functions to display np for both 0% and 3% per month.

(a) np = 28,000/(5000–1500) = 8 months

(b) 0 = –28,000 + (5000 – 1500)(P/A,3%,np)

n = 8 months: –28,000 + 3500(7.0197) = $–3431 too low

n = 10 months: –28,000 + 3500(8.5302) = $1856 too high

n = 9.3 months (interpolation)

(c) 0%: = NPER(0%,3500,–28000) displays 8.0 months

3%: = NPER(3%,3500,–28000) displays 9.3 months

Copyright 2021 © McGraw-Hill Education. All rights reserved. No reproduction or distribution without the
prior written consent of McGraw-Hill Education.
19
8.32 An investor wants to know how many years he must retain ownership of a
commercial property to make a current market return of 8% per year. The purchase
price is $60,000 with taxes of $1800 the first year, increasing by $100 each year
until sold. Assume the property must be retained for at least 2 years and that the
sales price is estimated at $90,000 for years 3 and beyond. (a) Determine the
payback period using factors. (b) (Spreadsheet exercise) Determine the payback
period using a spreadsheet.

(a) Set up the PW relation for n ≥ 3 years

0 = –60,000 – 1,800(P/A,8%,np) – 100(P/G,8%,np) + 90,000 (P/F,8%,np)

n PW, $
3 6,562
4 –274
5 – 6,672

By interpolation, PW = 0 at np = 3.96 years

(b) With a gradient, NPER function will not work directly. Enter cash flows and
use GOAL SEEK. Must go one year beyond payback period to obtain a
correct answer to accommodate the tax gradient for the last year of ownership.
Sample spreadsheet shows GOAL SEEK result forcing PW to 0.00 (cell D6)
be changing A6. Result is np = 3.95 years.

8.33 The Sundance Detective Agency has purchased new surveillance equipment with
the following estimates. The year index is k = 1, 2, 3, ...
First cost = $1050
Annual maintenance cost = $75 + 5k
Extra annual revenue = $250 + 50k
Salvage value = $600 for all years
Calculate the payback period to make a return of 10% per year.

Copyright 2021 © McGraw-Hill Education. All rights reserved. No reproduction or distribution without the
prior written consent of McGraw-Hill Education.
20
0 = –1,050 + 600(P/F,10%,np) + 175(P/A,10%,np) + 45(P/G,10%,np)

n = 3: $–59.20 too low


n = 4: $+111.55 too high

By interpolation, payback period is np = 3.3 years

8.34 Kahn Instruments is considering an investment of $500,000 in a new product line.


The company will make the investment only if it will result in a rate of return of
15% per year or higher. If the revenue is expected to be between $135,000 and
$165,000 per year for 5 years, (a) determine if the decision to invest is sensitive to
the projected range of income using a PW analysis. (b) If the decision is sensitive to
revenue, determine the revenue per year below which the investment would not
yield the 15% rate of return.

(a) $135,000: PW = –500,000 + 135,000(P/A,15%,5)


= –500,000 + 135,000(3.3522)
= $–47,453 (ROR < 15%)

$165,000: PW = –500,000 + 165,000(P/A,15%,5)


= –500,000 + 165,000(3.3522)
= $53,113 (ROR > 15%)

The decision to invest is sensitive to the revenue estimates.

(b) Factor: Let R = revenue to make PW = 0

0 = –500,000 + R(P/A,15%,5)
R = 500,000/3.3522
= $149,156

Spreadsheet: = – PMT(15%,5,–500000) displays $149,157.78

8.35 A young couple planning ahead for their retirement has decided that $2,600,000 is
the amount they will need to retire comfortably 20 years from now. For the past
5 years, they have been able to invest one of their salaries ($50,000 per year, which
includes employer contributions) while living off the other one. They plan to start a
family sometime in the next 10 years, and when they have done so, one of the
parents will quit working, causing the savings to decrease to $15,000 per year
thereafter. If they have gotten a rate of return of 10% per year on their investments,
and expect to continue at this ROR, is reaching their goal of $2.6 million in 20 more
years sensitive to when they have their first child (i.e., between now and 10 years
from now)? Use an FW analysis starting now with one year increments thereafter.

Copyright 2021 © McGraw-Hill Education. All rights reserved. No reproduction or distribution without the
prior written consent of McGraw-Hill Education.
21
Start family now:
FW = 50,000(F/A,10%,5)(F/P,10%,20) + 15,000(F/A,10%,20)
= 50,000(6.1051)(6.7275) + 15,000(57.2750)
= $2,912,728
Since FW > $2.6 million, their retirement goal is not sensitive to when they start
their family.

8.36 Because of international competition, a company that manufactures high-speed


submersible rotary indexing spindles must upgrade its production equipment to
reduce costs over a 6-year planning horizon. The company can invest $80,000 one
year from now, 2 years from now, or 3 years from now. Depending on when the
investment is made, the savings will vary. That is, the savings will be $26,000,
$30,000, or $38,000 per year if the investment is made 1, 2, or 3 years from now,
respectively. Will the timing of the investment affect the request to make at least
20% per year return? Use future worth analysis and solve using (a) factors, and (b)
a spreadsheet.

(a) Factors:
Invest in 1 year: FW = –80,000(F/P,20%,5) + 26,000(F/A,20%,5)
= –80,000(2.4883) + 26,000(7.4416)
= $–5582 (< 20% per year)

Invest in 2 years: FW = –80,000(F/P,20%,4) + 30,000(F/A,20%,4)


= –80,000(2.0736) + 30,000(5.3680)
= $–4848 (< 20% per year)

Invest in 3 years: FW = –80,000(F/P,20%,3) + 38,000(F/A,20%,3)


= –80,000(1.7280) + 38,000(3.6400)
= $80 (> 20% per year)

The timing will affect whether the company earns its MARR.

Conclusion: invest in 3 years.

(b) Spreadsheet: Use FV functions with cell referencing for years left and different
savings estimates. Return of 20% is met for year 3 investment.

Copyright 2021 © McGraw-Hill Education. All rights reserved. No reproduction or distribution without the
prior written consent of McGraw-Hill Education.
22
8.37 A machine currently used in manufacturing circuit board card locks has AW = $
−62,000 per year. A possible replacement is under consideration with a first cost of
$64,000 and an AOC of $38,000 per year for the next 3 years. Three different
engineers have given their opinion about what the salvage value of the new
machine will be 3 years from now: $10,000, $13,000, and $18,000. (a) Is the
decision to replace the machine sensitive to the salvage value estimates at the
MARR of 15% per year? (b) Write the form of the single-cell spreadsheet function
that will provide the AW values.

(a) AWcurrent = $–62,000

AW10,000 = –64,000(A/P,15%,3) – 38,000 + 10,000(A/F,15%,3)


= –64,000(0.43798) – 38,000 + 10,000(0.28798)
= $–63,151

AW13,000 = –64,000(A/P,15%,3) – 38,000 + 13,000(A/F,15%,3)


= –64,000(0.43798) – 38,000 + 13,000(0.28798)
= $–62,287

AW18,000 = –64,000(A/P,15%,3) – 38,000 + 18,000(A/F,15%,3)


= –64,000(0.43798) – 38,000 + 18,000(0.28798)
= $–60,847

The decision is sensitive to the salvage value estimates. If the salvage value will
be $18,000, the company should replace the existing machine. Otherwise, keep
the current one.
(b) The function form is = – PMT(15%,3,–64000,S) – 38000, where S is the
changing salvage estimate.

8.38 A biofuel subsidiary of Petrofac, Inc. is planning to borrow $12 million to acquire
and expand the product line of a small technology-based company. Due to the risk
involved, the rate on a 5-year loan is highly variable; it could be as low as 7%, as
high as 15%, but is expected to be 10% per year. Due to revenue projections, the
company will move forward only if the AW of total costs is below $5.7 million per
year. The M&O costs are estimated at $3.1 million per year. The anticipated sales
price of the company in 5 years could be $2.1 million if the interest rate is 7% or as
much as $2.5 million if the rate is 15%, but will most likely be about $2.3 million at
the 10% per year rate. Is the decision to move forward sensitive to the loan interest
rate and sales price estimates?

Required AW < $–5.7 million

7%: AW = –12,000,000(A/P,7%,5) – 3,100,000 + 2,100,000(A/F,7%,5)


= –12,000,000(0.24389) – 3,100,000 + 2,100,000(0.17389)
= $–5,661,511 (< $–5,700,000; acceptable)

Copyright 2021 © McGraw-Hill Education. All rights reserved. No reproduction or distribution without the
prior written consent of McGraw-Hill Education.
23
10%: AW = –12,000,000(A/P,10%,5) – 3,100,000 + 2,300,000(A/F,10%,5)
= –12,000,000(0.26380) – 3,100,000 + 2,300,000(0.16380)
= $–5,888,836 (> $–5,700,000; not acceptable)

15%: AW = –12,000,000(A/P,15%,5) – 3,100,000 + 2,500,000(A/F,15%,5)


= –12,000,000(0.29832) – 3,100,000 + 2,500,000(0.14832)
= $–6,309,040 (> $–5,700,000; not acceptable)

The decision is sensitive, since AW at 10% and 15% exceed maximum of


AW = $–5,700,000

Spreadsheet function format is = – PMT(i%,5,–12000000,sales_price) – 3100000

8.39 Your dad made the following estimates as he considered upgrading the air-
conditioning system in their home. Use AW analysis to determine the sensitivity of
the economic decision to interest rates of 4%, 6%, and 8% per year. Work this
problem by (a) factors, and (b) spreadsheet.

System 1 2

First cost, $ −10,000 −17,000


Annual operating cost, $ −600 −150
Salvage (disposal) value, $ −100 −300
New compressor and motor
cost at midlife, $ −1,750 −3,000
Life, years 8 12

(a) AW1 = –10,000(A/P,i%,8) – 600 – 100(A/F,i%,8) – 1750(P/F,i%,4)(A/P,i%,8)


AW2 = –17,000(A/P,i%,12) – 150 – 300(A/F,i%,12) – 3000(P/F,i%,6)(A/P,i%,12)

The decision is sensitive to MARR, changing at i = 6%.

MARR AW1 AW2 Selection


4% $–2318 $–2234 2
6% $–2444 $–2448 1
8% $–2573 $–2673 1

(b) Spreadsheet solution requires that the PW value is first determined using the
NPV function over the LCM of 24 years and then converting it to an AW value
using the PMT function.

Copyright 2021 © McGraw-Hill Education. All rights reserved. No reproduction or distribution without the
prior written consent of McGraw-Hill Education.
24
8.40 Ned Thompson Labs performs tests on superalloys, titanium, aluminum, and most
metals. Tests on metal composites that rely upon scanning electron microscope
results can be subcontracted or the labs can purchase new equipment. Evaluate the
sensitivity of the economic decision to purchase the equipment over a range of
±20% (in 10% increments) of the estimates for P, AOC, R, n, and MARR (range on
MARR is 12% to 18% in 1.5% increments). Use the AW method and plot the
results on a sensitivity graph. For which parameter(s) is the AW most sensitive?
Least sensitive?

First cost, P = $−180,000


Salvage, S = $20,000
Life, n = 10 year
Annual operating cost, AOC = $−30,000 per year
Annual revenue, R = $70,000 per year
MARR = 15% per year

Spreadsheet is for –20% to +20% changes in P, AOC, R, n and MARR. The


PMT function for a +20% change is detailed at the bottom of the spreadsheet.

AW is most sensitive to variations in revenue, R and least sensitive to variations in


life, n.

Copyright 2021 © McGraw-Hill Education. All rights reserved. No reproduction or distribution without the
prior written consent of McGraw-Hill Education.
25
8.41 Determine if the selection of system 1 or 2 is sensitive to variation in the return
required by management. The corporate MARR ranges from 8% to 16% per year.
In increments of 2%, use tabulated factors or a spreadsheet, as requested by your
instructor.

System 1 2

First cost, $ −50,000 −100,000


AOC, $ per year −6,000 −1,500
Salvage value, $ 30,000 0
Rework at midlife, $ −17,000 −30,000
Life, years 4 12

Factors: Determine AW at different MARR values between 8% and 16%.


Selection changes for MARR values between 12% and 14%.

AW1 = –50,000(A/P,i%,4) – 6,000 + 30,000(A/F,i%,4) –17,000(P/F,i%,2)(A/P,i%,4)

AW2 = –100,000(A/P,i%,12) – 1,500 – 30,000(P/F,i%,6)(A/P,i%,12)

MARR, % AW1 AW2 Selection


8 $–18,839 $–17,279 2
10 –19,742 –18,662 2
12 –20,647 –20,097 2
14 –21,555 –21,582 1

Copyright 2021 © McGraw-Hill Education. All rights reserved. No reproduction or distribution without the
prior written consent of McGraw-Hill Education.
26
16 –22,463 –23,110 1
Spreadsheet: PMT functions display AW values over 4 and 12 years, respectively.
Selection changes for MARR values between 12% and 14%.

8.42 Titan manufactures and sells gas-powered electricity generators. It can purchase a
new line of fuel injectors from either of two companies: A or B. The AOC and
annual savings estimates are available, but the savings estimate is unreliable at this
time. Use an AW analysis at MARR = 10% per year to determine if the selection
between A and B changes when the estimated savings varies as much as ±40% from
the best estimates, and if so, at what percentage in the estimate? Use tabulated
factors or a spreadsheet, as requested by your instructor.

Company A B

First cost, $ −50,000 −37,500


AOC, $ per year −7,500 −8,000
Savings best estimate,  15,000  13,000
$ per year
Salvage value, $ 5,000 3,700
Life, years 5 5

Factors: Determine AW values at different savings, s.

AWA = –50,000(A/P,10%,5) – 7500 + 5,000(A/F,10%,5) + s


= –50,000(0.2638) – 7500 + 5000(0.1638) + s
= –19,871 + s

AWB = –37,500(A/P,10%,5) – 8000 + 3700(A/F,10%,5) + s


= –37,500(0.2638) – 8000 + 3700(0.1638) + s

Copyright 2021 © McGraw-Hill Education. All rights reserved. No reproduction or distribution without the
prior written consent of McGraw-Hill Education.
27
= –17,286 + s
Percent Savings for A, Savings for
variation $ per year AWA B, AWB Selection
$ per year
–40% 9,000 $–10,871 7,800 $–9,486 B
–20 12,000 –7,871 10,400 –6,886 B
0 15,000 –4,871 13,000 –4,286 B
20 18,000 –1,871 15,600 –1,686 B
40 21,000 1,129 18,200 914 A
Selection changes when s = +40% of best estimate.

Spreadsheet: PMT functions display AW values with savings variation added to end
of function. Again, selection changes when s = +40% of best estimate.

8.43 DVH Technologies purchases several parts for the instruments it makes via a fixed-
price contract of $190,000 per year from a local supplier. The company is
considering making the parts inhouse through the purchase of equipment that will
have a first cost of $240,000 with an estimated salvage value of $30,000 after
5 years. The AOC is difficult to estimate, but company engineers have made
optimistic, most likely, and pessimistic estimates of $60,000, $85,000, and
$120,000, respectively. Use (a) spreadsheet functions, and (b) factors to determine
if the company should purchase the equipment under any of the AOC scenarios.
The MARR is 20% per year.

(a) Spreadsheet: Develop three PMT functions of the form


= – PMT(20%,5,–240000,30000) – AOC

$60,000: AWOptimistic display is $–136,220


$85,000: AWMostLikely display is $–161,220
$120,000: AWPessimistic display is $–196,220

At the pessimistic estimate ($120,000), select the contract. At others, select


the make option.

Copyright 2021 © McGraw-Hill Education. All rights reserved. No reproduction or distribution without the
prior written consent of McGraw-Hill Education.
28
(b) Factors:
AWContract = $–190,000

AWOptimistic = –240,000(A/P,20%,5) – 60,000 + 30,000(A/F,20%,5)


= –240,000(0.33438) – 60,000 + 30,000(0.13438)
= $–136,220 ( < $–190,000; purchase equipment)

AWMostLikely = –240,000(A/P,20%,5) – 85,000 + 30,000(A/F,20%,5)


= –240,000(0.33438) – 85,000 + 30,000(0.13438)
= $–161,220 ( < $–190,000; purchase equipment)

AWPessimistic = –240,000(A/P,20%,5) – 120,000 + 30,000(A/F,20%,5)


= –240,000(0.33438) – 120,000 + 30,000(0.13438)
= $–196,220 ( > $–190,000; do not purchase equipment)

The optimistic and most likely estimates favor purchasing the equipment, but
the pessimistic estimate does not.

8.44 Astor Engineering recently merged with another firm and could lease additional
office space or purchase its own building. The $30,000 per year lease agreement
will be a net, net, net lease, which means that the lessee (Astor) will pay the real
estate taxes on the leased space, the building insurance on the leased space, and the
common area maintenance. Since these costs are about the same if Astor owned the
building, they do not need to be considered in the analysis. A new building will cost
$880,000 to purchase, but there is considerable uncertainty about what it will be
worth in 20 years, which is the planning period selected. The individuals involved
in the discussion made optimistic, most likely, and pessimistic estimates of
$2,400,000, $1,400,000, and $900,000, respectively. Determine if Astor should
purchase the building under any of the estimated resale values at i = 10% per year.
Also, write the form of the spreadsheet function that will provide the AW values.

Let L = lease, O = optimistic, ML = Most likely, P = pessimistic

AWL = $–30,000 per year

AWP = –880,000(A/P,10%,20) + 900,000(A/F,10%,20)


= –880,000(0.11746) + 900,000(0.01746)
= $–87,651

AWML = –880,000(A/P,10%,20) + 1,400,000(A/F,10%,20)


= –880,000(0.11746) + 1,400,000(0.01746)
= $–78,920

AWO = –880,000(A/P,10%,20) + 2,400,000(A/F,10%,20)


= –880,000(0.11746) + 2,400,000(0.01746)
= $–61,461

Copyright 2021 © McGraw-Hill Education. All rights reserved. No reproduction or distribution without the
prior written consent of McGraw-Hill Education.
29
It would not be cost effective to purchase the building under any resale-value
scenario.

Spreadsheet: Function form is = – PMT(10%,20,–880000, resale estimate)

8.45 Holly Farms is considering two environmental chambers to accomplish detailed


laboratory confirmations of online bacteria tests in chicken meat for the presence of
E. coli 0157:H7 and Listeria monocytogenes. There is some uncertainty about how
long the D103 chamber will be useful. A realistic estimate is 3 years, but
pessimistic and optimistic estimates of 2 years and 6 years, respectively, are also
reasonable. The estimated salvage value will remain the same. Using an interest rate
of 10% per year, determine if any of the D103 estimates would result in a lower
cost than that of the 490G chamber for a 6-year planning period.

Chamber D103 Chamber 490G

Installed cost, P, $ −400,000 −200,000


AOC, $ per year  −4,000 −3,000
Salvage value at 10% of P, $   40,000   20,000
Life, years 2, 3, or 6 2

AW490G = –200,000(A/P,10%,2) – 3000 + 20,000(A/F,10%,2)


= –200,000(0.57619) – 3000 + 20,000(0.47619)
= $–108,714

AWD103 2-year life = –400,000(A/P,10%,2) – 4000 + 40,000(A/F,10%,2)


= –400,000(0.57619) – 4000 + 40,000(0.47619)
= $–215,428 (> $–108.714)

AWD103 3-year life = –400,000(A/P,10%,3) – 4000 + 40,000(A/F,10%,3)


= –400,000(0.40211) – 4000 + 40,000(0.30211)
= $–152,760 (> $–108,714)

AWD103 6-year life = –400,000(A/P,10%,6) – 4000 + 40,000(A/F,10%,6)


= –400,000(0.22961) – 4000 + 40,000(0.12961)
= $–90,660 (< $–108,714)

The D103 chamber would be more cost-effective than the G490 only under the
optimistic life estimate of 6 years.

8.46 When the country’s economy is expanding, AB Investment Company is optimistic


and expects a MARR of 15% for new investments. However, in a receding
economy, the expected return is 8%. Normally a 10% return is required. An

Copyright 2021 © McGraw-Hill Education. All rights reserved. No reproduction or distribution without the
prior written consent of McGraw-Hill Education.
30
expanding economy causes the estimates of asset life to go down about 20%, and a
receding economy makes the n values increase about 10%. Calculate and observe or
plot the sensitivity of PW values versus (a) the MARR, and (b) the life values for
the two plans detailed below, using the most likely estimates for the other factors.
(c) Considering all the analyses, under which scenario, if any, should plan M be
accepted?

Plan M Q

Initial investment, $ −100,000 −110,000


Cash flow, $/year +15,000 +19,000
Life, years 20 20

(a) MARR = 8% (Pessimistic)

PWM = –100,000 + 15,000(P/A,8%,20)


= –100,000 + 15,000(9.8181)
= $47,272

PWQ = –110,000 + 19,000(P/A,8%,20)


= –110,000 + 19,000(9.8181)
= $76,544

MARR = 10% (Most Likely)

PWM = –100,000 + 15,000(P/A,10%,20)


= –100,000 + 15,000(8.5136)
= $27,704

PWQ = –110,000 + 19,000(P/A,10%,20)


= –110,000 + 19,000(8.5136)
= $51,758

MARR = 15% (Optimistic)

PWM = –100,000 + 15,000(P/A,15%,20)


= –100,000 + 15,000(6.2593)
= $–6111

PWQ = –110,000 + 19,000(P/A,15%,20)


= –110,000 + 19,000(6.2593)
= $8927

Copyright 2021 © McGraw-Hill Education. All rights reserved. No reproduction or distribution without the
prior written consent of McGraw-Hill Education.
31
(b) n = 16: Expanding economy (Optimistic)

n = 20(0.80) = 16 years

PWM = –100,000 + 15,000(P/A,10%,16)


= –100,000 + 15,000(7.8237)
= $17,356

PWQ = –110,000 + 19,000(P/A,10%,16)


= –110,000 + 19,000(7.8237)
= $38,650

n = 20: Expected economy (Most likely)

PWM = $27,704 (From part (a))

PWQ = $51,758 (From part (a))

n = 22: Receding economy (Pessimistic)

n = 20(1.10) = 22 years

PWM = –100,000 + 15,000(P/A,10%,22)


= –100,000 + 15,000(8.7715)
= $31,573

PWQ = –110,000 + 19,000(P/A,10%,22)


= –110,000 + 19,000(8.7715)
= $56,659

Observing the plan M PW values (underlined), M is never better than plan Q.

8.47 Identify a fundamental reason why all decisions in engineering economics are
performed under risk.

Estimates are for the future in engineering economic evaluations. Decision making
under risk is, therefore, always present in any conclusion.

8.48 Your friend is a “betting man” and wants to double his $20 bill. (a) If you accept
the challenge from him that you won’t throw a 6 on one throw of a die, what are the
chances you will win $20? Conversely, what are the chances you will lose $20? (b)
In your answers, what have you assumed about the die?

Copyright 2021 © McGraw-Hill Education. All rights reserved. No reproduction or distribution without the
prior written consent of McGraw-Hill Education.
32
(a) P(win $20) = 1/6 = 0.167

P(lose $20) = 5/6 = 0.833

(b) You likely assumed the die is fair with a 1/6th chance of showing each number.

8.49 For each situation below, determine (1) if the variable is discrete or continuous, and
(2) if the information involves decision making under certainty or risk.
a. A friend in real estate tells you there is a 50-50 chance that the price per square
foot for new houses will increase 9% during the next 6 months.
b. Your manager informs the staff there is a 25% chance that sales will decrease
between 50 and 55 units next month.
c. Jane got paid yesterday, and $400 was taken out in income taxes.
d. There is a 20% chance of rain and a 30% chance of snow today.

(a) Continuous variable with risk


(b) Discrete variable with risk
(c) Discrete variable with certainty
(d) Discrete variable with risk

8.50 A survey of households included questions about the number of operating


automobiles N currently owned by people living at the residence, and the interest
rate i on the lowest-rate loan for the cars. The results for 100 households are shown.

Number of Cars, N Households

0 12
1 56
2 26
3 3
≥4 3

Loan Rate, i, % Households

0.0–2 22
2.01–4 10
4.01–6 12
6.01–8 42
8.01–10 8
10.01–12 6

Copyright 2021 © McGraw-Hill Education. All rights reserved. No reproduction or distribution without the
prior written consent of McGraw-Hill Education.
33
a. State whether each variable is discrete or continuous.
b. Numerically describe and plot the probability distributions for N and i.
c. From the data collected, determine the probability that a household has one or
two cars. Three or more cars.
d. Use the data for i to estimate the chances that the interest rate is between 7% and
11% per year.

(a) N is discrete since only specific values are mentioned; i is continuous from 0 to
12.

(b) The P(N) and P(i) values are calculated below.

N 0 1 2 3 4
P(N) .12 .56 .26 .03 .03

i 0-2 2-4 4-6 6-8 8-10 10-12


P(i) .22 .10 .12 .42 .08 .06

(c) P(N = 1 or 2) = P(N = 1) + P(N = 2)


= 0.56 + 0.26
= 0.82

P(N ≥ 3) = 0.03 + 0.03


= 0.06

(d) P(7%  i  11%) = P(6.01  i  12)


= 0.42 + 0.08 + 0.06
= 0.56
Chance is 56%

8.51 An officer of the state lottery commission has sampled lottery ticket purchasers
over a 1-week period at one location. The amounts distributed back to the
purchasers and the associated probabilities for 5000 tickets are as follows:

Distribution, $ 0 2 5 10 100
Probability 0.91 0.045 0.025 0.013 0.007

a. If a ticket cost $5, estimate the chances of receiving more than the cost of the
ticket.
b. Calculate the expected value of the distribution of dollars per ticket.
c. If tickets cost $2, determine the expected long-term income to the state per ticket,
based on this sample.

Copyright 2021 © McGraw-Hill Education. All rights reserved. No reproduction or distribution without the
prior written consent of McGraw-Hill Education.
34
(a) P(Distribution > $5) = P(Distribution = $10) + P(Distribution = $100)
= 0.013 + 0.007
= 0.020

Chances = 2%

(b) E($) = $P($) = 0.91(0) + ... + 0.007(100)


= 0 + 0.09 + 0.125 + 0.13 + 0.7
= $1.045

(c) 2.000 – 1.045 = 0.955

State’s long-term income is 95.5 cents per ticket

8.52 You just inherited the royalties from several hydraulically fractured (fracked) oil
wells. Data collected from similar wells in an established oilfield were used to
develop a probability-royalty relationship. Calculate (a) the expected value of
royalties per year, and (b) the probability that your royalties will be at least $12,600
per year.

Royalties R,
$/year 6,200 8,500 9,600 10,300 12,600 15,500
Probability,
P(R) 0.10 0.21 0.32 0.24 0.09 0.04

(a) E(R) = 6,200(0.10) + 8,500(0.21) + … + 12,600(0.09) + 15,500(0.04)


= $9,703

(b) P(R  12,600) = P(R = 12,600) + P(R =15,500)


= 0.09 + 0.04
= 0.13

8.53 An engineer was asked to determine whether the average air quality index (AQI) in
a volatile chemical mixing room was within OSHA guidelines. The following nine
AQI readings in ppm (parts per million) were collected:

181, 186, 180, 191, 183, 183, 196, 185, and 189

a. Determine the arithmetic mean.


b. Calculate the standard deviation.
c. Determine the percent of readings that fall within ±1 standard deviation from the
mean.
d. Verify your answers using spreadsheet functions.
e. The AQI categories and health concerns are shown below. Based on the value of
the upper limit of the range you determined in part (c), what can you conclude
about the air quality condition in the mixing room?

Copyright 2021 © McGraw-Hill Education. All rights reserved. No reproduction or distribution without the
prior written consent of McGraw-Hill Education.
35
Air Quality Index (AQI) Values Levels of Health Concern

When the AQI is in this range: ..air quality conditions are:

0 to 50 Good
51 to 100 Moderate
101 to 150 Unhealthy for Sensitive Groups
151 to 200 Unhealthy
201 to 300 Very Unhealthy
301 to 500 Hazardous

(a) X = (181 + 86 + … + 85 + 89)/9


= 186

(b) Reading Mean, X Xi – X (Xi – X)2


181 186 –5 25
186 186 0 0
180 186 –6 36
191 186 5 25
183 186 –3 9
183 186 –3 9
196 186 10 100
185 186 –1 1
189 186 3 9
0 214

s = √214/(9 –1) = 5.17

(c) Range for ±1s is 186 ± 5.17 = 180.83 to 191.17

Number of values in ±1s range = 7

Percent of values in ±1s range = 7/9 = 77.8%

Copyright 2021 © McGraw-Hill Education. All rights reserved. No reproduction or distribution without the
prior written consent of McGraw-Hill Education.
36
(d) Spreadsheet confirmation using AVERAGE and STDEV functions.

(e) Air quality is unhealthy, bordering on very unhealthy.

8.54 A discrete variable X that can take on integer values from 1 to 10 was sampled 50
times and resulted in the probability estimates shown. (a) Determine the sample
average and standard deviation. (b) Determine the values 1 and 2 standard deviations
from the mean. Of the 50 sample points, how many fall within the two ranges?

Xi 1 2 3 6 9 10
P(Xi) 0.2 0.2 0.2 0.1 0.2 0.1

(a) Convert P(X) data to frequency values to determine s.

X P(X) XP(X) f X2 fX2


1 .2 .2 10 1 10
2 .2 .4 10 4 40
3 .2 .6 10 9 90
6 .1 .6 5 36 180
9 .2 1.8 10 81 810
10 .1 1.0 5 100 500
4.6 1630

Sample average: X = 4.60

Sample variance: s2 = 1630 – 50 (4.6)2 = 11.67


49 49

s = 3.42

Copyright 2021 © McGraw-Hill Education. All rights reserved. No reproduction or distribution without the
prior written consent of McGraw-Hill Education.
37
(b) X ± 1s is 4.60 ± 3.42 = 1.18 and 8.02
25 values, or 50%, are in this range.

X ± 2s is 4.60 ± 6.84 = –2.24 and 11.44


All 50 values, or 100%, are in this range.

8.55 (Spreadsheet exercise) A newsstand manager is tracking Y, the number of weekly


magazines left on the shelf when the new edition is delivered. Data collected over a
30-week period are summarized by a discrete probability distribution. A colleague
determined that the expected value and standard deviation of these results are E(Y)
= 7.08 and sY = 3.23, respectively. Use a spreadsheet to plot the distribution and
indicate the expected value and one standard deviation on either side of it.

Copies, Y 3 7 10 12
P(Y) 1/3 1/4 1/3 1/12

Sample plot of the distribution, E(Y) = 7.08, and the points 1sY on either side
of E(Y) is shown.

E(Y) ± 1sY = 7.08 ± 3.23 = 3.85 and 10.31

8.56 Bennington Health purchased three dialysis systems each at an installed cost of
$100,000 for different locations in the city. The AOC over 2 years varied as shown.
(All values are costs.) The load factor in terms of percentage of clients at each
location is shown in decimal form. Find the expected AW of costs for each location
at i = 15% per year.

Copyright 2021 © McGraw-Hill Education. All rights reserved. No reproduction or distribution without the
prior written consent of McGraw-Hill Education.
38
Location North (N) South (S) West (W)

Load 0.50 0.35 0.15


AOC, $ per year
Year
 0 100,000 100,000 100,000
1 20,000 15,000 12,000
2 10,000 13,000 8,000

Find AW for each location, then E(AW) considering the load factor as the
probability that a client goes to a specific location.

AWN = [100,000 + 20,000(P/F,15%,1) + 10,000(P/F,15%,2)](A/P,15%,2)


= [100,000 + 17,392 + 7,561](0.61512)
= $76,861

AWS = [100,000 + 15,000(P/F,15%,1) + 13,000(P/F,15%,2)](A/P,15%,2)


= [100,000 + 13,044 + 9,829](0.61512)
= $75,582

AWW = [100,000 + 12,000(P/F,15%,1) + 8,000(P/F,15%,2)](A/P,15%,2)


= [100,000 + 10,435 + 6,049](0.61512)
= $71,652

E(AW) = 0.50(76,861) + 0.35(75,582) + 0.15(71,652)


= $75,632

8.57 (Spreadsheet exercise) A university wants to raise funds for scholarships using a
unique approach. The focus will be on alumni-owned companies. A marketing and
communications program will cost $300,000 up front. Depending on the type of
business (legal, industrial, IT, or financial), different contribution amounts each
year are estimated, as shown for a 4-year campaign period. The percentage of
alumni in each sector is known from records and shown as a probability for
receiving a contribution. Use expected value PW analysis, a MARR of 8% per year,
and a spreadsheet to determine if the program is economically justified.

Copyright 2021 © McGraw-Hill Education. All rights reserved. No reproduction or distribution without the
prior written consent of McGraw-Hill Education.
39
Sector Legal Industrial IT Financial
Probability 0.15 0.25 0.40 0.20

Annual Contribution Estimates, $


Year
0 −300,000 −300,000 −300,000 −300,000
1 90,000 70,000 120,000  75,000
2 90,000 75,000 100,000  90,000
3 80,000 80,000 90,000  108,000
4 70,000 85,000 80,000  129,600

Spreadsheet displays the PW values using the NPV function at 8%.

= B2*B10 + C2*C10 + D2*D10 + E2*E10 displays the average PW

PW = $+1449

Since PW > 0, program is justified, on the average

8.58 Jeri, a self-employed geo-tech engineering consultant, has recently experienced


financial problems in meeting monthly bills, in part, due to a “lumpy” income
pattern compared to a relatively uniform, but steadily increasing, expense pattern
throughout the year. The typical monthly income and expense pattern for the last
several years is reflected in the data shown. Jeri wants to better manage finances on
a monthly basis. Let I = income and E = expenses to determine the following
information. (Hint: You may want to use a spreadsheet to perform the analysis.)
a. Determine total I and total E for a typical year. Is there enough money annually
to cover all expenses?
b. Calculate monthly (I − E) and estimate what percentage of the year a negative
cash flow occurs.

Copyright 2021 © McGraw-Hill Education. All rights reserved. No reproduction or distribution without the
prior written consent of McGraw-Hill Education.
40
c. Determine the average monthly I, E, and (I − E) amounts. What percent of the
monthly income remains after all expenses are paid, on the average?
d. Determine the standard deviation of I, E, and (I − E). How many months have
(I − E) values outside the range of average ± one standard deviation?

Income −
Month Income, $ Expenses, $ Expenses, $

1 5,000 2,000   3,000


2 — 2,300 −2,300
3 10,000 2,600    7,400
4 — 2,900 −2,900
5 — 3,200 −3,200
6 5,000 3,500   1,500
7 — 5,000 −5,000
8 — 5,100 −5,100
9 — 5,200 −5,200
10 — 5,300 −5,300
11 10,000 5,400  4,600
12 25,000 5,500 19,500

(a) Row 14 displays annual income ($55,000) and expenses ($48,000) using the
SUM function. Yes, there is enough money to cover all expenses.

(b) Column D displays (I – E).

Percent with negative cash flow = (7/12)(100) = 58.3% of the months

(c) Row 15 uses the AVERAGE function.

Percent of income remaining = (583/4583)(100) = 12.7%

(d) Row 16 uses the STDEV function.

X ± s = 583 ± 7345
= –6,762 to 7,928

Copyright 2021 © McGraw-Hill Education. All rights reserved. No reproduction or distribution without the
prior written consent of McGraw-Hill Education.
41
Months outside range bounds = 1 (month 12)

8.59 When using Monte Carlo sampling to obtain random numbers from the probability
distributions of varying parameters, what is a fundamental assumption that must be
made?

The assumption is that the parameters are independent of each other. Each parameter
is a random variable and not affected by nor does it affect any other variable.

8.60 Use the RAND()*100 spreadsheet function to generate 100 values from a uniform
distribution with the range of 0 to 100. Then use other spreadsheet functions to
calculate (a) the average and compare the sample value to 50, and (b) the standard
deviation and compare the sample value to 28.87. These two values are correct for a
continuous uniform distribution with limits of 0 and 100.

As an example, a sample of 100 values generated the following results:

(a) Function = AVERAGE(A1:A100) resulted in 51.08331; close to 50

(b) Function = STDEV(A1:A100) resulted in 29.41508; close to 28.87

8.61 Use the Excel© help utility to determine what the following functions are designed
to do: (a) VLOOKUP, and (b) RANDBETWEEN.

(a) According to Excel help, VLOOKUP searches the first column of a of


cells, and then returns a value from any cell on the same row of the range.

(b) According to Excel, RANDBETWEEN displays a random integer number


between the two numbers specified. A new random integer number is
returned every time the worksheet is calculated.

Copyright 2021 © McGraw-Hill Education. All rights reserved. No reproduction or distribution without the
prior written consent of McGraw-Hill Education.
42
8.62 Carl, a colleague in Europe working for the same company that you do, estimated
the most likely cash flows after taxes (CFAT) for a project he is working on.

Year CFAT, $

0 −35,000
1–10 5,500

The PW value at the current MARR of 8% per year is

PW = −35,000 + 5500(P/A,8%,10) = $1905

Because of the changing economic scene, Carl believes the MARR will vary over a
relatively narrow range, as will the CFAT. He is willing to accept the other
estimates as certain. Use the following probability distribution assumptions for
MARR and CFAT to perform a simulation. Take a sample of 30 or more simulation
trials and use PW analysis to determine if the project is economically justified using
decision making under certainty. Under risk.

MARR Discrete uniform distribution over the range


7% to 10%
CFAT Continuous uniform distribution over the range
$4000 to $7000 for each year 1 to 10

A simulation similar to Example 8.17 is performed. MARR varies from 7% to 10%


with a 0.25 probability each. Use a lookup table for MARR (columns E and F)
coupled with the RAND() function. For CFAT, use RANDBETWEEN(4000,7000).
PW values (column M) for the simulation shown are positive 23 of the 30 trials.

Conclusion: The project appears economically viable under both certainty and risk.

Copyright 2021 © McGraw-Hill Education. All rights reserved. No reproduction or distribution without the
prior written consent of McGraw-Hill Education.
43
8.63 Janice is a process engineer for Upland Chemicals. Yesterday, she was handed the
following information about a piece of air quality sampling equipment that she
earlier requested be purchased for her department. The request was denied by the
Corporate Finance Manager based on the large negative PW value that the manager
calculated.
P = $−150,000
GI = $45,000 per year
E = 10% of P or $−15,000 per year
n = 10 years
S = 20% of P or $30,000
PW = $−39,664
Janice was quite aggravated when she determined the interest rate used by the
manager was 25% per year, well above the published corporate MARR of 10% to
justify required product or environmental quality equipment. In a meeting with you,
Janice shared some of her own estimates about first cost and expected life. Using
the following estimates and distribution assumptions, perform a simulation for
Janice that may help her in an effort to obtain approval to purchase the equipment.
Accepted as certain: GI = $45,000 per year
E = 10% of P
S = 20% of P
Varying amounts: P  Continuous uniform distribution over the range
$−140,000 to $−170,000

Copyright 2021 © McGraw-Hill Education. All rights reserved. No reproduction or distribution without the
prior written consent of McGraw-Hill Education.
44
n Discrete uniform distribution over the range 8 to 15 years
MARR Discrete uniform distribution; values from 10% through 15% have a 15%
chance each; values from 16% to 25% have a 1% chance each

A simulation patterned after that of Example 8.17 is shown below for a sample of
25 trials using the following functions.

Variable Column(s) Function format


MARR B and C = RAND() and = VLOOKUP(RN_#,E$13:F$28,2)
P I = RANDBETWEEN(–170000,–140000)
n L and M = RAND() and = VLOOKUP(RN_#,E$3:F$10,2)
PW O, row 8 = – PV(C8%,M8,45000+0.1*I8,–0.2*I8) + I8

Conclusion: This simulation run shows that, for the varying estimates that Janice
has presented, the equipment would be justified 9/25 or 76% of the
time with a PW > 0.

Comment: Results may vary widely over different simulation runs since the
random range on n is mostly above n = 10 years in the problem statement and
the random range for MARR values is lower than MARR = 25% used by the
manager to obtain PW = $–39,664.

Copyright 2021 © McGraw-Hill Education. All rights reserved. No reproduction or distribution without the
prior written consent of McGraw-Hill Education.
45
8.64 Cost and revenue data per year for a steel-joining process are shown. If the profit
was $100,000, the number of units sold was closest to:
a. < 8,000
b. 10,000
c. 11,500
d. 12,000
Fixed cost = $500,000 per year
Production cost per unit = $200
Revenue per unit = $250

Total profit/year = revenue/year – cost/year


100,000 = 250x – 200x – 500,000
x = 12,000
Answer is (d)

8.65 An engineer with EP Gas and Electric is evaluating alternative equipment for
maintaining gas wells using a no-return payback analysis. The minimum number of
years required to recover the initial investment is closest to:
a. 6
b. 5
c. 4
d. 3
First cost = $−185,000
n = 10 years
NCF = $42,000 for year 1, decreasing by $2500 per year thereafter
No salvage value

Try n = 3: –185,000 + 42,000 + 39,500 + 37,000 = $–66,500 too low


Try n = 4: –185,000 + 42,000 + 39,500 + 37,000 + 34,500 = $–32,000 too low
Try n = 5: –185,000 + 42,000 + 39,500 + 37,000 + 34,500 + 32,000 = 0 OK
Answer is (b)

8.66 Four mutually exclusive alternatives are evaluated using three estimates or
strategies (pessimistic, most likely, and optimistic) for several parameters. The
resulting PW values over the LCM are shown.

PW over LCM, $

Strategy 1 2 3 4

Pessimistic (P) 4,500 −6,000 3,700 −1,900


Most likely (ML) 5,100    −500 5,000   −100
Optimistic (O) 9,500  2,000 9,800   3,500

Copyright 2021 © McGraw-Hill Education. All rights reserved. No reproduction or distribution without the
prior written consent of McGraw-Hill Education.
46
The best alternative to select using only the pessimistic strategy is
a. 1
b. 2
c. 3
d. 4

Answer is (a)

8.67 A manufacturing process at Simplicity XP has a fixed cost of $40,000 per month. A
total of 100 units can be produced in one day at a cost of $3000 for materials and
labor. If the MARR is 12% per year compounded monthly, the number of units that
must be sold each month at $50 per unit to just break even is closest to:
a. 1000
b. 1500
c. 2000
d. 2500

Let x = breakeven units per month

0 = revenue – cost
0 = 50x – [40,000 + (3000/100)x]
20x = 40,000
x = 2000

Alternatively, QBE = 40,000/(50 – 3000/100)


= 2000
Answer is (c)

8.68 The landfill in Wellsburg has an area of 30 acres available for receiving waste from
the city of 40,000 people who generate 0.625 tons per person of municipal solid
waste (MSW) each year. If the fixed cost of the landfill is $300,000 per year and the
operating cost is $12 per ton, the landfill charge per ton of MSW to break even is
closest to:
a. $24
b. $28
c. $32
d. $39

Let r = charge per ton

Tons per year = 40,000(0.625) = 25,000 tons/year

Revenue = cost
25,000r = 300,000 + 12(25,000)
r = $24 per ton

Copyright 2021 © McGraw-Hill Education. All rights reserved. No reproduction or distribution without the
prior written consent of McGraw-Hill Education.
47
Alternatively, QBE = FC/(r – v)
25,000 = 300,000/(r – 12)
r = 300,000/25,000 + 12
= $24
Answer is (a)

8.69 A 2-lane road surface can be finished with concrete or asphalt. Concrete will cost
$2.3 million per mile and will last for 20 years. If signing, mowing, and winter
maintenance are not included, the basic maintenance cost for concrete and asphalt
roadways averages $48,300 and $77,400 per mile per year, respectively. The
interest rate is 10% per year. The maximum amount that can be spent on asphalt, if
it lasts 10 years, is closest to:
a. < $850,000
b. $940,565
c. $1,003,655
d. $1,481,100

–2,300,000(A/P,10%,20) – 48,300 = –x(A/P,10%,10) – 77,400


–2,300,000(0.11746) – 48,300 = –x(0.16275) – 77,400
x = $1,481,155
Answer is (d)

8.70 Process X has fixed costs of $40,000 per year and a variable cost of $60 per unit.
Process Y has an unknown fixed cost, but with this process 200 units can be
produced each month at a total variable cost of $2000. If the two processes break
even at a production rate of 2000 units per year, the annual fixed cost of Y is closest
to:
a. $60,000
b. $100,000
c. $120,000
d. $140,000

Let FCY = fixed cost for Y. Set cost relations equal at 2000 units per year.

Variable cost for Y = 2000/200 = $10/unit

40,000 + 60(2000) = FCY + 10(2000)


FCY = $140,000 per year
Answer is (d)

8.71 Two methods of weed control in an irrigation ditch are under consideration. Method
A involves lining at a first cost of $4,000, a life of 20 years, and M&O costs of
$3 per mile per year. Method B involves spraying a chemical that costs $40 per
gallon. One gallon will treat 8 miles, but the treatment must be applied 4 times per

Copyright 2021 © McGraw-Hill Education. All rights reserved. No reproduction or distribution without the
prior written consent of McGraw-Hill Education.
48
year. In determining the number of miles per year that results in breakeven, the
variable cost for method B is closest to:
a. $5/mile
b. $15/mile
c. $20/mile
d. $30/mile

VC = (40/8)(4)
= $20/mile
Answer is (c)

8.72 A call center has a capacity of 1,400,000 calls annually. The fixed cost of the center
is $775,000 with an average variable cost of $1 and revenue of $2.50 per call. The
percentage of the call capacity that must be placed each year to break even is
closest to:
a. 29%
b. 33%
c. 37%
d. 41%

QBE = 775,000/(2.50 – 1)
= 516,667 calls per year

% of capacity = (516,667/1,400,000)(100)
= 36.9%
Answer is (c)

8.73 John has woodworking equipment on which he does special-request jobs for people.
He paid $30,000 for the equipment, has an average AOC of $4000, and an average
income of $9800 per year. At a MARR of 20% per year, John’s payback period is
closest to:
a. 3 years
b. 15 years
c. 19 years
d. It will never pay off

–30,000 + (9800–4000)(P/A,20%,np) = 0
(P/A,20%,np) = 5.1724

There is no P/A factor this large for any n value, that is, n → ∞. From
observation, the net income of $5800 per year < $6000 that the 20% MARR
requires; the investment will never pay off.
Answer is (d)

Copyright 2021 © McGraw-Hill Education. All rights reserved. No reproduction or distribution without the
prior written consent of McGraw-Hill Education.
49
8.74 The shipping costs for fresh fruit items have been estimated and assigned the
probabilities shown. The expected value of the shipping costs is closest to:
a. $36.35
b. $39.21
c. $41.30
d. $45.10

Shipping Cost, $ 34 38 55
Probability 0.22 0.31 0.47

E($) = 0.22(34) + 0.31(38) + 0.47(55)


= 7.48 + 11.78 + 25.85
= $45.11
Answer is (d)

8.75 For the discrete income values and probabilities shown, the probability that the
income in any year will be greater than $8500 is closest to:
a. 0.38
b. 0.49
c. 0.69
d. 0.92

Income, $/year 6,200 8,500 9,600 10,300 12,600 15,500


Probability 0.15 0.23 0.32 0.24 0.09 0.04

P(Income > $8500) = 0.32 + 0.24 + 0.09 + 0.04


= 0.69
Answer is (c)

8.76 The net revenue from a candy product called Mummies-Lite has averaged $15,000

per month for the past 12 months. If the value of is $4,680,000, the
standard deviation value is closest to:
a. $38,100
b. $652
c. $958
d. $425,450

s = √4,680,000/(12–1)
= $652.3
Answer is (b)

Copyright 2021 © McGraw-Hill Education. All rights reserved. No reproduction or distribution without the
prior written consent of McGraw-Hill Education.
50
8.77 The president of Bullnose Shoes estimated the chances of different levels of
earnings for this calendar year as shown. The expected value of the earnings is
closest to:
a. $506,000
b. $493,000
c. $461,000
d. $402,000

Earnings, $ Chances

260,000 1 in 10
400,000 6 in 10
800,000 3 in 10

E = 0.1(260,000) + 0.6(400,000) + 0.3(800,000)


= $506,000
Answer is (a)

8.78 All of the following spreadsheet functions will correctly generate a random number,
except:
a. = RAND()
b. = RANDBETWEEN(50,45)
c. = RAND()*100
d. = RANDBETWEEN(−35,100)

Answer is (b)

Copyright 2021 © McGraw-Hill Education. All rights reserved. No reproduction or distribution without the
prior written consent of McGraw-Hill Education.
51
Solutions to end-of-chapter problems
Basics of Engineering Economy, 3rd edition
Leland Blank and Anthony Tarquin

Chapter 9
Replacement and Retention Decisions
9.1 In a replacement study, what is meant by “taking the nonowner’s viewpoint”?

In taking a nonowner’s viewpoint, the analysis is done from the perspective of


someone who does not own any of the assets under consideration. This means that in
order to acquire the presently-owned asset, it would have to be “bought” at its fair
market value. The costs associated with doing so represent the true cost of keeping
the presently-owned asset.

9.2 A robotic arm purchased 1 year ago for $85,000 costs more to operate than
anticipated. When purchased, it was expected to be used for 10 years with annual
maintenance costs of $37,000 and a salvage value of $10,000. However, last year, it
cost the company $46,000 to maintain it, and these costs are expected to escalate to
$48,000 this year and increase by $2000 each year thereafter. The salvage value is
now estimated to be $85,000 − $10,000k, where k is the number of years since the
arm was purchased. Also, it will be useful for a maximum of 5 more years.
Determine the values of P, AOC, n, and S for a replacement study performed now.

P = MV = 85,000 – 10,000(1) = $75,000


AOC = 48,000 + 2,000k where k = 1 to 5
n = 5 years
S = 85,000 – 10,000(6) = $ 25,000

9.3 A civil engineer who owns his own design/build/operate company purchased a small
crane 3 years ago at a cost of $65,000. At that time, it was expected to be used for
10 years and then traded in for its salvage value of $10,000. Due to increased
construction activities, the company would prefer to trade for a new, larger crane
now, which will cost $80,000. The company estimates that the old crane can be used,
if necessary, for another 3 years, at which time it would have a $21,000 estimated
market value. Its current market value is estimated to be $37,000, and if it is used for
another 3 years, it will have M&O costs (exclusive of operator costs) of $19,000 per
year. Determine the values of P, n, S, and AOC that should be used for the existing
crane in a replacement analysis.

P = MV = $37,000
AOC = $19,000 per year
n = 3 years
S = $21,000

Copyright 2021 © McGraw-Hill Education. All rights reserved. No reproduction or distribution without the
prior written consent of McGraw-Hill Education.
1
9.4 Newport Corporation purchased equipment for making pneumatic vibration isolators
at a cost of $90,000 two years ago. It has a market value that can be described by the
relation $90,000 − 8000k, where k is the years from time of purchase. Experience
with this type of equipment has shown that the operating cost for the first 4 years is
$65,000 per year, after which it increases by $6300 per year. The asset’s salvage
value was originally estimated to be $7000 after a predicted 10-year useful life.
Determine the values of P, S, and AOC if a replacement study is done (a) now and
(b) 1 year from now. Assume the expected retention period is only 1 additional year
beyond the time the replacement study is performed.

(a) P = 90,000 – 8000(2) = $74,000


S = 90,000 – 8000(3) = $66,000
AOC = $65,000 per year

(b) P = 90,000 – 8000(3) = $66,000


S = 90,000 – 8000(4) = $58,000
AOC = $65,000 per year

9.5 A piece of equipment that was purchased 2 years ago by Toshiba Imaging for
$50,000 was expected to have a useful life of 5 years with a $5000 salvage value. Its
performance was less than expected, and it was upgraded for $20,000 one year
ago. Increased demand now requires that the equipment be upgraded again for
another $17,000 so that it can be used for 3 more years. If upgraded, its M&O costs
will be $27,000 and it will have a $12,000 salvage after 3 years. Alternatively, it can
be replaced with new equipment priced at $65,000 with M&O costs of $14,000 per
year and a salvage value of $23,000 after 6 years. If replaced now, the existing
equipment will be sold for $7000. Determine the values of P, S, M&O, and n for the
defender in a replacement study.

P = 7000 + 17,000 = $24,000


S = $12,000
M&O = $27,000 per year
n = 3 years

9.6 For equipment that has a first cost of $10,000 and the estimated operating costs and
year-end salvage values shown, (a) determine the economic service life at i = 10%
per year using factors, and (b) write the PMT function to determine AW for year 4, if
net operating costs are entered into cells B2 through B6.

Copyright 2021 © McGraw-Hill Education. All rights reserved. No reproduction or distribution without the
prior written consent of McGraw-Hill Education.
2
Operating Salvage
Year Cost, $ Value, $
1 −1000 7000
2 −1200 5000
3 −1300 4500
4 −2000 3000
5 −3000 2000

(a) AW1 = –10,000(A/P,10%,1) – 1000 + 7000(A/F,10%,1) = $–5000

AW2 = –10,000(A/P,10%,2) – 1000(P/F,10%,1)(A/P,10%,2) + (5000 – 1200)


×(A/F,10%,2)
= $–4476

AW3 = –10,000(A/P,10%,3) – [1000(P/F,10%,1) +1200(P/F,10%,2)](A/P,10%,3)


+ (4500 – 1300)(A/F,10%,3)
= $–3819

AW4 = –10,000(A/P,10%,4) – [1000(P/F,10%,1) +1200(P/F,10% 2)


+ 1300(P/F,10%,3)](A/P,10%,4) + (3000 – 2000)(A/F,10%,4)
= $–3847

AW5 = –10,000(A/P,10%,5) – [1000(P/F,10%,1) + 1200(P/F,10% 2)


+ 1300(P/F,10%,3) + 2000(P/F,10%,4)](A/P,10%,5)
+ (2000 – 3000)(A/F,10%,5)
= $–3921

ESL is 3 years with AW = $–3819 per year

(b) Enter P and net operating costs for years 0-4 in cells B2 through B6. Function is

= – PMT(10%,4,NPV(10%,B3:B6)–10000)

9.7 To improve package tracking at a UPS transfer facility, conveyor equipment was
upgraded with RFID sensors at a cost of $345,000. The operating cost is expected to
be $148,000 per year for the first 3 years and $210,000 for the next 3 years. The
salvage value of the equipment is expected to be $140,000 for the first 3 years, but
due to obsolescence, it won’t have a significant value after that. The interest rate is
10% per year. (a) Determine the ESL and equivalent AW using tabulated factors, or
(b) use a spreadsheet to determine the two amounts, plus include a graph of years
versus total AW values.

Copyright 2021 © McGraw-Hill Education. All rights reserved. No reproduction or distribution without the
prior written consent of McGraw-Hill Education.
3
(a) Factors

AW1 = –345,000(A/P,10%,1) – 148,000 + 140,000(A/F,10%,1) = $–387,500

AW2 = –345,000(A/P,10%,2) – 148,000 + 140,000(A/F,10%,2) = $–280,119

AW3 = –345,000(A/P,10%,3) – 148,000 + 140,000(A/F,10%,3) = $–244,434

AW4 = –345,000(A/P,10%,4) – 148,000(P/A,10%,3)(A/P,10%,4)


–210,000(P/F,10%,4)(A/P,10%4) = $–270,197

AW5 = –345,000(A/P,10%,5) – 148,000(P/A,10%,3)(A/P,10%,5)


–210,000(P/A,10%,2)(P/F,10%,3)(A/P,10%,5) = $–260,337

AW6 = –345,000(A/P,10%,6) – 148,000(P/A,10%,3)(A/P,10%,6)


–210,000(P/A,10%,3)(P/F,10%,3)(A/P,10%,6) = $–253,813

The ESL is n = 3 years with AW = $–244,434

(b) Using a spreadsheet, AW = $–244,434 is minimum at n = 3 years.

9.8 As project engineer, you received the AW analysis below from the finance
department. It is for a new piece of equipment you ordered some months ago. You
were told the interest rate used was 10% per year, but no first cost or projected
salvage value were provided and you want to know them. Determine the values of P
and S using the AW values for year 3. Note: The AW values are equivalent values
through the given year, not costs for the single year.

Copyright 2021 © McGraw-Hill Education. All rights reserved. No reproduction or distribution without the
prior written consent of McGraw-Hill Education.
4
AW of AW of
Years AW of First Operating Cost, Salvage
Retained Cost, $ $ per Year Value, $
1 −51,700 −15,000 35,000
2 −27,091 −17,000 13,810
3 −18,899 −19,000 6,648
4 −14,827 −21,000 4,309
5 −12,398 −23,000 2,457

For P: –18,899 = –P(A/P,10%,3)


–18,899 = –P(0.40211)
P = $47,000

For S: 6648 = S(A/F,10%,3)


6648 = S(0.30211)
S = $22,005

9.9 The initial cost of a bridge that is expected to have a very long useful life is
$70 million. Maintenance can be done at 1-, 2-, 3-, or 4-year intervals, but the longer
the interval between servicing, the higher the cost. The cost of servicing is estimated
at $83,000, $91,000, $125,000, and $183,000 for intervals of 1 through 4 years,
respectively. What interval should be scheduled for maintenance to minimize the
overall equivalent annual cost? The interest rate is 8% per year.

Equivalent AW of the $70 million initial cost at 8% per year is constant at


$–5,600,000; only consider the maintenance cost for one cycle.

AW1 = –83,000(A/F,8%,1) = $–83,000


AW2 = –91,000(A/F,8%,2) = $–43,750
AW3 = –125,000(A/F,8%,3) = $–38,504
AW4 = –183,000(A/F,8%,4) = $–40,612

Lowest AW is for 3 years; maintenance should be scheduled at 3-year intervals.

9.10 Leroy purchased an AI-based robotics system with a first cost of $345,000, an AOC
of $148,000 per year, a $140,000 salvage value, and an ESL of 3 years. Rather than
trading after 3 years, he kept the system for a total of 5 years. Due to the wear factor
and expense of repairs, its AOC increased to $240,000 per year starting in year 4,
plus the salvage value went to zero. At i = 10% per year, what is the percentage
increase in the AW of costs that Leroy experienced?

Copyright 2021 © McGraw-Hill Education. All rights reserved. No reproduction or distribution without the
prior written consent of McGraw-Hill Education.
5
AW3 = –345,000(A/P,10%,3) – 148,000 + 140,000(A/F,10%,3)
= $–244,434

AW5 = –345,000(A/P,10%,5) – 148,000(P/A,10%,3)(A/P,10%,5)


– 240,000(P/A,10%,2)(P/F,10%,3)(A/P,10%,5)
= $–270,656

Percent increase = (270,656 – 244,434)/244,434 = 0.107 (10.7%)

9.11 (Spreadsheet exercise) A construction company bought a 180,000 metric ton earth
sifter at a cost of $65,000. The company expects to keep the equipment a maximum
of 7 years. The operating cost is expected to follow the series described by 40,000 +
10,000k, where k is the number of years since it was purchased (k = 1, …, 7). The
salvage value is estimated to be $30,000 for years 1 and 2 and $20,000 for years 3
through 7. At i = 10% per year, determine the ESL and equivalent AW of costs for
the sifter.
a. First, use a spreadsheet accompanied by a plot of AW of costs versus all
7 years.
b. Look at the factors that would be used to determine the AW values. Explain,
from a factor value viewpoint, why the AW value decreases significantly
between years 1 and 2.
c. Now for a sensitivity question using your spreadsheet. What happens to the ESL
and AW values if an enhanced version of the sifter is purchased for $90,000 and
the life, operating cost, and salvage estimates remain the same? Are the ESL
and AW values sensitive to this first-cost increase?

(a) Spreadsheet solution and plot show ESL is n = 2 years with AW = $–77,929

Capital recovery, year 2: = – PMT($B$1,$A5,$F$1,$B5)


AW of AOC, year 2: = – PMT($B$1,$A5,NPV($B$1,$C$4:$C5))

Copyright 2021 © McGraw-Hill Education. All rights reserved. No reproduction or distribution without the
prior written consent of McGraw-Hill Education.
6
(b) Form the relations to calculate AW for 1 and 2 years using factors.

AW1 = –65,000(A/P,10%,1) – 50,000 + 30,000(A/F,10%,1)


= –65,000(1.1000) –50,000 + 30,000(1.000)
= $–91,500

AW2 = –65,000(A/P,10%,2) – [50,000 + 10,000(A/G,10%,2)]


+ 30,000(A/F,10%,2)
= –65,000(0.57619) – [50,000 + 10,000(0.4762)] + 30,000(0.47619)
= $–77,929

The AW decreases from $–91,500 to $–77,929 because the A/P and A/F factor
values decrease significantly between n = 1 and n = 2 years as the first cost and
salvage amounts are distributed over 2, rather than 1, years.

(c) Use the spreadsheet with P = $–90,000. ESL is now 4 years and the AW curve is
flat after year 1 with AW4 = $–87,895. The ESL and AW are sensitive to the
first cost.

9.12 A large standby electricity generator in a hospital operating room has a first cost of
$70,000 and may be used for a maximum of 6 years. Its salvage value, which
decreases by 15% per year, is described by the equation S = 70,000(1 − 0.15)n,
where n is the number of years after purchase. The operating cost of the generator
will be constant at $7500 per year, and the interest rate is 12% per year. (a)
Determine the economic service life and associated AW values. (b) Write the PMT
function format to determine the AW value for year 3.

(a) Factors:

AW1 = –70,000(A/P,12%,1) – 7,500 + 59,500(A/F,12%,1) = $–26,400


AW2 = –70,000(A/P,12%,2) – 7,500 + 50,575(A/F,12%,2) = $–25,063
AW3 = –70,000(A/P,12%,3) – 7,500 + 42,989(A/F,12%,3) = $–23,905
AW4 = –70,000(A/P,12%,4) – 7,500 + 36,540(A/F,12%,4) = $–22,901
AW5 = –70,000(A/P,12%,5) – 7,500 + 31,059(A/F,12%,5) = $–22,030

Copyright 2021 © McGraw-Hill Education. All rights reserved. No reproduction or distribution without the
prior written consent of McGraw-Hill Education.
7
AW6 = –70,000(A/P,12%,6) – 7,500 + 26,400(A/F,12%,6) = $–21,273

ESL is 6 years with AW = $–21,273 per year

(b) Spreadsheet function: Using the term ‘year’ to identify the year of ownership,
if salvage values are entered in cells using the function = (70000*(0.85)^year),
the AW value is displayed using the PMT function:
= – PMT(12%,year,–70000,70000*(0.85^year)) –7500

9.13 Based on company records of similar equipment, a consulting aerospace engineer at


Aerospatiale estimated AW values for a presently owned, highly accurate steel rivet
inserter as shown. A challenger has ESL = 2 years and AW C = $−48,000 per year.
The MARR is 15% per year. (a) If the consultant must recommend a replace/retain
decision today, should the company keep the defender or purchase the challenger?
Why? (b) When should the next replacement evaluation take place, and under what
assumption?

If Retained This The AW Value Is,


Number of Years $ per Year
1 −62,000
2 −51,000
3 −49,000
4 −53,000
5 −70,000

(a) Purchase the challenger today because it’s AW of $–48,000 is lower


than the AW of the defender for any number of years of retention.

(b) Reevaluate in 2 years, assuming the challenger estimates do not change in the
meantime.

9.14 In planning a plant expansion, Medimmune can upgrade the existing controlled-
environment rooms or purchase new ones. The presently owned ones were
purchased 4 years ago for $250,000. They have a current “quick sale” value of
$20,000, but for an investment of $100,000 now, they would be adequate for up to
another 4 years. They could be sold for $40,000 at any time. Alternatively, new
controlled-environment rooms could be purchased at a cost of $270,000. They are
expected to have a maximum life of 10 years with a $50,000 salvage value at any
time. Use a MARR of 20% per year. (a) What are the ESL values for both options?
(b) Is it economically better to upgrade or replace?

Copyright 2021 © McGraw-Hill Education. All rights reserved. No reproduction or distribution without the
prior written consent of McGraw-Hill Education.
8
(a) Since the salvage values are the same for all years and no AOC series is
included, the ESL is the longest expected life.

ESLD = 4 years ESLC = 10 years

(b) AWD = –(100,000 + 20,000)(A/P,20%,4) + 40,000(A/F,20%,4)


= –120,000(0.38629) + 40,000(0.18629)
= $–38,903

AWC = –270,000(A/P,20%,10) + 50,000(A/F,20%,10)


= –270,000(0.23852) + 50,000(0.03852)
= $–62,474

Select the defender; upgrade rooms. Plan to keep them for 4 years, unless
estimates change or a new challenger is available.

9.15 A machine that cost $120,000 three years ago can be sold now for $54,000. Its
market value is expected to be $40,000 and $20,000 one year and two years from
now, respectively. Its operating cost was $18,000 for the first 3 years of its life, but
the M&O cost is expected to be $23,000 for the next 2 years. A new improved
machine that can be purchased for $138,000 will have an economic life of 5 years,
an operating cost of $9000 per year, and a salvage value of $32,000 whenever it is
replaced. At an interest rate of 10% per year, determine if the presently owned
machine should be replaced now, 1 year from now, or 2 years from now.

Find AW of defender for keeping one or two more years and compare against
AW of challenger

AWD1 = –54,000(A/P,10%,1) – 23,000 + 40,000(A/F,10%,1)


= –54,000(1.10) – 23,000 + 40,000(1.0)
= $–42,400

AWD2 = –54,000(A/P,10%,2) – 23,000 + 20,000(A/F,10%,2)


= –54,000(0.57619) – 23,000 + 20,000(0.47619)
= $–44,590

The ESLC is 5 years, since AOC is constant and the salvage value is the same for
all years.

AWC = –138,000(A/P,10%,5) – 9000 + 32,000(A/F,10%,5)


= –138,000(0.26380) – 9,000 + 32,000(0.16380)
= $–40,163

Replace with the challenger now

Copyright 2021 © McGraw-Hill Education. All rights reserved. No reproduction or distribution without the
prior written consent of McGraw-Hill Education.
9
9.16 You and a group of friends have purchased an annual lease at a commercial fishing
pond for the last several years. It is possible to renew the lease for up to 5 years.
Based on past costs, you calculated the AW values for different lease periods.

Current Lease AW Value,


Renewal, Years $ per Year
1 −9200
2 −8100
3 −8700
4 −8900
5 −9500

Another lease option, a few miles further from your home, is available with an
economic service life of 7 years and an AW of $−8600. Assuming all future costs
remain unchanged, when should you transfer to the new lease and location? You
value your recreation highly by using a MARR of 25% per year.

The ESL for current lease is 2 years with AW = $–8100, which is less than AWC.

You should not take the new lease. You should renew the current lease for two
more years and then renew it every two years thereafter, provided the costs do not
change.

9.17 A textile processing company is evaluating whether it should retain the current
bleaching process which uses chlorine dioxide or replace it with a proprietary
Genpure process. The relevant information for each process is shown. (a) Use an
interest rate of 15% per year to perform the replacement study. (b) Additionally,
write the PMT functions that display the information necessary to make the
decision.

Process Current Genpure


Original cost 6 years ago, $ −450,000 —
Investment cost now, $ — −600,000
Current market value, $ 25,000 —
Annual operating cost, $/year −190,000 −70,000
Remaining life, years 3 10
Salvage value at life end, $ 0 50,000

Copyright 2021 © McGraw-Hill Education. All rights reserved. No reproduction or distribution without the
prior written consent of McGraw-Hill Education.
10
(a) Since AOC is constant and the salvage value is the same for all years,
ESLD = 3 years and ESLC = 10 years.

AWD = –25,000(A/P,15%,3) – 190,000


= $–200,950

AWC = –600,000(A/P,15%,10) – 70,000 + 50,000(A/F,15%,10)


= $–187,088

Select the challenger

(b) Spreadsheet functions

Defender: = –PMT(15%,3,–25000) –190000 displays AWD = $–200,949

Challenger: = –PMT(15%,10,–600000,50000) – 70000 displays


AWC = $–187,089

9.18 Your employer, which manufactures vibration isolation platforms, is trying to


decide whether it should immediately upgrade the current assembly system of belts
and gears (the defender), which is rather labor-intensive, with a more highly
automated independent moving cart system (the challenger) 1 year from now. Some
components of the current system can be sold now for $9000, but they will be
worthless hereafter. The AOC of the existing system is $162,000 per year. The
moving cart system will cost $160,000 with a $50,000 salvage value after 1 year,
decreasing 30% per year thereafter until its expected life of 4 years. Its AOC is
estimated to be $128,000 the first year, increasing 20% each following year. If you
are asked to do a replacement analysis using an interest rate of 10% per year, which
alternative would you recommend?

No option of retention of defender more than one year; ESLD = 1 year

AWD = –9000(A/P,10%,1) – 162,000


= –9000(1.1000) –162,000
= $–171,900

Must determine ESL of challenger and the AWC. Use a spreadsheet to determine
ESLC = 3 years and AWC = $–210,459.

Copyright 2021 © McGraw-Hill Education. All rights reserved. No reproduction or distribution without the
prior written consent of McGraw-Hill Education.
11
If using factors for the AWC, the relation must account for the geometric
gradients. As an example for the 3-year AW,

3 years: AWC,3 = AP of initial investment – AW of AOC + AW of market value


= {–160,000 – 128,000{1 – [(1 + 0.2)/(1 + 0.1)]3}/(0.1 – 0.2)
+ 50,000{1 – [(1 – 0.3)/(1 + 0.1)]3}/(0.1 + 0.3)}×(A/P,10%,3)
= $–210,459

Comparing the AW values, retain the defender now; replace in one year with the
moving cart, if estimates do not change and no new challenger is identified.

9.19 A critical machine in BHP Billiton’s copper refining operation was purchased 7
years ago for $160,000. Last year a replacement study was performed with the
decision to retain it for 3 more years. The situation has changed. The equipment is
estimated to have a value of $8000 if “scavenged” for parts now or anytime in the
future. If kept in service, it can be minimally upgraded at a cost of $43,000 to make
it usable for up to 2 more years. Its operating cost is estimated at $22,000 the first
year and $29,000 the second year. Alternatively, the company can purchase a new
system, the challenger, that will have an AWC of $−48,063 over its ESL. Use a
MARR of 10% per year and annual worth analysis to determine when the company
should replace the machine and associated AW values.

No study period is specified; find defender ESL; compare with AWC = $–48,063

1 year: AWD,1 = – (8000 + 43,000)(A/P,10%,1) – 22,000 + 8000(A/F,10%,1)


= $–70,100

2 years: AWD,2 = – (8000 + 43,000)(A/P,10%,2) – 22,000(P/F,10%,1)(A/P,10%,2)


+ (8000 – 29,000)(A/F,10%,2)
= $–50,910

Defender ESL is n = 2 years with AWD = $–50,910; AWC is lower; replace the
machine now.

Copyright 2021 © McGraw-Hill Education. All rights reserved. No reproduction or distribution without the
prior written consent of McGraw-Hill Education.
12
9.20 (Spreadsheet exercise) A $500,000 piece of laser-driven, fruit cleaning equipment is
being rapidly challenged by new technology. Current estimates for the remaining
3 years of possible retention of this defender are:

Defender: Market value: $60,000 now, but zero thereafter


AOC estimates: $100,000 next year, increasing by $30,000 per year
thereafter
Retention period: Maximum of 3 more years

An external contractor will install and operate cutting-edge equipment in a turnkey


operation for $175,000 per year for up to 4 years. This price is guaranteed for
18 months, after which the contract price increases to $200,000 per year. The
MARR is 12% per year.
a. Perform the ESL and replacement analysis to retain the defender or bring in the
contractor.
b. Management did not want to abandon working equipment or have external
contract workers inside the plant. Therefore, the contract offer was declined
with the intent to keep the equipment for 3 full years and reconsider the
contract, knowing it will be more expensive. However, during the next year
there was an unexpected additional $100,000 operating cost, making AOC for
the year total $200,000. If all other estimates are unchanged, was the retention
decision correct or should the contract have been taken?

(a) Defender analysis in spreadsheet shows ESL is 2 years with AWD = $–149,653

Cell D5 function, as an example, is AW of MV and S + AW of AOC series


= – PMT(12%,$A5,$B$3,$B5) – PMT(12%,$A5,NPV(12%,$C$4:$C5)+0)

Comparing AWD = $–149,653 and AWC = $–175,000, retain the equipment for
2 years.
Contract will then cost $–200,000 per year when a new replacement study is
performed. (This follows middle branch in Figure 9.3)

Copyright 2021 © McGraw-Hill Education. All rights reserved. No reproduction or distribution without the
prior written consent of McGraw-Hill Education.
13
(b) Bottom portion of spreadsheet analysis shows ESL to be 3 years with
AWD = $–189,893 considering the unexpected AOC amount. Since this is
within the 18-month contract price guarantee of $–175,000, the contract
would have been cheaper economically.

9.21 Concurris Prototyping is committed to using the newest and finest equipment in its
labs. Accordingly, Wilma, a senior engineer, has recommended that a 2-year-old
piece of precision measurement equipment be replaced immediately. She believes it
can be demonstrated that the proposed equipment is economically advantageous at
a 15%-per-year return and a planning horizon of 5 years. Perform the replacement
analysis using the annual worth method, a 5-year study period, and the estimates
below. Was Wilma correct?

Equipment Current Proposed


Original purchase price, $ −30,000 −40,000
Current market value, $ 15,000 —
Remaining life, years 5 15
Estimated value in 5 years, $ 7,000 10,000
Salvage value after 15 years, $ — 5,000
AOC, $ per year −8,000 −3,000

AWD = –15,000(A/P,15%,5) – 8000 + 7000(A/F,15%,5)


= –15,000(0.29832) – 8000 + 7000(0.14832)
= $–11,437

AWC = –40,000(A/P,15%,5) – 3000 + 10,000(A/F,15%,5)


= –40,000(0.29832) – 3000 + 10,000(0.14832)
= $–13,450

Keep the defender; Wilma was not correct.

9.22 A machine purchased 3 years ago for $140,000 is currently too slow to satisfy
increased demand. The machine can be upgraded now for $70,000 or sold to a
smaller company for $40,000. The current machine will have an annual operating
cost of $85,000 per year and a $30,000 salvage value in 3 years. If upgraded, the
presently owned machine will definitely be retained for 3 more years. The
replacement, which will serve the company now and for at least 8 years, will cost
$220,000. Its salvage value will be $50,000 for years 1 through 5, $20,000 after
6 years, and, $10,000 thereafter. It will have an estimated operating cost of $65,000
per year. The company asks you to perform an economic analysis at 15% per year
using a 3-year planning horizon. Should the company replace the presently owned
machine now, or do it 3 years from now? What are the AW values?

Copyright 2021 © McGraw-Hill Education. All rights reserved. No reproduction or distribution without the
prior written consent of McGraw-Hill Education.
14
No ESL analysis; only 2 options: replace defender and buy challenger now, or
retain defender for 3 years.

AWD = – (40,000 + 70,000)(A/P,15%,3) – 85,000 + 30,000(A/F,15%,3)


= –110,000(0.43798) – 85,000 + 30,000(0.28798)
= $–124,538

AWC = –220,000(A/P,15%,3) – 65,000 + 50,000(A/F,15%,3)


= –220,000(0.43798) – 65,000 + 50,000(0.28798)
= $–146,957

Keep the defender

9.23 Two processes can be used for producing a polymer that reduces friction loss in
engines. Process K will use a presently owned machine that has a current market
value of $160,000, an operating cost of $7000 per month, and a salvage value of
$50,000 after 1 year and $40,000 after its maximum 2-year life. Used machines for
this type can be purchased and the same estimates can be used for a period of 1 or
2 years.
Process L will utilize a new machine that has a first cost of $210,000, an
operating cost of $5000 per month, and market values of $100,000 after 1 year,
$45,000 after 2 years, $25,000 after 3 years, and $20,000 after its maximum 4-year
life. You have been asked to determine which process is better using a MARR of
12% per year compounded monthly, and a study period of (a) 1 year, (b) 2 years,
and (c) 3 years. (d) Is the decision to retain K sensitive to different study periods?

(a) For 1-year study period

AWK,1 = –160,000(A/P,1%,12) – 7000 + 50,000(A/F,1%,12)


= –160,000(0.08885) –7000 + 50,000(0.07885)
= $–17,273

AWL,1 = –210,000(A/P,1%,12) – 5000 + 100,000(A/F,1%,12)


= –210,000(0.08885) –5000 + 100,000(0.07885)
= $–15,773

Select process L

(b) For 2-year study period

AWK,2 = –160,000(A/P,1%,24) – 7000 + 40,000(A/F,1%,24)


= –160,000(0.04707) –7000 + 40,000(0.03707)
= $–13,049

Copyright 2021 © McGraw-Hill Education. All rights reserved. No reproduction or distribution without the
prior written consent of McGraw-Hill Education.
15
AWL,2 = –210,000(A/P,1%,24) – 5000 + 45,000(A/F,1%,24)
= –210,000(0.04707) –5000 + 45,000(0.03707)
= $–13,217

Select process K marginally

(c) For 3-year study period, repurchase used machine for 1 year

AWK,3 = –160,000(A/P,1%,36) – 7000


+ (–160,000 + 40,000)(P/F,1%,24)(A/P,1%,36) + 50,000(A/F,1%,36)
= –160,000(0.03321) – 7000 – 120,000(0.7876)(0.03321)
+ 50,000(0.02321)
= $–14,292

AWL,3 = –210,000(A/P,1%,36) – 5000 + 25,000(A/F,1%,36)


= –210,000(0.03321) –5000 + 25,000(0.02321)
= $–11,394

Select process L, again

(d) Yes, decision to retain K is sensitive to the 1, 2 and 3 year periods.

9.24 ABB Communications is considering replacing equipment that had a first cost of
$300,000 five years ago. The company CEO wants to know if the equipment should
be replaced now or at any other time over the next 3 years to minimize the cost of
producing miniature background suppression sensors. Since the present equipment
or the proposed equipment can be used for any or all of the 3-year period, one of the
company’s industrial engineers produced AW cost information for the defender and
challenger as shown below. The values represent the annual costs of the respective
equipment if used for the indicated number of years. Use (a) a spreadsheet and (b)
factors to determine when the defender should be replaced to minimize the cost to
ABB for the 3-year study period using an interest rate of 10% per year.

The AW in $ per Year Is:


If Retained This
Number of Years Defender Challenger
1 −22,000 −29,000
2 −24,000 −26,000
3 −27,000 −25,000

There are four options:


1. Keep the defender for all 3 years,
2. Use the defender for 2 years and the challenger for 1 year,

Copyright 2021 © McGraw-Hill Education. All rights reserved. No reproduction or distribution without the
prior written consent of McGraw-Hill Education.
16
3. Use the defender for 1 years and the challenger for 2 years,
4. Use the challenger for all 3 years.

(a) Spreadsheet: List options of time in service and corresponding AW values;


determine PW values.

Select option 3 (retain D for 1 year, then switch to C); it has lowest PW.

(b) Factors: The PW for each option is as follows:

PW defender for 3 years = $–27,000(P/A,10%,3)


= $–27,000(2.4869)
= $–67,146
PW defender for 2, challenger for 1 = –24,000(P/A,10%,2) – 29,000(P/F,10%,3)
= –24,000(1.7355) – 29,000(0.7513)
= $–63,440

PW defender for 1, challenger for 2 = –22,000(P/F,10%,1)


– 26,000(P/A,10%,2)(P/F,10%,1)
= –22,000(0.9091) – 26,000(1.7355)(0.9091)
= $–61,022

PW challenger for 3 years = $–25,000(P/A,10%,3)


= $–25,000(2.4869)
= $–62,173

Lowest PW is $–61,022 (plan 3); keep the defender for 1 year and then replace
it with the challenger.

9.25 The table that follows shows present worth values of the costs associated with using
a presently owned machine (the defender) and a possible replacement (the
challenger) for different numbers of years. Determine when the defender should
be replaced using an interest rate of 10% per year and a 5-year study period. Show
solutions using (a) factors and (b) a spreadsheet.

Copyright 2021 © McGraw-Hill Education. All rights reserved. No reproduction or distribution without the
prior written consent of McGraw-Hill Education.
17
The PW Value in $ Is:
If in Service This
Number of Years Defender Challenger
1 −36,000 −89,000
2 −75,000 −96,000
3 −125,000 −102,000
4 −166,000 −113,000
5 −217,000 −149,000

(a) Factors: There are 6 options: retaining D zero to 5 years, while utilizing C for
5 to zero years
.
PW of D for zero years, C for 5 years = $–149,000

PW of D for 1 year, C for 4 years = –36,000 – 113,000(P/F,10%,1)


= –36,000 – 113,000(0.9091)
= $–138,728

PW of D for 2 years, C for 3 years = –75,000 – 102,000(P/F,10%,2)


= –75,000 – 102,000(0.8264)
= $–159,293

PW of D for 3 years, C for 2 years = –125,000 – 96,000(P/F,10%,3)


= –125,000 – 96,000(0.7513)
= $–197,125

PW of D for 4 years, C for 1 year = –166,000 – 89,000(P/F,10%,4)


= –166,000 – 89,000(0.6830)
= $–226,787

PW of D for 5 years, C for zero years = $–217,000

Lowest PW is $–138,728; keep the defender 1 year and then replace it with
the challenger.

(b) Spreadsheet: PW values are placed in the year cell prior to when the year
starts for challenger.

Lowest PW = $–138,727 for option E: defender for 1 year, followed by


challenger for 4 years)

Copyright 2021 © McGraw-Hill Education. All rights reserved. No reproduction or distribution without the
prior written consent of McGraw-Hill Education.
18
9.26 Digital Tech Dynamics purchased a new quality inspection system for $550,000.
The estimated salvage value was $50,000 after 10 years. Currently the expected
remaining life is 7 years with an AOC of $27,000 per year and an estimated salvage
value of $40,000. The new president has recommended early replacement of the
system with one that costs $400,000 and has a 12-year economic service life, a
$35,000 salvage value, and an estimated AOC of $50,000 per year. If the MARR
for the corporation is 12% per year, use factor-based relations to determine
the minimum trade-in value necessary now to make the president’s replacement
economically advantageous.

–RV(A/P,12%,7) – 27,000 + 40,000(A/F,12%,7) = –400,000(A/P,12%,12)


– 50,000 + 35,000(A/F,12%,12)

–RV(0.21912) – 27,000 + 40,000(0.09912) = –400,000(0.16144)


– 50,000 + 35,000(0.04144)

RV = $411,146

9.27 Huntington Medical Center purchased a used low-field MRI scanner 2 years ago for
$445,000. Its operating cost is $272,000 per year and it can be sold for $150,000
anytime in the next 3 years. The Center’s director is considering replacing the
presently owned MRI scanner with a state-of-the-art 3 Tasle machine that will cost
$2.2 million. The operating cost of the new machine will be $340,000 per year, but
it will generate extra revenue estimated to be $595,000 per year. The new unit can
probably be sold for $800,000 three years from now. You have been asked to
determine how much the presently owned scanner would have to be worth on the
open market for the AW values of the two machines to be the same over a 3-year
planning period. The Center’s MARR is 20% per year. Use (a) factors and (b) a
spreadsheet with GOAL SEEK to find the RV. Is the RV a reasonable market or
trade-in amount?

Copyright 2021 © McGraw-Hill Education. All rights reserved. No reproduction or distribution without the
prior written consent of McGraw-Hill Education.
19
(a) –RV(A/P,20%,3) – 272,000 + 150,000(A/F,20%,3) = –2,200,000(A/P,20%,3)
– 340,000 + 595,000 + 800,000(A/F.20%,3)

–RV(0.47473) – 272,000 + 150,000(0.27473) = –2,200,000(0.47473) + 255,000


+ 800,000(0.27473)

–RV(0.47473) = –338,832
RV = $713,735

(b) Spreadsheet uses PMT functions for AW of D and C, and GOAL SEEK to
determine RV = $713,726.

Probably can’t sell presently-owned MRI for anything close to $713,726, it cost
$445,000 two years ago. Therefore, keep presently-owned MRI.

9.28 For some years, Mel has contracted with several major pizza retailers for home
delivery services. He uses a MARR of 12% per year in all business dealings. His
current van, purchased 10 years ago for $75,000, can be used for 3 more years, with
an AOC of $63,000 and an estimated $25,000 trade-in value. A better-equipped van
will cost $130,000, have an economic life of 6 years, an estimated trade-in value of
$45,000, an AOC of $32,000 per year, and will generate an estimated $20,000 per
year additional revenue. On the basis of these estimates, what market value now for
the current van will make the new van equally attractive? Solve by spreadsheet or
factors, as requested by your instructor.

Factors: Set AWcurrent = AWnew with RV the unknown.

–RV(A/P,12%,3) – 63,000 + 25,000(A/F,12%,3) = –130,000(A/P,12%,6)


– 32,000 + 20,000 + 45,000(A/F,12%,6)

–RV(0.41635) – 63,000 + 25,000(0.29635) = –130,000(0.24323)


– 32,000 + 20,000 + 45,000(0.12323)

–RV(0.41635) = –17,516
RV = $42,072

Copyright 2021 © McGraw-Hill Education. All rights reserved. No reproduction or distribution without the
prior written consent of McGraw-Hill Education.
20
Spreadsheet: PMT functions determine the AW values; GOAL SEEK finds the
RV = $42,074 (cell B3) as the current van’s minimum trade-in required to
justify the new van.

9.29 Hydrochloric acid, which fumes at room temperatures, creates a very corrosive
work environment. A mixing machine working in this environment is deteriorating
fast and can be used for only 1 more year, at which time it will be scrapped. It was
purchased 3 years ago for $88,000 and its operating cost for the next year is
expected to be $63,000. A more corrosion-resistant challenger will cost $226,000
with an operating cost of $48,000 per year. It is expected to have a $60,000 salvage
value after its 10-year ESL. At an interest rate of 15% per year, what minimum
trade-in value will make the challenger economically attractive? Solve by
spreadsheet or factors, as requested by your instructor.

Spreadsheet: GOAL SEEK determines a minimum RV of $23,544 (cell B3) when


AWD is forced to equal AWC = $–90.076.

Factors:

–RV(A/P,15%,1) – 63,000 = –226,000(A/P,15%,10) – 48,000


+ 60,000(A/F,15%,10)

–RV(1.15) – 63,000 = –226,000(0.19925) – 48,000 + 60,000(0.04925)

–1.15RV = –90,076 + 63,000

Minimum RV = $23,544

Copyright 2021 © McGraw-Hill Education. All rights reserved. No reproduction or distribution without the
prior written consent of McGraw-Hill Education.
21
9.30 Clarksville Pottery has long sold a coffee cup style made inhouse using machinery
purchased 5 years ago for $110,000. The product line will be discontinued and the
machinery disposed of during the year of the largest estimated AW. Use the
following estimates to determine when the machinery should be sold at a MARR of
8% per year.

Market value: Current: $32,000


Future: $20,000 next year, decreasing $3000 per year thereafter
AOC: $40,000 and increasing at a rate of $2000 per year
Revenue: $80,000 and decreasing at a rate of $5000 per year
Retention: Maximum of 5 years

Use the guidelines in Section 9.4. A spreadsheet solution is shown. NCF series is
the sum of annual AOC and revenue estimates. The AW series uses two PMT
functions. For 5 years, the function is

= – PMT(8%,A7,–32000,C7) – PMT(8%,A7,NPV(8%,$F$3:F7)+0)

AW = $26,863 is the largest; retain for 2 more years, then abandon.

9.31 In a replacement study conducted last year, it was determined that the defender
should be kept for 3 more years. In reviewing the current costs, it appears that the
estimates for this year and next are still valid. The proper course of action is to:
a. Replace the existing asset now.
b. Replace the existing asset 2 years from now, as was determined last year.
c. Conduct a new replacement study using the new estimates.
d. Conduct a new replacement study using last year’s estimates

Answer is (b)

9.32 A consulting engineering firm purchased field test equipment 2 years ago for
$50,000 that was expected to have a useful life of 5 years with a $5000 salvage
value. Its performance was less than expected, and it was upgraded for $20,000 one
year ago. Increased demand now requires that the equipment be upgraded again for
another $16,000 so that it can be used for 3 more years. Its annual operating cost
will be $27,000 and it will have a $12,000 salvage after 3 years. If replaced now,
the existing equipment will be sold for $7,000. In conducting a replacement study,
the value to use for the defender’s first cost to calculate the annual worth is:
a. $−15,000
b. $−23,000

Copyright 2021 © McGraw-Hill Education. All rights reserved. No reproduction or distribution without the
prior written consent of McGraw-Hill Education.
22
c. $−50,000
d. $−70,000

P = 16,000 + 7,000
= $23,000
Answer is (b)

9.33 An engineer determined that the equivalent annual worth of an existing machine
over its remaining useful life of 3 years will be $−70,000 per year. It can be
replaced now or later with a machine that will have an AW of $−80,000 if it is kept
for 2 years or less, $−68,000 if it is kept between 3 and 4 years, and $−75,000 if it is
kept for 5 to 10 years. The company wants an analysis of what it should do for a
3-year planning period at an interest rate of 15% per year. The existing machine
should be replaced:
a. Now
b. One year from now
c. Two years from now
d. It should not be replaced

AWC for 3 years is $–68,000; AWD for 3 years is $–70,000; select C now
Answer is (a)

9.34 To determine the economic service life, you must calculate the AW for each year of
an asset’s useful life. Max just bought an old pickup truck for $10,000. If estimated
operating costs and year-end salvage values are as shown, and i = 10% per year, the
AW for keeping the truck for 2 years is closest to:
a. $−4475
b. $−4560
c. $−4728
d. $−4933

Operating Salvage
Year Costs, $ Value, $
1 −1000 7000
2 −1200 5000
3 −1500 4200
4 −2000 3000
5 −3000 2000

AW2 = –10,000(A/P,10%,2) – 1000(P/F,10%,1)(A/P,10%,2)


+ (5000 – 1200)(A/F,10%,2)
= $–4476
Answer is (a)

Copyright 2021 © McGraw-Hill Education. All rights reserved. No reproduction or distribution without the
prior written consent of McGraw-Hill Education.
23
9.35 Quality inspection equipment purchased 2 years ago for $50,000 was expected to
have a useful life of 5 years with a $5000 salvage value. Due to increased demand,
it was upgraded for $20,000 one year ago. A new product line now requires that the
equipment be upgraded again for another $25,000 so that it can be used for 3 more
years. After it is upgraded, its annual operating cost will be $35,000 and it will have
a $16,000 salvage value after 3 years. Alternatively, it can be replaced with new
equipment that will cost $79,000 with operating costs of $19,000 per year and a
salvage value of $19,000 after 3 years. If replaced now, the existing equipment will
be sold for $9000. The annual worth of this defender at an interest rate of 10% per
year is closest to:
a. $−30,522
b. $−32,221
c. $−43,840
d. $−50,835

AW = (–25,000 –9,000)(A/P,10%,3) – 35,000 + 16,000(A/F,10%,3)


= (–25,000 –9,000)(0.40211) – 35,000 + 16,000(0.30211)
= $–43,838
Answer is (c)

The following AW values are used in Problems 9.36 to 9.39.

Years AW of AW of
Retained Defender, $ Challenger, $
1 −145,000 −136,000
2 −96,429 −126,000
3 −63,317 −92,000
4  −39,321 −53,000
5 −49,570 −38,000

9.36 The economic service life of the challenger is:


a. 2 years
b. 3 years
c. 4 years
d. 5 years

Answer is (d)

Copyright 2021 © McGraw-Hill Education. All rights reserved. No reproduction or distribution without the
prior written consent of McGraw-Hill Education.
24
9.37 The economic service life of the defender is:
a. 2 years
b. 3 years
c. 4 years
d. 5 years

Answer is (c)

9.38 For a 1-year study period, the decision is to:


a. Retain the defender
b. Replace the defender now
c. Select the challenger after 1 more year
d. Do nothing

Answer is (b)

9.39 For a 4-year study period, the decision is to:


a. Retain the defender
b. Replace the defender now
c. Select the challenger after 1 more year
d. Perform another replacement study in 2 years

Answer is (a)

9.40 A small residential developer is analyzing a potential challenger to his current front-
end loader. The loader was purchased 6 years ago for $53,000. The most recent
economic information about the loader is shown below. In determining the
economic life of the vehicle, the AW for keeping the loader for 3 more years at an
interest rate of 8% per year is closest to:
a. $−8,385
b. $−9,214
c. $−10,950
d. $−12,343

Projected Operating
Year Market Value, $ Costs, $ per Year
0 29,000 —
1 28,000 −4000
2 22,000 −5000
3 17,000 −6000
4 13,000 −8400

Copyright 2021 © McGraw-Hill Education. All rights reserved. No reproduction or distribution without the
prior written consent of McGraw-Hill Education.
25
AW3 = –29,000(A/P,8%,3) – [4000 + 1000(A/G,8%,3)] + 17,000(A/F.8%,3)
= –29,000(0.38803) – [4000 + 1000(0.9487)] + 17,000(0.30803)
= $–10,965
Answer is (c)

9.41 A machine was purchased 3 years ago for $45,000; it can be sold today for $24,000.
The operating costs for this machine are $9000 per year, and it is expected to last 4
more years with a $5000 salvage value. Used machines similar to this one will be
available at any time in the future. A new machine that will perform the same
service can be purchased for $50,000, and it will have a life of 10 years with
operating costs of $28,000 per year and a $10,000 salvage value. The years that
should be used for the life, n, in calculating the AW of the presently owned
machine are:
a. 3
b. 4
c. 7
d. 10

Answer is (b)

9.42 The amounts below represent AW values if an asset is retained for the number of
years shown. The ESL is:
a. 1 year
b. 2 years
c. 3 years
d. 4 years

AW of AW of
Years AW of Operating Salvage
Retained First Cost, $ Cost, $ Value, $
1 −165,000 −36,000 99,000
2 −86,429 −36,000 38,095
3 −60,317 −42,000 18,127
4 −47,321 −43,000 6,464
5 −39,570 −48,000 3,276

AW1 = –165,000 – 36,000 + 99,000 = $–102,000


AW2 = –86,429 – 36,000 + 38,095 = $–84,334
AW3 = –60,317 – 42,000 + 18,127 = $–84,190
AW4 = –47,321 – 43,000 + 6,464 = $–83,857
AW5 = –39,570 – 48,000 + 3,276 = $–84,294
Answer is (d)

Copyright 2021 © McGraw-Hill Education. All rights reserved. No reproduction or distribution without the
prior written consent of McGraw-Hill Education.
26
9.43 The annual worth values for a defender, which can be replaced with a similar used
asset, and a challenger are estimated. The defender should be replaced:
a. Now
b. 1 year from now
c. 2 years from now
d. 3 years from now

AW Value, $ per Year


Number of
Years Retained Defender Challenger
1 −14,000 −21,000
2 −13,700 −18,000
3 −16,900 −13,800
4 −17,000 −15,600
5 −18,000 −17,500

No study period specified; perform ESL analysis first; ESLD = 2 years and ESLC
= 3 years. The AWD is lower; defender should be replaced 2 years from now
with a similar asset.
Answer is (c)

9.44 When performing an abandonment study, all of the following are correct, except:
a. Revenues are included in the analysis
b. Year of abandonment has the smallest AW value
c. First cost is the current market value
d. A maximum number of years of retention can be specified

Answer is (b)

Copyright 2021 © McGraw-Hill Education. All rights reserved. No reproduction or distribution without the
prior written consent of McGraw-Hill Education.
27
Solutions to end-of-chapter problems
Basics of Engineering Economy, 3rd edition
Leland Blank and Anthony Tarquin

Chapter 10
Effects of Inflation
10.1 At a time when the inflation rate is 8% per year, $100,000 (future dollars) 15 years
from now is equivalent to how many of today’s constant-value dollars?

CV dollars = 100,000/(1 + 0.08)15


= $31,524

10.2 If the inflation rate is 6% per year, how many (future) dollars will be required
10 years from now to buy the same things that $15,000 buys now?

Inflated dollars = 15,000(1 + 0.06)10


= $26,863

10.3 In 1951, your grandmother could purchase a loaf of bread for $0.14. The cost to you
in 2020 was $2.60. What was the average inflation rate per year over the 69 years
between 1951 and 2020?

2.60 = 0.14(1 + f)69


(1 + f)69 = 18.5714

Solution 1. 69[log (1 + f)] = log 18.5714


log (1 + f) = 0.01839
1 + f = 100.01839
f = 0.0432 (4.32% per year)

Solution 2. (1 + f)69 = 18.5714


1 + f = 18.57141/69
f = 18.57140.01449 – 1
= 0.0432 (4.32% per year)

10.4 In 2019, the average salary of petroleum engineers was $98,400. Predict what their
salary will be in 2028 if their salary increases only by the inflation rate. Assume the
inflation rate over this time period is constant at 2.5% per year.

Salary in 2028 = 98,400(1 + 0.025)9


= $122,888

Copyright 2021 © McGraw-Hill Education. All rights reserved. No reproduction or distribution without the
prior written consent of McGraw-Hill Education.
1
10.5 If the inflation rate is 5% per year, how many years will it take for the cost of an
item to double, if the price increases only by the inflation rate? Obtain the answer
using (a) an equation, and (b) Figure 1.3.

(a) Assume C1 is the cost today

2C1 = C1(1 + 0.05)n


(1 + 0.05)n = 2.000
n log 1.05 = log 2.000
n = 14.2 years

(b) Increase is 100%. Interpolate between 10 and 15 years in Figure 1.3

Increase = 10 + 5(100–63)/(108–63)
= 10 + 4.1
= 14.1 years

10.6 Assume that you want to retire 30 years from now with an amount of money that
will have the same value (purchasing power) as $1.5 million today. If you believe
the inflation rate will average 4% per year, determine the amount of future dollars
you will need.

Future dollars required = 1,500,000(1 + 0.04)30


= $4,865,096

10.7 A website determines how many dollars today have the same buying power as a
stated amount of money at the beginning of any year in the past. When the values
1920, 2019, and $1000 are entered for the beginning of the initial year, beginning of
the desired year, and initial amount, respectively, the result is $12,831. On the basis
of these numbers, what was the average annual inflation rate between 1920 and the
end of 2018? Solve by (a) equation, and (b) spreadsheet using GOAL SEEK.

(a) Future dollars = present dollars(1 + f)n


12,831 = 1000(1 + f)98
log 12.831 = 98 log(1 + f)
1.10826 = 98 log(1 + f)
log (1 + f) = 0.011309
1 + f = 100.011309
1 + f = 1.0264
f = 0.0264 (2.64% per year)

(b) Enter function = ((1 + C6)^98)*1000, where cell C6 contains the desired
inflation rate. Use GOAL SEEK to change C6 such that the relation equals
12831. Display is an f of 0.0264 or f = 2.64% per year.

Copyright 2021 © McGraw-Hill Education. All rights reserved. No reproduction or distribution without the
prior written consent of McGraw-Hill Education.
2
10.8 An engineer who is now 65 years old began planning for his retirement 40 years
ago. At that time, he thought that if he had $1 million when he retired, he would
have more than enough money to live his remaining life in luxury. Assume the
inflation rate over the 40-year time period was 2.69% per year, which is the
approximate average for the past 100 years. What was the constant-value dollar
amount of his $1 million 40 years ago, when he started his retirement plan?

CV dollars = 1,000,000/(1.0269)40
= $345,839

10.9 At an international meeting of consumer advocates for price control, a comparison


of CPI growth was made to understand the cost increases that inflation imposes on
citizens in the United States, Mexico, and Turkey. The annual CPI percentages are
shown for 2 years. (a) If the base spending level for the year 2016 was set at $1000
per household in each country for comparison purposes, develop a table and graph
of the resulting costs for 2017 and 2018. (b) What is the percentage difference in
2018 between the United States and Turkey?

CPI Value, %
Year USA Mexico Turkey
2017 2.10 6.77 11.92
2018 1.90 4.83 20.30

(a) Develop spreadsheet with relations of the format = (1 + CPI)*(previous_price)


and plot. Sample shown below.

(b) Difference in 2018 = (1346.4 – 1040.40)/1040.40)(100)


= 29.4%

Turkish consumers will pay 29.4% more during 2018 than in the USA.

Copyright 2021 © McGraw-Hill Education. All rights reserved. No reproduction or distribution without the
prior written consent of McGraw-Hill Education.
3
10.10 According to data from the National Association of Colleges and Employers
(NACE), engineering graduates’ average salaries at $66,521 were among the
highest for the class of 2018. You expect to graduate in 2023. What might you use
as an aimed-at starting salary when you graduate, if the assumed salary inflation
rate over the 5-year period is (a) 2.5% per year, and (b) twice this rate?

(a) Salary in 2023 = 66,521(1.025)5


= $75,262

(b) Salary in 2023 = 66,521(1.05)5


= $84,900

10.11 The inflation rate over a 10-year period for an automatic dishwasher that costs
$1000 now is shown below. (a) What will be the cost at the end of year 10? (b)
Do you get the same cost using an average inflation rate of 5% per year through
the 10-year period? Why?

Year Inflation Rate, %


1 10
2 0
3 10
4 0
5 10
6 0
7 10
8 0
9 10
10 0

(a) Cost in year 10 = 1000(1.10)(1.0)(1.10)(1.0)(1.10)(1.0)(1.10)(1.0)(1.10)(1.0)


= $1,610.51

(b) Cost in year 10 at 5% per year = 1000(1 + 0.05)10 = $1628.89

The cost is not the same, because there are different compound rates on
different amounts of money.

Copyright 2021 © McGraw-Hill Education. All rights reserved. No reproduction or distribution without the
prior written consent of McGraw-Hill Education.
4
10.12 α-β, Inc., a high-tech company in San Diego, whose stock trades on the NYSE
exchange, uses a MARR of 25% per year. If the chief financial officer (CFO) said
the company expects to make a real rate of return of 20% per year on its
investments over the next 3-year period, what is the company expecting the
annual inflation rate to be over that time period?

0.25 = 0.20 + f + 0.20f


1.20f = 0.05
f = 0.0417 (4.2% per year)

10.13 Cellgene Biometrics, a small biotech company, uses a MARR of 40% per year
when evaluating new investments. If the inflation rate in the bio field is 6% per
year, what will the real rate of return be, provided it realizes its optimistic MARR
objective?

0.40 = i + 0.06 + i(0.06)


1.06i = 0.34
i = 0.3208 (32.08% per year)

10.14 Calculate the present worth of $35,000 to be received 6 years from now, if the
predicted real rate of return is 15% per year and the inflation rate is 10% per year.

if = 0.15 + 0.10 + (0.15)(0.10) = 0.265 (26.5%)

P = 35,000(P/F,26.5%,6)
= 35,000[1/(1 + 0.265)6
= 35,000(0.24404)
= $8,541

10.15 Find the present worth sum of money that would be equivalent to the future
amounts of $5000 in year 6 and $7000 in year 8 if the real interest rate is 10% per
year and the inflation rate is 5% per year. Solve using the factors and their
equations with (a) an inflation-adjusted rate, and (b) the real interest rate.

(a) Inflation-adjusted rate

if = 0.10 + 0.05 + (0.10)(0.05) = 0.155 (15.5%)

PW = 5,000(P/F,15.5%,6) + 7,000(P/F,15.5%,8)
= 5,000(0.42122) + 7,000(0.31575)
= $4316.35

Copyright 2021 © McGraw-Hill Education. All rights reserved. No reproduction or distribution without the
prior written consent of McGraw-Hill Education.
5
(b) Real interest rate

CV5000 = 5000/(1.05)6
= $3731.09

CV7000 = 7000/(1.05)8
= $4737.88

PW = 3731.09(P/F,10%,6) + 4737.88(P/F.10%,8)
= 3731.09(0.5645) + 4737.88(0.4665)
= $4316.42

Same PW values, given small round-off errors.

10.16 How much money can the Eastman Land and Cattle Company afford to spend
now for a tractor trailer in lieu of spending $65,000 three years from now, if the
interest rate is 13% per year and the inflation rate is 7% per year? Solve (a) by
factors, and (b) a single-cell spreadsheet function.

(a) Factors: if = 0.13 + 0.07 + (0.13)(0.07) = 0.2091 (20.91%)

PW = 65,000(P/F,20.91%,3)
= 65,000(0.56574)
= $36,773
(b) Spreadsheet: Function = – PV(0.13+0.07+0.13*0.07,3,65000) displays a PW
of $36,773

10.17 The manager of the Pick ‘N’ Pak Food store is trying to determine how much can
be spent now to avoid spending an estimated $10,000 on freezer equipment 2
years from now. If the interest rate is 1.5% per month and the inflation rate is 1%
per month, what is this maximum amount? Solve (a) by factors, and (b) a single-
cell spreadsheet function.
(a) if = 0.015 + 0.01 + (0.015)(0.01) = 0.02515 (2.515% per month)
PW = 10,000(P/F,2.515%,24)
= $5,509
(b) Spreadsheet: Function = – PV(0.015+0.01+0.015*0.01,24,10000) displays a
PW of $5509

Copyright 2021 © McGraw-Hill Education. All rights reserved. No reproduction or distribution without the
prior written consent of McGraw-Hill Education.
6
10.18 Such a lucky girl! Helene was just told that she has inherited her great-
grandfather’s entire hedge fund investment account that contained $3 million. If
he started the account 50 years ago with a single deposit and never added even a
single dollar to the account, how much did he deposit? Assume that the account
increased at a rate of 10% per year and that the inflation rate during that time
period was 3% per year.
Inflation is not relevant, because we only want to know how much was deposited
50 years ago.
PW = 3,000,000(P/F,10%,50)
= 3,000,000(0.0085)
= $25,500

10.19 The makers of Lifestraw, a filter designed to allow the user to drink water safely
from a stream, have a financial decision to make. Implementing an automated
assembly process will cost $5000 per year for the next 6 years. The interest rate is
15% per year and the inflation rate is 5% per year. (a) What is the amount they
can spend now in lieu of these future costs? Solve using factors and a spreadsheet.
(b) If only $12,000 is available now, what is the maximum equivalent amount that
can be spent for each of the 6 years? How does this compare with the $5000
estimate?

(a) Factors: if = 0.15 + 0.05 + (0.15)(0.05) = 0.2075 (20.75%)

PW = 5,000(P/A,20.75%,6)
= 5,000(3.2645)
= $16,323 (from P/A formula)

Spreadsheet: = – PV(20.75%,6,5000) displays a PW of $16,323

(b) Function = – PMT(20.75%,6,12000) displays an AW of $3676 per year. This


is $1,324 per year less than the $5000 per year estimate.

10.20 (a) Find the present worth of the following estimated cash flows. As indicated,
some are expressed in future (then-current) dollars and others in current-value
(today’s) dollars. Use a real interest rate of 10% per year and an inflation rate of
6% per year. (b) (Spreadsheet exercise) Here’s a challenge. Write the single-cell
spreadsheet function to determine PW.

Year Cash Flow, $ Expressed as


0 16,000 Today’s
3 40,000 Then-current
4 12,000 Then-current
7 26,000 Today’s

Copyright 2021 © McGraw-Hill Education. All rights reserved. No reproduction or distribution without the
prior written consent of McGraw-Hill Education.
7
(a) Use if for then-current (future) dollars and i for today’s (current-value) dollars
if = 0.10 + 0.06 + (0.10)(0.06) = 16.6%
PW = 16,000 + 40,000(P/F,16.6%,3) + 12,000(P/F,16.6%,4)
+ 26,000(P/F,10%,7)
= 16,000 + 40,000(0.63082) + 12,000(0.54101) + 26,000(0.51316)
= $61,067
(b) = 16000 – PV(16.6%,3,40000) – PV(16.6%,4,12000) – PV(10%,7,26000)

10.21 May-Kit Metal Fabricators, Inc. is located in a relatively high inflation country.
Two machines under consideration have estimated cash flows as shown. (a) Use a
PW analysis to select one machine at i = 15% and f = 12% per year. (b)
(Spreadsheet exercise) The fab manager believes the AOC for A is quite high,
plus he favors this alternative. Determine the maximum AOC for A to reach
breakeven between A and B.
Machine A B
First cost, $ −10,000 −20,000
AOC, $ per year −8,000 −3,000
Salvage value, $ 3,000 6,000
Life, years 10 10
(a) if = 0.15 + 0.12 + (0.15)(0.12) = 0.288 (28.8%)
PWA = –10,000 – 8,000(P/A,28.8%,10) + 3,000(P/F,28.8%,10)
= –10,000 – 8,000(3.19588) + 3,000(0.07959)
= $–35,328
PWB = –20,000 – 3,000(P/A,28.8%,10) + 6,000(P/F,28.8%,10)
= –20,000 – 3,000(3.19588) + 6,000(0.07959)
= $–29,110
Select machine B
(b) Fastest approach uses GOAL SEEK to change AOC for A (cell B1) while
forcing PWA to equal PWB = –29110 in cell B6. Display to breakeven is:
Maximum AOC for A of $6054 per year

Copyright 2021 © McGraw-Hill Education. All rights reserved. No reproduction or distribution without the
prior written consent of McGraw-Hill Education.
8
10.22 You have been offered the opportunity to invest in one of the two programs
outlined below: railroads or olive trees. Select one on the basis of their capitalized
costs with adjustments made for inflation. The M&O and proceeds are one
investor’s share of the annual costs and projected end-of-project proceeds.
Revenues are expected to be the same for both. Use i = 12% and f = 3% per year.

Alternative Railroads Olive Trees


Investment now, $ −18,500 −9,000
M&O, $ per year −250 −1,000
End-of-program proceeds, $ 10,500 8,200
Life, years ∞ 10

if = 0.12 + 0.03 + (0.12)(0.03) = 15.36%

For railroads, determine CC of M&O; add the investment

CCRailroads = –18,500 – 250/0.1536


= $–20,128

For olive trees, first find AW; then divide by if

AWOlives = –9000(A/P,15.36%,10) – 1000 + 8200(A/F,15.36%,10)


= –9000(0.20199) – 1000 + 8200(0.0484)
= $–2421

CCOlives = –2421/0.1536
= $–15,762

Select olive tree investment

10.23 A company that makes hand and power tools has developed a fund of $280,000
now for possibly replacing its thermoforming equipment. If the replacement isn’t
needed for 3 years, how much will the company have available provided the fund
earns at the market rate of 10% per year when the inflation rate is 4% per year?

F = 280,000(F/P,10%,3)
= 280,000(1.3310)
= $372,680

10.24 In order to encourage its employees to save money for retirement, a large
pharmaceutical company offers a guaranteed rate of return of 10% per year, no
matter what the going market rate might be. If you, as a newly-hired engineer,
invest $5000 per year for 10 years when the inflation rate averages 4% per year,
how much will be in your account at the end of the 10-year period? Also, what is
the percent increase over the total of $50,000 that you placed in the program?

Copyright 2021 © McGraw-Hill Education. All rights reserved. No reproduction or distribution without the
prior written consent of McGraw-Hill Education.
9
F = 5000(F/A,10%,10)
= 5000(15.9374)
= $79,687

Percent increase = (29,687/50,000)(100) = 59.4%

10.25 Well-managed companies set aside funds to pay for emergencies that inevitably
arise in the course of doing business. A commercial solid waste recycling and
disposal company has placed 1% of its after-tax income into such a fund for 5
years. The market rate of return averages 12% per year and the inflation rate is
4% per year. (a) What is the purchasing power in CV dollars of the accumulated
amount if the aftertax income averages $18 million per year? (b) What was the
real rate of return earned by the fund?

(a) Accumulated amount, F = 0.01(18,000,000)(F/A,12%,5)


= 180,000(6.3528)
= $1,143,504

Purchasing power in CV = 1,143,504/(1 + 0.04)5


= $939,877

(b) By Equation [10.9]

i = (0.12 – 0.04)/(1.04) = 0.0769 (7.69% per year)

10.26 The strategic plan of a solar energy company that manufactures high-efficiency
solar cells includes an expansion of its physical plant in 4 years. As the engineer
in charge of facilities, you estimate that the expenditure required now is
$8,000,000, but in 4 years the cost will be higher by an amount equal to the
annual inflation rate. If the company sets aside $7,000,000 now and it earns at 7%
per year, you want to inform the finance committee of the maximum inflation rate
allowed to ensure that exactly the right amount of money is available for the
expansion. What is that inflation rate? Solve by (a) factors, and (b) a spreadsheet
with GOAL SEEK.

(a) 8,000,000(1 + f)4 = 7,000,000(F/P,7%,4)


8,000,000(1 + f)4 = 7,000,000(1.3108)
8,000,000(1 + f)4 = 9,175,600
(1 + f)4 = 1.14695
4 log (1+f) = 0.05954
log (1 + f) = 0.01489
1 + f = 100.01489
1 + f = 1.03487
f = 0.0348 (3.48% per year)

Copyright 2021 © McGraw-Hill Education. All rights reserved. No reproduction or distribution without the
prior written consent of McGraw-Hill Education.
10
(b) Use GOAL SEEK to change cell C2, the inflation rate, such that the FW of
cost equals FW of fund value (cell B5).

Display shows a maximum inflation rate of 3.49% per year

10.27 To cover themselves from lawsuits, most medical doctors in private practice carry
liability insurance. Assume the annual premium for liability insurance costs
$40,000 today for a coverage of $25 million. (a) If the premium increases only by
the inflation rate of 6% per year when the market rate of return is 10% per year,
estimate the annual premium 10 years from now. (b) Dr. Kirchner is complaining
that only 5 years ago, the same coverage cost $18,000 per year. Use a spreadsheet
to determine the rate of inflation over the last 5-year period.

(a) Premium = 40,000(1 + 0.06)10


= 40,000(1.79085)
= $71,634

(b) Place a ‘guessed at’ rate in a cell, say, B2. Develop the function
= – PV($B$2,5,40000) and use GOAL SEEK to force the display to be
$18,000. The response is an f value of 17.32% per year.

10.28 (a) How many future dollars will you need 5 years from now to maintain the
buying power of $50,000 now, if the deflation rate is 3% per year? (b) Why is
persistent deflation a potential danger to the economic health of a country?
(a) Find F in future (then-current) dollars using f = –3.0%
F = 50,000(1 – 0.03)5
= 50,000(0.85873)
= $42,937
(b) In short, money becomes ‘tighter’. People do not spend as readily, there is a
lack of capital to improve and build new industry, fewer jobs are available,
personal and business credit becomes less available, etc.

Copyright 2021 © McGraw-Hill Education. All rights reserved. No reproduction or distribution without the
prior written consent of McGraw-Hill Education.
11
10.29 Five years ago, an industrial engineer deposited $10,000 into an account and left
it undisturbed through now. The account is now worth $25,000. (a) What was the
overall ROR during that time? (b) If the inflation over 5 years was 4% per year,
what was the real ROR? (c) What is the purchasing power of the $25,000 with
respect to the purchasing power of dollars 5 years ago?

(a) 25,000 = 10,000(F/P,i*,5)


(F/P,i*,5) = 2.5000
i* = 20.1%

Spreadsheet function = RATE(5,–10000,25000) provides the i* value

(b) 0.201 = i + 0.04 + i(0.04)


1.04i = 0.161
i = 0.1548 (15.48%)

(c) Purchasing power = 25,000/(1 + 0.04)5 = $20,548

10.30 Timken Roller Bearing is a manufacturer of seamless tubes for drill bit collars.
The company is planning to add larger capacity robotic arms to one of its
assembly lines 3 years from now. If done now, the cost of the equipment with
installation will be $2.4 million. If the company’s real MARR is 15% per year,
what is the equivalent amount the company could spend 3 years from now in
future dollars? Inflation is 2.8% per year.

First, calculate MARRf to consider inflation

MARRf = 0.15 + 0.028 + (0.15)(0.028) = 0.1822 (18.22%)

FW = 2,400,000(F/P,18.22%,3)
= 2,400,000(1 + 0.1822)3
= $3,965,374

10.31 If the inflation rate is 6% per year and you want to earn a real return of 10% per
year, how many future dollars must you receive 10 years from now for an
investment of $10,000 today?

if = 0.10 + 0.06 + (0.10)(0.06) = 0.166 (16.6%)

F = 10,000(F/P,16.6%,10)
= 10,000(1 + 0.166)10
= $46,450

Copyright 2021 © McGraw-Hill Education. All rights reserved. No reproduction or distribution without the
prior written consent of McGraw-Hill Education.
12
10.32 The company you work for signed a contract to control access to company
grounds and corporate offices. The contract was for $140,000 the first year,
renewable each year for up to a total of 5 years at the same cost, plus a percentage
increase equal to the inflation rate for the preceding year. Your boss asked you to
calculate the expected cost in the 5th year. Assuming the inflation rate is 3% for
the next 3 years and 5% for the fourth, what is your estimate?

Cost, year 5 = 140,000(1 + 0.03)3(1 + 0.05) = $160,631

10.33 An engineer planning for her son’s college education made deposits into a high-
risk stock hedge fund every time she earned extra money from side consulting
jobs. The deposits and their timing are as follows:

Year Deposit, $
0 5,000
3 8,000
4 9,000
7 15,000
11 16,000
17 20,000

a. The account increased at a market rate of 15% per year while inflation
averaged 3% per year over the entire period. What was the purchasing power
in terms of year 0 constant-value dollars immediately after the last deposit?

b. (Spreadsheet exercise) Calculate the purchasing power if the account actually


earned less at 6% per year and inflation was higher than expected at 4% per
year. Now, compare the purchasing power with the actual amount she
deposited over the 17 years.

(a) Find F, then determine purchasing power by dividing by (1 + f)n

F = 5000(F/P,15%,17) + 8000(F/P,15%,14) + 9000(F/P,15%,13)


+ 15,000(F/P,15%,10) + 16,000(F/P,15%,6) + 20,000
= 5000(10.7613) + 8000(7.0757) + 9000(6.1528)
+ 15,000(4.0456) + 16,000(2.3131) + 20,000
= $283,481
Purchasing power = 283,481/(1.03)17 = $171,511
(b) Purchasing power in CV dollars is $61,762, which is less than the total amount
deposited of $73,000.

Copyright 2021 © McGraw-Hill Education. All rights reserved. No reproduction or distribution without the
prior written consent of McGraw-Hill Education.
13
10.34 To retire at a decent age and move to Hawaii, an engineer plans to trust her
account to an investment firm that promises to make a real ROR of 10% per year,
after commissions and fees. Inflation has a historic rate of 4% per year and the
account balance is currently $422,000. (a) To retire in 15 years with no further
deposits, determine the account balance in (future) dollars to realize a real 10%
per year return. (b) Write a single-cell spreadsheet function to display the answer.

(a) Account will have to grow at rate of if

if = 0.10 + 0.04 + (0.10)(0.04) = 0.144 (14.4%)

F = 422,000(F/P,14.4%,15)
= 422,000(7.52299)
= $3,174,701

(b) Spreadsheet function: = – FV(0.1+0.04+0.1*0.04,15,422000) displays


an F value of $3,174,701

10.35 Hammond Enterprises offers an e-commerce service that is managed inhouse at


an equivalent AW of $3.50 million based on past NCF values. An international,
outsourcing contractor can provide improved services for a guaranteed 5-year
period starting at $4.5 million per year with an annual increase of 3% on the
contract cost, assuming the same current revenue base of $8.3 million per year.
Estimates to increase revenue to $9.5 million per year and remain an inhouse
provider over a 5-year study period require an initial investment of $5 million and
an AOC of $3 million. At a market MARR of 13.4% per year, assuming an
industrywide inflation of 5% per year, select the economically better alternative
with inflation (a) not considered, and (b) considered. (c) With your selection,
what is expected to happen to revenue? (d) (Spreadsheet exercise) Perform the
analysis using a spreadsheet.

Copyright 2021 © McGraw-Hill Education. All rights reserved. No reproduction or distribution without the
prior written consent of McGraw-Hill Education.
14
(a) Inflation not considered. Perform make/buy analysis on basis of real MARR.
(Monetary values in $ million units.)

MARR = (0.134 – 0.05)/(1.05) = 0.08 (8.00%)

AWin-house = investment + enhanced revenue – AOC


= –5(A/P,8%,5) + (9.5 – 3.0)
= –5(0.25046) + 6.5
= $5.25 ($5.25 million)

AWoutsource = contract cost + current revenue


= –4.5{1 – [(1.03)/(1.08)]5/(0.08 – 0.03)}(A/P,8%,5) + 8.3
= –4.5{4.22035}(0.25046) + 8.3
= $3.54 ($3.54 million)

Select in-house with the larger AW value


(b) Inflation considered. Perform analysis at MARRf = 13.4%

AWin-house = –5(A/P,13.4%,5) + (9.5 – 3.0)


= –5(0.28709) + 6.5
= $5.06 ($5.06 million)

AWoutsource = current revenue – contract cost


= –4.5{1 – [(1.03)/(1.134)]5/(0.134 – 0.03)}(A/P,13.4%,5+ 8.3
= –4.5{3.67127}(0.28709) + 8.3
= $3.56 ($3.56 million)

Select in-house, again

(c) Plan on increased annual revenue from $8.3 to $9.5 million with the in-house
selection.

(d) Spreadsheet solutions indicate that in-house is selected in both cases.

Copyright 2021 © McGraw-Hill Education. All rights reserved. No reproduction or distribution without the
prior written consent of McGraw-Hill Education.
15
10.36 Your project manager asks you to evaluate alternatives A and B on the basis of
their PW values using a real MARR of 10% per year and an inflation rate of 3%
per year (a) without any adjustment for inflation, and (b) with inflation
considered. Also, write the spreadsheet functions that will display the correct PW
values. (c) (Spreadsheet exercise) Your manager clearly wants alternative A to be
selected. If inflation is steady at 3% per year, what real return, i, would A have to
generate each year to make the choice between A and B indifferent? Maintain the
real MARR of 10% for alternative B.

Machine A B
First cost, $ −31,000 −48,000
AOC, $ per year −28,000 −19,000
Salvage, $ 5,000 7,000
Life, years 5 5

(a) Without inflation considered

PWA = –31,000 – 28,000(P/A,10%,5) + 5000(P/F,10%,5)


= –31,000 – 28,000(3.7908) + 5000(0.6209)
= $–134,038

PWB = –48,000 – 19,000(P/A,10%,5) + 7000(P/F,10%,5)


= –48,000 – 19,000(3.7908) + 7000(0.6209)
= $–115,679

Select machine B

Spreadsheet functions: PWA: = – PV(10%,5,–28000,5000) – 31000


PWB: = – PV(10%,5,–19000,7000) – 48000

(b) With inflation considered

if = 0.10 + 0.03 + (0.10)(0.03) = 0.133 (13.3%)

PWA = –31,000 – 28,000(P/A,13.3%,5) + 5000(P/F,13.3%,5)


= –31,000 – 28,000(3.4916) + 5000(0.5356)
= $–126,087
PWB = –48,000 – 19,000(P/A,13.3%,5) + 7000(P/F,13.3%,5)
= –48,000 – 19,000(3.4916) + 7000(0.5356)
= $–110,591
Select machine B, again
Spreadsheet functions: PWA: = – PV(13.3%,5,–28000,5000) – 31000
PWB: = – PV(13.3%,5,–19000,7000) – 48000

Copyright 2021 © McGraw-Hill Education. All rights reserved. No reproduction or distribution without the
prior written consent of McGraw-Hill Education.
16
(c) Considering inflation at 3%, maintain i = 10% for B; use GOAL SEEK to
force PW difference to be 0 while changing the real return i for A (cell F2). A
real i of 17.7% per year is required for breakeven.

10.37 Johnson Thermal Products uses austenitic nickelchromium alloys to manufacture


resistance heating wire. The company is considering a new annealing-drawing
process to reduce costs. If the new process will cost $3.1 million now, how much
must be saved each year to recover the investment in 5 years if the company’s
MARR is a real 10% per year and the inflation rate is 3% per year?

if = 0.10 + 0.03 + (0.10)(0.03) = 0.133 (13.3%)

A = 3,100,000(A/P,13.3%,5)
= 3,100,000(0.2864)
= $887,840 per year

10.38 MetroKlean, LLC, an oil field hazardous-waste cleaning company, borrowed $2.5
million to finance start-up costs for a site reclamation project. How much must the
company receive each year in revenue to earn a real ROR of 20% per year for the
5-year project period, if inflation is 5% per year?

if = 0.20 + 0.05 + (0.20)(0.05) = 0.260 (26.0%)

A = 2,500,000(A/P,26%,5)
= 2,500,000(0.3795)
= $948,750 per year

Copyright 2021 © McGraw-Hill Education. All rights reserved. No reproduction or distribution without the
prior written consent of McGraw-Hill Education.
17
10.39 In wisely planning for your retirement, you invest $25,000 per year for 20 years
into a 401K tax-deferred account. Assume you make a real return of 10% per year
when the inflation rate averages 2.8% per year. (a) How many future dollars will
you have in the account immediately after your last deposit? (b) How much will
you be able to withdraw each year for 10 years, starting one year after your last
deposit?

(a) Market ROR: if = 0.10 + 0.028 + (0.10)(0.028) = 0.1308 (13.08%)

F = 25,000(F/A,13.08%,20)
= 25,000(81.7076)
= $2,042,690

(b) A = 25,000(F/A,13.08%,20)(A/P,13.08%,10)
= 25,000(81.7076)(0.18488)
= $377,653 per year

10.40 A European-based cattle genetics engineering research lab is planning for a major
expenditure on research equipment. The lab needs $6 million of CV dollars so it
can make the acquisition 4 years from now. The inflation rate is steady at 5% per
year. (a) How many future dollars will be needed when the equipment is acquired,
if purchasing power is maintained? (b) What is the required amount of the annual
deposit into a fund that earns the market rate of 10% per year to ensure that the
amount calculated in part (a) is accumulated? (b) Write a single-cell function that
immediately displays the correct annual deposit required.

(a) To maintain purchasing power, use f to find future dollars.

F = 6,000,000(F/P,5%,4)
= 6,000,000(1.2155)
= $7,293,000

(b) Use market rate to find A

A = 7,293,000(A/F,10%,4)
= 7,293,000(0.21547)
= $1,571,423 per year

(c) Function: = – PMT(10%,4,–FV(5%,4,6000000)) displays A = $1,571,437

10.41 A multinational software security firm is planning an overseas expansion 3 years


from now that will cost $50 million of today’s dollars. Due to a robust economy
in Europe in this area, the cost is expected to increase by 15% per year in each of
the next 3 years. Assuming the inflation rate is 4% per year, (a) determine the
required annual deposit into a fund that earns the market rate of 10% per year to

Copyright 2021 © McGraw-Hill Education. All rights reserved. No reproduction or distribution without the
prior written consent of McGraw-Hill Education.
18
ensure that the amount needed in 3 years will be available. (b) Write the
spreadsheet function that displays the annual deposit directly.
(a) Use 15% rate to determine cost of expansion, then find A using market rate.
F = 50,000,000(F/P,15%,3)
= 50,000,000(1.5209)
= $76,045,000
A = 76,045,000 (A/F,10%,3)
= 76,045,000 (0.30211)
= $22,973,955 per year
(b) Function: = – PMT(10%,3,– FV(15%,3,50000000)) displays A = $22,973,943

10.42 Assume the market interest rate is 8% per year and inflation averages 4% per
year. (a) Calculate the perpetual equivalent annual worth in future dollars for
years 1 through ∞ for an income of $50,000 now and $5000 per year thereafter.
(b) If the amounts had been quoted in CV dollars, what is the annual worth in
future dollars?

(a) Use market rate to find AW

AW = 50,000(0.08) + 5000
= $9000

(b) For CV dollars, first find PW using the real rate i; then find AW using if

i = (0.08– 0.04)/1.04 = 0.0385 (3.85%)

PW = 50,000 + 5000/0.0385
= $179,870

AW = 179,870(0.08)
= $14,390

10.43 For a market interest rate of 12% per year and an inflation rate of 7% per year, the
real interest rate per year is closest to:

a. 4.7% c. 12%
b. 7% d. 19.8%

0.12 = i + 0.07 + (i)(0.07)


1.07i = 0.05
i = 0.0464 (4.64%)
Answer is (a)

Copyright 2021 © McGraw-Hill Education. All rights reserved. No reproduction or distribution without the
prior written consent of McGraw-Hill Education.
19
10.44 If the market ROR is 16% per year when the real ROR is 9% per year, the
inflation rate is closest to:

a. 6.4% c. 9%
b. 7% d. 15.6%

0.16 = 0.09 + f + 0.09(f)


1.09f = 0.07
f = 0.064 (6.4%)
Answer is (a)

10.45 A modern 3D printer has a cost today of $20,000. The future worth of an
equivalent printer 10 years from now at an inflation rate of 4% and an interest rate
of 9% per year is closest to:

a. $29,000 c. $67,900
b. $47,300 d. $70,080

if = 0.09 + 0.04 + (0.09)(0.04) = 0.1336 (13.36%)

F = 20,000(F/P, 13.36%,10)
= 20,000(1 + 0.1336)10
= $70,086
Answer is (d)

10.46 A used Porsche 911, your favorite European sports car, has a cost today of
$140,000. Because of high demand, its cost is expected to increase by 2X the
inflation rate. If the inflation rate is 3% and the market interest rate is 8% per
year, the cost 4 years from now is estimated to be closest to:
a. $177,000 c. $190,470
b. $189,500 d. $253,000
Cost in 4 years = 140,000(1 + 2×0.03)4
= $176,747
Answer is (a)

10.47 If the market rate today on foreign stock investments is 12% per year and the
inflation rate is 5% per year, the number of future dollars seven years in the future
that will have the same purchasing power as $2000 now is closest to:

a. $905 c. $2810
b. $1421 d. $4420

Future dollars = 2000(1 + 0.05)7


= $2814
Answer is (c)

Copyright 2021 © McGraw-Hill Education. All rights reserved. No reproduction or distribution without the
prior written consent of McGraw-Hill Education.
20
10.48 Construction equipment has a cost today of $40,000. If its cost has increased only
by the inflation rate of 6% per year when the market interest rate was 10% per
year, its cost 10 years ago was closest to:

a. $15,420 c. $27,405
b. $22,300 d. $71,630

Cost 10 years ago = 40,000/(1 + 0.06)10


= $22,336
Answer is (b)

10.49 Abby made an investment of only $1000 25 years ago. The amount available to
her 10 years from now at the reasonable market ROR of 5% per year, given an
inflation rate of 2% per year, is closest to:

a. $3085 c. $5510
b. $4430 d. $15,000

F = 1000(F/P,5%,35)
= 1000(5.5160)
= $5516
Answer is (c)

10.50 Your favorite pickup is a well-equipped F-150 truck. The cost 4 years ago was
$29,350. If the cost increased only by the inflation rate and the price today is
$35,675, the inflation rate over the 4-year period was closest to:

a. 3% c. 5%
b. 4% d. 6%

35,675 = 29,350(1 + f)4


(1 + f)4 = 1.2155
(1 + f) = 1.21550.25
f = 0.05 (5.0%)
Answer is (c)

10.51 If you are promised $65,000 five years from now, the present worth of the
purchasing power today in CV dollars at a real ROR of 4% per year, when the
inflation rate is 3% per year, is closest to:
a. $27,600 c. $33,100
b. $29,800 d. $46,100

if = 0.04 + 0.03 + (0.04)(0.03) = 0.712 (7.12%)

PW = 65,000/(1.0712)5
= $46,085
Answer is (d)

Copyright 2021 © McGraw-Hill Education. All rights reserved. No reproduction or distribution without the
prior written consent of McGraw-Hill Education.
21
10.52 If the inflation rate is 8% per year, the purchasing power 10 years from now of
$20,000 future dollars with respect to today’s (CV) dollars is closest to:

a. $8178 c. $8900
b. $8423 d. $9270

Purchasing power today = 20,000/(1 + 0.08)10


= $9264
Answer is (d)

10.53 For a real interest rate of 1% per month and an inflation rate of 1% per month, the
nominal inflated interest rate per year is closest to:

a. 2% c. 24%
b. 12% d. 24.12%

if per month = 0.01 + 0.01 + (0.01)(0.01) = 0.0201 (2.01%)

Nominal if per year = 2.01(12)


= 24.12%
Answer is (d)

10.54 (Spreadsheet exercise, if instructed) You own and operate Braxon Consultants. A
new CNC system will cost $350,000 now, have an AOC of $60,000, produce
revenue estimated at $200,000 per year, have a useful life of 10 years, and bring a
salvage of 20% of first cost. You expect a real MARR of 8% per year and realize
that inflation averages 3.5% per year. If you calculate the difference between the
AW values with and without inflation considered, the difference is closest to:

a. $5,000 c. $12,000
b. $10,000 d. $15,000

Without inflation

Function: = – PMT(8%,10,–350000,0.2*350000) – 60000 + 200000


displays AW = $92,672

Factors: AW = –350,000(A/P,8%,10) – 60,000 + 200,000


+ 0.2(350,000)(A/F,8%,10)
= –350,000(0.14903) + 140,000 + 70,000(0.06903)
= $92,672

Copyright 2021 © McGraw-Hill Education. All rights reserved. No reproduction or distribution without the
prior written consent of McGraw-Hill Education.
22
With inflation: MARRf = 0.08 + 0.035 + 0.08(0.035) = 0.1178

Function: = – PMT(11.78%,10,–350000,0.2*350000) – 60000 + 200000


displays AW = $82,644

Factors: AW = –350,000(A/P,11.78%,10) – 60,000 + 200,000


+ 0.2(350,000)(A/F,11.78%,10)
= –350,000(0.17539) + 140,000 + 70,000(0.05759)
= $82,644

Difference = 92,672 – 82,644


= $10,028
Answer is (b)

Copyright 2021 © McGraw-Hill Education. All rights reserved. No reproduction or distribution without the
prior written consent of McGraw-Hill Education.
23
Solutions to end-of-chapter problems
Basics of Engineering Economy, 3rd edition
Leland Blank and Anthony Tarquin

Chapter 11
Cost Estimation and Allocation
11.1 Classify the following cost elements as first-cost (FC) components or annual
operating cost (AOC) components for a piece of equipment on the shop floor:
supplies, insurance, equipment cost, utility cost, installation, delivery charges, labor
cost.

Supplies: AOC Installation: FC


Insurance: AOC Delivery charges: FC
Equipment cost: FC Labor cost: AOC
Utility cost: AOC

11.2 Explain the difference between a cost estimate that is accurate and one that is
precise.

Accurate means the estimate focuses on the correct value, that is, the actual
number when it is known. Precise means the sample estimates are tightly
clustered around a center or average point.

11.3 Total solids measurements from samples of sludge taken from a belt filter press
yielded the following results: 13.2%, 14.6%, and 13.4%. If the actual value was
13.5%, what was the percentage error of the average of the total solids
measurements? Is this an under- or overestimate?

Average = (13.2 + 14.6 + 13.4)/3 = 13.7%

% error = [(13.7/13.5) – 1](100) = 1.48% (an over-estimate)

11.4 An industrial engineer collected data about the time required by various teams to
assemble a 5-ton residential air-conditioning unit. Each team was evaluated five
times. Team A had times of 1.5, 1.4, 1.2, 1.4, and 1.2 hours. Team B had times of
1.5, 1.3, 1.1, 1.1, and 1.3 hours. Team C had times of 1.5, 1.4, 1.4, 1.5, and 1.3
hours. If range is used as the sole criterion for determining precision, which team’s
results were the most precise and what was the range of their results?

Range, team A = 1.5 – 1.2 = 0.3


Range, team B = 1.5 – 1.1 = 0.4
Range, team C = 1.5 – 1.3 = 0.2

Team C had most precise results with range of 0.2 hours

Copyright 2021 © McGraw-Hill Education. All rights reserved. No reproduction or distribution without the
prior written consent of McGraw-Hill Education.
1
11.5 Three friends—Marjorie, Jane, and Joe—went to a county fair and paid a play fee
of $10 each to enter a game in which each makes one estimate of the number of
chocolate M&Ms® in a glass jar. The one closest within 50 candies or ±5% of the
actual count wins $100. Assuming no other people entered the game, who gets the
$100 or the $30 play fee if the measure of “closest” is (a) being correct as measured
by absolute difference, and (b) being correct measured by percentage error? The
guesses are: Marjorie 950, Jane 756, and Joe 699. The actual number of candies in
the jar is 839. (c) For another round and an additional $10 each, the hawker gives
the trio a second chance to win by pooling their three estimates and using the
average with the measure of “closest” being the ±5% percentage error criterion.
Does the trio get to split the $100 prize? (d) If they win in either round, what are the
net winnings of the trio?

(a) Absolute difference:

Marjorie: │950 – 839│ = 111


Jane: │756 – 839│ = 83
Joe: │699 – 839│ = 140

No winner, since none are within 50 of actual; hawker keeps the $30.

(b) Percentage error:

Marjorie: [(950/839) – 1](100) = +13.2%


Jane: [(756/839) – 1](100) = –9.9%
Joe: [(699/839) – 1](100) = –16.7%

No winner, since none are within ±5% of actual; hawker keeps the $30.

(c) Percentage error of sample average:

Average = (950 + 756 + 699)/3 = 801.7

Percentage error = [(801.7/839.0) – 1]100 = –4.4%

Yes, they win, since the error is within ±5% of actual.

(d) First round: net winnings are 100 – 30 = $70 for the trio

Second round: net winnings are 100 – 60 = $40 for the trio

Copyright 2021 © McGraw-Hill Education. All rights reserved. No reproduction or distribution without the
prior written consent of McGraw-Hill Education.
2
11.6 To evaluate the performance of certified water testing laboratories, the appropriate
governing agencies send standard solutions to the labs with concentrations
unknown to the lab. In analyzing four samples with actual concentrations of 12.3,
72.2, 130.6, and 210.4 mg/L of total hardness, laboratory X reported results of 12.5,
70.2, 128.3, and 214.7 mg/L, respectively. The respective results from laboratory Y
were 11.3, 71.2, 129.6, and 209.4 mg/L. Which laboratory results were the more
accurate based on the sum of the absolute differences between the actual and test
concentrations?

Lab X, sum of absolute differences = │12.5 – 12.3│+│70.2 – 72.2│


+│128.3 – 130.6│ + │214.7 – 210.4│
= 0.2 + 2.0 + 2.3 + 4.3
= 8.8

Lab Y, sum of absolute differences = │11.3 – 12.3│+│71.2 – 72.2│


+│129.6 – 130.6│+│209.4 – 210.4│
=1+1+1+1
=4
Lab Y is more accurate

11.7 A preliminary estimate for the cost of a parking garage can be made using the unit
cost method. If the cost per parking space is $19,700, what is the estimated cost for
a 600-space garage?

Cost = 19,700(600)
= $11,820,000

11.8 Use the unit cost method to determine the preliminary cost of a guardrail for a
hillside highway, if a total of 420 linear feet will be required at a cost of $58.19 per
linear foot for material, equipment, and installation.

Cost = 420(58.19)
= $24,440

11.9 The University of Texas Southwestern (UTS) hospital has over 1.3 million square
feet and has 532 beds. It cost $800 million to build. Parkland Memorial (PM),
Dallas County’s public hospital, has about 2 million square feet and contains 862
beds. If the cost per bed for PM was the same as that of the UTS hospital, and its
cost was $1.3 billion, how many beds would it have?

UTS cost/bed = 800,000,000/532


= $1,503,759

PM beds = 1,300,000,000/1,503,759
= 865

Copyright 2021 © McGraw-Hill Education. All rights reserved. No reproduction or distribution without the
prior written consent of McGraw-Hill Education.
3
11.10 Your parents are in the process of having a new house designed. A total of 150
cubic yards of foundation concrete must be poured for the current 1-story design.
The equipment required to pour the concrete by experienced workers is 2 gasoline
engine vibrators and 1 concrete pump. The vibrators cost $76 per day and the
pump costs $580 per day. The contractor states that only 1 day of equipment
rental is needed to pour the foundation. (a) Your dad asked you to estimate the
cost of the equipment per cubic yard of concrete. (b) A 2-story house will have a
smaller footprint and require only 90 cubic yards of concrete. Now estimate the
equipment cost for him, assuming partial- day charges for the equipment are not
possible. What is the percent change in cost per cubic yard going from a 1-story to
2-story design?

(a) Equipment cost per day = 2(76) + 580


= $732

Cost per cubic yard = 732/150


= $4.88 per cubic yard

(b) Cost per cubic yard = 732/90


= $8.13 per cubic yard

Percent increase = (8.13 – 4.88)/4.88)(100%) = + 66.6%

11.11 Cost estimates for clinics can be made using costs based on either unit area
(square feet) or unit volume (cubic feet). If the unit area cost is $400 per square
foot and the average height of the ceilings is 9 feet, what is the unit volume cost?

Cost/volume = 400/[(1ft2)(9 ft)]


= $44.44 per ft3

11.12 The Department of Defense uses an area cost factor (ACF) to compensate for
variations in construction costs in different parts of the country and world. If a
cold storage warehouse cost $1,150,000 in Rapid City, South Dakota, where the
ACF is 0.93, and the cost of a similar facility built in Andros Island, The
Bahamas, cost $2,500,000, what is the ACF for Andros Island?

1,150,000/2,500,000 = 0.93/ACF
ACF = 2.02

Copyright 2021 © McGraw-Hill Education. All rights reserved. No reproduction or distribution without the
prior written consent of McGraw-Hill Education.
4
11.13 A 130,000-square-foot building is to be constructed on a university campus. You
just received two sets of estimates: physical area and furnishings percentages, and
the per unit cost factors, as shown below. Estimate the cost of the building and its
furnishings.

Area, % of Cost per


Type of Usage 130,000 ft2, $
Classrooms 30 125
Laboratories 40 185
Offices 30 110
Furnishings—labs 25 150
Furnishings—all others 75 25

Estimate,
Type Area, % Unit cost, $ $ million
Classrooms 39,000 125 4.8750
Lab 52,000 185 9.6200
Office 39,000 110 4.2900
Furnishings-labs 32,500 150 4.8750
Furnishings-other 97,500 25 2.4375
Total 26.0975

Estimate is $26,097,500

11.14 A construction cost index identified as CCX measures the construction cost in
each state as a percentage of the national average. From historical data, you
discover that the national average construction cost of middle schools is $10,200
per student. (a) If the CCX index for Texas is 0.721, what is the estimated cost of
an 800-student school? (b) What is the CCX index for California if an 800-student
school cost $9,114,000?

(a) Cost in Texas = 10,200(800)(0.721)


= $5,883,360

(b) 9,114,000 = 10,200(800)(CCX)


CCX = 1.117

11.15 A consulting engineering firm wants to make a preliminary cost estimate for the
design/construct of an e-commerce warehouse facility in the south of the country.
The firm completed a similar project in 2012 that had a construction cost of $68
million, and it wants to use the ENR Construction Cost Index (CCI) to update the
cost. If the index value in 2012 was 8802 and today it is 11,981, determine the
estimated cost of the facility today. (Note: CCI values may be different on its
website.)

Ct = 68,000,000(11,981/8802)
= $92,559,418

Copyright 2021 © McGraw-Hill Education. All rights reserved. No reproduction or distribution without the
prior written consent of McGraw-Hill Education.
5
11.16 An engineer who owns a construction company that specializes in large commercial
projects noticed that material costs increased at a rate of 1.5% per month over the
past 12 months. If a material cost index were created for each month of this year
with the value of the index set at 100 at the beginning of the year, what is the value
of the index at the end of July? Express your answer to two decimal places.

Price increased at 1.5% per month for 7 months.


Increase = (1 + 0.015)7 –1
= 0.10984 (10.984%)

Index value = 100(1.10984)


= 110.98

11.17 Electropneumatic general-purpose pressure transducers convert supply pressure to


regulated output pressure in direct proportion to an electrical input signal. The
cost of a transducer was $410 in 2005 when the relevant cost index was at 789.6.
If the price increased exactly in proportion to the index, what is the cost estimate
today if the index value is now 3461.3?

Cost = 410(3461.3/789.6)
= $1797

11.18 The construction cost index for New York City had a value of 12,381.40 when the
value for Pittsburgh was 7341.32. If a construction contractor in Pittsburgh bid on
a job that would total $94.0 million in Pittsburgh, determine the equivalent bid he
should make in the New York City solicitation.

Value in NYC = (94.0 million)(12,381.40/7341.32)


= $158.53 million

11.19 A contractor purchased pump equipment costing $40,000 when the FRED pumps,
compressors, and equipment index was 265.2. He remembers purchasing the same
equipment for $25,000 some years ago, but he does not remember how many
years ago. If the index increased by 2.38% per year since the previous purchase
and the equipment increased in price exactly in proportion to the index, (a) how
many years ago did he purchase the pump, and (b) what was the approximate
value of the index in that year?

(a) 40,000 = 25,000(F/P,2.38%,n)


40,000 = 25,000(1 + 0.0238)n
1.60 = (1.0238)n
log 1.60 = n log 1.0238
n = 20 years ago

(Note: Can also use GOAL SEEK to find n = 20 years.)

(b) Index value = 265.2/(1.0238)20 = 165.7

Copyright 2021 © McGraw-Hill Education. All rights reserved. No reproduction or distribution without the
prior written consent of McGraw-Hill Education.
6
11.20 The cost of chemical plant equipment was estimated to be $464.5 million when
the value of the CEPCI was 567.5. If the cost of similar equipment increased in
proportion to the index, what was the value of the index when the cost was
$400 million?

464.5 = 400(567.5/I0)
I0 = 488.7

11.21 A 0.75 million gallon per day (MGD) Induced Draft Packed Tower for air-
stripping trihalomethanes from drinking water costs $58,890. Estimate the cost of
a 2.0 MGD tower if the exponent in the cost-capacity equation is 0.57.

C2 = 58,890(2/0.75)0.57
= $103,002

11.22 Use the exponent values in Table 11.4 to estimate the cost for the following
equipment to be placed on an offshore drilling platform.
a. The cost of a 125 hp centrifugal pump, if a 200 hp pump costs $28,000.
b. The cost of a 1700 gallon stainless-steel tank, if a 900 gallon tank costs $4100.

(a) Cost = 28,000[(125/200)0.69


= $20,245

(b) Cost = 4100[(1700/900)0.67


= $6278

11.23 The onsite manager told you that the variable frequency drive (VFD) for a 300 hp
motor costs $20,000. Make the best cost estimate possible for the VFD for a
100 hp motor. Since the exponent in the cost-capacity equation is not available
where you are located, use the average value.

Use the six-tenths model; exponent = 0.60

20,000 = C1(300/100)0.60
20,000 = 1.93318C1
C1 = $10,346

11.24 The cost of a 68 m2 falling-film evaporator was 1.52 times the cost of a 30 m 2
unit. What exponent value in the cost-capacity equation yielded these results?

1.52C1 = C1(68/30)x
log 1.52 = x log 2.267
x = 0.51

Copyright 2021 © McGraw-Hill Education. All rights reserved. No reproduction or distribution without the
prior written consent of McGraw-Hill Education.
7
11.25 Instead of using a cost-capacity equation for relating project size and construction
cost, the Department of Defense uses a size adjustment factor (SAF) that is based on
the size relationship ratio (SRR). If an SRR of 2.0 has an SAF of 1.25, what must be
the exponent in the cost-capacity relation for C2 to be 1.25C1 when Q2 ∕Q1 = 2.0?

If C2 = 1.25C1, the cost-capacity relation is

1.25C1 = C1 (Q2/Q1)x

1.25 = (2.0)x
log 1.25 = x log 2.0
x = 0.32

11.26 The cost of an 8-inch pipe line placed in an open channel in 2016 was $16.13 per
LF. An 18-inch pipe cost $22.90 per LF. The cross-sectional area of the pipe is
considered the “capacity” in the costcapacity equation. (a) Determine the value of
the exponent in the cost-capacity equation that exactly relates the two pipe sizes.
(b) You learned that the FRED cost index has increased from its 2016 value to
280.0 now. What is your updated cost estimate for an 18-inch pipe?

(a) Area of 8” pipe = π(8/12)2/4


= 0.349 ft2

Area of 18” pipe = π(18/12)2/4


= 1.767 ft2

22.90 = 16.13(1.767/0.349)x
1.42 = 5.063x
log 1.42 = x log 5.063
0.152 = 0.704x
x = 0.216

(b) FRED index in 2016 was 247.0

C2 = 16.13(1.767/0.349)0.216(280.0/247.0)
= 16.13(1.42)(1.134)
= $25.97 per LF

11.27 A 100,000 barrel per day (BPD) fractionation tower cost $1.2 million in 2010.
Use the chemical engineering plant cost index (CEPCI) to estimate the cost for a
450,000 BPD plant cost. The index (annual value) is now 607.0, and the exponent
in the cost-capacity equation is 0.67.

Use Equation [11.4]. CEPCI in 2010 was 550.8

Cost = (1.2 million)[(450,000/100,000)0.67](607.0/550.8)


= $3.623 million

Copyright 2021 © McGraw-Hill Education. All rights reserved. No reproduction or distribution without the
prior written consent of McGraw-Hill Education.
8
11.28 Your brother operates a small testing business. He wants to purchase a mini-wind
tunnel for calibrating vane or hotwire anemometers to complete a contract with a
local aircraft parts manufacturer. The cost was $3750 in 2010 when the relevant
equipment index was 1457.4. Now the index value is 1820.6. Help him estimate
the cost of a tunnel twice as large. The cost-capacity equation exponent is 0.89.
Cost = 3750(2)0.89 (1820.6/1457.4)
= $8,681

11.29 In 2020, a military engineer estimated the cost for a classified laser-guided device to
be $376,900. At that time, the engineer used cost indices for the years 2012 and
2020, with values of 1302 and 1536, respectively, plus the cost-capacity equation
with an exponent value of 0.61. If the original equipment had only one-fourth the
capacity of the new equipment, what was the cost of the original equipment in 2012?
Let C1 = cost in 2012
376,900 = C1(1536/1302)(4)0.61
C0 = $137,148

11.30 Estimate the current cost of a 1000 horsepower compressor if a 200 horsepower
unit cost $160,000 ten years ago and the equipment cost index has increased by
35%. Find the correct exponent in Table 11.4 for the cost-capacity equation.
Exponent is 0.32; use Equation [11.4]
C2 = 160,000 (1000/200)0.32(1.35/1)
= $361,513

11.31 The cost of equipment for manufacturing mineral products in a harsh environment
is $2.3 million. If the overall cost factor for this type of facility is 2.25, what is the
estimated total plant cost?
CT = 2.25(2,300,000)
= $5,175,000

11.32 The total cost for a production plant that supplies hydrogen to the refining
industry is estimated at $55.4 million. The equipment is expected to cost
$17.8 million. What is the overall cost factor for this type of plant?
h = 55.4/17.8
= 3.11

11.33 A chemical engineer at Tesoro Refining estimated the total cost for a diesel fuel
desulfurization system to be $13.39 million. If the direct cost factor is 1.55 and
the indirect cost factor is 0.43, what is the total equipment cost? Both factors
apply to delivered-equipment cost.
13,390,000 = (1 + 1.55 + 0.43)CE
CE = $4,493,290

Copyright 2021 © McGraw-Hill Education. All rights reserved. No reproduction or distribution without the
prior written consent of McGraw-Hill Education.
9
11.34 The delivered-equipment cost for a fully equipped CNC machining system is
$4.6 million. The direct cost factor is 1.52 and the indirect cost factor is 0.31.
Estimate the total plant cost if the indirect cost factor applies to (a) the delivered-
equipment cost only, and (b) the total direct cost.

(a) h = 1 + 1.52 + 0.31 = 2.83

CT = 2.83 (4,600,000)
= $13,018,000

(b) h = 1 + 1.52 = 2.52

CT = [4,600,000(2.52)](1.31)
= $15,185,520

11.35 Zia Homes is contracted to frame 32 wooden houses of the same design in a new
subdivision of 300 homes. The fourth unit took 400 hours, but the company
expects to cut about 10% off the time for each replication of the same design. Use
the learning curve assumption of a constant decrease with each doubling of
production to estimate past completion times (units 1 and 2) and the completion
time for units 8 and 32. Round to the nearest hour.

Use a 0.9 learning rate for each doubled production. (Times are rounded.)

Unit 2: 400/0.9 = 444 hours


Unit 1: 444/0.9 = 494 hours
Unit 8: 400(0.9) = 360 hours
Unit 32: 360(0.9)2 = 292 hours

11.36 The Department of the Navy estimates that the learning rate for shipbuilding is
80%. Assuming the 80% rate applies to nuclear submarines, how long should it
take to build the 12th vessel, if the first one took 62 months to complete?

T12 = T1Ns

s = log 0.80/log 2
= –0.322

T12 = (62)12– 0.322


= 27.9 months

Copyright 2021 © McGraw-Hill Education. All rights reserved. No reproduction or distribution without the
prior written consent of McGraw-Hill Education.
10
11.37 As an engineer with KBR Construction, you have been asked to predict the
completion time per unit to build 125 field-ready office/storage units in the plant.
The Red Cross uses the units in international hunger and medical relief efforts.
Use a spreadsheet to plot the learning curves on arithmetic and log-log scales in
25-unit increments for a 92% learning rate. The first-unit took 100 hours to
complete.

s = log 0.92/log 2 = –0.120; TN = 100(N)– 0.12; spreadsheet graphs are for


arithmetic and log-log scales.

11.38 As an engineer with KBR Construction, you have been asked to estimate the cost
per unit to build 125 field-ready office/medical units in the plant for use by the
Red Cross. (a) Use a spreadsheet to plot the cost curve on an arithmetic scale in
25- unit increments for a 92% learning rate. The first unit took 100 hours to
complete and the cost is contractually fixed at $300 per hour for all units. (b)
Explain how you will determine the total estimated cost for all 125 units.

(a) s = log 0.92/log 2 = –0.120

Time relation, hours per unit: TN = 100(N)– 0.12

Cost relation, $ per unit: CN = 300(TN)

Copyright 2021 © McGraw-Hill Education. All rights reserved. No reproduction or distribution without the
prior written consent of McGraw-Hill Education.
11
(b) Determine TN for (N = 1, 2, …, 125), add them and multiply by $300.

11.39 The company you work for currently allocates insurance costs on the basis of cost
per direct labor hour. This indirect cost component for the year is budgeted at
$36,000. If the annual direct labor hours for departments A, B, and C are expected
to be 2000, 8000, and 5000, respectively, determine the allocation to each
department.

Total direct labor hours = 2000 + 8000 + 5000


= 15,000 hours

Indirect cost rate = 36,000/15,000 = $2.40 per hour

Allocation:
Dept. A = 2000(2.40) = $4,800

Dept. B = 8000(2.40 = $19,200

Dept. C = 5000(2.40) = $12,000

11.40 Company XSell has a processing department with 10 stations. Because of the
nature and use of 3 of these stations, each is considered a separate cost center for
IDC allocation. The remaining 7 are grouped as one cost center, CC150. Operating
hours are used as the allocation basis for all stations. A total of $250,000 is
allocated to the department for next year. (a) Use the data collected this year to
determine the IDC rate for each center. (b) If accounting decides to use a blanket
rate for all departments, determine the value of this rate. How does the blanket rate
compare with the rates for CC100 and CC110 calculated previously?

IDC Operating
Cost Center Allocated, $ Hours
CC100 25,000 800
CC110 50,000 200
CC120 75,000 1200

Copyright 2021 © McGraw-Hill Education. All rights reserved. No reproduction or distribution without the
prior written consent of McGraw-Hill Education.
12
CC150 100,000 1600
(a) Rate for CC100 = 25,000/800 = $31.25 per hour
Rate for CC110 = 50,000/200 = $250.00 per hour
Rate for CC120 = 75,000/1200 = $62.50 per hour
Rate for CC190 = 100,000/1600 = $62.50 per hour

(b) Total IDC allocated = $250,000


Total operating hours = 3,800

Blanket IDC rate = 250,000/3,800


= $65.79 per hour

Comparison for CC100: Blanket is approximately 2X the cost center rate


Comparison for CC110: Blanket is approximately 1/4th the cost center rate

11.41 Milford Industries provides medical equipment to oncology and surgical units in
major hospitals. Milford allocates indirect costs to 5 departments and redistributes
the IDC for Quality Assurance and Engineering to the other 3 departments
monthly. The table summarizes IDC allocation and direct labor (DL) hours for
one month. Determine the (a) indirect cost rate, and (b) IDC allocation for each of
the 3 departments.

IDC Actual
Department Allocation, $ DL Hours
Production 30,000 500
Subassemblies 20,000 1000
Final assembly 10,000 600
Quality assurance 10,000
Engineering 25,000

(a) Total IDC to allocate = 10,000 + 25,000 = $35,000


Basis, total DL hours = 500 + 1000 + 600 = 2100 hours

IDC rate = 35,000/2100


= $16.67 per hour

(b) IDC Allocation (rounded):


Production: 500(16.67) = $ 8,335
Subassemblies: 1000(16.67) = $16,670
Final assembly: 600(16.67) = $10,002

11.42 The mechanical components division manager asks you to recommend a


make/buy decision on a major automotive subassembly that is currently
outsourced for a total of $3.9 million this year. This cost is expected to continue
rising at a rate of $300,000 per year. For the make option, equipment will cost
$3 million, have a salvage of $0.5 million, a life of 6 years, and an estimated
$1.5 million per year for all direct costs. Your manager asks that both direct and

Copyright 2021 © McGraw-Hill Education. All rights reserved. No reproduction or distribution without the
prior written consent of McGraw-Hill Education.
13
indirect costs be included when the make alternative is evaluated. Typical indirect
rates, bases, and expected usage are shown. Perform the AW evaluation at a
MARR = 12% per year over a 6-year study period. Show both (a) factor, and
(b) spreadsheet solutions.

Expected
Department Basis Rate Usage
X Direct labor cost $2.40 per $ $450,000
Y Materials cost $0.50 per $ $850,000
Z Number of $20 per inspection 4,500
inspections

(a) Factors: Make: Indirect cost computation

Rate, $ Usage Annual cost, $


Dept (1) (2) (3) = (1)(2)
X 2.40 450,000 1,080,000
Y 0.50 850,000 425,000
Z 20.00 4500 90,000
$/ 1,595,000
year

AWmake = –3,000,000(A/P,12%,6) + 500,000(A/F,12%,6) – 1,500,000


– 1,595,000
= –3,000,000(0.24323) + 500,000(0.12323) –3,095,000
= $–3,763,075

AWbuy = –3,900,000 – 300,000(A/G,12%,6)


= –3,900,000 – 300,000(2.1720)
= $–4,551,600

Select Make alternative

(b) Spreadsheet: Determine AWMake using the function format

= –PMT(12%,6,–3000000,500000) – 1500000 – IDC

Select Make alternative

Copyright 2021 © McGraw-Hill Education. All rights reserved. No reproduction or distribution without the
prior written consent of McGraw-Hill Education.
14
Copyright 2021 © McGraw-Hill Education. All rights reserved. No reproduction or distribution without the
prior written consent of McGraw-Hill Education.
15
11.43 Using an automobile as a cost center and miles traveled as the cost driver, classify
the following cost components as either direct or indirect costs: gasoline, garage
rental, license plate fee, insurance, tires, inspection fee, oil change.

Cost Component Type of Cost


Gasoline Direct
Garage rental Indirect
License plate fee Indirect
Insurance Indirect
Tires Direct
Inspection fee Indirect
Oil change Direct

11.44 For many years, Sweet Trucking has allocated truck maintenance and repair
(M&R) costs to its three divisions using a traditional basis of number of trucks
traveling through repair centers within each region. The allocation, using a
blanket IDC rate, is shown for last year when $2.31 million was allocated.

Region Trucks Rate Allocation, $


Denver 25,000 $44/ 1,100,000
truck
Albuquerque 18,000 $44/ 792,000
truck
Oklahoma City 9,500 $44/ 418,000
truck

Alberto, the M&R supervisor for Sweet, wants to switch to the ABC method
where the cost driver is a combination of load carried and age of trucks that pass
through repair centers. The measure he recommends is total tonnage passing
through a center times the average age of trucks passing through the center.
Reallocate the $2.31 million using the data below and compare the center-
bycenter allocation of traditional and ABC methods.

Region Load Recorded, Average Truck


1000 Tons Age, Years
Denver 800 12.5
Albuquerque 1200 15.8
Oklahoma City 2500 6.0
Determine load-age basis; rate (rounded) and allocation.

Region Load×Age Rate, $/Basis Allocation, $


Denver 10,000 52.55 525.500

Albuquerque 18,960 52.55 996,348


Oklahoma City 15,000 52.55 788,250

Copyright 2021 © McGraw-Hill Education. All rights reserved. No reproduction or distribution without the
prior written consent of McGraw-Hill Education.
16
Total 43,960 2,310,100
Rate = 2,310,000/(Total load×age)
= 2,310,000/43,960
= $52.55
Comparison: Denver allocation (lower loads and older trucks) decreased to about
50% when ABC is applied. Oklahoma City allocation (heavy loads
and newer trucks) increased substantially.

11.45 Alamo Power historically allocates IDC for its safety program to generation
facilities in cities A and B based on the number of employees. Last year,
$300,000 was distributed and the employee count was 800 in city A and 450 in
city B. Implementation of the ABC method took place this year to allocate IDC
on the basis of number of accidents. City A reported 325 events and city B had
142 accidents reported. Determine the following:

a. Cost driver for the ABC method


b. Allocation based on number of employees
c. Allocation based on number of accidents
d. The number of accidents per employee and its impact on the ABC allocation to
the cities

(a) Cost driver is number of accidents

(b) IDC rate = 300,000/(800+450)


= $240 per employee

City A: 800(240) = $192,000


City B: 450(240) = $108,000

(c) IDC rate = 300,000/(325+142)


= $642.40 per accident

City A: 325(642.40) = $208,780


City B: 142(642.40) = $91,220

(d) Accident rate:

City A: 325/800 = 0.41 accidents per employee


City B: 142/450 = 0.32 accidents per employee

Accident rate for city B is lower; therefore, the allocation is lower for B using
ABC compared to the traditional basis of number of employees.

Copyright 2021 © McGraw-Hill Education. All rights reserved. No reproduction or distribution without the
prior written consent of McGraw-Hill Education.
17
ADDITIONAL PROBLEMS AND FE EXAM REVIEW QUESTIONS

11.46 The most accurate measurement is the one that is:

a. Most repeatable
b. Most consistent
c. Most difficult to get
d. Closest to the right answer

Answer is (d)

11.47 A ceramic crucible weighed on an analytical balance yielded a result of 50.3214 g.


If the true weight is 50.4100 g, the percentage error is closest to:

a. −2.02% c. −0.18%
b. −1.09% d. −1.8%

Percentage error = [(50.3214/50.4100) – 1] × 100


= – 0.176%
Answer is (c)

11.48 Corrugated steel pipe with a diameter of 60 inches had a cost of $49.20 per foot in
Philadelphia, Pennsylvania, when the materials cost index (MCI) had a value of
2583.52. When the MCI had a value of 2150.00, the cost per foot of the 60-inch
pipe was closest to:

a. $40.94 c. $49.36
b. $45.94 d. $59.12

49.20 = C0(2583.52/2150.00)
C0 = $40.94 per foot
Answer is (a)

11.49 The 20-city Building Cost Skilled Labor Index has a value of 7458.80 now when
the skilled labor wage rate is $41.40 per hour. If the skilled labor wage rate was
$30.65 per hour 10 years ago, the skilled labor index value at that time was
closest to:

a. 10,074.86 c. 5522.03
b. 7436.12 d. 4027.83

41.40 = 30.65(7458.80/I0)
I0 = 5522.03
Answer is (c)

Copyright 2021 © McGraw-Hill Education. All rights reserved. No reproduction or distribution without the
prior written consent of McGraw-Hill Education.
18
11.50 The exponent in the cost-capacity equation for pumps is 0.64. If the cost for a 200
horsepower (hp) pump with controller was $22,000, the cost of a similar 500 hp
pump is closest to:

a. $12,240 c. $33,780
b. $28,650 d. $39,550

C2 = 22,000(500/200)0.64
= $39,546
Answer is (d)

11.51 If the cost of a high-speed assembly-line robot is $95,000 and the cost for one
with triple the capacity is $160,000, the value of the exponent in the cost-capacity
equation is closest to:

a. 0.41 c. 0.51
b. 0.47 d. 0.58

160,000 = 95,000(3)x
log 1.684 = x log 3
0.2263 = 0.4771x
x = 0.474
Answer is (b)

11.52 The equipment for applying specialty coatings that provide a high angle of skid
for the paperboard and corrugated box industries has a delivered cost of $390,000.
If the overall cost factor for the complete system is 2.96, the total plant cost
estimate is approximately:

a. $954,400 c. $1,154,000
b. $1,054,400 d. $1,544,400

CT = 2.96(390,000)
= $1,154,400
Answer is (c)

11.53 The total plant cost for manufacturing CO 2 warning devices that have a compact
gas detector and a signaling unit is $1,154,400. If the overall cost factor for the
plant was 2.61, the delivered-equipment cost was closest to:

a. $442,300 c. $593,700
b. $501,200 d. $613,900

1,154,400 = 2.61(CE)
CE = $442,299
Answer is (a)

Copyright 2021 © McGraw-Hill Education. All rights reserved. No reproduction or distribution without the
prior written consent of McGraw-Hill Education.
19
11.54 The delivered-equipment cost for setting up a production and assembly line for
high-sensitivity, gas-damped accelerometers is $650,000. If the direct cost and
indirect cost factors are 1.82 and 0.31, respectively, and both factors apply to
delivered- equipment cost, the total plant cost estimate is approximately:
a. $2,034,500 c. $1,384,500
b. $1,734,500 d. $1,183,000
CT = (1 + 1.82 + 0.31)(650,000)
= $2,034,500
Answer is (a)

11.55 If the processing cost decreases by a constant 4% every time the output doubles,
the slope parameter of the learning curve is closest to:
a. −0.009 c. 0.991
b. −0.059 d. −1.699
s = log 0.96/log 2
= –0.0177/0.30103
= –0.059
Answer is (b)

11.56 The first golf-cart chassis off a new fabrication line took 24.5 seconds to paint. If
during the first 100 units, a learning rate of 95% is assumed, the paint time for
unit 10 is closest to:

a. 20.7 seconds c. 2.75 seconds


b. 0.46 minutes d. 15.5 seconds
s = log 0.95/log 2 = –0.0222/0.30103 = –0.074
T10 = 24.5(10)–0.074 = 20.66 seconds
Answer is (a)

11.57 The IT department allocates indirect costs to user departments on the basis of
CPU time at the rate of $2000 per second. For the first quarter, the two heaviest
use departments logged 900 and 1300 seconds, respectively. If the IT indirect
budget for the year is $8.0 million, the percentage of this year’s allocation
consumed by these two departments is closest to:
a. 32%
b. 22.5%
c. 55%
d. Not enough information to determine
Allocation = (900 + 1300)(2000) = $4.4 million
Percent allocated = (4.4/8.0 million)(100) = 55%
Answer is (c)

Copyright 2021 © McGraw-Hill Education. All rights reserved. No reproduction or distribution without the
prior written consent of McGraw-Hill Education.
20
11.58 If engineering change order is the activity for an application of the ABC method
of indirect cost allocation, the most reasonable cost driver(s) may be:
1 – Number of changes processed
2 – Size of the workforce
3 – Management cost to process the change orders

a. 1 c. 3
b. 2 d. 1 and 3

Answer is (a)

11.59 If a hospital cafeteria is the activity to receive indirect cost allocation for the year,
cost driver(s) for the ABC method that seem reasonable may be:
1 – Number of cafeteria employees
2 – Number of meals
3 – Hospital patient volume

a. 1 c. 3
b. 2 d. 1 and 2

Answer is (d)

11.60 A blanket IDC rate is correctly applied when the following is/are present:
1 – The same basis is used for multiple cost centers
2 – An easy to calculate and use IDC rate is desirable
3 – Sensitivity to different functions and contributions is needed

a. 1 c. 3
b. 2 d. 1 and 2

Answer is (a)

Copyright 2021 © McGraw-Hill Education. All rights reserved. No reproduction or distribution without the
prior written consent of McGraw-Hill Education.
21
Solutions to end-of-chapter problems
Basics of Engineering Economy, 3rd edition
Leland Blank and Anthony Tarquin

Chapter 12
Depreciation Methods
12.1 Yvonne drives for Uber, Lyft, and other ride-sharing companies; plus she operates
an e-commerce franchise. Calculate her federal taxes on income of $150,000,
expenses of $45,000, and depreciation of $60,000 at a tax rate of 25%.

Taxes = (150,000 – 45,000 – 60,000)(0.25)


= $11,250

12.2 Puritan Cement Products placed a new sand sifter into production 3 years ago. It
had an installed cost of $100,000, a life of 5 years, and an anticipated salvage of
$20,000. Book depreciation charges for the 3 years are $40,000, $24,000, and
$14,000, respectively. Determine the book value after 3 years.

BV3 = 100,000 – (40,000 + 24,000 + 14,000)


= $22,000

12.3 Cyber Manufacturing is purchasing a complete video borescope system for


applications that require work in places that eyes cannot see. The purchase price is
$8000, shipping and delivery is $300, installation cost is $1200, tax recovery period
is 5 years, book depreciation period is 10 years, salvage value is estimated to be
$500, operating cost (with technician) will be $45,000 per year. For MACRS
depreciation of the system, what are the values of B, S, and n?

B = 8000 + 300 + 1200 = $9500


S = 0 (S is always 0 for MACRS)
n = 5 years

12.4 Cyber Source Computer Services paid P = $40,000 for its networked computer
system. Both tax and book depreciation accounts are maintained. The annual tax
depreciation rate is based on the previous year’s book value (BV), while the book
depreciation rate is based on the original first cost (P). Use the rates listed to (a)
determine the difference between the tax and book depreciation values in year 2,
and (b) plot the annual and book depreciation values (years 1 to 4) for each method
using a spreadsheet.

Copyright 2021 © McGraw-Hill Education. All rights reserved. No reproduction or distribution without the
prior written consent of McGraw-Hill Education.
1
Year of Ownership

Depreciation Rate 1 2 3 4
Tax, % of BV 40 40 40 40
Book, % of P 25 25 25 25

(a) Tax depreciation: D1 = 40,000(0.40)


= $16,000
BV1 = 40,000 – $16,000
= $24,000

D2 = 24,000(0.40)
= $9600
BV2 = 24,000 – 9,600
= $14,400

Book depreciation: D1 & D2 are both = 40,000(0.25)


= $10,000
BV2 = 40,000 – 10,000 – 10,000
= 20,000

Difference = 20,000 – 14,400 = $5600 higher for book depreciation

(b) Spreadsheet solution with graphs

12.5 Stahmann Products paid $350,000 for a numerical controller and had it installed at a
cost of $50,000. The recovery period was 7 years with an estimated salvage value
of 10% of the original purchase price. Stahmann sold the system 4 years after it was
purchased for $45,000. State the numerical values for the following: remaining life
at sale time, market value at sale time, book value at sale time if 65% of the basis
had been depreciated.

Copyright 2021 © McGraw-Hill Education. All rights reserved. No reproduction or distribution without the
prior written consent of McGraw-Hill Education.
2
Remaining life = 3 years
Market value = $45,000
Book Value = $400,000(1 – 0.65) = $140,000

12.6 KTT Chomerics manufactures transient voltage suppressors with high-power surge
capability. The company uses the straight line method for book depreciation
purposes. Newly acquired equipment has a first cost of $240,000 with a 3-year life
and a $30,000 salvage value. Determine the depreciation charge and book value for
year 2.

Depreciation is the same each year

D = (240,000 – 30,000)/3 = $70,000

BV2 = 240,000 – 2(70,000) = $100,000

12.7 Goodson Healthcare purchased a new sonogram imaging unit for $300,000 and a
truck body and chassis for an additional $100,000 to make the unit mobile. The
unit-truck system will be depreciated as one asset. The functional life is 8 years, and
the salvage is estimated to be 10% of the purchase price of the imaging unit
regardless of the number of years of service. Use classical straight line depreciation
to determine the salvage value, annual depreciation, and book value after 4 years of
service.

B = $400,000
S = 0.1(300,000) = $30,000

Depreciation is the same each year

D = (400,000 – 30,000)/8 = $46,250

BV4 = 400,000 – 4(46,250) = $215,000

12.8 Pneumatics Engineering purchased a machine that had a first cost of $40,000, an
expected useful life of 8 years, a recovery period of 10 years, and a salvage value of
$10,000. The operating cost of the machine is expected to be $15,000 per year. If
the inflation rate is 6% per year and the MARR is 11% per year, determine the
straight line (a) depreciation amount for year 3, and (b) book value for year 3.

(a) D3 = (40,000 – 10,000)/10 = $3000

(b) BV3 = 40,000 – 3(3000) = $31,000

Copyright 2021 © McGraw-Hill Education. All rights reserved. No reproduction or distribution without the
prior written consent of McGraw-Hill Education.
3
12.9 Your self-employed friend tells you that he is able to legally use depreciation write-
off on his income tax return for his car, since he uses it part time in his business
travel. He said the car is book depreciated over a 5-year period by the SL method
and after 3 years it has BV3 = $62,000 with an annual depreciation of $26,000.
Determine (a) the first cost of the car, and (b) what he assumed to be the trade-in
value after 5 years.

(a) D3 = 26,000
BV3 = 62,000 = P – 3(26,000)

P = 62,000 + 78,000
= $140,000

(b) 26,000 = (140,000 – S)/5


S = $10,000

12.10 The Lee Company of Westbrook, Connecticut, manufactures pressure relief


inserts for use where zero leakage is required. A machine purchased 3 years ago
has been book depreciated by the straight line method using a 5-year useful life.
The book value at the end of year 3 is $30,000, and the company assumed that the
machine would be worthless at the end of its 5-year useful life. (a) What is the
book depreciation charge each year, and (b) what was the basis for depreciation
purposes? (c) (Spreadsheet exercise) Use the SLN function to plot the book value
for 5 years if S = 0, as assumed above, and if S is 15% of the basis.

(a) BV5 = 0; determine SL depreciation for each of the last 2 years

Dt = 30,000/2
= $15,000 per year

(b) 15,000 = (B – 0)/5


B = $75,000

(c) Plots for BV with S = 0 (column C) and S = 0.15(75,000) = $11,250 (column E)


using = SLN(75000,S,5) for the depreciation amount

Copyright 2021 © McGraw-Hill Education. All rights reserved. No reproduction or distribution without the
prior written consent of McGraw-Hill Education.
4
12.11 An asset owned by Photon Environmental is book depreciated by the SL method
over a 5-year period with book values of $296,000 and $224,000 in years 2 and 3,
respectively. Determine (a) the salvage value used in the calculation, and (b) the
asset’s basis.

(a) D is the change in book values; then 2 years of depreciation reduces BV to S

D = 296,000 – 224,000
= $72,000 per year

S = 224,000 – 2(72,000) = $80,000

(b) (B – 80,000)/5 = 72,000


B = $440,000

12.12 You are curious about the original cost of the digital imaging equipment you use
at the First National Bank, where you work part time while in college. Accounting
cannot tell you the cost, but they know the annual depreciation over an 8-year
period is $18,900 per year. If all items are straight line depreciated and the
salvage is always 25% of the first cost, estimate the original cost.

D = 18,900
18,900 = (P – 0.25P)/8
0.75P = 8(18,900)
P = $201,600

12.13 A straight line depreciated asset has a basis of $200,000, a salvage value of
$10,000, and a recovery period of 7 years. Write two separate single-cell
spreadsheet functions: one to display the depreciation each year, and another to
display the book value for year 5.

Spreadsheet function = SLN(200000,10000,7) displays a D of $27,142.86

Function = 200000 – 5*SLN(200000,10000,7) displays a BV5 of $64,285.71

12.14 Equipment for immersion cooling of electronic components has an installed value
of $182,000 with an estimated trade-in value of $40,000 after 15 years. For years
2 and 10, use DDB book depreciation to determine the depreciation by (a)
equations, and (b) spreadsheet functions.

(a) Equation: Calculating to 3 decimals, d = 2/15 = 0.133

D2 = 0.133(182,000)(1 – 0.133)1
= $20,987

Copyright 2021 © McGraw-Hill Education. All rights reserved. No reproduction or distribution without the
prior written consent of McGraw-Hill Education.
5
D10 = 0.133(182,000)(1 – 0.133)9
= $6700

(b) Spreadsheet: = DDB(182000,40000,15,2,2) displays D2 = $21,031 (rounded)


= DDB(182000,40000,15,10,2) displays D10 = $6,694 (rounded)

12.15 An asset that has a first cost of $200,000 and an expected salvage value of 10% of
the first cost is book depreciated over a 10-year period. Determine the book value
in year 4 for (a) DDB, and (b) DB at 175% of the SL rate.

(a) d = 2/10 = 0.20

BV4 = 200,000(1 – 0.20)4


= $81,920

(b) d = 1.75/10 = 0.0.175

BV4 = 200,000(1 – 0.175)4


= $92,650

12.16 Determine the original basis of a machine that is used for making spill
containment pallets if its book value in year 3 is $25,000. The machine has a
5-year recovery period and the DDB method is applied.

d = 2/5 = 0.40

BV3 = 25,000 = B(1 – 0.4)3


B = $115,740

12.17 A video recording system was purchased 3 years ago at a cost of $30,000. A
5-year recovery period and DDB depreciation have been used to write off the
basis. The system is to be replaced this year with a trade-in value of $5000. What
is the difference between the book value and the trade-in value?

d = 2/5 = 0.40

BV3 = 30,000(1 – 0.40)3


= $6480

Difference = 6480 – 5000


= $1480

Trade-in value is lower than book value

Copyright 2021 © McGraw-Hill Education. All rights reserved. No reproduction or distribution without the
prior written consent of McGraw-Hill Education.
6
12.18 Checker LLC has purchased a $100,000 asset to be located in its Italian facility.
Either of two classical depreciation models is allowed: SL depreciation over
5 years with a salvage of $10,000 or DDB depreciation over 6 years with no
salvage considered. The asset is expected to be discarded after 5 years.
(a) (Spreadsheet exercise) Use the SLN and DDB functions to plot the book value
curves and determine which method has a lower BV after 5 years. (b) Use
equations to answer the same question.

(a) Spreadsheet shows graph; straight line is lower with BV5 = S = $10,000

(b) Equations: SL: BV5 = S = $10,000 (by definition)

DDB: d = 2/6 = 0.3333

BV5 = 100,000(1 – 0.3333)5


= $13,172

Straight line BV5 = S = $10,000 is lower than DDB BV5 = $13,172

12.19 An engineer with Accenture was asked by her client to help him understand the
difference between DDB and 150% DB depreciation. Answer his questions if B =
$180,000, n = 12 years, and S = $30,000.
a. What are the book values after 12 years for both methods?
b. How do the estimated salvage and the two implied book values after 12 years
compare in value?
c. Which of the two methods, when calculated correctly considering S =
$30,000, write off more of the first cost over the 12 years?

(a) DDB: d = 2/12 = 0.1667

Copyright 2021 © McGraw-Hill Education. All rights reserved. No reproduction or distribution without the
prior written consent of McGraw-Hill Education.
7
BV12 = B(1 – d)12 = 180,000(1 – 0.1667)12
= $20,188 (less than S = $30,000)

Copyright 2021 © McGraw-Hill Education. All rights reserved. No reproduction or distribution without the
prior written consent of McGraw-Hill Education.
8
Since BV12 < S, no depreciation is allowed beyond $30,000; for DDB

BV12 = $30,000

150% DB: d = 1.5(1/12) = 0.125

BV12 = 180,000(1 – 0.125)12


= $36,255

(b) S = $30,000 is between the two implied salvages of $20,188 and $36,255.

(c) DDB writes off more since all B – S = $150,000 is depreciated.


150% DB writes off less since it will stop at BV12 = $36,255

12.20 A company just purchased an intelligent robot, which has a first cost of $280,000.
Since the robot is unique in its capabilities, the company expects to be able to sell
it in 4 years for $200,000.
a. If M&O costs are $100,000 per year, determine the MACRS depreciation in
year 2. Assume the recovery period for robots is 5 years and the MARR is
12% when the inflation rate is 4% per year.
b. Determine the book value of the robot at the end of year 2.

(a) D2 = 280,000(0.32)
= $89,600

    (b) BV2 = 280,000 – 280,000(0.20 + 0.32)


= 280,000 – 145,600
= $134,400

12.21 An automated shuttle retriever system has a first cost of $350,000, a depreciable
life of 5 years, and an expected salvage value of 20% of the first cost. Using the
MACRS depreciation rates of 20.00, 32.00, 19.20, and 11.52% for years 1, 2, 3,
and 4, respectively, determine (a) the depreciation for year 3, and (b) the book
value at the end of year 4.

(a) D3 = 350,000(0.1920)
= $67,200

(b) BV4 = 350,000 – 350,000(0.20 + 0.32 + 0.192 + 0.1152)


= $60,480

12.22 In 2020, the Del Norte Brick Co. constructed a new bridge across the Rio Grande
River for improved access to the company’s property. The cost of the bridge was
$850,000, and it has a 15-year recovery period for MACRS depreciation.
Determine (a) the depreciation for year 5, and (b) the book value after 10 years.

Copyright 2021 © McGraw-Hill Education. All rights reserved. No reproduction or distribution without the
prior written consent of McGraw-Hill Education.
9
(a) From Table 12-2, d5 = 6.93%

D5 = 0.0693(850,000) = $58,905

(b) BV10 = B – total depreciation through year 10


= 850,000(1 – 0.050 – 0.095 – 0.0855 – 0.0770 – 0.0693 – 0.0623
– 0.0590 – 0.0590 – 0.0591 – 0.0590)
= $276,080

12.23 A plant manager for a large cable company knows that the remaining invested
value of quality assurance equipment is more closely approximated when the
equipment is depreciated linearly by the SL method compared to a rapid write-off
method like MACRS. Therefore, he keeps two sets of books, one for tax purposes
(MACRS) and one for equipment-management purposes (SL). For an asset that
has a first cost of $80,000, a depreciable life of 5 years, and a salvage value equal
to 25% of the first cost, determine the difference in the book values shown in the
two sets of books at the end of year 4.

Straight line: D = [80,000 – 0.25(80,000)]/5


= $12,000 per year

BV4 = 80,000 – 4(12,000) = $32,000

MACRS: BV4 = 80,000 – 80,000(0.20 + 0.320 + 0.1920 + 0.1152)


= 80,000 – 66,176
= $13,824

Difference = 32,000 – 13,824 = $18,176 higher for SL

12.24 Animatics Corp. of Santa Clara, California, makes small servo systems with built-
in controllers, amplifiers, and encoders so that they can control entire machines.
The company purchased an asset 2 years ago that has a 5-year recovery period.
Depreciation by the MACRS method for year 2 is $24,320.
a. What was the first cost of the asset?
b. How much was the depreciation in year 1?
c. (Spreadsheet exercise) Develop the complete MACRS depreciation and book
value schedule using the VDB function.

(a) MACRS rate is d2 = 0.32

B = 24,320/0.32 = $76,000

(b) MACRS rate is d1 = 0.20

D1 = 76,000(0.20) = $15,200

Copyright 2021 © McGraw-Hill Education. All rights reserved. No reproduction or distribution without the
prior written consent of McGraw-Hill Education.
10
(c) The function for D is = VDB(76000,0,5,MAX(0,t–1.5), MIN(5,t–0.5),2) where
t = 1, 2, …, 5.

12.25 A company report stated that a $140,000 asset purchased 3 years ago has a current
MACRS book value that is 57.6% of the asset’s basis. (a) Determine the recovery
period used. (b) Determine the depreciation next year using the VDB spreadsheet
function.

(a) Total depreciation for 3 years = 100% – 57.6% = 42.4%

Look in MACRS rate table for column that has total depreciation of 42.4% for
the first three years

Try n = 7: Sum = 14.29 +24.49 +17.49 = 56.27% (too high)


Try n = 10: Sum = 10.0 +18.0 +14.4 = 42.4% (correct)

Recovery period is n = 10 years

(b) Function for D4 is = VDB(140000,0,10,MAX(0,4–1.5),MIN(10,4–0.5),2)


Display is D4 = $16,128

12.26 Fairfield Properties owns real property that is MACRS depreciated with n = 39
years. They paid $3.4 million for the apartment complex and hope to sell it after
10 years of ownership for 50% more than the book value at that time. Determine
the anticipated profit, that is, the difference between the probable selling price and
the purchase price.

Use rates for real property with n = 39 years

D1 = 0.01391(3,400,000) = $47,294
D2 to D10 = 0.02564(3,400,000) = $87,176

Total depreciation for 10 years = 47,294 + 9(87,176) = $831,878

BV10 = 3,400,000 – 831,878 = $2,568,122

Anticipated selling price is 1.5(BV10) = $3,852,183

Profit = 3,852,183 – 3,400,000 = $452,183

Copyright 2021 © McGraw-Hill Education. All rights reserved. No reproduction or distribution without the
prior written consent of McGraw-Hill Education.
11
12.27 Three years ago, you purchased woodworking equipment for your hobby
activities and side-line business in cabinetry. The cost was $70,000, no salvage
was expected, and the estimated capacity was 10,000 operating hours. Determine
the annual depreciation and book value using the unit-of-production method if the
usage has been 3800, 2850, and 5510 hours per year.

Dt = (hours per year t/10,000)(70,000)

Number
of
Year, t hours Dt, $ BVt, $
1 3800 26,600 43,400
2 2850 19,950 23,450
3 5510 23,450* 0

*D3 = 5510/10,000(70,000) = $38,570 is too large; only the remaining


BV of $23,450 can be depreciated in year 3.

12.28 (Spreadsheet exercise) Pinnacle Consultants purchased a new Toyota hybrid car
for employees use for $35,000 with the expectation to retain it for 5 years or
100,000 miles. No salvage is considered for depreciation purposes. The question
is which book depreciation method to use: DDB for 5 years or UOP for 100,000
miles. The miles driven each year for the last automobile are listed. Use a
spreadsheet to plot the book values for both methods and determine which method
may remove the $35,000 faster.

Year 1 2 3 4 5
Miles, ×1000 15 22 16 18 25

Plots show that DDB does decrease BV faster, but UOP removed slightly more of
the first cost ($33,600 vs $32,278).

Copyright 2021 © McGraw-Hill Education. All rights reserved. No reproduction or distribution without the
prior written consent of McGraw-Hill Education.
12
12.29 Sensing equipment to measure carbon monoxide levels across the refinery cost
$160,000. A salvage of $15,000 after its 10-year useful life is estimated.
Determine the depreciation amounts for years 2 and 7 for the SYD method using
(a) equations, and (b) spreadsheet functions.

(a) Equations: B = $160,000; n = 10; S = $15,000, and SUM = 55

D2 = 10 – 2 + 1 (160,000 – 15,000) = $23,727


55

D7 = 10 – 7 + 1 (160,000 – 15,000) = $10,545


55

(b) Functions:

D2: = SYD(160000,15000,10,2) displays $23,737

D7: = SYD(160000,15000,10,7) displays $10,545

12.30 (Spreadsheet exercise) If B = $400,000, n = 8 years, S = 10% of B, and MARR =


12% per year, use the depreciation and book value schedule for the SYD method
to determine the following:
a. Percent of the basis removed in the first 50% of useful life.
b. Depreciation rate for each year 1 through 8.
c. Comparison of book values after 10 years for SYD, SL, and MACRS
methods.

The schedule is shown in the spreadsheet using the SYD function.

(a) Depreciation total for 4 years is $260,000, which represents

(260/400) *100 = 65% of B

Copyright 2021 © McGraw-Hill Education. All rights reserved. No reproduction or distribution without the
prior written consent of McGraw-Hill Education.
13
(b) Annual depreciation rates (column E) are calculated using the function
= (8 – year + 1)/SUM * 100, where SUM = 8(9)/2.

(c) SYD: BV10 = S = $40,000 (by equation per the method)


SL: BV10 = S = $40,000 (by equation per the method)
MACRS: BV10 = 0 (required by method)

12.31 For a country that allows switching between declining balance (not DDB) and
straight line, determine if a switch is advisable in year 2. The asset has a first cost
of $100,000, a 5-year recovery period, and a $10,000 salvage value.

DB method: d = 1/5 = 0.20

D2 = 0.20(100,000)(1 – 0.20)1 = $16,000

SL method: Need BV1 from DB method with d = 0.20

BV1 = 100,000(1 – 0.2)1 = $80,000

New SL depreciation

D2 = (80,000 – 10,000)/4 = $17,500

SL has larger depreciation by 17,500 – 16,000 = $1500; switch to SL is advisable

12.32 Henry has an assignment from his boss at Czech Glass and Wood Sculpting to
evaluate depreciation methods for writing off the $200,000 first cost of a newly
acquired Trotec CO2 laser system for engraving and cutting. Productive life is
8 years and salvage is estimated at $10,000. Henry wants to compare the PW of
depreciation at i = 10% per year for DDB-to-SL switching with MACRS for n = 7
years to determine which is the preferred method. Perform the analysis using
tabulation or a spreadsheet, as requested by your instructor.

Tabulation: Use the switching procedure for DDB-to-SL. DDB d = 0.25. Two
example computations for switching follow.

Year 2: DDB depreciation is 0.25(150,000) = $37,500 (selected)


SL depreciation is (150,000 – 10,000)/7 = $20,000

Year 6: DDB depreciation is 0.25(47,461) = $11,865


SL depreciation is (47,461 – 10,000)/3 = $12,487 (selected)

Switch in year 6

Copyright 2021 © McGraw-Hill Education. All rights reserved. No reproduction or distribution without the
prior written consent of McGraw-Hill Education.
14
DDB-to-SL switch MACRS
Year DDB SL Selecte BV Rate Depr
d
0 200,000
1 50,000 23,750 50,000 150,000 0.1429 28,580
2 37,500 20,000 37,500 112,500 0.2449 48,980
3 28,125 17,083 28,125 84,375 0.1749 34,980
4 21,094 14,875 21,094 63,281 0.1249 24,980
5 15,820 13,320 15,820 47,461 0.0893 17,860
6 11,865 12,487 12,487 34,974 0.0892 17,840
7 12,487 12,487 22,487 0.0893 17,860
8 12,487 12,487 10,000 0.0446 8,920

Compare PW of selected and MACRS depreciation values. MACRS has the


larger present worth; MACRS is preferred.

DDB-to-SL: PWD = 50,000(P/F,10%,1) + … + 12,487(P/F,10%,8)


= $141,089

MACRS: PWD = 28,580(P/F,10%,1) + … + 8,920(P/F,10%,8)


= $144,283

Spreadsheet: Apply the VDB function; MACRS is preferred with the larger PW.

12.33 ConocoPhillips alkylation processes are licensed to produce high-octane, low-


sulphur blend stocks domestically and internationally. Halliburton Industries has
newly licensed alkylation equipment costing $1 million per system at its Moscow,
Houston, and Abu Dhabi refinery service operations. Russia requires a 10-year,
straight line recovery with a 10% salvage value. The United States allows a 7-year
MACRS recovery with no salvage considered. The United Arab Emirates allows a
7-year recovery with switching from DDB to SL method and no salvage

Copyright 2021 © McGraw-Hill Education. All rights reserved. No reproduction or distribution without the
prior written consent of McGraw-Hill Education.
15
considered. Which of the country’s methods has the largest PW of depreciation at
i = 15% per year?

Copyright 2021 © McGraw-Hill Education. All rights reserved. No reproduction or distribution without the
prior written consent of McGraw-Hill Education.
16
This problem is best worked using a spreadsheet. Hand solution is relatively
easy, but quite time consuming.

SL: Dt = 900,000/10 = $90,000 per year


MACRS: Rates from n = 7 table
DDB-to-SL switch: Use VDB to switch in year 5 of a 7-year recovery

Switching to SL from DDB (in the UAE) offers the largest PWD value.

12.34 A coal mine purchased 3 years ago for $7 million was estimated to contain
4,000,000 tons of coal. During the past 3 years the tonnage of coal removed was
21,000, 18,000, and 20,000 tons, respectively. The gross income obtained in these
3 years was $257,000 for the first year, $320,000 for the second year, and $340,000
for the third year. Determine (a) the cost depletion allowance for year 2, and
(b) the percentage of the purchase price removed from the books through year 3.

pt = 7,000,000/4,000,000
= $1.75 per ton

(a) Cost Depletion, Year 2 = 1.75(18,000)


= $31,500

(b) Total depletion is sum of depletion for 3 years

Cost Allowance: Year 1: 1.75(21,000) = 36,750


Year 2: 1.75(18,000) = 31,500
Year 3: 1.75(20,000) = 35,000
Total = $103,250

% depletion in 3 years = 103,250/7,000,000


= 0.015 (1.5%)

Copyright 2021 © McGraw-Hill Education. All rights reserved. No reproduction or distribution without the
prior written consent of McGraw-Hill Education.
17
12.35 Four years ago, International Uranium Mines paid $350 million for rights to
remove uranium ore (pitchblende) for refining into U3O8 (triuranium octaoxide)
for use in nuclear reactor fuels. Of the estimated 6.5 million pounds of ore
available, the yearly amounts removed in pounds were 275,000; 250,000;
320,000; and 425,000. Uranium prices per pound over the 4 years were $70, $69,
$73, and $75, respectively. For an annual percentage depletion rate of 22%, (a)
what was the total depletion amount over the 4-year period, and (b) what
percentage of the original investment was depleted in the first 4 years?

(a) Annual percentage depletion is 22% of gross income (GI).

Price,
Year Pounds $ per pound GI, $ Depletion, $
1 275,000 70 19,250,000 4,235,000
2 250,000 69 17,250,000 3,795,000
3 320,000 73 23,360,000 5,139,200
4 425,000 75 31,875,000 7,012,500
Tota 20,181,700
l

(b) Percent depleted: (20,181,700/350,000,000)(100) = 5.77%

12.36 For the last 10 years, Am-Mex Coal has used the cost depletion factor of $2500
per 100 tons to write off the investment of $35 million in its Pennsylvania
anthracite coal mine. Depletion thus far totals $24.8 million. A new study to
appraise mine reserves indicates that no more than 800,000 tons of salable coal
remains. If estimated gross income is expected to be $8.50 million on a
production level of 72,000 tons, determine next year’s (a) cost, and (b) percentage
depletion amounts. The percentage depletion allowance is 10%.

Remaining investment = 35.0 – 24.8 million


= $10.2 million

(a) Cost depletion: pt = 10,200,000 /800,000 = $12.75 per ton


Allowance = 72,000(12.75)
= $918,000

(b) Percentage depletion: Rate is 10% of GI


GI is $8,500,000
Allowance = 0.10(8,500,000)
= $850,000

12.37 NA Forest Resources purchased forest acreage for $500,000 from which an
estimated 200 million board feet of lumber are recoverable. The company will sell
the lumber for $0.10 per board foot. No lumber will be sold for the next 2 years

Copyright 2021 © McGraw-Hill Education. All rights reserved. No reproduction or distribution without the
prior written consent of McGraw-Hill Education.
18
because an environmental impact statement must be completed before harvesting
can begin. In years 3 to 10, however, the company expects to remove 20 million
board feet per year. The company’s taxable income on timber harvesting has
averaged $170,000 per year. Applying the cost depletion method, determine the
depletion (a) for year 2, (b) for year 5, and (c) total for all 10 years.

(a) There is no depletion deduction in year 2 because no timber will be


harvested until year 3.

(b) pt = 500,000/200
= $2500 per million board feet

Depletion, year 5 = 2500(20) = $50,000

(c) Ten-year total = 8(50,000) = $400,000, since there is no depletion for years
and 2.

12.38 Carrolton Oil and Gas, an independent oil and gas producer, is approved to use a
20% of gross income depletion allowance. The write-off last year was $700,000
on its horizontal directional drill wells. Determine the estimated total reserves in
barrels if the volume pumped last year amounted to 1% of the total and the
delivered-product price averaged $75 per barrel.

Percentage depletion = 0.20(GI) = 700,000


GI = $3,500,000

Let N = number of barrels

GI = N × price
3,500,000 = N × 75
N = 46,667

Reserves = 46,667/0.01
= 4,666,700 barrels

12.39 Hardwick Mines operates a stone and gravel quarry. During the first 2 years the
tonnage extracted each year was 60,000 and 65,000 tons. The quarry, estimated to
contain a total of 2.5 million tons of usable material, had an initial cost of
$3.2 million. The company reported a per-ton gross income (GI) of $35 and
$40 for the 2 years.
a. Determine the depletion charge each year, using the larger of the values for
the two depletion methods. Assume all depletion amounts are less than 50%
of taxable income.
b. If the quarry reserve has been reevaluated and is estimated now to contain a
remaining total of 1.5 million tons, determine the new cost depletion rate.

Copyright 2021 © McGraw-Hill Education. All rights reserved. No reproduction or distribution without the
prior written consent of McGraw-Hill Education.
19
(a) Cost depletion: pt = $3.2/2.5 million = $1.28 per ton

Percentage depletion: 5% of gross income

Tonnage GI
for Cost Per-ton for Percentage
Year Depletion GI, $ Depletion, $___
1 60,000 35 2,100,000
2 65,000 40 2,600,000

Cost Depletion Percentage


Year $1.28×tons Depletion, 5%×GI Selected
1 76,800 105,000 Percentage
2 83,200 130,000 Percentage

(b) Remaining investment = 3,200,000 – (105,000 + 130,000) = $2,965,000

New pt = $2.965 million/1.5 million tons = $1.98 per ton

12.40 Standard straight line depreciation of a $100,000 asset takes place over a 7-year
recovery period. If the salvage value is 20% of first cost, the book value at the end
of 3 years is closest to:
a. $57,140
b. $65,715
c. $11,430
d. $80,000

BV3 = 100,000 – 3(80,000/7) = $65,714


Answer is (b)

12.41 If SL and DDB depreciation rates for 5-year recovery periods are calculated, the
rates (in percent) for the second year are:
a. 20% and 40%
b. 40% and 24%
c. 20% and 24%
d. 20% and 16%

SL: d2 = 20%
DDB: d = 2/5 = 0.40; d2 = 40%(1 – 40%)1 = 24%
Answer is (c)

Copyright 2021 © McGraw-Hill Education. All rights reserved. No reproduction or distribution without the
prior written consent of McGraw-Hill Education.
20
12.42 All of the following types of real property are depreciable, except:
a. Warehouses
b. Land
c. Office buildings
d. Test facilities

Answer is (b)

12.43 A machine with a 5-year life has a first cost of $20,000, a $2000 salvage value,
and an AOC of $8000 per year. According to the classical straight line method,
the depreciation in year 2 is nearest to:
a. $2800
b. $3600
c. $4500
d. $5300

D = (20,000 – 2000)/5
= $3600 per year
Answer is (b)

12.44 A machine with a 10-year life has a basis of $40,000, a $5000 salvage value, and
an AOC of $7000 per year. If the MACRS depreciation rates for years 1, 2, and 3
are 10.00%, 18.00%, and 14.40%, respectively, the depreciation for year 3 is
closest to:
a. $5800
b. $7200
c. $8500
d. $9300

D3 = 40,000(0.144)
= $5760
Answer is (a)

12.45 The depreciation in year 4 for a 5-year DDB-depreciated asset is $3000. The first
cost of the asset is closest to:
a. $8,000
b. $25,000
c. $35,000
d. $120,000

d = 2/5 = 0.4

3000 = 0.4(B)(1 – 0.4)3


B = $34,722
Answer is (c)

Copyright 2021 © McGraw-Hill Education. All rights reserved. No reproduction or distribution without the
prior written consent of McGraw-Hill Education.
21
12.46 An automated assembly robot that cost $400,000 has a depreciable life of 5 years
with a $100,000 salvage value. If the MACRS depreciation rates for years 1, 2,
and 3 are 20%, 32%, and 19.2%, respectively, the book value of the robot at the
end of year 3 is nearest to:
a. $102,000
b. $115,000
c. $128,000
d. $174,000

BV3 = 400,000 – 400,000(0.20 + 0.32 + 0.192)


= $115,200
Answer is (b)

12.47 A delivery truck with a basis of $155,000 is depreciated using the unit-of-
production method with an estimated life of 200,000 miles. Recorded miles for
the first 2 years are 31,100 and 48,450 miles. The total depreciation for the
2 years is closest to:
a. $61,650
b. $75,100
c. $102,650
d. $190,500

UOP depreciation rate = 155,000/200,000 = $0.775 per mile

D1 + D2 = 0.775(31,100 + 48,450)
= $61,651
Answer is (a)

12.48 A coal mine purchased for $5 million has enough coal to operate for 10 years. The
AOC is estimated at $200,000 per year. The coal is expected to sell for $150 per
ton, with annual production expected to be 10,000 tons. Coal has a depletion
percentage rate of 10%. The percentage depletion for year 6 is closest to:
a. $75,000
b. $100,000
c. $125,000
d. $150,000

Percentage depletion = 10,000(150)(0.10)


= $150,000
Answer is (d)

Copyright 2021 © McGraw-Hill Education. All rights reserved. No reproduction or distribution without the
prior written consent of McGraw-Hill Education.
22
12.49 Rayonier Forrest Resources purchased a tract of land for $700,000. It contains a
total of 25,000 harvestable timber trees. The value of the land, over and above the
value of the timber, is estimated to be $200,000. In the first year of operation,
Rayonier harvested 7000 trees. The cost depletion for year 1 is closest to:
a. $20,000
b. $70,000
c. $100,000
d. $140,000

pt = (700,000 – 200,000)/25,000 = $20.00/tree

Cost depletion = 20.00(7000) = $140,000


Answer is (d)

12.50 A construction stone quarry in central Texas can use a percentage depletion rate
of 5% of gross income or a cost depletion rate of $1.28 per ton. The quarry’s first
cost was $3.2 million; estimated total tonnage is 2.5 million tons; tonnage this
year is 65,000; gross income is $40 per ton. The cost depletion method amount is
closest to:
a. $130,000
b. $105,500
c. $94,600
d. $83,200

pt = $1.28 per ton

Cost depletion, year 1 = 65,000(1.28)


= $83,200
Answer is (d)

Copyright 2021 © McGraw-Hill Education. All rights reserved. No reproduction or distribution without the
prior written consent of McGraw-Hill Education.
23
Solutions to end-of-chapter problems
Basics of Engineering Economy, 3rd edition
Leland Blank and Anthony Tarquin

Chapter 13
After-Tax Economic Analysis
13.1 Divisions of Doubleday Computers and Merritt-Douglas Computing make
competing products for the commercial IT market worldwide. Use the data for each
company to determine (a) the effective total tax rate Te for Doubleday, and (b) the
taxable income (in millions) for Merritt-Douglas.

Doubleday M-D
Sales, $ 2.8 million 4.7 million
Other revenue, $ 900,000 250,000
Expenses, $ 1.4 million 3.1 million
Depreciation, $ 850,000 970,000
Total of state and local tax rate, % 9.2 7.5
Average federal tax rate, % 21 21

(a) Te = 9.2 + (1–0.092)(21)


= 28.27%

(b) In millions units: TI = 4.7 + 0.25 – 3.1 – 0.97


= $0.88 ($880,000)

13.2 In 2018, its first year of operation, Borsberry Construction had a federal tax bill of
$84,000. What was the company’s taxable income?

Taxes = TI(T)
84,000 = TI(0.21)
TI = $400,000

13.3 In 2020, Carl is thinking of purchasing stock in Harrison Engineering and 3-D
Imaging. From the annual report, he deduced that GI = $4.9 million, E = $2.1
million, and D = $1.4 million. If the average federal tax rate is 21% and state/local
tax rates total 9.8%, estimate (a) federal income taxes, and (b) the percent of GI that
the federal government takes in income taxes. (c) If Carl had performed the same
analysis in 2017 and the data were the same, what answers would he have gotten?

Copyright 2021 © McGraw-Hill Education. All rights reserved. No reproduction or distribution without the
prior written consent of McGraw-Hill Education.
1
Copyright 2021 © McGraw-Hill Education. All rights reserved. No reproduction or distribution without the
prior written consent of McGraw-Hill Education.
2
(a) Te = 9.8 + (1 – 0.098)(21%) = 28.74%

TI = 4.9 – 2.1 – 1.4 = $1.4 million

Tax estimate = 1,400,000(0.2874) = $402,360

(b) Percent of GI: 402,360/4,900,000 = 0.0821 (8.21%)

(c) Use 9.8% of TI for state and local, which are deductible for federal tax
computation; use Table 13.1 rates for federal taxes; use

State and local taxes = 0.098(1,400,000)


= $137,200

Federal TI basis = 1,400,000 – 137,200


= $1,262,800

Federal taxes = 113,900 + 0.34(1,262,800–335,000)


= $429,352

Total taxes = 137,200 + 429,352


= $566,552

Percent of GI: 566,552/4,900,000 = 0.1156 (11.56%)

13.4 Last year, an investor in rental property had gross income of $160,000 with the
following expenses: maintenance $22,000, insurance $5000, management $10,000,
utilities $16,000, and debt service (interest) $19,000. Income taxes totaled $8000.
What was the net operating income for the year?

Interest and taxes are not included in NOI.

NOI = 160,000 – 22,000 – 5000 – 10,000 – 16,000


= $107,000

13.5 Helical Products makes machined springs with elastic redundant elements so that a
broken spring will continue to function. The company has gross income of
$450,000 with expenses of $260,000 and depreciation of $58,000. (a) Approximate
the company’s total taxes for an effective tax rate of 26%. (b) What was the state
tax rate if Helical paid a flat 21% federal tax rate on TI?

(a) TI = 450,000 – 260,000 – 58,000


= $132,000

Approximate taxes = 132,000(0.26)


= $34,320

Copyright 2021 © McGraw-Hill Education. All rights reserved. No reproduction or distribution without the
prior written consent of McGraw-Hill Education.
3
(b) 0.26 = ts + (1 – ts)(0.21)
0.05 = (1 – 0.21)ts
ts = 0.633 (6.33%)

13.6 Last year, Marylynn opened Baron’s Appliance Sales and Service. Her tax
accountant provided the year’s results:

Gross income = $320,000


Business expenses = $149,000
MACRS depreciation = $95,000
Average federal tax rate = 21%
Average state tax rate = 6%
City and county flat tax rates combined = 4.5%

Determine the following for Marylynn:


a. Taxable income.
b. Estimate the federal income taxes without considering state and lower level
taxes.
c. Using an effective tax rate, estimate the percent of the GI needed to pay all
income taxes: federal, state, city, and county.

(a) TI = 320,000 – 149,000 – 95,000 = $76,000

(b) Federal tax rate = 21%

Taxes = (76,000)(0.21) = $15,960

(c) Te = 10.5 + (1 – 0.105)(21) = 29.3%

Tax estimate = 76,000(0.293) = $22,268

Percent of GI = (22,268/320,000)(100) = 6.96%

13.7 Sameh, a U.S. citizen and an engineer living outside the country while working for
GE International, Inc., files his U.S. income tax return as “Unmarried.” This last
year, his salary was $103,000 and his retirement investment portfolio provided net
supplemental dividends of $10,500. (a) Use Table 13.2 rates to determine his
federal taxes, assuming no state or other income taxes. Only a $12,000 standard
deduction can be taken in filing his U.S. tax return. (b) If all investment dividends
are directly reinvested (meaning he receives no actual cash proceeds, but must pay
taxes on these earnings), Sameh receives only his salary. What percent of his salary
is taken in paying U.S. federal taxes, though he lives outside the country?

(a) TI = GI – standard deduction


= 103,000 + 10,500 – 12,000
= $101,500

Copyright 2021 © McGraw-Hill Education. All rights reserved. No reproduction or distribution without the
prior written consent of McGraw-Hill Education.
4
Taxes = 14,089.50 + (0.24)(101,500 – 82,500)
= $18,649.50

(b) Percentage of salary = (18,649.50/103,000)(100) = 18.11%

13.8 Last year Jennifer was single; this year she is married to Manish. Salaries,
deductions, and other information are listed for the two years. Determine the federal
income taxes for each when they file as “Single” and now as “Married filing
jointly” using Table 13.2 rates. Is there a financial benefit or penalty for being
married and filing as such? What is the size of the benefit or penalty?

Jennifer, Manish, Jennifer/Manish,


Last Year Last Year This Year
Salary, $ 75,000 75,000 150,000
Other income, $   5,000   5,000 10,000
Standard deduction, $ 12,000 12,000 24,000
Filing status Single Single Married filing jointly

Single: Use tax rates for filing ‘Single’ in Table 13-2(a).

TI = 75,000 + 5,000 – 12,000 = $68,000


Taxes = 4,453.50 + 0.22(68,000 – 38,700)
= $10,899.50

Total taxes are 2(10,899.50) = $21,799

Married: Use tax rates for ‘Married, filing jointly’ in Table 13-2(b).

TI = 150,000 + 10,000 – 24,000 = $136,000


Taxes = 8,907 + 0.22(136,000 – 77,400)
= $21,799

Difference is $0. There is no marriage penalty at this level of income.

13.9 It is a known fact that the U.S. graduated tax rates require people with higher
income levels to pay more federal taxes. A common question: Just how much more
of total income and TI does it take? Consider the salary, investment dividend, and
other income data shown for two families; both file as ‘Married filing jointly’. The
standard deduction per family is $24,000 and there is no personal exemption
allowed as of the year 2018 per current tax law. For each family, determine the
percent of (a) total family income, and (b) taxable income consumed by federal
taxes. Solve by spreadsheet or equation, as instructed.

Copyright 2021 © McGraw-Hill Education. All rights reserved. No reproduction or distribution without the
prior written consent of McGraw-Hill Education.
5
Family Carrolton Thompson
Salaries, $1000 65 290
Dividends, $1000 12 58
Other income 0 14

Spreadsheet analysis indicates that the percentage of total income increases from
7.76% to 19.76%, about 2.5X more, for the larger income family. Similarly, the
larger TI family has about 2X more of TI consumed by federal taxes.

13.10 Estimate the approximate after-tax rate of return for a project that has a before-tax
ROR of 18%. Assume the company’s effective tax rate is 27% and it uses
MACRS depreciation for an asset that has a $40,000 salvage value.

After-tax ROR = 18(1– 0.27) = 13.1%

13.11 Approximate the after-tax ROR on a project that had a first cost of $500,000, a
salvage value of 20% of the first cost after five years and annual CFBT of
$230,000. Assume the company had a 25% effective tax rate.

Using the RATE function,

Before-tax ROR: 0 = –500,000 + 230,000(P/A,i*,5) + 100,000(P/F,i*,5)


i* = 38.48% (spreadsheet)

After-tax ROR = 38.48(1 – 0.25)


= 28.86%

13.12 Estimate the CFAT for a company that has taxable income of $120,000,
depreciation of $133,350, and an effective tax rate of 25% per year.

Copyright 2021 © McGraw-Hill Education. All rights reserved. No reproduction or distribution without the
prior written consent of McGraw-Hill Education.
6
Use the following relations:

TI = GI – E – D
CFBT = GI – E = TI + D

Taxes = TI(Te) = 120,000(0.25) = $30,000

CFAT = CFBT – taxes


= TI + D – taxes
= 120,000 + 133,350 – 30,000
= $223,350

The information in the right column is used in problems 13.13 through 13.17.

After 4 years of use, the Henkel Corporation has decided to replace some of the capital
equipment used on its Dial bath soap line. Cash flow data for the 4 years are tabulated in
$1000 units. After-tax MARR was 10% per year, Te was 25%, and MACRS 3-year
depreciation was used.

Year 0 1 2 3 4
Purchase, $ −1900
Gross income, $  800    950  600  300
Expenses, $ −100 −150 −200 −250
Salvage, $   700

13.13 Use the CFBT and PW value to determine if the equipment investment exceeded
the MARR.

Estimate before-tax MARR, Tabulate CFBT, and calculate PW; Monetary values
in $1000 units.

Before-tax MARR = 10%/(1–0.25) = 13.3%

Yea GI, $ E, $ P and S, $ CFBT, $


r
0 –1900 –1900
1 800 –100 700
2 950 –150 800
3 600 –200 400
4 300 –250
700 750
PW = – 1900 +
700(P/F,13.3%,1) + … + 750(P/F,13.3%,4)
= –1900 + 700(0.88261) + 800(0.77900) + 400(0.68756) + 750(0.60685)

Copyright 2021 © McGraw-Hill Education. All rights reserved. No reproduction or distribution without the
prior written consent of McGraw-Hill Education.
7
= $71.19 ($71,190)

Copyright 2021 © McGraw-Hill Education. All rights reserved. No reproduction or distribution without the
prior written consent of McGraw-Hill Education.
8
(Note: PW = $71,193 using NPV function at 13.3%)

Equipment is justified before taxes are considered.

13.14 Calculate MACRS depreciation and estimate the CFAT series over the 4 years.
Remember to include the salvage in year 4 as income that increases taxes.
Develop CFAT using (a) tabulation, and (b) a spreadsheet.

(a) Tabulation: Determine MACRS depreciation, taxes and CFAT. Assume negative

tax will increase CFAT and PW. Monetary values are in $1000 units.

TI = GI – E – D
CFAT = CFBT – taxes
Year GI E P and S CFBT Rate D TI Taxes CFAT
0 $–1900 $–1900 $–1900
1 $800 $–100 700 33.33% $633 $ 67 $17 683
2 950 –150 800 44.45 845 –45 –11 811
3 600 –200 400 14.81 281 119 30 370
4 300 –250 700 750 7.41 141 609 152 598

(b) Spreadsheet: CFAT values (column F, rows 2-6) are more accurate than
tabulation. CFBT values taken from tabulation. Salvage of
$700,000 increases TI in year 4 from $50,000 to $750,000.

13.15 Utilize the CFAT and PW value to determine if the investment was justified with
taxes considered.

Determine PW of CFAT at 10%.

PW = –1900 + 683(P/F,10%,1) + … + 598(P/F,10%,4)


= –1900 + 683(0.9091) + 811(0.8264) + 370(0.7513) + 598(0.6830)
= $77.54 ($77,540)

(Note: PW = $78,042 using NPV function at 10%.)

Equipment is also justified when taxes are considered.

Copyright 2021 © McGraw-Hill Education. All rights reserved. No reproduction or distribution without the
prior written consent of McGraw-Hill Education.
9
13.16 Compare the after-tax ROR values using both methods—approximated from the
CFBT values using the before-tax ROR and Te, and directly from the CFAT series.

CFBT approximation: Use PW relation on tabulated CFBT series to get before


tax i*

0 = –1900 + 700(P/F,i*,1) + 800(P/F,i*,2) + 400(P/F,i*,3) + 750(P/F,i*,4)

Before-tax i* = 15.14% (using IRR function)

After-tax estimated ROR = 15.14(1– 0.25)


= 11.36%

CFAT series ROR: Use PW relation on tabulated CFAT series to get after-tax i*

0 = –1900 + 683(P/F,i*,1) + 811(P/F,i*,2) + 370(P/F,i*,3) + 598(P/F,i*,4)

After-tax i* = 12.01% (using IRR function)

This is slightly higher than the 11.36% approximated from the CFBT values.

13.17 (Spreadsheet exercise) Develop a spreadsheet that includes cash flow, PW, and
ROR values for both before-tax and after-tax analyses. Was the equipment
purchase justified in both analyses? (Note: If you worked 13.14(b), the CFAT
series is already developed.)

Spreadsheet displays PW and i* values. Purchase is justified before and after-taxes


are considered.

Before-taxes: MARR = 13.3%; PW @ 13.3% = $71,193; i* = 15.14%

After-taxes: MARR = 10%; PW @ 10% = $78,042; i* = 12.01%

Copyright 2021 © McGraw-Hill Education. All rights reserved. No reproduction or distribution without the
prior written consent of McGraw-Hill Education.
10
13.18 Fill in the missing values in the table below for the CFBT, D, TI, Taxes, and
CFAT columns. Depreciation amounts are based on 3-year MACRS depreciation
and the effective tax rate is 25%.

P and S,
Year GI, $ E, $ CFBT, $ D, $ TI, $ Taxes, $ CFAT, $
$
0 −1900 −1900 −1900
1 800 −100   0  700 633   67 17   683
2 950 −150   0 ? ? −45 ?      811
3 600 −200   0  400 281 ? 30 ?
4 300 −250   0    50 ? −91 −23     73

Missing values are shown in bold red.

CFBT2 = 950 – 150 = $800


D2 = 0.4445(1900) = $845
D4 = 0.0741(1900) = $141
TI3 = 600 – 200 – 281 = $119
Taxes2 = –45(0.25) = $–11
CFAT3 = 400 – 30 = $370

Year GI, $ E, $ P and S, $ CFBT, $ D, $ TI, $ Taxes, CFAT, $


$
0 –1900 –1900 –1900
1 800 –100 0 700 633 67 17 683
2 950 –150 0 800 845 –45 –11 811
3 600 –200 0 400 281 119 30 370
4 300 –250 0 50 141 –91 –23 73

13.19 Estimate the gross income for Bling Enterprises, which reports a CFAT of $2.5
million, $900,000 in expenses, $900,000 in depreciation charges, and has an
effective tax rate of 26.4%.

Solve relations for GI

CFBT = CFAT + taxes


GI – E = CFAT + (GI – E – D)(Te)

GI = [CFAT + E(1– Te) – DTe]/ (1– Te)


= [2,500,000 + 900,000(0.736) – 900,000(0.264)]/0.736
= $3,973,913

Copyright 2021 © McGraw-Hill Education. All rights reserved. No reproduction or distribution without the
prior written consent of McGraw-Hill Education.
11
13.20 Advanced Anatomists, Inc., researchers in medical science, is contemplating a
commercial venture concentrating on proteins based on the new X-ray technology
of free-electron lasers. To recover the huge investment needed, an annual
$2.5 million CFAT is needed. The federal tax rate is 21%. However, state taxing
authorities will levy an 8% tax on TI. Over a 3-year period, the deductible
expenses and depreciation are estimated to total $1.3 million the first year,
increasing by $500,000 per year thereafter. Of this, 50% is expenses and 50% is
depreciation. Help the CFO estimate the required gross income for each year.

GI can be determined from the data provided.

CFBT = CFAT + taxes


GI – E = CFBT = CFAT + (GI – E – D)(Te)

Solve for GI to obtain a general relation for each year t.

GIt = [CFAT + Et(1–Te) – DtTe]/ (1–Te)

where CFAT = $2.5 million


Te = 8% + (1–0.08)(21%) = 27.3% (0.273)

1– Te = 72.7% (0.727)

Year 1: D1 = E1 = 1,300,000/2 = $650,000


GI1 = [2,500,000 + 650,000(0.727) – 650,000(0.273)]/0.727
= $3,844,704

Year 2: D2 = E2 = 1,800,000/2 = $900,000


GI2 = [2,500,000 + 900,000(0.727) – 900,000(0.273)]/0.727
= $4,000,825

Year 3: D3 = E3 = 2,300,000/2 = $1,150,000


GI3 = [2,500,000 + 1,150,000(0.727) – 1,150,000(0.273)]/0.727
= $4,156,946

13.21 Two years ago, on the recommendation of its construction engineers, United
Homebuilders purchased dumpsters, a front-end loader, and truck to carry off
construction debris, rather than subcontracting the service. Information is listed
for the project with depreciation determined using 5-year MACRS.

Year 0 1 2
First cost, $ −350,000
Savings, $ 150,000 150,000
Expenses, $ −25,000 −25,000

Copyright 2021 © McGraw-Hill Education. All rights reserved. No reproduction or distribution without the
prior written consent of McGraw-Hill Education.
12
Depreciation, $ 70,000 112,000
a. The United Homebuilders president does not want to continue ownership, as
he prefers subcontracting such services. What must the realizable market
value of the currently- owned equipment be to equal the current book value?
b. If savings are considered the equivalent of gross income, determine the CFBT
series and rate of return over the 2-year period. Assume the market value from
(a) is paid by an exporter of used heavy equipment.
c. Determine CFAT and after-tax ROR for the 2 years of ownership. Use the
information from previous parts. Let Te = 25%. (Hint: At this point, do not
consider the realized market value when TI and taxes are determined. See
problem 13.32 for further analysis.)

(a) Find BV after 2 years of MACRS depreciation

BV2 = 350,000 – 70,000 – 112,000


= $168,000

(b) Solve PW relation of CFBT values for i*, or use IRR function

Year Savings, $ Expenses, $ P and S, $ CFBT, $


0 –350,000 –350,000
0=– 1 150,000 –25,000 0 125,000 350,000 +
2 150,000 –25,000 168,000 293,000
125,000(P/F,i*,1) + 293,000(P/F,i*,2)

i* = 11.1% (using IRR function)

(c) Solve PW relation of CFAT values for i*, or use IRR function

Year Savings E P and S D TI Taxes CFAT


0 –350,000 –350,000
1 150,000 –25,000 70,000 55,000 13,750 111,250
2 150,000 –25,000 168,000 112,000 13,000 3,250 289,750

0 = –350,000 + 111,250(P/F,i*,1) + 289,750(P/F,i*,2)

i* = 8.3% (using IRR function)

13.22 (Spreadsheet exercise) You work for Tamimi Industries, which purchased
robotics equipment for $500,000 six years ago. The equipment is in place today,
has a total 10-year useful life, no salvage value, and a 5-year MACRS recovery
period. The effective tax rate is 21% and the actual cash flow and depreciation
amounts are shown. (a) Use a spreadsheet to tabulate CFBT, CFAT, EBIT, and i*
before and after taxes for 6 years of ownership. Is the after-tax approximation
using the before-tax rate within 2% of the calculated after-tax i*, using a

Copyright 2021 © McGraw-Hill Education. All rights reserved. No reproduction or distribution without the
prior written consent of McGraw-Hill Education.
13
round-off to one decimal? (b) What is one obvious difference between the CFBT
and EBIT series?

Year GI, $ Expenses, $ Depreciation, $


1 250,000 −120,000 100,000
2 280,000 −120,000 160,000
3 200,000 −122,000 96,000
4 260,000 −124,000 57,600
5 260,000 −126,000 57,600
6 180,000 −128,000 28,800

(a) Before-tax i* = 11.1% and after-tax i* = 9.1%

After-tax ROR approximation: 11.1(1 – 0.21) = 8.8%


2% range of calculated after-tax i*: 9.1 ± 0.2 is 8.9 to 9.3%

Approximation is just below the lower range bound

(b) EBIT, also called NOI, is calculated for years 1 to n and does not include the P
or S amounts.

13.23 Though capital gains and losses can make significant differences in CFAT estimates
in the year that a depreciable asset is salvaged, as a matter of practice, gains and
losses are generally neglected when the evaluation is performed. Why is this?
Identify one recommended exception to this practice for each of gains and losses.

Gains and losses occur at the time an asset is disposed of. Predicting the amount
of change in TI and taxes before the asset is purchased is not reliable and,
therefore, now very useful to the economic evaluation. Exceptions are land and
high-investment buildings (capital gains) and assets that are known to likely be
sacrificed at a significant loss when they are acquired or replaced (capital loss).

Copyright 2021 © McGraw-Hill Education. All rights reserved. No reproduction or distribution without the
prior written consent of McGraw-Hill Education.
14
13.24 Last month, a company specializing in wind power plant design and engineering
made a capital investment of $400,000 in physical simulation equipment that will
be used for at least 5 years and then sold for approximately 25% of its first cost.
By law, the assets are MACRS depreciated using a 3-year recovery period.
a. Explain why there is a predictable tax implication when the assets are sold.
b. By how much will the sale cause TI and taxes to change in year 5?

The information on the next page is used in problems 13.25 through 13.28.

The same asset is purchased in three countries with different tax laws and practices,
as outlined here. Use an after-tax MARR of 9% per year.

Country 1 2 3
First cost, $ −100,000 −100,000 −100,000
GI − E, $/year   25,000   25,000   25,000
Estimated salvage, $ 0 in year 5 0 in year 5 20,000 in year 5
Depreciation method SL with n = 5 MACRS with n = 3 DDB with n = 5
Depreciation recapture Not taxed Taxed as TI Taxed as TI
Tax rate, Te, % 30 30 30
Life, years 5 5 5
Actual SP and year, $ 20,000; year 5 20,000; year 5 20,000; year 5

(a) Selling for $100,000 after 5 years will cause depreciation recovery, since
MACRS will depreciate the BV to zero after 4 years.

(b) TI will increase by the depreciation recapture of $100,000

DR = SP – BV = 100,000 – 0 = $100,000

Taxes will increase by TI(Te) = 100,000(Te)

13.25 For Country 1, SL depreciation is $20,000 per year. Determine the (a) CFAT
series, and (b) PW of depreciation, taxes, and CFAT series using the 9% MARR.

(a)
Yea
r GI – E P and SP D TI Taxes CFAT
0 –100,000   –100,000
1 25,000 20,000 5,000 1,500 23,500
2 25,000 20,000 5,000 1,500 23,500
3 25,000 20,000 5,000 1,500 23,500
4 25,000 20,000 5,000 1,500 23,500

Copyright 2021 © McGraw-Hill Education. All rights reserved. No reproduction or distribution without the
prior written consent of McGraw-Hill Education.
15
5 25,000 20,000 20,000 5,000 1,500 43,500
(b) PWD = 20,000(P/A,9%,5) = 20,000(3.8897)
= $77,794

PWtax = 1500(P/A,9%,5)
= $5835

PWCFAT = –100,000 + 23,500(P/A,9%,5) + 20,000(P/F,9%,5)


= –100,000 + 23,500(3.8897) + 20,000(0.6499)
= $4406

There is no tax levied by country 1 on the DR of $20,000 in year 5

13.26 For Country 2, MACRS depreciation for the 4 years is $33,333, $44,444,
$14,815, and $7,407, respectively. Determine the (a) CFAT series, and (b) PW of
depreciation, taxes, and CFAT series using the 9% MARR.

(a)
Year GI – E P and SP D TI Taxes CFAT
0 –100,000   –100,000
1 25,000 33,333 –8,333 –2,500 27,500
2 25,000 44,444 –19,444 –5,833 30,833
3 25,000 14,815 10,185 3,056 21,944
4 25,000 7,407 17,593 5,278 19,722
5 25,000 20,000 0 45,000 13,500 31,500

(b) PWD = 33,333(P/F,9%,1) + … + 7407(P/F,9%,4)


= $84,675

In year 5, there is depreciation recapture

DR = SP – BV = 20,000 – 0 = $20,000 = S

TI is $20,000 larger in year 5 in country 2

TI = GI – E – D + DR
= 25,000 – 0 + 20,000
= $45,000

PWtax = –2500(P/F,9%,1) – 5833(P/F,9%,2) + … + 13,500(P/F,9%,5)


= $7669

PWCFAT = –100,000 + 27,500(P/F,9%,1) +…+ 31,500(P/F,9%,5)


= –100,000 + 27,500(0.9174) + …+ 31,500(0.6499)
= $2569

Copyright 2021 © McGraw-Hill Education. All rights reserved. No reproduction or distribution without the
prior written consent of McGraw-Hill Education.
16
13.27 For Country 3, DDB depreciation for the 5 years is $40,000, $24,000, $14,400,
$1,600, and 0, respectively. Determine the (a) CFAT series, and (b) PW of
depreciation, taxes, and CFAT series.

(a)
Year GI – E P and SP D TI Taxes CFAT
0 –100,000   –100,000
1 25,000 40,000 –15,000 –4,500 29,500
2 25,000 24,000 1,000 300 24,700
3 25,000 14,400 10,600 3,180 21,820
4 25,000 1,600 23,400 7,020 17,980
5 25,000 20,000 0 25,000 7,500 37,500

(b) PWD = 40,000(P/F,9%,1) + … + 1600(P/F,9%,4)


= $69,150

In year 5, there is no depreciation recapture, since DDB took the value down to
S = $20,000 and the asset was sold for this amount.

PWtax = –4500(P/F,9%,1) + 300(P/F,9%,2) + … + 7,500(P/F,9%,5)


= $8427

PWCFAT = –100,000 + 29,500(P/F,9%,1) + … + 37,500(P/F,9%,5)


= –100,000 + 29,500(0.9174) + … + 37,500(0.6499)
= $1811

13.28 (Spreadsheet exercise) Develop a spreadsheet that calculates the PW values for
each country requested above. Which country offers the lowest PW of taxes?

NPV function displays PW @ 9% values for D, Taxes, and CFAT series

PWtax is lowest for country 1; no DR taxation is one reason.

Copyright 2021 © McGraw-Hill Education. All rights reserved. No reproduction or distribution without the
prior written consent of McGraw-Hill Education.
17
13.29 You use a car in a majority of your sole-proprietorship engineering consulting
business. This business asset cost you $80,000 new and has been depreciated
according to MACRS over a 5-year period. At the end of year 4, you bought a
replacement vehicle and traded for a net positive $15,000 on the old car.
Determine if depreciation recapture or a capital loss is present and, if so, how
much.

Total depreciation: 20% + 32% + 19.2% + 11.52% = 82.72%

BV4 = 80,000 – 80,000(0.8272)


= $13,824

BV4 < SP of $15,000; there is depreciation recapture

DR = 15,000 – 13,824
= $1176

13.30 An automated assembly robot that cost $300,000 has a recovery period of 5 years
with an expected $50,000 salvage value. The MACRS depreciation rates for years
1, 2, 3, and 4 are 20.0%, 32.0%, 19.2%, and 11.52%, respectively. (a) What is the
depreciation recapture, capital gain, or capital loss, provided the robot was sold
after 3 years for $80,000? (b) How is this amount taxed?

Copyright 2021 © McGraw-Hill Education. All rights reserved. No reproduction or distribution without the
prior written consent of McGraw-Hill Education.
18
(a) Total depreciation: 20% + 32% + 19.2% = 71.2%

BV3 = 300,000 – 300,000(0.712)


= $86,400

BV3 > SP of $80,000; there is a capital loss

CL = 86,400 – 80,000 = $6400

(b) The CL can offset CG; a CL is not a direct reduction in TI

13.31 Cheryl, a CVE student who is working on an ISE minor, is studying depreciation
and taxes in her engineering management course. The assignment is to
demonstrate (1) that shorter recovery periods require the same total taxes as
longer periods, and (2) that PWtax is lower for shorter recovery periods. Help her
demonstrate these two facts using these estimates developed for one asset and a
6-year study period: P = $65,000; S = $5000 whenever it is sold; GI = $32,000
per year; AOC = $10,000 per year; SL = depreciation; MARR = 12% per year;
and Te = 31%. The recovery period can be either 3 or 6 years. Solve using
(a) equations, and (b) a spreadsheet.

(a) Equations: Let AOC = Expenses

(1) Recovery over 3 years

D = (65,000 – 5,000)/3 = $20,000 per year

Years 1-3: Taxes = (GI – E – D)( Te )


= (32,000 – 10,000 – 20,000)(0.31)
= $620

Years 4-6: Taxes = (GI – E)( Te )


= (32,000 – 10,000)(0.31)
= $6820

Total taxes = 3(620) + 3(6820) = $22,320

PWtax = 620(P/A,12%,3) + 6820(P/A,12%,3)(P/F,12%,3)


= 620(2.4018) + 6820(2.4018)(0.7118)
= $13,149

Copyright 2021 © McGraw-Hill Education. All rights reserved. No reproduction or distribution without the
prior written consent of McGraw-Hill Education.
19
(2) Recovery over 6 years

D = (65,000 – 5,000)/6 = $10,000 per year

Years 1-6: Taxes = (GI – E – D)( Te )


= (32,000–10,000–10,000)(0.31)
= $3720

Total taxes = 6(3720) = $22,320 (Same as for 3-year recovery period)

PWtax = 3720(P/A,12%,6)
= 3720(4.1114)
= $15,294

Recovery in 3 years has a lower PWtax value; total taxes are the same

(b) Spreadsheet: Taxes are the same ($22,320), but PWtax is smaller for the 3-year
recovery period ($13,148 vs $15,294).

13.32 Review the situation in problem 13.21. In part (c), you were asked to not consider
the realized market value when calculating TI and taxes. Now, consider the fact
that the equipment was sold for $188,000 after 2 years of service. Determine the
CFAT series and after-tax ROR for the 2 years of ownership. The initial data are
repeated here.

Year 0 1 2
First cost, $ −350,000
Savings, $ 150,000 150,000
Expenses, $ −25,000 −25,000
Depreciation, $ 70,000 112,000

Copyright 2021 © McGraw-Hill Education. All rights reserved. No reproduction or distribution without the
prior written consent of McGraw-Hill Education.
20
RD = SP – BV2 = 188,000 – (350,000 – 70,000 – 112,000)
= $ 20,000

Solve PW relation of CFAT values for i*, or use IRR function

Year Savings E P and S D DR TI Taxes CFAT


0 –350,000 –350,000
1 150,000 –25,000 70,000 55,000 13,750 111,250
2 150,000 –25,000 188,000 112,000 20,000 33,000 8,250 304,750

PW = –350,000 + 111,250(P/F,i*,1) + 304,750(P/F,i*,2)

i* = 10.5% (using IRR function)

13.33 (Spreadsheet exercise) James Engineering, Inc. wants to determine a method for
book depreciation of newly acquired assets. Use a 6-year study period to
determine the total taxes and PW of taxes for the depreciation methods SL, SYD,
MACRS, and DDB for the information below. Using the criterion of PW of taxes,
rounded to the nearest one hundreds, what is the numerical difference between the
four methods of depreciation?

B = $200,000              S = 0
Recovery period = 5 years    i = 10% per year
Te = 24%
CFBT = $500,000 for year 1, decreasing by $50,000 per year thereafter

Spreadsheet uses SLN, SYD, VDB and DDB functions for depreciation over
5 years. Only MACRS rates extend over 6 years. Total taxes are $492,000 for all
methods, except DDB (Tax total is $495,732), since less than B = $200,000 is
removed by DDB.

PW of taxes are lowest for SYD since MACRS takes 6 years to depreciate; all
PW values are lower than that for the SL method.

The PW of taxes, when rounded, range from $367,700 to $370,000. The numerical
difference is $2300. There is no significant difference between the methods using
this criterion.

Copyright 2021 © McGraw-Hill Education. All rights reserved. No reproduction or distribution without the
prior written consent of McGraw-Hill Education.
21
13.34 In a replacement study between a defender and a challenger, there may be a
capital gain or loss when the defender asset is sold. (a) How is the gain or loss
calculated, and (b) how does the gain or loss affect the AW values?

(a) For a capital gain, it is the difference between the sales price and the basis
(first cost) of the asset.
For a capital loss, it is the difference between sales price and the asset’s book
value.

(b) The AW of the challenger is affected in year 0 by the capital gains tax. If it is a
capital loss, the netting of losses against gains can affect AW.

13.35 In an after-tax replacement study of cost alternatives involving one challenger and
one defender, how will a capital loss affect the AW of each alternative when
selling the defender?

A capital loss will result in reduced taxes to the company. This tax savings is
applied to the challenger, since the savings is realized only if the challenger is
bought. Thus, a capital loss will render the challenger more attractive.

13.36 The market value of a presently-owned machine in a wind turbine manufacturing


plant is $130,000. It has annual operating costs of $70,000 with no salvage value
after its remaining life of 3 years. The depreciation for the next 3 years will be
$69,960, $49,960, and $35,720. Assume the effective tax rate is 25% and its after-
tax MARR is 12%. A PW relation for comparing the defender against a
challenger that also has a 3-year life is being developed. Determine the after-tax
cash flow value for year 2 only used in the PW relation.

TI2 = –70,000 – 49,960 = –119,960


Taxes2 = –119,960(0.25) = $–29,990 (tax savings)

CFAT2 = –70,000 + 29,990


= $–40,010

Copyright 2021 © McGraw-Hill Education. All rights reserved. No reproduction or distribution without the
prior written consent of McGraw-Hill Education.
22
13.37 The information for a defender and challenger is shown below. All monetary
values are in $1000 units. Assume the assets will be salvaged at their original
salvage estimates. Since no revenues are estimated, all taxes are negative and
considered “savings” in the cash flow computations. Use an after-tax MARR of
12% per year, Te = 21%, and a study period of 4 years to perform a replacement
analysis (a) by factors, and (b) by spreadsheet.

Defender Challenger
First cost, $1000 −45 −24
Estimated S at purchase, $1000  5  0
Market value now, $1000   35 —
AOC, $1000 per year −7 −8
Depreciation method SL MACRS
Recovery period, years   8  3
Useful life, years   8  5
Years owned   3 —

(a) Find after-tax PW of costs; DR is present on the defender trade in; no capital
gains or losses present

Defender:
SL depreciation is (45,000–5000)/8 = $5000

Annual tax = (–E – D)(Te)


= (–7000 – 5000)(0.21)
= $–2520 (savings)

CFAT = CFBT – taxes


= –7000 – (–2520)
= $–4480

PWD = –35,000 + 5000(P/F,12%,4) – 4480(P/A,12%,4)


= –35,000 + 5000(0.6355) – 4480(3.0373)
= $–45,430

Challenger:
MACRS depreciation over n = 5, but only 4 years apply. Defender trade-in
DR must be included.

Defender BV3 = 45,000 – 3(5000) = $30,000


SP = $35,000
DR = SP – BV = 5,000

Copyright 2021 © McGraw-Hill Education. All rights reserved. No reproduction or distribution without the
prior written consent of McGraw-Hill Education.
23
Tax on DR = 5,000(0.21) = $1050

Challenger first cost = –24,000 – 1050 = $–25,050

MACRS depreciation is based on $24,000 first cost

Year E P and S Rate D TI Taxes CFAT


0 –25,050 –25,050
1 –8000 0.3333 8,000 –16,000 –3360 –4,640
2 –8000 0.4445 10,668 –18,668 –3920 –4,080
3 –8000 0.1481 3,554 –11,554 –2426 –5,574
4 –8000 0 0.0741 1,778 –9,778 –2053 –5,947

PWC = –25,050 – 4,640(P/F,12%,1) – … – 5.947(P/F,12%,4)


= $–40,193

Select the challenger with a lower PW of cost.

(b) Spreadsheet: Select the challenger with a smaller PW of costs.

13.38 Apple Crisp Foods signed a contract some years ago for maintenance services on
its fleet of trucks and cars. The contract is up for renewal now for a period of
1 year or 2 years only. The contract quote is $300,000 per year if taken for 1 year
and $240,000 per year if taken for 2 years. The VP for Finance wants to renew the
contract for 2 years without further analysis, but the VP for Engineering believes
it is more economical to perform the maintenance inhouse. Since much of the
fleet is aging and must be replaced in the near future, a fixed 3-year study period

Copyright 2021 © McGraw-Hill Education. All rights reserved. No reproduction or distribution without the
prior written consent of McGraw-Hill Education.
24
has been agreed upon. The estimates for the inhouse (challenger) alternative are
as follows:

First cost, $ −800,000


AOC, $ per year −120,000
Life, years 4
Estimated selling price Loses 25% of P annually
(year 1: $600,000; year
3: $200,000; year 4: 0)
MACRS depreciation 3-year recovery period

The effective tax rate is 35% and the after-tax MARR is 10% per year. Perform an
after-tax AW analysis, and determine which VP has the better economic strategy
over the next 3 years.

Study period is fixed at 3 years

1. Succession options

Option Defender Challenger


1 2 years 1 year
2 1 2
3 0 3

2. Find AW for defender and challenger for 1, 2 and 3 years of retention.

Defender

AWD1 = $300,000 AWD2 = $240,000

Challenger

No tax effect if contract (the defender) is cancelled. Calculate CFAT for 1, 2, and
3 years of ownership. Tax rate is 35%. There is DR each year.
Tax
Year E, $ d D, $ BV, $ SP, $ DR, $ TI, $ savings, $ CFAT, $
0 – – – 800,000 – – – – –800,000
1 –120,000 0.333 266,640 533,360 600,000 66,640 –320,000 –112,000 592,000
2 –120,000 0.445 355,600 177,760 400,000 222,240 –253,360 – 88,676 368,676
3 –120,000 0.148 118,480 59,280 200,000 140,720 – 97,760 – 34,216 114,216

Copyright 2021 © McGraw-Hill Education. All rights reserved. No reproduction or distribution without the
prior written consent of McGraw-Hill Education.
25
TI = –E – D + DR
Year 1: TI = –120,000 – 266,640 + 66,640 = $–320,000
Year 2: TI = –120,000 – 355,600 + 222,240 = $–253,360
Year 3: TI = –120,000 – 118,480 + 140,720 = $– 97,760

CFAT = – E + SP – taxes

Year 1: – 120,000 + 600,000 – (–112,000) = $592,000


Year 2: – 120,000 + 400,000 – (–88,676) = $368,676
Year 3: – 120,000 + 200,000 – (–34,216) = $114,216

AWC1 = – 800,000(A/P,10%,1) + 592,000


= – 800,000 (1.10) + 592,000
= $– 288,000

AWC2 = – 800,000(A/P,10%,2) + [592,000(P/F,10%,1)


+ 368,676(P/F,10%,2)](A/P,10%,2)
= – 800,000(0.57619) + [592,000(0.9091) + 368,676(0.8264)](0.57619)
= $24,696

AWC3 = – 800,000(A/P,10%,3) + [592,000(P/F,10%,1) + 368,676(P/F,10%,2)


+ 114,216(P/F,10%,3)](A/P,10%,3)
= – 800,000(0.40211) + [592,000(0.9091) + 368,676(0.8264)
+ 114,216(0.7513)](0.40211)
= $51,740

Selection of best option: Determine AW for each option; select best AW.

Summary of cost/year and project AW

Cost per year, $


Option 1 2 3 AW___
1 – 240,000 –240,000 –288,000 –254,493
2 –300,000 24,696 24,696 – 94,000
3 51,740 51,740 51,740 + 51,740
Conclusion: Select option 3; replace now with the challenger. Engineering VP has
the better economic strategy.

13.39 You provide consulting services as a sole proprietor business owner. You know it
is economically difficult to justify the purchase of a new car when your currently
owned vehicle is operating okay; however, you decide to try, anyway. The
information you collected about your current, 5-year-old car versus a new, fuel-
efficient hybrid, safety-loaded car is shown on the next page. The AOC is
calculated at 25,000 miles driven per year, with costs of 60¢/mile (current car)
and 48¢/mile (new car, based on better mileage and lower maintenance cost).

Copyright 2021 © McGraw-Hill Education. All rights reserved. No reproduction or distribution without the
prior written consent of McGraw-Hill Education.
26
Once this decision is made, you plan to retain the current car for 5 more years;
retention of the new car may vary, as described below. The low trade-in offer
from the dealership on the current car of $2000, well below the Kelley Blue
Book® value you found online, is a “cash now” offer, not a reduction in the cost of
the new car. You take no annual depreciation on your car.
At i = 8% per year, perform an AW-based replacement analysis and select
the economically better alternative, under the following assumptions:
a. You are serious about retaining the new car 10 years.
b. You may get the “new car bug” and decide to trade again in 5 years, but you
are not concerned about possible future trade-in values on the new car in this
analysis.
c. You want to know the maximum cost of a new car that will make the
alternatives economically break even in 5 years.
d. You believe you can find a way to use the perceived capital loss (market value
− offered trade-in) on the current car to a full tax advantage if the new car
option is chosen with a 5-year retention period. You have an effective 22% tax
rate.

Current Car New Car


Market value, $   10,000
First cost, $ −35,000
AOC, $ per year −15,000 −12,000
Original useful life, years      10 10
Retention time, years 5 varies
Offered trade-in, $  2,000

(a) Defender: P = $10,000; n = 5 years

AWD = –10,000(A/P,8%,5) – 15,000


= $–17,505

Challenger: P = $–35,000; n = 10 years

AWC = –35,000(A/P,8%,10) – 12,000


= $–17,216

Select the new car by a $289 per year AW advantage

(b) AWD = $–17,505

AWC = –35,000(A/P,8%,5) – 12,000


= $–20,766

Retain the current car by a $3261 per year AW advantage

Copyright 2021 © McGraw-Hill Education. All rights reserved. No reproduction or distribution without the
prior written consent of McGraw-Hill Education.
27
(c) Set AW relations for n = 5 equal and let PC = new car first cost

AWD = AWC
–10,000(A/P,8%,5) – 15,000 = PC(A/P,8%,5) – 12,000
PC = –5504.6/0.25046
= $–21,978

(d) AWD = $–17,505

AWC = [–35,000 + 0.22(10,000–2,000)](A/P,8%,5) – 12,000


= $–20,325

Again, retain the current car. Makes little difference since the possible tax
advantage is distributed over 5 years.

13.40 Electrical generators produce not only electricity, but also heat from conductor
resistance and from friction losses in bearings. A company that manufactures
generator coolers for nuclear and gas turbine power plants undertook a plant
expansion through financing that had a debt-equity mix of 45-55. If $18 million
came from mortgages and bond sales, what was the total amount of the financing?

The debt portion of $18 million represents 45% of the total

Total amount of financing = 18,000,000/0.45


= $40,000,000

13.41 Determine the debt-to-equity mix when Applied Technology bought out
Southwest Semiconductor using financing as follows: $12 million from
mortgages, $5 million from retained earnings, $10 million from cash on hand, and
$20 million from bonds.

D-E mix: Debt = 12 + 20 = $32 million


Equity = 5 + 10 = $15 million

% debt = (32/(32 + 15))(100) = 68%


% equity = (15/47)(100) = 32%

D-E mix is 68-32

13.42 Business and engineering seniors are comparing methods of financing their
college education during their senior year. The business student has $30,000 in
student loans that comes due at graduation. Interest is an effective 4% per year.
The engineering senior owes $50,000: 50% from his parents with no interest due,
and 50% from a credit union loan. This latter amount is also due at graduation
with an effective rate of 7% per year.

Copyright 2021 © McGraw-Hill Education. All rights reserved. No reproduction or distribution without the
prior written consent of McGraw-Hill Education.
28
a. What is the D-E mix for each student?
b. If their grandparents pay the loans in full at graduation, what is the amount on
each check they write for each graduate?
c. When grandparents pay the full amount at graduation, what percent of the
total amount due does the interest represent?

(a) Business: all debt; D-E = 100 to 0

Engineering: all debt; D-E = 100 to 0

(b) Business: FW = 30,000(F/P,4%,1)


= 30,000(1.04)
= $31,200

Check is for $31,200 to student loan office

Engineering: FW = 25,000(1) + 25,000(F/P,7%,1)


= 25,000(1 + 1.07)
= $51,750

Two checks: $25,000 to parents and $26,750 to credit union

(c) Business: 4%

Engineering: 0.5(0%) + 0.5(7%) = 3.5%

13.43 Two public corporations, First Engineering and Midwest Development, each
show capitalization of $175 million in their annual reports. The balance sheet for
First Engineering indicates total debt of $87 million, and that of Midwest
indicates net worth of $62 million. Determine the D-E mix for each company.

First Engineering: Fraction debt = 87/175 = 0.497

Fraction equity = (175-87)/175 = 0.503

Basically, a 50-50 D-E mix


Midwest Development: Fraction debt = (175-62)/175 = 0.646

Fraction equity = 62/175 = 0.354

Approximately, a 65-35 D-E mix

Copyright 2021 © McGraw-Hill Education. All rights reserved. No reproduction or distribution without the
prior written consent of McGraw-Hill Education.
29
13.44 Forest Products, Inc. invested $50 million. The company’s overall D-E mix is
60-40. What is the return on the company’s equity capital if the net income is
$5 million on a revenue base of $6 million?

In $ million units,

Equity capital = 50(0.40) = $20

Return on equity capital = 5/20 = 0.25 (25%)

13.45 Gerald Factories manufactures tri-axial accelerometers for space-restricted


applications. The financing profile, with interest rates, is as follows: $3 million in
stock sales at 15% per year, $4 million in bonds at 9%, and $6 million in retained
earnings at 7% per year. Determine the following: (a) WACC, and (b) inflation-
adjusted MARR, if Gerald expects a real ROR of 5% per year and inflation has
averaged 3.5% per year.

(a) In $ million units,

Total financing = 3 + 4 + 6 = $13

WACC = (3/13)(0.15) + (4/13)(0.09) + (6/13)(0.07)


= 0.0946 (9.46%)

(b) Use Equation [10.11] for MARRf to determine the inflation-adjusted MARR
after adding the 5% ROR to WACC.

MARR = WACC = ROR


= 9.46 + 5.0
= 14.46%

MARRf = MARR + f + MARR(f)


= 0.1446 + 0.035 + 0.1446(0.035)
= 0.1847 (18.47%)

13.46 Midac Corporation wants to arrange for $37.5 million in capital to finance
manufacturing a new consumer product. The current plan is 60% equity capital
and 40% debt financing. Calculate (a) the WACC, and (b) MARR for the
following scenario:

Equity capital: 60%, or $22.5 million, via common stock sales, with 40% of this
amount paying dividends at a rate of 5% per year, and the remaining
60% retained earnings, which currently earn 9% per year.
Debt capital: 40%, or $15 million, obtained through two sources: bank loans for
$10 million borrowed at 8% per year, and the remainder in
convertible bonds at an estimated 10% per year bond dividend rate.
Rate of return: 8% above the average cost of capital with no inflation considered.

Copyright 2021 © McGraw-Hill Education. All rights reserved. No reproduction or distribution without the
prior written consent of McGraw-Hill Education.
30
(a) WACC = cost of debt capital + cost of equity capital
= (0.4)[0.667(8%) + 0.333(10%)] + (0.6)[(0.4)(5%) + (0.6)(9%)]

= 0.4[8.667%] + 0.6 [7.4%]


= 7.907%

(b) MARR = WACC + ROR


= 7.9 + 8.0
= 15.9% (rounded)

13.47 Assume you are reviewing your investment portfolio. One public corporation in
which you own common stock reported a before-tax WACC of 11.1% for the year
in its report to stockholders. The common stock that you own has averaged a total
return of 7% per year over the last three years. The annual report also mentions
that projects within the corporation are 50% funded by its own capital. Estimate
the company’s (a) before-tax cost of debt capital, (b) after-tax cost of debt capital
if Te is 25%, and (c) after-tax WACC.

(a) Let x = Before-tax cost of debt capital

WACC = 11.1% = 0.5(7%) + (1– 0.5)(x)


x = (11.1 – 3.5)/0.5
= 15.2%

(b) After-tax cost = 15.2(1 –.25)


= 11.4%

(c) Debt capital has a tax advantage

After-tax WACC = 0.5(7%) + 0.5(11.4%)


= 9.2%

13.48 Brad and Zack are twin brothers in many ways, but not identical in their personal
financial management. They both decided to buy the same new car and had to
provide some financial information to qualify for a $30,000 car loan. Here is a
snapshot of their situations.

Brother Brad Zack


Debt, $ 28,000 200,000
Investments, $ 250,000 50,000
Credit score 725 (Good) 590 (Low)

Copyright 2021 © McGraw-Hill Education. All rights reserved. No reproduction or distribution without the
prior written consent of McGraw-Hill Education.
31
A person’s credit score can range from 300 to 850. Their car loan rates,
based on credit scores and financial history, are predicted to be 3.6% per year
(Brad) and 10.7% (Zack). These rates are per year compounded monthly and a
total of 60 payments will be made.
You are sharing an evening meal with them and you wish to help them
understand the differences in their debt versus investments (equity) status and the
monthly loan payments they will have to make. Determine the following for
them:
a. Current D-E mixes, assuming no other significant financial amounts are
unreported.
b. Expected monthly payment and the total amount they will pay over the
5 years.
c. D-E mixes for each brother after the loans are approved.

(a) Brad: Total financials = $278,000

Fraction debt = 28,000/278,000 = 0.10


Fraction equity = 250,000/278,000 = 0.90

D-E mix is 10-90

Zack: Total financials = $250,000

Fraction debt = 200,000/250,000 = 0.80


Fraction equity = 50,000/250,000 = 0.20

D-E mix is 80-20

(b) Brad’s loan: Monthly payment: AW = 30,000(A/P,3.6%/12,60)


= $547.10

Alternatively, spreadsheet function: = – PMT(3.6%/12,60,30000)

Total amount paid: 547.10(60) = $32,826.00

Zack’s loan: Monthly payment: AW = 30,000(A/P,10.7%/12,60)


= $647.79

Alternatively, spreadsheet function: = – PMT(10.7%/12,60,30000)

Total amount paid: 647.79(60) = $38,867.40

Copyright 2021 © McGraw-Hill Education. All rights reserved. No reproduction or distribution without the
prior written consent of McGraw-Hill Education.
32
(c) Add $30,000 of debt and total financials for each brother

Brad: Total financials = $308,000

Fraction debt = 58,000/308,000 = 0.19


Fraction equity = 250,000/308,000 = 0.81

D-E mix is 19-81

Zack: Total financials = $280,000

Fraction debt = 230,000/280,000 = 0.82


Fraction equity = 50,000/280,000 = 0.18

D-E mix is 82-18

13.49 Use the template in Example 13.12 to accept or reject the following proposal
developed by Mighty Foundation, Inc. (a) Use the information below. (b) Change
the inflation rate to 5% per year. All other parameters remain the same.

Asset Depreciation Financing Evaluation


First cost: Method: SL Loan: $500,000 MARR: 12%
$1,500,000
Useful life: 8 years Recovery: 8 years Length: 6 years Inflation rate: 0%
Salvage: $150,000 Interest rate: 4% Effective tax rate: 21%
GI: $500,000/year Compounding: Annual
AOC: $200,000/year

(a) Enter the parameter values into the Figure 13.12 spreadsheet. The depreciation
method (SL) requires the SLN function in column F (calculations section).
Adjust the loan calculations for 6 years at 4% per year compounded annually
(columns G through J in calculations section). GOAL SEEK is used in column
J (analysis section) to determine i* = 12.52% for f = 0% (cell H38).

Conclusion: Accept the proposal, since i* > MARR = 12%

Copyright 2021 © McGraw-Hill Education. All rights reserved. No reproduction or distribution without the
prior written consent of McGraw-Hill Education.
33
(b) If f = 5%, the situation changes. Now if* = 16.86% (cell H38)

Conclusion: Reject the proposal, since if* < MARRf = 17.60%

Copyright 2021 © McGraw-Hill Education. All rights reserved. No reproduction or distribution without the
prior written consent of McGraw-Hill Education.
34
13.50 Beto has a son, Jaime, a senior in engineering, who is considering a new car that
costs $40,000, with $35,000 of the purchase cost provided as a car loan at 9% per
year compounded monthly for 5 years. If the estimated trade-in 8 years from now
is $5000, and money is currently earning 12% per year, determine if this is an
economically justified deal. Don’t consider any inflation or depreciation in the
analysis. Jaime has an annual income of $63,000, average expenses of $55,000
per year, and an effective tax rate of 13% per year. These are not expected to
change in the near future. Assuming the loan interest can be used to reduce
taxable income, is this an economically justified deal?

Enter the parameters into the spreadsheet from Example 13.12 (shown below).
There is no depreciation or inflation considered (f = 0%) and Te is 13%. The market
rate of 12% is the MARR, and GOAL SEEK displays i* = 11.32%. The economic
conclusion is a close one:

Conclusion: Reject the deal, since i* = 11.32% < MARR = 12%.

13.51 While an engineering manager may prefer to use CFAT estimates to evaluate the
AW of a project, a financial manager may select AW of EVA estimates. Why are
these preferences predictable?

A finance manager likes EVA because it indicates the project’s enhancement to


the monetary worth of the corporation. Engineering managers like CFAT because
it indicates actual cash flows for the project.

Copyright 2021 © McGraw-Hill Education. All rights reserved. No reproduction or distribution without the
prior written consent of McGraw-Hill Education.
35
13.52 An asset with a first cost of $300,000 is depreciated by the MACRS method using
a 5-year recovery period. Determine the monetary value added to the corporation
by the asset in year 2 of its service if NOPAT is $70,000 and the company uses an
after-tax interest rate of 9%. The MACRS depreciation rates for years 1 and 2 are
20% and 32%, respectively.

BV1 = 300,000 – 300,000(0.20) = $240,000

EVA2 = NOPAT – (after-tax interest rate)(BV1)


= 70,000 – (0.09)(240,000)
= $48,400

13.53 In conducting an EVA analysis for year 2 for a newly introduced product line,
Bethune, Inc., which manufactures preassembled blower packages and other
water treatment components, determined the EVA to be $28,000. The company
uses an after-tax interest rate of 14% and a Te of 24%. The initial investment
capital required for the new product was $550,000 and all equipment is 3-year
MACRS depreciated. Bethune’s CEO knew that the gross income was $700,000,
but he asked you to find out how much expense was associated with the new
product line for year 2.

Find BVt –1 = BV1; solve for NOPAT; find TI from NOPAT; solve for E from TI.

BV1 = 550,000 – 550,000(0.3333) = $366,685

EVA2 = 28,000 = NOPAT – (0.14)(366,685)

NOPAT = $79,336

NOPAT = 79,336 = TI(1 – 0.24)


TI = $104,389

D2 = 550,000(0.4445)
= $244,475

104,389 = GI – E2 – D2
= 700,000 – E2 – 244,475
E2 = $351,136

13.54 (Spreadsheet exercise) Cardenas and Moreno Engineering is evaluating a large


flood control program for several southern cities. One component is a 4-year
project for a special-purpose transport ship crane for use in building permanent
storm-surge protection against hurricanes on the East Coast. The estimates are
P = $300,000, S = 0, n = 3 years. MACRS depreciation with a 3-year recovery is
indicated. GI and E are estimated at $200,000 and $80,000, respectively, for each

Copyright 2021 © McGraw-Hill Education. All rights reserved. No reproduction or distribution without the
prior written consent of McGraw-Hill Education.
36
of 4 years, Te = 23%, and the after-tax MARR is 10% per year. The CFAT has
been calculated. (a) Determine the AW values of both the CFAT and the EVA
series by spreadsheet, unless instructed otherwise. They should have the same
value. (b) Monetarily, after how many years does the ship crane begin to
contribute positive value to the company?

Year GI, $ E, $ P, $ D, $ TI, $ Taxes, $ CFAT, $


0 −300,000 −300,000
1 200,000 −80,000 99,990   20,010    4,602   115,398
2 200,000 −80,000 133,350 −13,350 −3,071 123,071
3 200,000 −80,000 44,430    75,570   17,381   102,619
4 200,000 −80,000 22,230    97,770  22,487   97,513

(a) The spreadsheet verifies that the AW values are the same. Some functions for
year 4 are detailed.

(b) CFAT shows a big cost in year 0 and positive cash flows thereafter. EVA
shows nothing in year 0 and after 2 years the value-added terms turn positive,
indicating a positive contribution to the company’s worth.

13.55 (Spreadsheet exercise) Triple Play Innovators Corporation (TPIC) plans to offer
IPTV (Internet Protocol TV) service to North American customers. Perform an
AW analysis of the EVA series for the two alternative suppliers that bid for
hardware and software contracts. Let Te = 30% and after-tax MARR = 8%; use
SL depreciation (for simplicity) and a study period of 8 years. Use a spreadsheet,
unless instructed to perform factor-based calculations.

Copyright 2021 © McGraw-Hill Education. All rights reserved. No reproduction or distribution without the
prior written consent of McGraw-Hill Education.
37
Bidder’s Country United States Malaysia
First cost, $ 4.2 million 3.6 million
Recovery period, years 8 5
Salvage value, $ 0 0
NOI, $ per year 1,500,000 in year 1; increasing
by 300,000 per year up to 8 years

Depreciation is SL: United States: 4.2 million/8 = $525,000


Malaysia: 3.6 million/5 = $720,000

Net operating income: NOI = GI – E

Spreadsheet solution: The CFAT series is shown also for information only.

The Malaysian bidder indicates a larger AW of EVA; however, the difference is


quite small, given the size of the order.

13.56 A graduated income tax system means:


a. only taxable incomes above a certain level pay any taxes
b. a higher flat rate goes with all of the taxable income
c. higher tax rates go with higher taxable incomes
d. rates are indexed each year to keep up with inflation

Answer is (c)

Copyright 2021 © McGraw-Hill Education. All rights reserved. No reproduction or distribution without the
prior written consent of McGraw-Hill Education.
38
13.57 Charlotte has an effective tax rate of 22%. This year she reported the following:
salary of $175,000, investment income of $4000, expenses of $15,000, and a
standard deduction of $12,000. Approximate federal income taxes are closest to:
a. $11,750
b. $27,750
c. $33,450
d. $36,740

Taxes = (175,000 + 4,000 – 12,000)(0.22) = $36,740


Answer is (d)

13.58 Ric and Mariam (R&M) file taxes as “married filing jointly.” Their total income
for one year was $185,000. Harold and Jan (H&J) file as “single.” Their total
incomes were $90,000 and $95,000, respectively. Standard deductions of $12,000
for singles and $24,000 for married applied. If the year was 2018, the couple that
paid more taxes and the difference between their total taxes were closest to:
a. R&M, $200
b. R&M, $10
c. H&J, $10
d. H&J, $200

Ric and Mariam: TI = 185,000 – 24,000 = $161,000

R&M taxes = 8.907 + 0.22(161,000 – 77,400)


= $27,299

Harold: TI = 90,000 – 12,000 = $78,000

Taxes = 4,453.50 + 0.22(78,000 – 38,700)


= $13,099.50

Jan: TI = 95,000 – 12,000 = $83,000

Taxes = 14,089.50 + 0.24(83,000 – 82,500)


= $14,209.50

H&J taxes = 13,099.50 + 14,209.50


= $27,309

H&J pay more by $10


Answer is (c)

Copyright 2021 © McGraw-Hill Education. All rights reserved. No reproduction or distribution without the
prior written consent of McGraw-Hill Education.
39
13.59 The after-tax analysis for a $60,000 investment with associated GI − E is shown
for the first 2 years only. If the effective tax rate is 24%, the value for taxable
income (TI) for year 1 is closest to:
a. $11,750
b. $16,670
c. $27,750
d. $36,742

Year Investment GI − E TI Taxes CFAT


0 −60,000 −60,000
1 30,000 ?   26,000
2 35,000 15,000 3600   31,400

CFAT = GI – E – TI(0.24)
26,000 = 30,000 – TI(0.24)
TI = (30,000 – 26,000)/0.24
= $16,667
Answer is (b)

13.60 If the after-tax ROR for a cash flow series is 11.2% and the corporate effective tax
rate is 25%, the approximated before-tax ROR is closest to:
a. 6.8%
b. 14.9%
c. 18.4%
d. 28.7%

Before-tax ROR = After-tax ROR/(1– Te)


= 11.2%/(1–0.25)
= 14.9%
Answer is (b)

13.61 If the federal tax rate is 21% and the state tax rate is 7%, the effective tax rate is
closest to:
a. 31.5%
b. 29.5%
c. 27.4%
d. 26.5%

Te = 7 + (1 – 0.07)(21) = 26.5%
Answer is (d)

Copyright 2021 © McGraw-Hill Education. All rights reserved. No reproduction or distribution without the
prior written consent of McGraw-Hill Education.
40
13.62 If all values carry a + sign, cash flow before taxes (CFBT) is represented by the
equation:
a. gross income − operating expenses − depreciation − initial investment +
salvage value
b. gross income − operating expenses − depreciation + salvage value
c. gross income − operating expenses − initial investment + salvage value
d. gross income − operating expenses + initial investment + salvage value

Answer is (c)

13.63 Pennington Oil is in the 28% effective tax bracket. It had gross income of $470
million in each of the last two years. Total deductions were $160 million and
$120 million for years 1 and 2, respectively. The difference in income taxes in the
two years was closest to:
a. $11 million
b. $20 million
c. $50 million
d. $40 million

Tax difference = (160,000,000 – 120,000,000)(0.28)


= $11,200,000
Answer is (a)

13.64 A small manufacturing company with a gross income of $360,000 has the
following expenses: M&O $76,000, insurance $7000, labor $110,000, utilities
$29,000. If debt service is $37,000 and taxes are $9000, the net operating income
is closest to:
a. $92,000
b. $101,000
c. $138,000
d. $174,000

NOI = GI – E
= 360,000 – 76,000 – 7000 – 110,000 – 29,000
= $138,000
Answer is (c)

13.65 An asset purchased for $100,000 with S = $20,000 after 5 years was depreciated
using the 5-year MACRS rates. Expenses average $18,000 per year and the
effective tax rate is 30%. The asset is actually sold after 5 years of service for
$22,000. MACRS rates in years 5 and 6 are 11.52% and 5.76%, respectively. The
after-tax cash flow from the sale is closest to:
a. $27,760
b. $17,130
c. $26,870
d. $20,585

Copyright 2021 © McGraw-Hill Education. All rights reserved. No reproduction or distribution without the
prior written consent of McGraw-Hill Education.
41
BV5 = 100,000(0.0576) = $5760

DR = 22,000 – 5760 = $16,240


Tax on DR = 16,240(0.30) = $4872

Cash flow = 22,000 – 4872


= $17,128
Answer is (b)

13.66 The Wilkins Company has maintained a 50-50 D-E mix for capital investments.
Equity capital costs 11%; however, debt capital that historically costs 9% has now
increased by 20%. If Wilkins does not want to exceed its past weighted average
cost of capital (WACC) and it is forced to go to a D-E mix of 75-25, the
maximum cost of equity capital that Wilkins can accept is closest to:
a. 8%
b. 10.9%
c. 7.6%
d. 9.2%

Let x = cost of equity capital

WACC = (equity fraction)(cost of equity) + (debt fraction)(cost of debt)


10% = 0.25(x) + 0.75[9%(1.2)]

x = (10% – 8.1%)/0.25 = 7.6%


Answer is (c)

Copyright 2021 © McGraw-Hill Education. All rights reserved. No reproduction or distribution without the
prior written consent of McGraw-Hill Education.
42
Solutions to end-of-chapter problems
Basics of Engineering Economy, 3rd edition
Leland Blank and Anthony Tarquin

Chapter 14
Alternative Evaluation Including
Noneconomic Attributes
14.1 St. Mark’s Hospital administrators want to understand the level of satisfaction of
the experience that patients had during their stay in the hospital. The results of
several small group discussions with outgoing patients will be used to identify key
attributes for a full-fledged online survey and analysis to be conducted over the next
year. Some of the topics mentioned during the small group discussions are
summarized below. Use them to identify 4 or 5 possible key attributes for the online
survey.

Small Group Topics


Cleanliness of room and facilities
Noise level of the hospital environment
Response time to a call for assistance
Understanding and friendliness of nurses and doctors
Time to rest and sleep without disturbances
Explanation of what is being done or administered to me
Listening to my comments about pain, laying position, etc.
Concern of cost over what my insurance will cover

Examples of key attributes may be:


Response time to patient calls
Noise factor of the environment
Understanding dimension of staff
Cleanliness/sanitation
Explanation of drugs and treatments administered

14.2 Write 3 or 4 key attributes that you used to determine which major to pursue in
college. If you switched majors one or more times, include an attribute that helped
in making your final choice.

No examples provided since this is an individually-determined answer.

14.3 Why is a public sector project a good example of multiple-attribute evaluation?

Public sector projects attract stakeholders with many different views about
noneconomic attributes making selection of an alternative more complex.

Copyright 2021 © McGraw-Hill Education. All rights reserved. No reproduction or distribution without the
prior written consent of McGraw-Hill Education.
1
14.4 Three alternatives are being evaluated based on six different attributes, all of which are
considered of equal importance. Determine the weight to assign to each attribute.

Wi = 1/6 = 0.1667

14.5 A consultant asked the president of Toyota to assign importance values from 0 to
100 to five attributes that will be included in a multiple-attribute evaluation of a
newly-designed steering mechanism for all Lexus cars. Determine the weight of
each attribute using the president’s importance scores.

Attribute Importance Score


1. Safety 50
2. Cost 40
3. Impact 60
4. Environmental 30
5. Acceptability 70

S = Σsi = 50 + 40 + 60 + 30 + 70 = 250

W1 = 50/250 = 0.20
W2 = 40/250 = 0.16
W3 = 60/250 = 0.24
W4 = 30/250 = 0.12
W5 = 70/250 = 0.28
1.00

14.6 Jill rank ordered 10 attributes in increasing importance and identified them as A, B, …,
J, with a value of 1 assigned to A, 2 to B, etc. (a) Determine the sum of the scores. (b)
Calculate the weight for attribute D. (c) Answer the two questions above if Jill decides
that attribute D has the same importance as J, but all other scores stay the same.

(a) S = 1 + 2 + 3 +… + 10
= n(n+1)/2
= 10(11)/2
= 55

(b) WD = 4/55 = 0.073

(c) S = 1 + 2 + 3 + 10 + 5 + … + 10
= 61

WD = 10/61 = 0.164

Copyright 2021 © McGraw-Hill Education. All rights reserved. No reproduction or distribution without the
prior written consent of McGraw-Hill Education.
2
14.7 A committee of four people submitted the following statements about the attributes
to be used in a weighted attribute evaluation. Use the statements to determine the
normalized weights of each attribute, if scores are assigned between 0 and 10, with
10 indicating the most important factor.

Attribute Statement
1. Flexibility The most important factor
2. Safety 50% as important as uptime
3. Uptime One-half as important as flexibility
4. Speed As important as uptime
5. Rate of return Twice as important as safety

Importance scores with a 10 for attribute 1

Attribute Importance, sj Logic


1 10 Most important (10)
2 2.5 0.5(5) = 2.5
3 5 1/2(10) = 5
4 5 2(2.5) = 5
5 5 2(2.5) = 5
27.5

Wi = si/27.5

Attribute, i Wi
1 0.364
2 0.090
3 0.182
4 0.182
5 0.182
1.000

14.8 Carol and Larry are arranging a birthday party for their to-be 10-year-old daughter,
Yvonne. They asked Yvonne to help design the event by stating her preferences
about several key aspects (attributes) of the party. Use her responses to apply
importance scores from 0 to 100 to each aspect, and determine their sum.

Copyright 2021 © McGraw-Hill Education. All rights reserved. No reproduction or distribution without the
prior written consent of McGraw-Hill Education.
3
Attribute Statement
1. Location 50% as important as video and games
2. Timing One-half as important as location
3. Videos and games available The most important factor
4. Number of invitees Half as important as timing
5. Who is invited The most important factor
6. Cost Give it a score of 10
7. Type of food Twice as important as timing

Attribute Importance Score


1. Location 50 = 0.5(100)
2. Timing 25 = 0.5(50)
3. Videos and games available 100
4. Number of invitees 12.5 = 0.5(25)
5. Who is invited 100
6. Cost 10
7. Type of food 50 = 2(25)

Total = 347.5

14.9 A new restaurant has opened with the expressed goals of providing reasonably
priced food and great customer service. Using an integer scale of −2 to +2, the
manager has “value rated” his view of how they are meeting these two goals, plus
three other attributes: food taste, food temperature, and wait time to be seated.
Separately, two customers were asked to rate their server on the same attributes.
Customer 2, rating Server B, had exactly the opposite ratings when compared to
Customer 1, rating Server A. (a) Complete the value rating for Customer 2. (b)
Using simple addition of the ratings by each individual, determine, on the basis of
this sketchy feedback, if customers appear to agree that the restaurant is meeting its
two primary goals. (c) For Customer 1 and 2 separately, determine the required
total for the two expressed goals (attributes) to make each customer’s total for all
five value ratings equal to that made by the manager.

Server A, Server B,
Attribute Manager Customer 1 Customer 2
Reasonable price +2 0 0
Customer service +2 −2 ?
Taste of food  0 −1 ?
Temperature of food  0 +2 ?
Wait time to seating +1 −1 ?

Copyright 2021 © McGraw-Hill Education. All rights reserved. No reproduction or distribution without the
prior written consent of McGraw-Hill Education.
4
(a) Answers shown in bold red.
Server A, Server B,
Attribute Manager Customer 1 Customer 2
Reasonable price +2 0 0
Customer service +2 –2 +2
Taste of food 0 –1 +1
Temperature of food 0 +2 –2
Wait time to seating +1 –1 +1
Totals +5 –2 +2

(b) Customers do not agree with the manager’s value ratings; goals not being
attained.

(c) Price plus service ratings must total to +5 for each customer to equal the
manager’s total of 5 for all attributes.

14.10 Thomas used the online chat function for two retailers, Better Buy (BB) and
Quick Serv (QS). He had developed four attributes and predetermined weights to
evaluate his online experience. He then used a 0 to 5 (5 is highest) scale to assign
value ratings to each retailer. Use his data to determine the evaluation measure R
for each retailer. Which retailer offered a better experience for Thomas?

Value Rating, Vij

Attribute, i Wi BB QS
1. Clear information 0.5 3 1
2. Wait time 0.2 5 2
3. Courtesy of agent 0.2 2 4
4. Language usage 0.1 1 4

Calculate Rj component for each retailer and add. BB has a higher evaluation
measure (3.0 vs 2.1), indicating a better experience.

BB QS
Attribute, i Wi Vi(BB) Ri(BB) Vi(QS) Ri(QS)
1. Clear information 0.5 3 1.5 1 0.5
2. Wait time 0.2 5 1.0 2 0.4
3. Courtesy of agent 0.2 2 0.4 4 0.8
4. Language usage 0.1 1 0.1 4 0.4
Rj totals 3.0 2.1

Copyright 2021 © McGraw-Hill Education. All rights reserved. No reproduction or distribution without the
prior written consent of McGraw-Hill Education.
5
14.11 An airport baggage handling department has evaluated two proposals for baggage
delivery conveyor systems. A present worth analysis at i = 15% per year of
estimated revenues and costs resulted in PWA = $460,000 and PWB = $395,000. In
addition to this economic measure, three more attributes were independently
assigned a relative importance score from 0 to 100 by the department manager
and a senior team supervisor.

Importance Scores
Attribute Manager Supervisor
1. Economics 80 25
2. Durability 35 80
3. Safety 30 100
4. Maintainability 20 90

Separately, you have used the four attributes to value rate the two proposals on a
scale of 0 to 1.0 as shown in the following table. (The economic attribute was
rated using the PW values.)

Value Rating
Attribute Proposal A Proposal B
1. Economics 1.00 0.90
2. Durability 0.35 0.50
3. Safety 1.00 0.20
4. Maintainability 0.25 1.00

Select the better proposal using each of the following methods:


a. Weighted evaluation of the department manager
b. Weighted evaluation of the team supervisor
c. Present worth

(a) Calculate Rj using department manager scores. Result is 0.76 vs 0.70; select
proposal A.

Wi = Importance score
Sum

Copyright 2021 © McGraw-Hill Education. All rights reserved. No reproduction or distribution without the
prior written consent of McGraw-Hill Education.
6
Attribute, Importance Rj
i by Manager Wi A B__
1 80 0.48 0.48 0.43
2 35 0.21 0.07 0.11
3 30 0.18 0.18 0.04
4 20 0.12 0.03 0.12
165 0.76 0.70

(b) Calculate Rj using the team supervisor scores. Result is 0.59 vs 0.59; either
proposal is acceptable.

Attribute, Importance Rj
i by Supervisor Wi A B__
1 25 0.08 0.08 0.07
2 80 0.27 0.09 0.14
3 100 0.34 0.34 0.07
4 90 0.31 0.08 0.31
295 0.59 0.59

(c) Select A, since PWA is larger

Conclusion: Two methods indicate A, but the supervisor’s score


indicates indifference between A and B.

14.12 The importance values (0 to 100) for five attributes are shown below. The weight
to assign to attribute 1 is:
a. 0.16
b. 0.20
c. 0.22
d. 0.25

Attribute Importance Score


1 55
2 45
3 85
4 30
5 60

S = Σsi = 55 + 45 + 85 + 30 + 60 = 275
W1 = 55/275 = 0.20
Answer is (b)

Copyright 2021 © McGraw-Hill Education. All rights reserved. No reproduction or distribution without the
prior written consent of McGraw-Hill Education.
7
14.13 For eight attributes rank ordered in terms of increasing importance, the weighting
of the 6th attribute is closest to:
a. 0.17
b. 0.14
c. 0.08
d. 0.03

S = 1 + 2 + 3 +… + 8 = 8(9)/2 = 36
WC = 6/36 = 0.167
Answer is (a)

14.14 All of the following are excellent attribute identification approaches, except
a. Employing small group discussions
b. Using the same attributes that competing entities use
c. Getting input from experts with relevant experience
d. Surveying the stakeholders

Answer is (b)

14.15 If attributes of first cost, safety, and environmental concerns had importance
scores of 100, 75, and 50, respectively, the weight for environmental concerns is
closest to:
a. 0.44
b. 0.33
c. 0.22
d. 0.11

Wenv = 50/(100+75+50)
= 0.22
Answer is (c)

14.16 Consumers have completed a study to assign importance scores and value ratings
(0 to 100) to two consumer discretionary products, namely toasters (O and P) on
the bases of cost and reliability. The results are shown below. The conclusion of
the study, using the R evaluation measure taken to one decimal, is that:
a. O is clearly better
b. P is clearly better
c. P is marginally better
d. P is marginally poorer

Copyright 2021 © McGraw-Hill Education. All rights reserved. No reproduction or distribution without the
prior written consent of McGraw-Hill Education.
8
Value Ratings
Attribute Weight Product O Product P
1. Cost 0.6 73 60
2. Reliability 0.4 60 80

Product O Product P
Attribute, i Weight ViO RiO ViP RiP
1. Cost 0.6 73 43.8 60 36.0
2. Reliability 0.4 60 24.0 80 32.0
R totals 67.8 68.0
Answer is (c)

Copyright 2021 © McGraw-Hill Education. All rights reserved. No reproduction or distribution without the
prior written consent of McGraw-Hill Education.
9

You might also like